You are on page 1of 235

Mc lc

Li ni u 4
Cc thnh vin tham gia bin son 5
1 Cc bt ng thc kinh in 6
1.1 Bt ng thc gia trung bnh cng v trung bnh nhn (AM-GM). . . . . . . . . 6
1.2 Bt ng thc gia trung bnh cng v trung bnh iu ho (AM-HM). . . . . . . 6
1.3 Bt ng thc Cauchy - Schwarz. . . . . . . . . . . . . . . . . . . . . . . . . . . . 6
1.4 Bt ng thc Holder. . . . . . . . . . . . . . . . . . . . . . . . . . . . . . . . . . 7
1.5 Bt ng thc Chebyshev. . . . . . . . . . . . . . . . . . . . . . . . . . . . . . . . 7
1.6 Bt ng thc Minkowski. . . . . . . . . . . . . . . . . . . . . . . . . . . . . . . . 7
1.7 Bt ng thc Schur. . . . . . . . . . . . . . . . . . . . . . . . . . . . . . . . . . . 7
1.8 Bt ng thc Vornicu - Schur. . . . . . . . . . . . . . . . . . . . . . . . . . . . . 8
1.9 Bt ng thc Bernoulli. . . . . . . . . . . . . . . . . . . . . . . . . . . . . . . . . 8
1.10 Ba tiu chun SOS thng gp. . . . . . . . . . . . . . . . . . . . . . . . . . . . . 9
2 Mt s nh gi quen thuc 9
3 Tuyn tp bt ng thc 10
3.1 Bi 1.1 n bi 1.40 . . . . . . . . . . . . . . . . . . . . . . . . . . . . . . . . . . . 10
3.2 Bi 2.1 n bi 2.40 . . . . . . . . . . . . . . . . . . . . . . . . . . . . . . . . . . . 39
3.3 Bi 3.1 n bi 3.40 . . . . . . . . . . . . . . . . . . . . . . . . . . . . . . . . . . . 59
3.4 Bi 4.1 n bi 4.40 . . . . . . . . . . . . . . . . . . . . . . . . . . . . . . . . . . . 80
3.5 Bi 5.1 n bi 5.40 . . . . . . . . . . . . . . . . . . . . . . . . . . . . . . . . . . . 104
3.6 Bi 6.1 n bi 6.40 . . . . . . . . . . . . . . . . . . . . . . . . . . . . . . . . . . . 132
3.7 Bi 7.1 n bi 7.40 . . . . . . . . . . . . . . . . . . . . . . . . . . . . . . . . . . . 148
3.8 Bi 8.1 n bi 8.40 . . . . . . . . . . . . . . . . . . . . . . . . . . . . . . . . . . . 168
3.9 Bi 9.1 n bi 9.40 . . . . . . . . . . . . . . . . . . . . . . . . . . . . . . . . . . . 193
3.10 Bi 10.1 n bi 10.40 . . . . . . . . . . . . . . . . . . . . . . . . . . . . . . . . . 211
3
Li ni u
Bin vn mi nhp nh vi nhng con sng dt vo b, thuyn vn mi lnh nh theo tng con
sng i vo i dng, v trong t lin cuc sng vn c nhiu bt cp cn ang xy ra,. . . , tt
c nhng iu u l cc bt ng thc trong phm tr c th ca tng lnh vc. Trong ton
hc cng vy ni n bt ng thc l chng ta ni n mt lp bi ton kh m n cha bn
trong c nhiu li gii p l k lm say m bit bao nhiu ngi.
Trong thi i cng ngh thng tin vi vic kt ni internet bn c th giao lu hc hi c rt
nhiu v cc phng php lm bi bt ng thc, hoc hc hi vi nhiu cun sch v bt ng
thc ang by bn trn th trng nhng c mt cun sch bt ng thc hay vi s hi t
tinh hoa kin thc ca nhiu ngi th iu chnh l im mnh ca cun sch bt ng thc
m cc bn ang cm trn tay.
"Tuyn Tp Bt ng Thc" vi khong bn trm bi ton bt ng thc chn lc c gi ti
t cc bn tr, cc thy c gio yu ton trn mi min ca t quc, bao gm cc bi ton
bt ng thc mi sng to, cc bi ton bt ng thc kh, cc bi ton bt ng thc hay v
th v m cc bn tr mun chia s vi mi ngi. iu to nn s hp dn, tnh cp nht v
thi i ca cun sch ny.
Bn c hy nhm nhi vi nhng li gii hay, nhng tng c o, nhng sng kin l k trong
cch gii tng bi ton t rt kinh nghim hc tp cho mnh, gip cho bn thm yu, thm
tin vo vic gii nhiu bi ton bt ng thc.
Vi tinh thn lm vic nghim tc, ham hc hi nhm bin tp xin c gi li cm n su
sc ti tt c cc bn tham gia gi bi v gii bi, ng thi cng xin by t s cm
n v knh trng ti thy gio Chu Ngc Hng - THPT Ninh Hi - Ninh Thun nhit
tnh c vn k thut latex. Nhm bin tp cng xin gi li cm n ti ban qun tr din n
http://forum.mathscope.org/index.php c v, ng vin anh em trong qu trnh lm vic
ngy hm nay chng ta c mt cun sch hay, c gi tr cao v kin thc chuyn mn m li hon
ton min ph v ti chnh.
"TUYN TP BT NG THC" chnh thc c pht hnh trn cng ng mng nhng
ngi yu ton, t thi mt lung gi mi em li nhiu iu mi l cho hc sinh, l ti
liu tham kho hu ch cho gio vin trong vic ging dy v hc tp bt ng thc.
Do thi gian gp rt v trnh c hn, d rt c gng song nhng sai st l kh trnh khi rt
mong nhn c s thng cm, chia s, gp ca cc bn nhm bin tp hon thin cun sch
tt hn. Mi kin ng gp xin gi v a ch hoangquan9@gmail.
Thay mt nhm bin son, ti xin chn thnh cm n!
H Ni, ngy 10 thng 8 nm 2011
i din nhm bin son
Ch bin
Hong Minh Qun-Batigoal
4
Cc thnh vin tham gia bin son
Ni dung
Hong Minh Qun - THPT Ngc To - H Ni.
Tng Hi Tun - THPT Nguyn c Cnh - TP. Thi Bnh.
L c Cnh - THPT Chuyn L Hng Phong-Nam nh.
o Thi Hip - PTNK - HQG HCM.
Phm Tun Huy - PTNK - HQG HCM.
Phm Quang Hng - THPT Cao B Qut - H Ni.
Phm Tin Kha - THPT Chuyn L Hng Phong - TP. HCM.
Nguyn Vn Khnh - THPT Chuyn Bc Ninh - TP. Bc Ninh.
Nguyn Th Nguyn Khoa - THCS Nguyn Tri Phng - TP. Hu.
Mc c Tr - Hi Dng.
L
A
T
E
X
H tr k thut Latex
1. Chu Ngc Hng - THPT Ninh Hi -Ninh Thun.
2. Cc thnh vin trong nhm bin son.
Trnh by ba
Hong Minh Qun - THPT Ngc To - H Ni.
5
1 Cc bt ng thc kinh in
1.1 Bt ng thc gia trung bnh cng v trung bnh nhn (AM-
GM).
Nu a
1
, a
2
, . . . , a
n
l cc s thc khng m, th
a
1
+ a
2
+ . . . + a
n
n
n

a
1
a
2
. . . a
n
.
ng thc xy ra khi v ch khi a
1
= a
2
= . . . = a
n
.
1.2 Bt ng thc gia trung bnh cng v trung bnh iu ho (AM-
HM).
Nu a
1
, a
2
, . . . , a
n
l cc s thc dng, th
a
1
+ a
2
+ . . . + a
n
n

n
1
a
1
+
1
a
2
+ . . . +
1
a
n
.
ng thc xy ra khi v ch khi a
1
= a
2
= . . . = a
n
.
Thc cht y l mt h qu trc tip ca bt ng thc Cauchy - Schwarz. Hai trng hp thng
c s dng nht ca bt ng thc ny l khi n = 3 hay n = 4.
Vi n = 3, ta c
a + b + c
3

3
1
a
+
1
b
+
1
c
,
1
a
+
1
b
+
1
c

9
a + b + c
.
Vi n = 4, ta c
a + b + c + d
4

4
1
a
+
1
b
+
1
c
+
1
d
,
1
a
+
1
b
+
1
c
+
1
d

16
a + b + c + d
.
1.3 Bt ng thc Cauchy - Schwarz.
Dng s cp ca n c pht biu nh sau:
Nu a
1
, a
2
, . . . , a
n
v b
1
, b
2
, . . . , b
n
l cc s thc tu , th
(a
1
b
1
+ a
2
b
2
+ . . . + a
n
b
n
)
2
(a
2
1
+ a
2
2
+ . . . + a
2
n
)(b
1
+ b
2
+ . . . + b
2
n
).
ng thc xy ra khi v ch khi
a
1
b
1
=
a
2
b
2
= . . . =
a
n
b
n
, trong ta s dng quy c: nu mu
bng 0 th t cng bng 0.
Trong nh gi trn, chn a
i
=
x
i

y
i
,b
i
=

y
i
vi x
i
, y
i
R; y
i
> 0, ta thu c bt ng thc
Cauchy - Schwarz dng phn thc:
Nu x
1
, x
2
, . . . , x
n
l cc s thc v y
1
, y
2
, . . . , y
n
, l cc s thc dng, th
x
2
1
y
1
+
x
2
2
y
2
+ . . . +
x
2
n
y
n

(x
1
+ x
2
+ . . . + x
n
)
2
y
1
+ y
2
+ . . . + y
n
.
ng thc xy ra khi v ch khi
x
1
y
1
=
x
2
y
2
= . . . =
x
n
y
n
.
6
1.4 Bt ng thc Holder.
Cho x
ij
(i = 1, 2, . . . , m; j = 1, 2, . . . , n) l cc s thc khng m. Khi ta c
m

i=1
_
n

j=1
x
ij
_1
m

j=1
_
m

i=1
x
1
m
ij
_
.
Tng qut hn, nu p
1
, p
2
, . . . , p
n
l cc s thc dng tho mn p
1
+ p
2
+ . . . + p
n
= 1, th
m

i=1
_
n

j=1
x
ij
_
p
i

j=1
_
m

i=1
x
p
i
ij
_
.
1.5 Bt ng thc Chebyshev.
Cho hai dy s thc a
1
a
2
. . . a
n
v b
1
, b
2
, . . . , b
n
. Khi
1. Nu b
1
b
2
. . . b
n
th n
n

i=1
a
i
b
i

_
n

i=1
a
i
__
n

i=1
b
i
_
;
2. Nu b
1
b
2
. . . b
n
th n
n

i=1
a
i
b
i

_
n

i=1
a
i
__
n

i=1
b
i
_
.
1.6 Bt ng thc Minkowski.
Cho hai dy s dng a
1
, a
2
, . . . , a
n
v b
1
, b
2
, . . . , b
n
. Vi mi r 1, ta c
_
n

i=1
(a
i
+ b
i
)
r
_1
r

_
n

i=1
a
r
i
_1
r
+
_
n

i=1
b
r
i
_1
r
.
Trng hp r = 2 l trng hp thng c s dng nht ca bt ng thc Minkowski. Khi
ta c

_
n

i=1
(a
i
+ b
i
)
2

_
n

i=1
a
2
i
+

_
n

i=1
b
2
i
.
1.7 Bt ng thc Schur.
Cho cc s thc khng m a, b, c. Khi vi mi s thc dng r, ta c
a
r
(a b)(a c) + b
r
(b a)(b c) + c
r
(c a)(c b) 0.
ng thc xy ra khi v ch khi a = b = c, hoc a = 0 v b = c, hoc cc hon v tng ng.
Hai trng hp thng c s dng nht ca bt ng thc Schur l r = 1 v r = 2.
Vi r = 1, ta c bt ng thc Schur bc ba
a
3
+ b
3
+ c
3
+ 3abc ab(a + b) + bc(b + c) + ca(c + a),
(a + b + c)
3
+ 9abc 4(a + b + c)(ab + bc + ca),
(b c)
2
(b + c a) + (c a)
2
(c + a b) + (a b)
2
(a + b c) 0,
7
a
2
+ b
2
+ c
2
+
9abc
a + b + c
2(ab + bc + ca),
a
b + c
+
b
c + a
+
c
a + b
+
4abc
(a + b)(b + c)(c + a)
2.
Vi r = 2, ta thu c bt ng thc Schur bc bn
a
4
+ b
4
+ c
4
+ abc(a + b + c) ab(a
2
+ b
2
) + bc(b
2
+ c
2
) + ca(c
2
+ a
2
).
1.8 Bt ng thc Vornicu - Schur.
Vi mi s thc a, b, c v x, y, z 0, bt ng thc
x(a b)(a b) + y(b c)(b a) + z(c a)(c b) 0
ng nu mt trong cc iu kin sau c tho mn
1. a b c v x y;
2. a b c v z y;
3. a b c v x + z y;
4. a b c 0 v ax by;
5. a b c 0 v cz by;
6. a b c 0 v ax + cz by;
7. x, y, z l di ba cnh ca mt tam gic;
8. x, y, z l bnh phng di ba cnh ca mt tam gic;
9. ax, by, cz l di ba cnh ca mt tam gic;
10. ax, by, cz l bnh phng di ba cnh ca mt tam gic;
11. Tn ti mt hm li t : I R
+
, trong I l tp xc nh ca a, b, c, sao cho x =
t(a), y = t(b), z = t(c).
1.9 Bt ng thc Bernoulli.
Nu 1 hoc 0 th (1 + x)

1 + x, x > 1.
Nu 0 1 th (1 + x)

1 + x, x > 1.
8
1.10 Ba tiu chun SOS thng gp.
Gi s a b c v c: S
a
(b c)
2
+ S
b
(c a)
2
+ S
c
(a b)
2
0(S
a
, S
b
, S
c
l cc hm cha
bin a, b, c).
Khi bt ng thc ng nu tha mn mt trong cc tiu chun.
1.S
b
0, S
b
+ S
c
0, S
b
+ S
a
0.
2.Vi a, b, c > 0 tha mn S
b
0, S
c
0, a
2
S
b
+ b
2
S
a
0.
3.S
b
0, S
c
0, S
a
(b c) + S
b
(a c) 0
2 Mt s nh gi quen thuc
1 Vi mi s thc a, b, ta lun c
2(a
2
+ b
2
) (a + b)
2
Chng minh. rng
2(a
2
+ b
2
) (a + b)
2
= (a b)
2
0,
do ta c iu phi chng minh.
ng thc xy ra khi v ch khi a = b. 2
2 Vi mi s thc a, b, c, ta lun c
a
2
+ b
2
+ c
2
ab + bc + ca
Chng minh. rng
a
2
+ b
2
+ c
2
(ab + bc + ca) =
1
2
[(a b)
2
+ (b c)
2
+ (c a)
2
] 0,
do vy ta c iu phi chng minh.
ng thc xy ra khi v ch khi a = b = c. 2
Lu . T nh gi ny ta suy ra
(a + b + c)
2
3(ab + bc + ca),
v
3(a
2
+ b
2
+ c
2
) (a + b + c)
2
.
3 Vi mi s thc dng a, b, c, ta lun c
1
a
+
1
b
+
1
c

9
a + b + c
Chng minh. y l mt kt qu c cp trn. Li gii c th s dng bt ng thc
AM-HM hoc Cauchy - Schwarz. ng thc xy ra khi v ch khi a = b = c. 2
9
3 Tuyn tp bt ng thc
3.1 Bi 1.1 n bi 1.40
1.1 Cho x, y, z l cc s thc dng tha mn x + y + z = 1. Chng minh rng:
8
x
+ 8
y
+ 8
z
4
x+1
+ 4
y+1
+ 4
z+1
Li gii. t a = 2
x
, b = 2
y
, c = 2
z
. Khi iu kin cho c vit li thnh
a, b, c > 0; abc = 2
x+y+z
= 64,
v ta cn chng minh
a
3
+ b
3
+ c
3
4(a
2
+ b
2
+ c
2
).
rng ta c ng thc
a
3
+ 32 6a
2
= (a 4)
2
(a + 2),
t s dng gi thit a > 0 ta suy ra a
3
+ 32 6a
2
. Thit lp cc bt ng thc tng t cho
b v c v cng v theo v cc bt ng thc thu c, ta c
a
3
+ b
3
+ c
3
+ 96 6(a
2
+ b
2
+ c
2
).
Nh vy kt thc chng minh ta cn ch ra rng
6(a
2
+ b
2
+ c
2
) 4(a
2
+ b
2
+ c
2
) + 96,
hay 2(a
2
+b
2
+c
2
) 96. Tuy nhin bt ng thc ny ng theo bt ng thc AM-GM cho ba
s:
2(a
2
+ b
2
+ c
2
) 2.3
3

a
2
b
2
c
2
= 6
3

4096 = 96.
Nh vy php chng minh n y hon tt.2
1.2 Cho a, b, c l cc s thc tho mn a 4, b 5, c 6 v a
2
+ b
2
+ c
2
= 90. Tm gi tr
nh nht ca biu thc:
P = a + b + c
Li gii. t a = m+ 4, b = n + 5, c = p + 6, khi m, n, p 0 v t gi thit a
2
+b
2
+c
2
= 90
ta suy ra
m
2
+ n
2
+ p
2
+ 8m + 10n + 12p = 13.
rng ta c ng thc sau
(m+n +p)
2
+ 12(m+n +p) = (m
2
+n
2
+p
2
+ 8m+ 10n + 12p) + 2(mn +np +pm+ 2m+n).
n y ta s dng cc gi thit cho c
(m + n + p)
2
+ 12(m + n + p) 13,
t ta suy ra m+n +p 1. Thay m = a 4, n = b 5, p = c 6 ta suy ra a +b +c 10 hay
P 16.
10
Cui cng, vi a = 4, b = 5, c = 7 (tho mn cc iu kin cho) ta c P = 16 nn ta kt lun
16 l gi tr nh nht ca biu thc P.
Php chng minh hon tt. 2
1.3 Cho x, y, z l cc s thc tho mn xy + yz + 3zx = 1. Tm gi tr nh nht ca biu
thc:
P = x
2
+ y
2
+ z
2
Li gii. t a =
9 + 3

17
4
v b =
3 +

17
4
, khi a = 3b v a +1 = 2b
2
= c =
13 + 3

17
4
. p
dng bt ng thc AM-GM ta thu c cc bt ng thc sau
x
2
+ b
2
y
2
2bxy,
by
2
+ z
2
2byz,
a(z
2
+ x
2
) 2azx.
n y ta cng v theo v cc bt ng thc thu c c
(a + 1)(x
2
+ z
2
) + 2b
2
y
2
2b(xy + yz) + 2azx,
hay c(x
2
+ y
2
+ z
2
) 2b(xy + yz + 3zx). T ta thay cc gi tr ca xy + yz + 3zx, b v c
c
P = x
2
+ y
2
+ z
2

17 3
2
.
Cui cng, vi x = z =
1
4

17
v y =
_
13

17 51
34
(tho mn gi thit) th P =

17 3
2
nn ta
kt lun

17 3
2
l gi tr nh nht ca biu thc P.
Php chng minh hon tt.2
1.4 Cho a, b, c l cc s thc dng tho mn a + b + c = 1. Chng minh rng:
a
7
+ b
7
a
5
+ b
5
+
b
7
+ c
7
b
5
+ c
5
+
c
7
+ a
7
c
5
+ a
5

1
3
Li gii. Trc ht ta c ng thc sau
2(a
7
+ b
7
) (a
2
+ b
2
)(a
5
+ b
5
) = (a b)
2
(a + b)(a
4
+ a
3
b + a
2
b
2
+ ab
3
+ b
4
),
do vy t gi thit a, b 0 ta suy ra
a
7
+ b
7
a
5
+ b
5

a
2
+ b
2
2
.
Hon ton tng t ta cng c
b
7
+ c
7
b
5
+ c
5

b
2
+ c
2
2
v
c
7
+ a
7
c
5
+ a
5

c
2
+ a
2
2
. n y ta cng v theo
v ba bt ng thc thu c c
a
7
+ b
7
a
5
+ b
5
+
b
7
+ c
7
b
5
+ c
5
+
c
7
+ a
7
c
5
+ a
5
a
2
+ b
2
+ c
2
.
11
Nh vy kt thc chng minh ta cn ch ra rng
a
2
+ b
2
+ c
2

1
3
.
Tuy nhin bt ng thc trn ng do
a
2
+ b
2
+ c
2

1
3
= a
2
+ b
2
+ c
2

(a + b + c)
2
3
=
(a b)
2
+ (b c)
2
+ (c a)
2
3
0.
Nh vy php chng minh n y hon tt.2
1.5 Cho a, b, c l cc s thc dng. Chng minh rng:
b
2
c
a
3
(b + c)
+
c
2
a
b
3
(c + a)
+
a
2
b
c
3
(a + b)

1
2
(a + b + c)
Li gii. Ta p dng AM-GM cho ba s nh sau:
b
2
c
a
3
(b + c)
+
b + c
4bc
+
1
2b
3
3

b
2
c
a
3
(b + c)
.
(b + c)
4bc
.
1
2b
=
3
2a
,
t ta suy ra
b
2
c
a
3
(b + c)

3
2a

3
4b

1
4c
.
Thit lp hai bt ng thc tng t v cng li, ta suy ra
b
2
c
a
3
(b + c)
+
c
2
a
b
3
(c + a)
+
a
2
b
c
3
(a + b)

_
3
2

3
4

1
4
_
(a + b + c) =
1
2
(a + b + c).
Php chng minh hon tt.2
1.6 Cho a, b, c l cc s thc khng m. Chng minh rng:
(a + b + c)
3
6

3(a b)(b c)(c a)


Li gii. Bt ng thc ban u mang tnh hon v gia cc bin nn khng mt tnh tng qut,
ta gi s a = max {a, b, c}.
Vi a b c th v phi l biu thc khng dng, trong khi v tri l biu thc khng m nn
bt ng thc cn chng minh hin nhin ng. Do vy ta xt trng hp a c b. Khi bnh
phng hai v ta thu c bt ng thc tng ng sau:
(a + b + c)
6
108[(a b)(b c)(c a)]
2
.
rng cc bin khng m, v vi vic sp th t nh trn th
[(a b)(b c)(c a)]
2
= [(a b)(c b)(a c)]
2
(a c)
2
a
2
c
2
.
n y ta p dng bt ng thc AM-GM c
4(a c)
2
a
2
c
2
= (a c)
2
.2ac.2ac
[(a c)
2
+ 2ac + 2ac]
3
27
=
(a + c)
6
27
,
t ta suy ra
[(a b)(b c)(c a)]
2

(a + c)
6
108
,
12
v nh vy ta chng minh c bt ng thc ban u v
(a + b + c)
6
(a + c)
6
108[(a b)(b c)(c a)]
2
.
Php chng minh hon tt.2
1.7 Cho a, b, c l cc s thc dng tho mn a + b + c =
1
a
+
1
b
+
1
c
. Chng minh rng:
2(a + b + c)

a
2
+ 3 +

b
2
+ 3 +

c
2
+ 3
Li gii. D thy bt ng thc cn chng minh tng ng vi mi bt ng thc trong dy
sau
(2a

a
2
+ 3) + (2b

b
2
+ 3) + (2c

c
2
+ 3) 0,
a
2
1
2a +

a
2
+ 3
+
b
2
1
2b +

b
2
+ 3
+
c
2
1
2c +

c
2
+ 3
0,
a
2
1
a
2 +
_
1 +
3
a
2
+
b
2
1
b
2 +
_
1 +
3
b
2
+
c
2
1
c
2 +
_
1 +
3
c
2
0.
Cc bt ng thc trn u mang tnh i xng gia cc bin nn khng mt tnh tng qut ta
hon ton c th gi s a b c. Khi khng kh ta suy ra
a
2
1
a

b
2
1
b

c
2
1
c
v
1
2 +
_
1 +
3
a
2

1
2 +
_
1 +
3
b
2

1
2 +
_
1 +
3
b
2
.
Nh vy theo bt ng thc Chebyshev ta c
a
2
1
a
2 +
_
1 +
3
a
2
+
b
2
1
b
2 +
_
1 +
3
b
2
+
c
2
1
c
2 +
_
1 +
3
c
2

1
3
_

a
2
1
a
_
_
_
_
_

1
2 +
_
1 +
3
a
2
_
_
_
_
Nhng theo gi thit ta li c

a
2
1
a
= (a + b + c)
_
1
a
+
1
b
+
1
c
_
= 0
nn ta suy ra
a
2
1
a
2 +
_
1 +
3
a
2
+
b
2
1
b
2 +
_
1 +
3
b
2
+
c
2
1
c
2 +
_
1 +
3
c
2
0, v v vy bt ng thc cho
cng ng.
Php chng minh hon tt.2
1.8 Cho a, b, c l cc s thc dng tho mn a + b + c = 3. Chng minh rng:
ab

c
2
+ 3
+
bc

a
2
+ 3
+
ca

b
2
+ 3

3
2
13
Li gii. Trc ht rng
ab + bc + ca
(a + b + c)
2
3
=
_
(a b)
2
+ (b c)
2
+ (c a)
2
6
_
0,
do t gi thit ta suy ra ab + bc + ca 3. Nh vy
ab

c
2
+ 3

ab

c
2
+ ab + bc + ca
=
ab
_
(c + a)(b + c)
.
n y ta p dng bt ng thc AM-GM c
ab

c
2
+ 3

1
2
_
ab
c + a
+
ab
b + c
_
.
Thit lp hai bt ng thc tng t v cng li, ta suy ra dy cc nh gi sau
ab

c
2
+ 3
+
bc

a
2
+ 3
+
ca

b
2
+ 3

1
2
__
ab
c + a
+
bc
c + a
_
+
_
bc
a + b
+
ca
a + b
_
+
_
ca
b + c
+
ab
b + c
__
,
ab

c
2
+ 3
+
bc

a
2
+ 3
+
ca

b
2
+ 3

a + b + c
2
,
t vi lu a + b + c = 3 ta suy ra bt ng thc cho l ng.
Php chng minh hon tt.2
1.9 Cho a, b, c l cc s thc dng thay i bt k. Chng minh rng:
_
b + c
a
+
c + a
b
+
a + b
c
_
2
4(ab + bc + ca)
_
1
a
2
+
1
b
2
+
1
c
2
_
Li gii 1. D thy rng bt ng thc ban u tng ng vi mi bt ng thc trong dy
sau
[ab(a + b) + bc(b + c) + ca(c + a)]
2
4(a + b + c)(a
2
b
2
+ b
2
c
2
+ c
2
a
2
)

a
2
b
2
(a + b)
2
+ 2abc[

a(a + b)(a + c)] 4


_

a
3
b
3
+ abc[

ab(a + b)]
_
Tuy nhin rng

a
2
b
2
(a + b)
2
4(

a
3
b
3
) =

a
2
b
2
(a b)
2
0
v
2abc[

a(a + b)(a + c)] 4


_
abc[

ab(a + b)]
_
= 2abc[a
3
+ b
3
+ c
3
+ 3abc

ab(a + b)] 0,
do bt ng thc ban u l ng. Php chng minh n y hon tt.2
Li gii 2. Bt ng thc ban u mang tnh hon v gia cc bin, nn khng mt tnh tng
qut, ta gi s b = max {a, b, c}.
Ta p dng bt ng thc AM-GM nh sau
_
b + c
a
+
c + a
b
+
a + b
c
_
2
=
__
a
b
+
b
a
+
a
c
_
+
_
b
c
+
c
b
+
c
a
__
2
4
_
a
b
+
b
a
+
a
c
__
b
c
+
c
b
+
c
a
_
.
14
Nh vy kt thc chng minh, ta cn ch ra rng
_
a
b
+
b
a
+
a
c
__
b
c
+
c
b
+
c
a
_
(ab + bc + ca)
_
1
a
2
+
1
b
2
+
1
c
2
_
.
Tuy nhin bng php bin i tng ng ta c
(b a)(b c)
ca
0,
l mt nh gi ng do ta gi s b = max {a, b, c}.
Php chng minh n y hon tt.2
Li gii 3. Bt ng thc ban u mang tnh i xng gia cc bin nn khng mt tnh tng
qut, ta gi s b nm gia a v c.
Ta p dng bt ng thc AM-GM nh sau:
4(ab + bc + ca)
_
1
a
2
+
1
b
2
+
1
c
2
_

_
ab + bc + ca
ca
+ ca
_
1
a
2
+
1
b
2
+
1
c
2
__
2
.
Nh vy kt thc chng minh, ta cn ch ra rng
b + c
a
+
c + a
b
+
a + b
c

ab + bc + ca
ca
+ ca
_
1
a
2
+
1
b
2
+
1
c
2
_
.
Thc hin php bin i tng ng ta c bt ng thc
(a b)(b c)
b
2
0,
tuy nhin y li l mt nh gi ng do ta gi s b nm gia a v c.
Php chng minh n y hon tt.2
Nhn xt. Li gii u tin khng mang nhiu ngha lm, v n n thun ch l bin i tng
ng km theo mt cht tinh trong s dng cc nh gi quen thuc v c bn. y ta bn
thm v hai li gii bng AM-GM.
Ta nhn thy rng pht biu ca bi ton c dng "Chng minh rng A
2
4BC" ( y
A =
_
b + c
a
+
c + a
b
+
a + b
c
_
2
, B = ab + bc + ca v C =
1
a
2
+
1
b
2
+
1
c
2
. Nhn xt ny kh c
bit, n gip ta lin tng n mt nh gi quen thuc sau bng AM-GM:
(x + y)
2
4xy x, y 0.
Do vy, mt cch t nhin ta ngh ra hai hng gii quyt bi ton trn bng AM-GM:
1. Biu din A = X +Y , vi X v Y l hai i lng thch hp, sau p dng bt ng thc
AM-GM c A
2
4XY , t i chng minh XY BC; hoc
15
2. Biu din BC =
B
D
.CD, vi D l mt i lng thch hp, sau p dng bt ng thc
AM-GM c 4BC
_
B
D
+ CD
_
2
, t i chng minh A
B
D
+ CD.
y ta hiu cm t "thch hp" l nh th no? Lu rng mt trong nhng iu cn
trong mi chng minh bt ng thc l cn phi n gin ho bt ng thc cn chng minh. Ta
c th tm cch gim bc, chun ho iu kin, . . ., nhng tu chung li, ta lun mun bt ng
thc cn chng minh tr nn n gin nht c th, t p dng nh nhng cc nh gi
quen thuc hoc bin i tng ng. y ta tm cch thu gn nh gi sau cng theo kiu
trit tiu mt lng ng k cc phn t chung, tc l nh gi XY BC hoc A
B
D
+CD,
cc i lng X, Y, D c chn sao cho hai v ca bt ng thc c nhiu phn t chung
ta rt gn. C th:
Hng 1. Trc tin ta vit li A v khai trin tch BC nh sau:
A =
b
a
+
c
a
+
c
b
+
a
b
+
a
c
+
b
c
= X + Y,
BC =
a
c
+
c
b
+
b
a
+
a
b
+
b
c
+
c
a
+
ca
b
2
+
ab
c
2
+
bc
a
2
.
rng trong BC c phn t
ca
b
2
, nn ta cn c
a
b
v
c
b
X v Y tng ng:
X =
a
b
+ . . . , Y =
c
b
+ . . .
Mt khc, trong BC c phn t
a
b
, m Y c
c
b
nn ta cn phn t
a
c
trong X:
X =
a
b
+
a
c
+ . . . , Y =
c
b
+ . . .
Tip tc, trong BC c phn t
ab
c
2
, nn ta cn c
a
c
v
b
c
X v Y tng ng:
X =
a
b
+
a
c
+ . . . , Y =
c
b
+
b
c
+ . . .
Tip tc nh vy ta s tm c hai i lng X, Y chng hn nh sau:
X =
a
b
+
b
a
+
a
c
, Y =
b
c
+
c
b
+
c
a
,
v ta c c li gii th hai. Cn lu rng y khng phi l cch chn duy nht.
Hng 2. Xt hiu sau
A
B
D
CD =
b + c
a
+
c + a
b
+
a + b
c

ab + bc + ca
D
D
_
1
a
2
+
1
b
2
+
1
c
2
_
.
rng trong hiu trn th h s ca bin b bng
1
c
+
1
a

c + a
D
,
nh vy tm cch thu gn bt ng thc, ti sao ta khng cho h s ca bin b bng khng?
C th, nu chn D = ca th
16
A
B
D
CD =
b + c
a
+
c + a
b
+
a + b
c

ab + bc + ca
ca
ca
_
1
a
2
+
1
b
2
+
1
c
2
_
=
(a b)(b c)
b
2
,
v nh vy ta c li gii th ba.
1.10 Cho a, b, c l cc s thc dng tho mn a + b + c = 1. Tm gi tr ln nht ca biu
thc:
P = ab + bc + ca +
5
2
[(a + b)

ab + (b + c)

bc + (c + a)

ca]
Li gii. Trc ht ta p dng bt ng thc AM-GM nh sau:
2(a + b)
2
+ 2ab =
(a + b)
2
2
+
(a + b)
2
2
+
(a + b)
2
2
+
(a + b)
2
2
+ 2ab 5
5
_
ab(a + b)
8
8
v
(a + b)
3
(2

ab)
3
= 8(

ab)
3
,
t kt hp hai bt ng thc ny c
2(a + b)
2
+ 2ab 5(a + b)

ab.
Thit lp hai bt ng thc tng t v cng li, ta suy ra
5[(a + b)

ab + (b + c)

bc + (c + a)

ca] 4(a
2
+ b
2
+ c
2
) + 6(ab + bc + ca)
n y ta cng thm 2(ab + bc + ca) vo mi v c
2(ab + bc + ca) + 5[(a + b)

ab + (b + c)

bc + (c + a)

ca] 4(a + b + c)
2
,
t ta suy ra P 2(a + b + c)
2
= 2.
Cui cng, vi a = b = c =
1
3
(tho mn iu kin) th P = 2 nn ta suy ra 2 l gi tr ln nht
ca biu thc P.
Php chng minh hon tt.2
1.11 Cho a, b, c l cc s thc dng tho mn
1
a
+
1
b
+
1
c
16(a+b +c). Chng minh rng:
1
(a + b + 2

a + c)
3
+
1
(b + c + 2

b + a)
3
+
1
(c + a + 2

c + b)
3

8
9
Li gii. Trc ht ta p dng bt ng thc AM-GM nh sau:
a + b +
_
a + c
2
+
_
a + c
2
3
3
_
(a + b)(a + c)
2
,
t ta suy ra
1
(a + b + 2

a + c)
3

2
27(a + b)(a + c)
.
17
Cng v theo v bt ng thc ny vi hai bt ng thc tng t cho ta
1
(a + b + 2

a + c)
3
+
1
(b + c + 2

b + a)
3
+
1
(c + a + 2

c + b)
3

4(a + b + c)
27(a + b)(b + c)(c + a)
.
Hn na, theo mt kt qu quen thuc, ta li c
(a + b)(b + c)(c + a)
8
9
(a + b + c)(ab + bc + ca),
do vy
1
(a + b + 2

a + c)
3
+
1
(b + c + 2

b + a)
3
+
1
(c + a + 2

c + b)
3

1
6(ab + bc + ca)
. ()
n y ta s dng gi thit v nh gi c bn (ab + bc + ca)
2
3abc(a + b + c) c
16(a + b + c)
1
a
+
1
b
+
1
c

3(a + b + c)
ab + bc + ca
,
t suy ra ab + bc + ca
3
16
. Kt hp vi () ta suy ra
1
(a + b + 2

a + c)
3
+
1
(b + c + 2

b + a)
3
+
1
(c + a + 2

c + b)
3

8
9
.
Php chng minh n y hon tt.2
Nhn xt.
1. C th thy nh gi ban u a +b +
_
a + c
2
+
_
a + c
2
3
3
_
(a + b)(a + c)
2
chnh l im
mu cht gii quyt bi ton. Thc ra nh gi ny khng kh ngh ti v bi ngm
gi cho chng ta phi p dng bt ng thc AM-GM cho ba s.
2. Sau khi nh gi bng AM-GM, ta c th s dng lun gi thit a v bt ng thc
thun nht sau:
(a + b + c)
(a + b)(b + c)(c + a)

3(ab + bc + ca)
8abc(a + b + c)
.
Bt ng thc ny c th c chng minh bng nhiu cch khc nhau.
1.12 Cho a, b, c l cc s thc dng tho mn a + b + c =
1
a
+
1
b
+
1
c
. Chng minh rng:
5(a + b + c) 7 + 8abc
Li gii. Trc ht t gi thit ta c
a + b + c =
1
a
+
1
b
+
1
c

9
a + b + c
,
t suy ra a + b + c = 3.
Cng t gi thit ta c ab +bc +ca = abc(a +b +c), t y ta suy ra bt ng thc sau l tng
ng vi bt ng thc cn chng minh
5(a + b + c)
2
7(a + b + c) + 8(ab + bc + ca).
18
rng ta c nh gi c bn sau:
(a + b + c)
2
3(ab + bc + ca),
do vy c kt lun cho bi ton ta cn ch ra rng
5(a + b + c)
2
7(a + b + c) +
8(a + b + c)
2
3
,
hay a + b + c 3, l mt nh gi ng do ta chng minh trn.
Do vy bt ng thc ban u c chng minh xong. Bi ton kt thc.2
1.13 Cho a, b, c l cc s thc dng tho mn
1
a
+
1
b
+
1
c
16(a+b +c). Chng minh rng:
1
2 + a
2
+
1
2 + b
2
+
1
2 + c
2
1
Li gii. Bt ng thc cn chng minh tng ng vi
a
2
2 + a
2
+
b
2
2 + b
2
+
c
2
2 + c
2
1.
p dng bt ng thc Cauchy - Schwarz, ta c
a
2
2 + a
2
+
b
2
2 + b
2
+
c
2
2 + c
2

(a + b + c)
2
a
2
+ b
2
+ c
2
+ 6
.
Nh vy kt thc chng minh ta cn ch ra rng
(a + b + c)
2
a
2
+ b
2
+ c
2
+ 6
1.
Thc hin php khai trin tng ng ta c ab + bc + ca 3. Tuy nhin bt ng thc ny
ng nh vo gi thit ca bi ton. Lu rng t gi thit ta c
ab + bc + ca = abc(a + b + c),
v theo mt nh gi quen thuc th abc(a + b + c)
(ab + bc + ca)
2
3
, t ta suy ra
ab + bc + ca
(ab + bc + ca)
2
3
,
hay ab + bc + ca 3. Php chng minh n y hon tt.2
1.14 Cho a, b, c, d l cc s thc dng tho mn a +b +c +d = 1. Tm gi tr nh nht ca
biu thc:
P =
1
a
2
+ b
2
+ c
2
+ d
2
+
1
abc
+
1
bcd
+
1
cda
+
1
dab
Li gii. K hiu

l tng hon v. Trc ht ta s dng AM-GM v gi thit c cc nh


gi sau:
abcd
_
a + b + c + d
4
_
4
=
1
256
,
ab + ac + ad + bc + bd + cd
3(a + b + c + d)
2
8
=
3
8
.
Kt hp cc nh gi ny vi bt ng thc Cauchy - Schwarz ta suy ra c cc bt ng thc
sau:
19
1.
1
a
2
+ b
2
+ c
2
+ d
2
+

1
4ab

7
2
a
2
+ b
2
+ c
2
+ d
2
+

4ab
=
49
(a + b + c + d)
2
+ 2

ab

49
1 + 2.
3
8
= 28,
2. 7

1
4ab

7.6
2

4ab

7.36
4.
3
8
= 168.
Mt khc p dng bt ng thc AM-GM cho bn s ta li c

a
bcd
4
_
1
4abcd
4

_
1
1
256
= 64.
Kt hp ba bt ng thc va chng minh trn, ta suy ra
1
a
2
+ b
2
+ c
2
+ d
2
+ 2

1
ab
+

a
bcd
28 + 168 + 64 = 260.
Hn na, s dng gi thit a + b + c + d = 1 ta suy ra
P =
1
a
2
+ b
2
+ c
2
+ d
2
+ (a + b + c + d)
_
1
abc
+
1
bcd
+
1
cda
+
1
dab
_
=
1
a
2
+ b
2
+ c
2
+ d
2
+ 2

1
ab
+

a
bcd
.
Do vy P 260.
Cui cng, vi a = b = c = d =
1
4
(tho mn iu kin) th P = 260 nn ta suy ra 260 l gi tr
nh nht ca biu thc P.
Php chng minh hon tt.2
1.15 Cho x, y, z l cc s thc dng tho mn xyz = 1. Chng minh rng:
18
_
1
x
3
+ 1
+
1
y
3
+ 1
+
1
z
3
+ 1
_
(x + y + z)
3
Li gii. S dng gi thit, d thy bt ng thc cn chng minh tng ng vi mi bt ng
thc trong dy sau:
18
_
3
x
3
x
3
+ 1

y
3
y
3
+ 1

z
3
z
3
+ 1
_
(x + y + z)
3
,
18
_
x
2
x
2
+ yz
+
y
2
y
2
+ zx
+
z
2
z
2
+ xy
_
+ (x + y + z)
3
54. ()
p dng bt ng thc Cauchy - Schwarz, ta c
x
2
x
2
+ yz
+
y
2
y
2
+ zx
+
z
2
z
2
+ xy

(x + y + z)
2
x
2
+ y
2
+ z
2
+ xy + yz + zx
.
20
Nh vy nu k hiu V T() l v tri ca bt ng thc () th ta c
V T()
18(x + y + z)
2
x
2
+ y
2
+ z
2
+ xy + yz + zx
+ (x + y + z)
3
.
n y ta p dng bt ng thc AM-GM c
V T() 2

18(x + y + z)
5
x
2
+ y
2
+ z
2
+ xy + yz + zx
.
Nh vy kt thc chng minh, ta cn ch ra rng
(x + y + z)
5

81
2
(x
2
+ y
2
+ z
2
+ xy + yz + zx).
Trc ht ta p dng bt ng thc AM-GM nh sau:
(x+y +z)
6
= [(x
2
+y
2
+z
2
) +(xy +yz +zx) +(xy +yz +zx)]
3
27(x
2
+y
2
+z
2
)(xy +yz +zx)
2
.
Hn na, theo mt kt qu quen thuc ta c (xy + yz + zx)
2
3xyz(x + y + z), do
(x + y + z)
6
81xyz(x
2
+ y
2
+ z
2
)(x + y + z),
hay (x + y + z)
5
81(x
2
+ y
2
+ z
2
) do xyz = 1. Nh vy ta cn ch ra rng
2(x
2
+ y
2
+ z
2
) x
2
+ y
2
+ z
2
+ xy + yz + zx.
Tuy nhin bng php bin i tng ng ta thu c
1
2
[(a b)
2
+ (b c)
2
+ (c a)
2
] 0,
l mt bt ng thc hin nhin ng. Do vy bt ng thc ban u c chng minh.
Bi ton kt thc.2
1.16 Cho a, b, c l cc s thc dng tho mn a
4
+ b
4
+ c
4
= 3. Chng minh rng:
a
2
b + c
+
b
2
c + a
+
c
2
a + b

3
2
Li gii. Ta s i chng minh
a
2
b + c
+
b
2
c + a
+
c
2
a + b

3
2
4
_
a
4
+ b
4
+ c
4
3
,
t s dng gi thit suy ra kt lun cho bi ton. Tht vy, p dng bt ng thc Holder,
ta c
_
a
2
b + c
+
b
2
c + a
+
c
2
a + b
_
2
[a
2
(b + c)
2
+ b
2
(c + a)
2
+ c
2
(a + b)
2
] (a
2
+ b
2
+ c
2
)
3
.
Hn na, theo mt kt qu quen thuc, ta c
2(a
2
+ b
2
) (a + b)
2
,
21
t y ta thit lp hai nh gi tng t c
_
a
2
b + c
+
b
2
c + a
+
c
2
a + b
_
2
[2a
2
(b
2
+ c
2
) + 2b
2
(c
2
+ a
2
) + 2c
2
(a
2
+ b
2
)] (a
2
+ b
2
+ c
2
)
3
,
hay
a
2
b + c
+
b
2
c + a
+
c
2
a + b

1
2

(a
2
+ b
2
+ c
2
)
3
a
2
b
2
+ b
2
c
2
+ c
2
a
2
.
Nh vy kt thc chng minh ta cn ch ra rng

(a
2
+ b
2
+ c
2
)
3
a
2
b
2
+ b
2
c
2
+ c
2
a
2
3
4
_
a
4
+ b
4
+ c
4
3
.
Thc hin php bin i tng ng ta thu c
(a
2
+ b
2
+ c
2
)
6
27(a
4
+ b
4
+ c
4
)(a
2
b
2
+ b
2
c
2
+ c
2
a
2
)
2
.
Tuy nhin bt ng thc trn ng nu ta p dng bt ng thc AM-GM nh sau:
(a
2
+ b
2
+ c
2
)
6
= [(a
4
+ b
4
+ c
4
) + (a
2
b
2
+ b
2
c
2
+ c
2
a
2
) + (a
2
b
2
+ b
2
c
2
+ c
2
a
2
)]
3
27(a
4
+ b
4
+ c
4
)(a
2
b
2
+ b
2
c
2
+ c
2
a
2
)
2
Php chng minh n y hon tt.2
1.17 Cho a, b, c l cc s thc dng tho mn a + b + c = 3. Chng minh rng:
a
a + b + 1
+
b
b + c + 1
+
c
c + a + 1
1
Li gii. S dng gi thit, ta thy rng cc bt ng thc sau l tng ng vi bt ng thc
cn chng minh
a
4 c
+
b
4 a
+
c
4 b
1,
a(4 a)(4 b) + b(4 b)(4 c) + c(4 c)(4 a) (4 a)(4 b)(4 c),
a
2
b + b
2
c + c
2
a + abc 4.
Bt ng thc trn mang tnh hon v gia cc bin nn khng mt tnh tng qut, ta gi s c
nm gia a v b. Khi
a(a c)(b c) 0.
Thc hin php khai trin ta c a
2
b+c
2
a a
2
c+abc. T y ta cng thm i lng (b
2
c+abc)
vo hai v c
a
2
b + b
2
c + c
2
a + abc a
2
c + b
2
c + 2abc = c(a + b)
2
.
n y ta p dng AM-GM nh sau:
c(a + b)
2
=
1
2
2c(a + b)(a + b)
(2c + a + b + a + b)
3
2.27
= 4,
t suy ra a
2
b + b
2
c + c
2
a + abc 4, tc l bt ng thc ban u c chng minh.
22
Bi ton hon tt.2
1.18 Cho a, b, c l cc s thc khng m tho mn a + b + c = 1. Chng minh rng:
25
27
(1 4ab)
2
+ (1 4bc)
2
+ (1 4ca)
2
3
Li gii.
1. Chng minh (1 4ab)
2
+ (1 4bc)
2
+ (1 4ca)
2
3.
Trc ht ta c
1 = a + b + c a + b 2

ab,
t suy ra 1 4ab. n y ta s dng gi thit cc bin khng m c
0 1 4ab 1,
t m (1 4ab)
2
1. Thit lp hai nh gi tng t v cng li ta c ngay iu phi
chng minh.
2. Chng minh (1 4ab)
2
+ (1 4bc)
2
+ (1 4ca)
2

25
27
.
D thy bt ng thc trn tng ng vi mi bt ng thc trong dy sau:
3 8(ab + bc + ca) + 16(a
2
b
2
+ b
2
c
2
+ c
2
a
2
)
25
27
,
ab + bc + ca 2(a
2
b
2
+ b
2
c
2
+ c
2
a
2
)
7
27
.
rng ta c ng thc sau
ab 2a
2
b
2

5
9
_
ab
1
9
_

7
81
= 2
_
ab
1
9
_
2
,
do ta suy ra ab 2a
2
b
2

5
9
_
ab
1
9
_
+
7
81
. n y ta thit lp hai nh gi tng t
v cng li c
ab + bc + ca 2(a
2
b
2
+ b
2
c
2
+ c
2
a
2
)
5
9
_
ab + bc + ca
1
3
_

7
27
.
Hn na, theo mt kt qu quen thuc ta c ab +bc +ca
(a + b + c)
2
3
=
1
3
, do vy ta suy
ra
ab + bc + ca 2(a
2
b
2
+ b
2
c
2
+ c
2
a
2
)
7
27
,
tc l bt ng thc ban u c chng minh.
Tm li ta chng minh c
25
27
(1 4ab)
2
+ (1 4bc)
2
+ (1 4ca)
2
3. Php chng minh
hon tt.2
1.18 Cho x, y, z l cc s thc dng tho mn xy + yz + zx = 1. Chng minh rng:
1
1 + xy + z
2
+
1
1 + yz + x
2
+
1
1 + zx + y
2

9
5
Li gii. t x =
1
a
, y =
1
b
, z =
1
c
. Khi s dng gi thit xy + yz + zx = 1, ta thy rng
23
1
1 + xy + z
2
=
xy + yz + zx
x
2
+ xy + xz + 2yz
=
1
ab
+
1
bc
+
1
ca
1
a
2
+
1
ab
+
1
ac
+
2
bc
=
a(a + b + c)
2a
2
+ ab + bc + ca
,
do bt ng thc cho tng ng vi

a
2a
2
+ ab + bc + ca

9
5(a + b + c)
.
Nhn c hai v ca bt ng thc ny vi ab + bc + ca v ch rng
a(ab + bc + ca)
2a
2
+ ab + bc + ca
= a
2a
3
2a
2
+ ab + bc + ca
,
ta c
2

a
3
2a
2
+ ab + bc + ca
+
9(ab + bc + ca)
5(a + b + c)
a + b + c.
p dng bt ng thc Cauchy - Schwarz, ta c

a
3
2a
2
+ ab + bc + ca

(

a
2
)
2

a(2a
2
+ ab + bc + ca)
=
(

a
2
)
2
6abc + (

a)(2

a
2

ab)
.
(1)
Mt khc, t bt ng thc c bn (ab + bc + ca)
2
3abc(a + b + c), ta li c
3abc
(ab + bc + ca)
2
a + b + c
. (2)
Kt hp (1) v (2), ta suy ra

a
3
2a
2
+ ab + bc + ca

(

a
2
)
2
(

a)
2(

ab + bc + ca)
2
+ (

a)
2
(2

a
2

ab)
.
=
(

a
2
)(

a)
2

a
2
+ 3

ab
.
Cui cng ta ch cn chng minh
2(a
2
+ b
2
+ c
2
)(a + b + c)
2(a
2
+ b
2
+ c
2
) + 3(ab + bc + ca)
+
9(ab + bc + ca)
5(a + b + c)
a + b + c.
Sau khi khai trin v rt gn, ta c bt ng thc hin nhin ng
(ab + bc + ca)(a
2
+ b
2
+ c
2
ab bc ca) 0.
Bi ton c chng minh xong.2
24
1.19 Cho a, b, c l cc s thc dng tho mn a + b + c =
1
a
+
1
b
+
1
c
. Chng minh rng:
(b + c a)(c + a b)(a + b c) 1
Li gii 1. Bt ng thc cn chng minh mang tnh i xng gia cc bin, do khng mt
tnh tng qut, ta gi s a b c. Khi a + b c 0 v c + a b 0.
Nu b +c a < 0 th bt ng thc hin nhin ng do (b +c a)(c +ab)(a+b c) 0 < 1. Do
ta ch cn gii quyt bi ton trong trng hp b +c a 0. Lc ny ta t x = b +c a, y =
c + a b, z = a + b c. Khi ta vit li iu kin nh sau
x, y, z 0; x + y + z =
2
x + y
+
2
y + z
+
2
z + x
,
v ta cn chng minh
xyz 1.
Ta s gii quyt bi ton bng phng php phn chng. Tht vy, gi s rng xyz > 1. Khi
s dng bt ng thc AM-GM, ta suy ra
x + y + z =
2
x + y
+
2
y + z
+
2
z + x

1

xy
+
1

yz
+
1

zx
,
hay

x +

y +

xyz(x + y + z). Hn na, ta cng c xyz > 1 nn

x +

y +

z > x + y + z.
Tuy nhin theo bt ng thc AM-GM, ta li c

x
x + 1
2
. Ta thit lp thm hai nh gi
tng t na c
x + y + z + 3
2

x +

y +

z > x + y + z,
hay x + y + z < 3. Nhng y l mt nh gi sai v theo mt kt qu quen thuc, ta c
x + y + z =
2
x + y
+
2
y + z
+
2
z + x

9
x + y + z
,
dn ti x + y + z 3. Mu thun ny chng t iu gi s ban u l sai, do vy xyz 1.
Php chng minh hon tt.2
Li gii 2. Bt ng thc cn chng minh mang tnh i xng gia cc bin, do khng mt
tnh tng qut, ta gi s a b c. Khi a + b c 0 v c + a b 0.
Nu b +c a < 0 th bt ng thc hin nhin ng do (b +c a)(c +ab)(a+b c) 0 < 1. Do
ta ch cn gii quyt bi ton trong trng hp b +c a 0. Lc ny ta t x = b +c a, y =
c + a b, z = a + b c. Khi ta vit li iu kin nh sau
x, y, z 0; x + y + z =
2
x + y
+
2
y + z
+
2
z + x
,
v ta cn chng minh
xyz 1.
25
Ta s gii quyt bi ton bng phng php phn chng. Tht vy, gi s rng xyz > 1. Khi ,
t gi thit, ta suy ra
(x + y + z)
2
(xy + yz + zx) = 2(x + y + z)
2
+ 2(xy + yz + zx) + xyz(x + y + z). ()
Tuy nhin, theo bt ng thc AM-GM v theo iu gi s trn, ta c cc nh gi
xy + yz + zx 3
3
_
x
2
y
2
z
2
> 3,
x + y + z 3
3

xyz > 3,
do vy ta suy ra
2(x + y + z)
2
(xy + yz + zx)
3
> 2(x + y + z)
2
,
2(x + y + z)
2
(xy + yz + zx)
9
> 2(xy + yz + zx),
(x + y + z)
2
(xy + yz + zx)
9
> xyz(x + y + z).
Cng v theo v cc nh gi trn li, ta c
(x + y + z)
2
(xy + yz + zx) > 2(x + y + z)
2
+ 2(xy + yz + zx) + xyz(x + y + z),
tri vi (). Mu thun ny chng t iu gi s ban u l sai, do vy xyz 1.
Php chng minh hon tt.2
1.20 Cho a, b, c l cc s thc dng tho mn a + b + c = 3. Chng minh rng:
1
5a
2
+ ab + bc
+
1
5b
2
+ bc + ca
+
1
5c
2
+ ca + ab

3
7
Li gii. p dng bt ng thc Cauchy - Schwarz, ta c
1
5a
2
+ ab + bc
+
1
5b
2
+ bc + ca
+
1
5c
2
+ ca + ab
=

cyc
(b + c)
2
(b + c)
2
(5a
2
+ ab + bc)

4(a + b + c)
2

cyc
(b + c)
2
(5a
2
+ ab + bc)
.
Theo , ta cn chng minh rng
4(a + b + c)
2

cyc
(b + c)
2
(5a
2
+ ab + bc)

3
7
.
S dng gi thit a + b + c = 3, ta thy rng bt ng thc trn tng ng vi
28(a + b + c)
4
27[

cyc
(b + c)
2
(5a
2
+ ab + bc)].
Sau khi khai trin v rt gn, ta c
28

a
4
+ 58

cyc
a
3
b + 85

cyc
ab
3
156

a
2
b
2
+ 15abc(a + b + c).
26
chng minh bt ng thc ny, trc ht ta ch n cc nh gi c bn sau (thu c bng
bt ng thc AM-GM):

cyc
a
3
b +

cyc
ab
3
2

a
2
b
2
,

a
4
+

cyc
ab
3

cyc
a
3
b +

cyc
ab
3
2

a
2
b
2
,

a
4

a
2
b
2
abc(a + b + c).
T ta suy ra
58

cyc
a
3
b + 58

cyc
ab
3
116

a
2
b
2
,
27

a
4
+ 27

cyc
ab
3
54

a
2
b
2
,

a
4
+ 14

a
2
b
2
15abc(a + b + c).
Cng v theo v cc nh gi trn, ta thu c bt ng thc cn chng minh.
Bi ton kt thc.2
1.21 Cho a, b, c l cc s thc dng thay i bt k. Chng minh rng:
b + c
2a
2
+ bc
+
c + a
2b
2
+ ca
+
a + b
2c
2
+ ab

6
a + b + c
Li gii. Nhn c hai v ca bt ng thc cho 4(a + b + c), ta c
4(b + c)(a + b + c)
2a
2
+ bc
+
4(c + a)(a + b + c)
2b
2
+ ca
+
4(a + b)(a + b + c)
2c
2
+ ab
24.
Do
4(b + c)(a + b + c)
2a
2
+ bc
=
(a + 2b + 2c)
2
2a
2
+ bc

a
2
2a
2
+ bc
nn ta c

(a + 2b + 2c)
2
2a
2
+ bc
24 +

a
2
2a
2
+ bc
.
Bt ng thc ny c suy ra bng cch cng hai bt ng thc
a
2
2a
2
+ bc
+
b
2
2b
2
+ ca
+
c
2
2c
2
+ ab
1,
(a + 2b + 2c)
2
2a
2
+ bc
+
(b + 2c + 2a)
2
2b
2
+ ca
+
(c + 2c + 2b)
2
2c
2
+ ab
25.
Do
a
2
2a
2
+ bc
=
1
2

bc
2(2a
2
+ bc)
nn bt ng thc th nht tng ng vi
bc
2a
2
+ bc
+
ca
2b
2
+ ca
+
ab
2c
2
+ ab
1,
ng v theo bt ng thc Cauchy - Schwarz

bc
2a
2
+ bc

_

bc
_
2

bc(2a
2
+ bc)
= 1.
27
By gi ta s chng minh bt ng thc th hai. y l bt ng thc i xng nn khng mt
tnh tng qut, ta gi s c = min{a, b, c}. t t =
b + c
2
, ta s chng minh
(a + 2b + 2c)
2
2a
2
+ bc
+
(b + 2c + 2a)
2
2b
2
+ ca

2(3t + 2c)
2
2t
2
+ tc
. ()
S dng bt ng thc Cauchy - Schwarz, ta c
(a + 2b + 2c)
2
2a
2
+ bc
+
(b + 2c + 2a)
2
2b
2
+ ca

[b(a + 2b + 2c) + a(b + 2c + 2a)]
2
b
2
(2a
2
+ bc) + a
2
(2b
2
+ ca)
=
2(4t
2
ab + 2tc)
2
2a
2
b
2
3abtc + 4t
3
c
.
V tc ab t
2
nn
2a
2
b
2
3abtc (2t
4
3t
3
c) = (t
2
ab)(2t
2
+ 2ab 3tc) 0,
t dn n
(a + 2b + 2c)
2
2a
2
+ bc
+
(b + 2c + 2a)
2
2b
2
+ ca

2(4t
2
ab + 2tc)
2
2a
2
b
2
3abtc + 4t
3
c

2(3t
2
+ 2tc)
2
2t
4
3t
3
c + 4t
3
c
=
2(3t + 2c)
2
2t
2
+ tc
.
Mt khc, ta li c
(c + 2c + 2b)
2
2c
2
+ ab

(4t + c)
2
t
2
+ 2c
2
. ()
Kt hp hai nh gi () v (), ta a bi ton v vic chng minh
2(3t + 2c)
2
2t
2
+ tc
+
(4t + c)
2
t
2
+ 2c
2
25.
Sau khi thu gn, ta c bt ng thc hin nhin ng
c(31t + 16c)(t c)
2
t(2t + c)(t
2
+ 2c
2
)
0.
Bi ton c chng minh xong.2
1.22 Cho a, b, c, d l cc s thc khng m tha mn a
2
+b
2
+c
2
+d
2
= 1. Chng minh rng:
a
b
2
+ 1
+
b
c
2
+ 1
+
c
d
2
+ 1
+
d
a
2
+ 1

4(a

a + b

b + c

c + d

d)
2
5
Li gii. p dng bt ng thc Cauchy - Schwarz, ta c
a
b
2
+ 1
+
b
c
2
+ 1
+
c
d
2
+ 1
+
d
a
2
+ 1
=
a
3
a
2
b
2
+ a
2
+
b
3
b
2
c
2
+ b
2
+
c
3
c
2
d
2
+ c
2
+
d
3
d
2
a
2
+ d
2

(a

a + b

b + c

c + d

d)
2
a
2
+ b
2
+ c
2
+ d
2
+ a
2
b
2
+ b
2
c
2
+ c
2
d
2
+ a
2
d
2
.
28
Nh vy, kt thc chng minh, ta cn ch ra rng
a
2
+ b
2
+ c
2
+ d
2
+ a
2
b
2
+ b
2
c
2
+ c
2
d
2
+ a
2
d
2

5
4
,
hay (a
2
+ c
2
)(b
2
+ d
2
)
1
4
. Tuy nhin y li l nh gi ng v theo bt ng thc AM-GM:
(a
2
+ c
2
)(b
2
+ d
2
)
(a
2
+ c
2
+ b
2
+ d
2
)
2
4
=
1
4
,
do vy bt ng thc ban u c chng minh xong.
Bi ton kt thc.2
1.23 Cho x,y,z l cc s thc thuc on [0, 1]. Chng minh rng:
x
3
_
1 + y
3
+
y
3

1 + z
3
+
z
3

1 + x
3

3
3

1 + xyz
Li gii. Do x, y, z [0, 1] nn ta c
x
3
_
1 + y
3
+
y
3

1 + z
3
+
z
3

1 + x
3

1
3
_
1 + y
3
+
1
3

1 + z
3
+
1
3

1 + x
3
.
rng theo bt ng thc Holder, ta c nh gi sau vi mi s thc dng a, b, c:
(a + b + c)
3
9(a
3
+ b
3
+ c
3
),
hay (a + b + c)
3
_
9(a
3
+ b
3
+ c
3
). S dng nh gi ny, ta c
1
3
_
1 + y
3
+
1
3

1 + z
3
+
1
3

1 + x
3

9
_
1
1 + y
3
+
1
1 + x
3
+
1
1 + z
3
_
.
Nh vy, kt thc chng minh, ta cn ch ra rng
1
1 + y
3
+
1
1 + x
3
+
1
1 + z
3

3
1 + xyz
. ()
rng vi hai s thc a, b thay i trong on [0, 1] ta lun c
1
1 + a
2
+
1
1 + b
2

2
1 + ab
=
(ab 1)(a b)
2
(1 + a
2
)(1 + b
2
)(1 + ab)
0.
S dng nh gi ny, ta c
1
1 + x
3
+
1
1 + y
3
+
1
1 + z
3
+
1
1 + xyz

2
1 +
_
x
3
y
3
+
2
1 +
_
z
4
xy

4
1 + xyz
.
Do vy nh gi () c chng minh, dn n bt ng thc ban u ng.
Php chng minh hon tt.2
1.24 Cho a, b, c l cc s thc dng thay i bt k. Chng minh rng:
a
2
b + c
+
b
2
a + c
+
c
2
a + b

a + b + c
2
29
Li gii 1. p dng bt ng thc Cauchy-Schwartz, ta c
a
2
b + c
+
b
2
a + c
+
c
2
a + b

(a + b + c)
2
2(a + b + c)
=
a + b + c
2
.
Php chng minh hon tt. 2
Li gii 2. p dng bt ng thc AM-GM cho hai s dng, ta c
a
2
b + c
+
b + c
4
a.
Cng v theo v nh gi ny vi hai nh gi tng t khc, ta c:
a
2
b + c
+
b
2
a + c
+
c
2
a + b
+
a + b + c
2
a + b + c,
t ta thu c bt ng thc cn chng minh.
Bi ton kt thc.2
Li gii 3. Bt ng thc ban u mang tnh i xng gia cc bin, do khng mt tnh tng
qut, ta gi s a b c. Khi ta c
1
b + c

1
a + c

1
a + b
.
Nh vy, theo bt ng thc Chebyshev, ta c
a
2
b + c
+
b
2
a + c
+
c
2
a + b

1
3
.(a
2
+ b
2
+ c
2
).(
1
a + b
+
1
b + c
+
1
a + c
).
n y ta p dng hai nh gi c bn x
2
+ y
2
+ z
2

(x + y + z)
2
3
v
1
x
+
1
y
+
1
z

9
x + y + z
c
a
2
b + c
+
b
2
a + c
+
c
2
a + b

1
3
.
(a + b + c)
2
3
.
9
2(a + b + c)
=
a + b + c
2
.
Php chng minh hon tt.2
1.25 Cho a, b, c l cc s thc dng thay i bt k. Chng minh rng:
4

3 + a
4
+
4

3 + b
4
+
4

3 + c
4

4
_
108(a + b + c)
Li gii. p dng bt ng thc Holder, ta c
(1 + 3)(1 + 3)(1 + 3)(a
4
+ 3) (a + 3)
4
,
t suy ra
4

3 + a
4

3 + a
4

64
. Thit lp cc nh gi tng t v cng li, ta c
4

3 + a
4
+
4

3 + b
4
+
4

3 + c
4

9 + a + b + c
4

64
.
Hn na, theo bt ng thc AM-GM, ta c
9 + a + b + c = 3 + 3 + 3 + (a + b + c) 4
4
_
27(a + b + c),
30
nh vy
4

3 + a
4
+
4

3 + b
4
+
4

3 + c
4

4
4
_
27(a + b + c)
4

64
=
4
_
108(a + b + c).
Php chng minh hon tt.2
1.26 Cho a, b l cc s thc dng tho mn ab 1. Chng minh rng:
1
1 + a
2
+
1
1 + b
2

2
1 + ab
Li gii. Thc hin php bin i tng ng, ta thu c dy cc nh gi sau:
2 + a
2
+ b
2
a
2
b
2
+ a
2
+ b
2
+ 1

2
1 + ab
,
2 + 2ab + a
3
b + b
3
a + a
2
+ b
2
2a
2
b
2
2a
2
2b
2
2 0,
(ab 1)(a b)
2
0.
nh gi cui cng ng do ab 1, do vy bt ng thc ban u c chng minh.
Bi ton kt thc.2
1.27 Cho a, b, c l cc s thc dng tho mn a + b + c = 1. Chng minh rng:
c + ab
a + b
+
a + bc
b + c
+
b + ac
a + c
2
Li gii. rng ta c
c + ab = c(a + b + c) + ab = (c + a)(c + b),
do vy bt ng thc cn chng minh tng ng vi
(c + a)(c + b)
a + b
+
(b + a)(b + c)
a + c
+
(a + b)(a + c)
b + c
2.
p dng nh gi c bn x
2
+ y
2
+ z
2
xy + yz + zx, ta thy nh gi trn ng do
(c + a)(c + b)
a + b
+
(b + a)(b + c)
a + c
+
(a + b)(a + c)
b + c
b + c + a + b + c + a = 2.
Php chng minh hon tt.2
1.28 Cho x, y, z l cc s thc dng tho mn 2x + 3y + z = 1. Tm gi tr nh nht ca
biu thc:
P = x
3
+ y
3
+ z
3
Li gii. p dng bt ng thc Holder, ta c
P(2

2 + 3

3 + 1)
2
= (x
3
+ y
3
+ z
3
)(2

2 + 3

3 + 1)(2

2 + 3

3 + 1)
(2x + 3y + z)
3
= 1.
31
Nh vy P
1
(2

2 + 3

3 + 1)
2
.
Cui cng, vi x =

2
2

2 + 3

3 + 1
,y =

3
2

2 + 3

3 + 1
v z =
1
2

2 + 3

3 + 1
(tho mn iu
kin) th P =
1
(2

2 + 3

3 + 1)
2
nn ta kt lun
1
(2

2 + 3

3 + 1)
2
l gi tr nh nht ca biu
thc P.
Bi ton kt thc.2
1.29 Cho a, b, c l cc s thc tho mn a
2
+ab +b
2
= 3. Tm gi tr nh nht v gi tr ln
nht ca biu thc
P = a
2
ab 3b
2
Li gii. Vi b = 0 th t gi thit ta suy ra a
2
= 3, t biu thc P c gi tr l 3.
Vi b = 0, xt biu thc
Q =
P
3
=
a
2
ab 3b
2
a
2
+ ab + b
2
=
x
2
x 3
x
2
+ x + 1
,
trong x =
a
b
. T y ta suy ra
(Q1)x
2
+ (Q + 1)x + Q + 3 = 0.
Coi l mt phng trnh theo n x. Xt bit thc ca phng trnh trn, ta thy rng
phng trnh trn c nghim th
(Q + 1)
2
4(Q1)(Q + 3) 0,
t y ta suy ra
3 4

3
3
Q
3 + 4

3
3
. Hn na, do P = 3Q nn ta c
3 4

3 P 3 + 4

3.
Cui cng, vi a =
_
2

3 v b =
_
2 +

3 th P = 3 4

3; vi a =
_
2 +

3 v
b =
_
2

3 th P = 3 + 4

3 nn ta kt lun 3 4

3 v 3 + 4

3 ln lt l gi tr nh
nht v gi tr ln nht ca biu thc P.
Bi ton kt thc.2
1.30 Cho a, b, c l cc s thc tho mn a
2
+ 2b
2
= 3c
2
. Chng minh rng:
1
a
+
2
b

3
c
Li gii. T gi thit, ta suy ra (3c)
2
= (a
2
+ 2b
2
)(1 + 2). T y ta p dng bt ng thc
Cauchy - Schwarz c
(3c)
2
(a + 2b)
2
,
t suy ra 3c a + 2b. ()
Hn na, cng theo bt ng thc Cauchy - Schwarz, ta c
1
a
+
1
b
+
1
b

9
a + 2b
. ()
32
Kt hp hai nh gi () v (), ta thu c bt ng thc cn chng minh.
Bi ton kt thc.2
1.31 Cho x, y, z l cc s thc khng m tho mn x
2
+ y
2
+ z
2
= 3. Tm gi tr ln nht
ca biu thc:
P = xy + yz + zx +
5
x + y + z
Li gii. rng
P =
(x + y + z)
2
x
2
y
2
z
2
2
+
5
x + y + z
=
(x + y + z)
2
2
+
5
x + y + z

3
2
,
t t t = x + y + z, ta a bi ton v vic tm gi tr ln nht ca biu thc
Q = t
2
+
10
t
.
rng t nh gi x
2
+y
2
+z
2
(x +y +z)
2
3(x
2
+y
2
+z
2
), ta suy ra

3 t 3, do vy
t
2
+
10
t

37
3
=
(t 3)(3t
2
+ 9t 10)
3t
0.
Nh vy Q
37
3
, v v P =
Q
2

3
2
nn
P
37
6

3
2
=
14
3
.
Cui cng, vi x = y = z = 1 (tho mn iu kin) th P =
14
3
nn ta kt lun
14
3
l gi tr ln
nht ca biu thc P.
Bi ton kt thc.2
1.32 Cho x, y l cc s thc dng tho mn 2y > x. Chng minh rng:
1
x
3
(2y x)
+ x
2
+ y
2
3
Li gii. Ta thy rng
1
x
3
(2y x)
+ x
2
+ y
2
=
1
x
2
(2xy x
2
)
+ +x
2
+ (y
2
+ x
2
x
2
),
v v x
2
+ y
2
2xy theo bt ng thc AM-GM nn
1
x
3
(2y x)
+ x
2
+ y
2

1
x
2
(2xy x
2
)
+ x
2
+ (2xy x
2
).
n y ta p dng bt ng thc AM-GM mt ln na c
1
x
3
(2y x)
+ x
2
+ y
2
3
3

1
x
2
(2xy x
2
)
.x
2
.(2xy x
2
) = 3.
Php chng minh hon tt.2
33
1.33 Cho a, b, c l cc s thc dng tho mn ab + bc + ca = 2abc. Chng minh rng:
1
a(2a 1)
2
+
1
b(2b 1)
2
+
1
c(2c 1)
2

1
2
Li gii. t m =
1
a
; n =
1
b
; p =
1
c
. Khi iu kin cho tng ng vi m + n + p = 2 (
rng t y ta c m, n, p < 2), v bt ng thc cho c vit li thnh
m
3
(2 m)
2
+
n
3
(2 n)
2
+
p
3
(2 p)
2

1
2
.
p dng bt ng thc AM-GM, ta c
m
3
(2 m)
2
+
2 m
8
+
2 m
8

3m
4
,
t suy ra
m
3
(2 m)
2
m
1
2
. Thit lp hai nh gi tng t cho n v p v cng li, ta c
m
3
(2 m)
2
+
n
3
(2 n)
2
+
p
3
(2 p)
2
m + n + p
3
2
=
1
2
.
Php chng minh hon tt.2
1.34 Cho a, b, c l cc s thc khng m tho mn a + 2b + 3c = 4. Chng minh rng:
(a
2
b + b
2
c + c
2
a + abc)(ab
2
+ bc
2
+ ca
2
+ abc) 8
Li gii. p dng bt ng thc AM-GM, ta c
8(a
2
b + b
2
c + c
2
a + abc)(ab
2
+ bc
2
+ ca
2
+ abc) = 4(a
2
b + b
2
c + c
2
a + abc).2(ab
2
+ bc
2
+ ca
2
+ abc)
(a
2
b + b
2
c + c
2
a + 2ab
2
+ 2bc
2
+ 2ca
2
+ 3abc)
2
.
Hn na, ta cng c
(a + 2b)(b + 2c)(c + 2a) = 9abc + 2a
2
b + 2ac
2
+ 4a
2
c + 2b
2
c + 4b
2
a + 4c
2
b
2(a
2
b + b
2
c + c
2
a + 2ab
2
+ 2bc
2
+ 2ca
2
+ 3abc),
do vy 8(a
2
b + b
2
c + c
2
a + abc)(ab
2
+ bc
2
+ ca
2
+ abc)
_
(a + 2b)(b + 2c)(c + 2a)
2
_
2
. Mt khc,
theo bt ng thc AM-GM, ta c
4(a + 2b)(b + 2c)(c + 2a) = (a + 2b)(4b + 8c)(c + 2a)
_
3a + 6b + 9c
3
_
3
= (a + 2b + 3c)
3
= 64.
Nh vy, ta suy ra
8(a
2
b + b
2
c + c
2
a + abc)(ab
2
+ bc
2
+ ca
2
+ abc)
_
64
4.2
_
2
= 64,
hay (a
2
b + b
2
c + c
2
a + abc)(ab
2
+ bc
2
+ ca
2
+ abc) 8.
34
Php chng minh hon tt.2
1.35 Cho a, b, c l cc s thc dng thay i bt k. Chng minh rng:
ab
a + 3b + 2c
+
bc
b + 3c + 2a
+
ac
c + 3a + 2b

a + b + c
6
Li gii. S dng nh gi c bn
9
x + y + z

1
x
+
1
y
+
1
z
, ta c
9
a + 3b + 2c
=
9
(a + c) + (b + c) + 2b

1
a + c
+
1
b + c
+
1
2b
.
T ta suy ra
9ab
a + 3b + 2c

ab
a + c
+
ab
b + c
+
a
2
. Hon ton tng t, ta cng c
9bc
b + 3c + 2a

bc
b + a
+
bc
c + a
+
b
2
,
v
9ca
c + 3a + 2b

ca
c + b
+
ca
a + b
+
c
2
.
Cng v theo v cc nh gi trn, ta thu c
9ab
a + 3b + 2c
+
9bc
b + 3c + 2a
+
9ca
c + 3a + 2b

ca + ab
b + c
+
ab + bc
a + c
+
bc + ca
b + a
+
a + b + c
2
=
3(a + b + c)
2
,
t y ta c bt ng thc cn chng minh.
Bi ton kt thc.2
1.36 Cho a, b, c l cc s thc dng tho mn abc = 1. Chng minh rng:
1
a + b + 4
+
1
b + c + 4
+
1
c + a + 4

1
2
Li gii 1. t x =

a, y =

b, z =

c. Khi ta phi chng minh


1
x
2
+ y
2
+ 4
+
1
y
2
+ z
2
+ 4
+
1
z
2
+ x
2
+ 4

1
2
vi x, y, z > 0 v xyz = 1.
Do
1
x
2
+ y
2
+ 4
= 1
x
2
+ y
2
x
2
+ y
2
+ 4
= 1
(x + y)
2
+ (x y)
2
2(x
2
+ y
2
+ 4)
nn bt ng thc ny c th c
vit li thnh

(x + y)
2
x
2
+ y
2
+ 4
+

(x y)
2
x
2
+ y
2
+ 4
2.
Khng mt tnh tng qut, ta gi s x y z. S dng bt ng thc Cauchy - Schwarz, ta c

(x + y)
2
x
2
+ y
2
+ 4

[(x + y) + (y + z) + (z + x)]
2

(x
2
+ y
2
+ 4)
,
v

(x y)
2
x
2
+ y
2
+ 4

[x y + y z + x z]
2

(x
2
+ y
2
+ 4)
.
35
T y ta a bi ton v chng minh
2(x + y + z)
2
+ 2(x z)
2
2(x
2
+ y
2
+ z
2
) + 12,
hay 2(x z)
2
+ 4(xy + yz + zx 3) 0. Tuy nhin y li l nh gi ng do (x z)
2
0 v
theo bt ng thc AM-GM th
xy + yz + zx 3
3
_
x
2
y
2
z
2
= 3,
do vy bt ng thc ban u c chng minh xong.
Bi ton kt thc.2
Li gii 2. t x =
3

a, y =
3

b, z =
3

c. Khi x, y, z > 0; xyz = 1 v ta cn chng minh


1
x
3
+ y
3
+ 4
+
1
y
3
+ z
3
+ 4
+
1
z
3
+ x
3
+ 4

1
2
Vi ch ta c nh gi x
3
+ y
3
xy(x + y), ng thi li c 4 = 4xyz, ta a bi ton v vic
chng minh
1
xy(x + y + 4z)
+
1
yz(y + z + 4x)
+
1
zx(z + x + 4y)

1
2
,
hay
x + y
x + y + 4z
+
y + z
y + z + 4x
+
z + x
z + x + 4y
1.
p dng bt ng thc Cauchy-Schwartz, ta c
x + y
x + y + 4z
+
y + z
y + z + 4x
+
z + x
z + x + 4y

4(x + y + z)
2

(x + y)(x + y + 4z)
=
4(x + y + z)
2
2(x
2
+ y
2
+ z
2
) + 10(xy + yz + zx)
,
nh vy, kt thc chng minh, ta cn ch ra rng
4(x + y + z)
2
2(x
2
+ y
2
+ z
2
) + 10(xy + yz + zx),
hay x
2
+ y
2
+ z
2
xy + yz + zx. Tuy nhin y li l mt nh gi ng, do vy bt ng thc
ban u c chng minh xong.
Bi ton kt thc.2
1.37 Cho a, b, c l cc s thc dng tho mn ab + bc + ca = 1. Chng minh rng:
3
_
1
a
+ 6b +
3
_
1
b
+ 6c +
3
_
1
c
+ 6a
1
abc
Li gii. p dng bt ng thc Holder ta c
_
3
_
1
a
+ 6b +
3
_
1
b
+ 6c +
3
_
1
c
+ 6a
_
3

_
1
a
+ 6b +
1
b
+ 6c +
1
c
+ 6a
__
1

3
.3
__
1

3
.3
_
= 9
_
1
a
+
1
b
+
1
c
+ 6a + 6b + 6c
_
. ()
36
Hn na, s dng nh gi c bn xy + yz + zx
(x + y + z)
2
3
, ta c
abc
_
1
a
+
1
b
+
1
c
+ 6a + 6b + 6c
_
= ab + bc + ca + 6abc(a + b + c)
ab + bc + ca + 2(ab + bc + ca)
2
= 3,
do vy
1
a
+
1
b
+
1
c
+ 6a + 6b + 6c
3
abc
. Kt hp vi nh gi () trn, ta c
_
3
_
1
a
+ 6b +
3
_
1
b
+ 6c +
3
_
1
c
+ 6a
_
3

27
abc
,
t ta ly cn bc ba hai v thu c bt ng thc cn chng minh.
Bi ton kt thc.2
1.38 Cho a, b, c l cc s thc dng tho mn abc = 1. Chng minh rng:
1
1 + a + b
+
1
1 + b + c
+
1
1 + a + c
1
Li gii. t x =

a, y =

b, z =

c. Khi ta phi chng minh


1
x
2
+ y
2
+ 1
+
1
y
2
+ z
2
+ 1
+
1
z
2
+ x
2
+ 1
1
vi x, y, z > 0 v xyz = 1.
Do
1
x
2
+ y
2
+ 1
= 1
x
2
+ y
2
x
2
+ y
2
+ 1
= 1
(x + y)
2
+ (x y)
2
2(x
2
+ y
2
+ 1)
nn bt ng thc ny c th c
vit li thnh

(x + y)
2
x
2
+ y
2
+ 1
+

(x y)
2
x
2
+ y
2
+ 1
4.
Khng mt tnh tng qut, ta gi s x y z. S dng bt ng thc Cauchy - Schwarz, ta c

(x + y)
2
x
2
+ y
2
+ 1

[(x + y) + (y + z) + (z + x)]
2

(x
2
+ y
2
+ 1)
,
v

(x y)
2
x
2
+ y
2
+ 1

[x y + y z + x z]
2

(x
2
+ y
2
+ 1)
.
T y ta a bi ton v chng minh
(x + y + z)
2
+ (x z)
2
2(x
2
+ y
2
+ z
2
) + 3.
Mt khc, theo bt ng thc AM-GM, ta li c
3 = 3
3
_
x
2
y
2
z
2
xy + yz + zx,
do vy ta ch cn phi chng minh
(x + y + z)
2
+ (x z)
2
2(x
2
+ y
2
+ z
2
) + xy + yz + zx.
37
Sau khi thu gn, ta c bt ng thc hin nhin ng
(x y)(y z) 0.
Bi ton do c chng minh xong.2
1.39 Cho a, b, c l cc s thc dng tho mn abc = 1. Chng minh rng:
a
3
b(c + 2)
+
b
3
c(a + 2)
+
c
3
a(b + 2)
1
Li gii. p dng bt ng thc AM-GM, ta c
a
3
b(c + 2)
+
b
3
+
c + 2
9
a.
Lp hai bt ng thc tng t v cng li, ta c
a
3
b(c + 2)
+
b
3
c(a + 2)
+
c
3
a(b + 2)
+
a + b + c
3
+
a + b + c + 6
9
a + b + c,
hay
a
3
b(c + 2)
+
b
3
c(a + 2)
+
c
3
a(b + 2)

5(a + b + c)
9

2
3
.
Mt khc cng theo bt ng thc AM-GM th a + b + c 3
3

abc = 3, do vy
a
3
b(c + 2)
+
b
3
c(a + 2)
+
c
3
a(b + 2)

5
3

2
3
= 1.
Php chng minh hon tt.2
1.40 Cho a, b, c l cc s thc khng m thay i bt k. Chng minh rng:
a
3
+ b
3
+ c
3
+ 3abc ab(a + b) + bc(b + c) + ca(c + a)
Li gii. Bt ng thc ban u mang tnh i xng gia cc bin, nn khng mt tnh tng
qut, ta gi s a = max{a; b; c}. Khi thc hin bin i tng ng, ta thu c dy bt
ng thc tng ng vi bt ng thc cn chng minh
a(a b)(a c) + b(b a)(b c) + c(c a)(c b) 0,
(a b)(a
2
ac b
2
+ bc) + c(a c)(b c) 0,
(a b)
2
(a + b c) + c(a c)(b c) 0.
nh gi cui cng ng do a = max{a; b; c}, do vy bt ng thc ban u c chng minh
xong.
Bi ton kt thc.2
38
3.2 Bi 2.1 n bi 2.40
2.1 Cho a, b, c l cc s thc dng thay i bt k. Chng minh rng:
a
(b + c)
2
+
b
(a + c)
2
+
c
(a + b)
2

9
4(a + b + c)
Li gii. Bt ng thc ban u tng ng vi
(a + b + c)
_
a
(b + c)
2
+
b
(a + c)
2
+
c
(a + b)
2
_

9
4
.
t k =
a
b + c
+
b
a + c
+
c
a + b
. Ta thy rng
(a+b+c)
_
a
(b + c)
2
+
b
(a + c)
2
+
c
(a + b)
2
_
=
a
2
(b + c)
2
+
b
2
(a + c)
2
+
c
2
(a + b)
2
+
a
b + c
+
b
a + c
+
c
a + b
,
v theo mt nh gi quen thuc th
a
2
(b + c)
2
+
b
2
(a + c)
2
+
c
2
(a + b)
2

k
2
3
, do vy
(a + b + c)
_
a
(b + c)
2
+
b
(a + c)
2
+
c
(a + b)
2
_

k
2
3
+ k
Ta li c ch rng k
3
2
theo bt ng thc Nesbitt, do
(a + b + c)
_
a
(b + c)
2
+
b
(a + c)
2
+
c
(a + b)
2
_

9
4.3
+
3
2
=
9
4
.
Php chng minh hon tt.2
2.2 Cho a, b, c l cc s thc dng thay i bt k. Chng minh rng:
a

a
2
+ 8bc
+
b

b
2
+ 8ac
+
c

c
2
+ 8ab
1
Li gii. p dng bt ng thc Cauchy-Schwartz ta c
a

a
2
+ 8bc
+
b

b
2
+ 8ac
+
c

c
2
+ 8ab

(a + b + c)
2
a

a
2
+ 8bc + b

b
2
+ 8ac + c

c
2
+ 8ab
.
Mt khc, cng theo bt ng thc Cauchy - Schwarz, ta c
a

a
2
+ 8bc + b

b
2
+ 8ac + c

c
2
+ 8ab =

a
3
+ 8abc +

b
3
+ 8abc +

c
3
+ 8abc

_
(a + b + c)(a
3
+ b
3
+ c
3
+ 24abc),
do vy
a

a
2
+ 8bc
+
b

b
2
+ 8ac
+
c

c
2
+ 8ab

(a + b + c)
2
_
(a + b + c)(a
3
+ b
3
+ c
3
+ 24abc)
=

(a + b + c)
3
a
3
+ b
3
+ c
3
+ 24abc
.
39
Nh vy, kt thc chng minh, ta cn ch ra rng
(a + b + c)
3
a
3
+ b
3
+ c
3
+ 24abc,
hay (a + b)(b + c)(c + a) 8abc. Tuy nhin y l mt nh gi ng v theo bt ng thc
AM-GM, ta c
(a + b)(b + c)(c + a) 2

ab.2

bc.2

ca = 8abc,
do vy bt ng thc ban u c chng minh xong.
Bi ton kt thc.2
2.3 Cho a, b, c l cc s thc dng tho mn ng thi c a v 3a
2
+4b
2
+5c
2
= 12. Chng
minh rng:
1
a
+
1
b
+
1
c
3
Li gii. T gi thit, ta c
4a
2
+ 4b
2
+ 4c
2
= 12 + a
2
c
2
12,
nh vy a
2
+ b
2
+ c
2
3. T y ta cng c
a + b + c
_
3(a
2
+ b
2
+ c
2
) 3,
v v vy ta chng minh c bt ng thc ban u v
1
a
+
1
b
+
1
c

9
a + b + c

9
3
= 3.
Bi ton kt thc.2
2.4 Cho a, b, c l cc s thc dng thay i bt k. Chng minh rng:
a
b
+
b
c
+
c
a

a + c
b + c
+
b + a
c + a
+
c + b
a + b
Li gii 1. t
X =
1 +
a
b
2
, Y =
1 +
b
c
2
, Z =
1 +
c
a
2
.
S dng bt ng thc Holder, ta thu c
_
1 +
a
b
_
_
1 +
b
c
_
_
1 +
c
a
_

_
1 +
3
_
a
b
.
b
c
.
c
a
_
3
= 8,
t ta suy ra XY Z 1.
By gi ta thc hin bin i bt ng thc cho nh sau
_
a
b

a + c
b + c
_
+
_
b
c

b + a
c + a
_
+
_
c
a

c + b
a + b
_
0,
c(a b)
b(b + c)
+
a(b c)
c(c + a)
+
b(c a)
a(a + b)
0,
40
a
b
1
1 +
b
c
+
b
c
1
1 +
c
a
+
c
a
1
1 +
a
b
0.
rng
a
b
1
1 +
b
c
=
2X 1 1
2Y
=
X 1
Y
,
do vy bt ng thc cui c th vit li thnh
X 1
Y
+
Y 1
Z
+
Z 1
X
0,
tng ng
X
Y
+
Y
Z
+
Z
X

1
X
+
1
Y
+
1
Z
.
S dng bt ng thc AM-GM, ta c
3

X
Y
=

_
X
Y
+
X
Y
+
Z
X
_
3

3
_
ZX
Y
2
= 3
3

XY Z

1
Y
= 3

1
Y
.
Nh vy bt ng thc ban u c chng minh xong.
Bi ton kt thc.2
Li gii 2. Thc hin bin i tng t nh cch 1, ta cn chng minh
c(a b)
b(b + c)
+
a(b c)
c(c + a)
+
b(c a)
a(a + b)
0.
Khng mt tnh tng qut, ta gi s b l s nm gia a v c. Khi (b a)(b c) 0. rng
b(c a) = c(a b) a(b c),
v vy bt ng thc trn c th vit li thnh
c(a b)
_
1
b(b + c)

1
a(a + b)
_
+ a(b c)
_
1
c(c + a)

1
a(a + b)
_
0,
tng ng
c[(a b)
2
(a + b) + b(a b)(a c)]
ab(a + b)(b + c)
+
[(b c)(a c)(a + c) + a(b c)
2
]
c(c + a)(a + b)
0.
Bt ng thc cui ny ng do
(a b)(a c) = (a b)
2
(b a)(b c) 0,
v
(b c)(a c) = (b c)
2
(b a)(b c) 0,
do vy bt ng thc ban u c chng minh xong.
Bi ton kt thc.2
Nhn xt.
41
1. Lu rng bt ng thc sau ng vi a, b, c v k l cc s thc dng:
a
b
+
b
c
+
c
a

ka + c
kb + c
+
kb + c
kc + a
+
kc + b
ka + b
.
Vi k = 1, ta thu c bi ton trn.
2. Ring vi trng hp k = 1, ta c th chng minh bi ton da trn bt ng thc sau (y
l mt bi trong Belarusian Mathematical Olympiad 1998): Cho a, b, c l cc s thc dng.
Chng minh rng
a
b
+
b
c
+
c
a

a + b
b + c
+
b + c
a + b
+ 1.
Vic chng minh cng nh p dng xin dnh cho bn c.
2.5 Cho a, b, c l cc s thc dng tho mn a + b + c = 3. Chng minh rng:

a +

b +

c ab + bc + ca
Li gii. Bt ng thc cho tng ng vi
a
2
+ b
2
+ c
2
+ 2

a + 2

b + 2

c a
2
+ b
2
+ c
2
+ 2(ab + bc + ca) = 9.
p dng bt ng thc AM-GM, ta c
a
2
+ 2

a 3a.
Lp cc bt ng thc tng t v cng li, ta c
a
2
+ b
2
+ c
2
+ 2

a + 2

b + 2

c 3(a + b + c) = 9.
Php chng minh hon tt.2
2.6 Cho a, b, c l cc s thc dng tho mn ab + bc + ca = 1. Chng minh rng:
1
abc
+
4
(a + b)(b + c)(c + a)

9

3
2
Li gii. p dng bt ng thc AM-GM, ta c
1
abc
+
4
(a + b)(b + c)(c + a)
=
1
2abc
+
1
2abc
+
4
(a + b)(b + c)(c + a)
3
3

1
a
2
b
2
c
2
(a + b)(b + c)(c + a)
= 3
3

1
abc(ab + ac)(bc + ba)(ca + cb)
.
Mt khc, cng theo bt ng thc AM-GM, ta c hai nh gi:
a
2
b
2
c
2

(ab + bc + ca)
3
27
,
v
(ab + bc)(bc + ca)(ca + ab)
8(ab + bc + ca)
3
27
,
42
t s dng gi thit ta suy ra abc
1
3

3
v (ab + bc)(bc + ca)(ca + ab)
8
27
. Do vy
1
abc
+
4
(a + b)(b + c)(c + a)
3
3

27.3

3
8
=
9

3
2
.
Php chng minh hon tt.2
2.7 Cho x, y, z l cc s thc tho mn x + y + z = 0, trong c hai s cng du. Chng
minh rng:
(x
2
+ y
2
+ z
2
)
3
(x
3
+ y
3
+ z
3
)
2
6
Li gii. Khng mt tnh tng qut, gi s x, y l hai s cng du, tc l xy 0. Vi iu kin
z = x y, ta c
(x
2
+ y
2
+ z
2
)
3
(x
3
+ y
3
+ z
3
)
2
=
8(x
2
+ y
2
+ xy)
3
9x
2
y
2
(x + y)
2
.
Nh vy, nu ta t x
2
+ y
2
= m v xy = n ( rng m 2n) th ta cn chng minh
8(m + n)
3
9n
2
(m + 2n)
6,
hay
4m
3
+ 4n
3
+ 12m
2
n + 12n
2
m 27n
2
m + 54n
3
.
Bt ng thc trn mang tnh thun nht gia cc bin, do ta cho n = 1, lc ny m 2 v ta
cn chng minh
4m
3
+ 12m
2
15m50 0.
Tuy nhin bng bin i tng ng, ta c (m2)
_
m
5
2
_
2
0. y l mt nh gi ng
do m 2, do vy bt ng thc ban u c chng minh xong.
Bi ton kt thc.2
2.8 Cho a, b, c l cc s thc dng thay i trong on [0, 1]. Chng minh rng:

abc +
_
(1 a)(1 b)(1 c) 1
Li gii. p dng bt ng thc Cauchy - Schwarz ta c

abc +
_
(1 a)(1 b)(1 c)
_
(a + 1 a)[bc + (1 b)(1 c)] =

2bc b c + 1.
Nh vy, kt thc chng minh, ta cn ch ra rng
2bc b + c.
Tuy nhin y l mt nh gi ng v theo gi thit v bt ng thc AM-GM th
2bc 2

bc b + c,
do bt ng thc ban u c chng minh xong.
43
Bi ton kt thc.2
2.9 Cho a, b, c l cc s thc khng m thay i bt k. Chng minh rng:
2
_
(ab + bc + ca)

3.
3
_
(a + b)(b + c)(c + a)
Li gii. rng ta c ng thc
(a + b)(b + c)(c + a) = (a + b + c)(ab + bc + ca) abc.
Mt khc, theo cc nh gi quen thuc, ta c
a + b + c
_
3(ab + bc + ca),
v
abc
_
(ab + bc + ca)
3
27
,
do vy
(a + b)(b + c)(c + a) (ab + bc + ca)
_
3(ab + bc + ca)
_
(ab + bc + ca)
3
27
=
8
_
(ab + bc + ca)
3
3

3
.
T y, ly cn bc ba hai v, ta thu c bt ng thc cn chng minh.
Bi ton kt thc.2
2.10 Cho a, b, c l cc s thc i mt phn bit. Chng minh rng:
a
2
+ b
2
a
2
2ab + b
2
+
a
2
+ c
2
a
2
2ac + c
2
+
b
2
+ c
2
b
2
2bc + c
2

5
2
Li gii. Dy bt ng thc sau tng ng vi bt ng thc cn chng minh
(a + b)
2
+ (a b)
2
(a b)
2
+
(b + c)
2
+ (b c)
2
(b c)
2
+
(c + a)
2
+ (c a)
2
(c a)
2
5,
_
a + b
a b
_
2
+
_
b + c
b c
_
2
+
_
c + a
c a
_
2
2.
t x =
a + b
a b
, y =
b + c
b c
, z =
c + a
c a
. rng ta c ng thc
xy + yz + zx =
(a + b)(b + c)
(a b)(b c)
+
(b + c)(c + a)
(b c)(c a)
+
(c + a)(a + b)
(c a)(a b)
=
(a + b)(b + c)(c a) + (b + c)(c + a)(a b) + (c + a)(a + b)(b c)
(a b)(b c)(c a)
= 1
Hn na, ta cng c (x + y + z)
2
0, do vy
x
2
+ y
2
+ z
2
2(xy + yz + zx) = 2.
T y ta thu c bt ng thc cn chng minh.
44
Bi ton kt thc.2
2.11 Cho a, b l cc s thc khng m tho mn a + b
4
5
. Chng minh rng:
_
1 a
1 + a
+
_
1 b
1 + b
1
_
1 a b
1 + a + b
Li gii. Dy bt ng thc sau l tng ng vi bt ng thc cn chng minh
1 a
1 + a
+
1 b
1 + b
+ 2

(1 a)(1 b)
(1 + a)(1 + b)

1 a b
1 + a + b
+ 1 + 2
_
1 a b
1 + a + b
,
2(1 ab)
1 + ab + a + b
+ 2
_
1 + ab a b
1 + ab + a + b

2
1 + a + b
+ 2
_
1 a b
1 + a + b
.
t u = ab; v = a + b. Khi u, v 0 v ta cn chng minh
2(1 u)
1 + u + v
+ 2
_
1 + u v
1 + u + v

2
1 + v
+ 2
_
1 v
1 + v
.
Thc hin bin i tng ng, ta c dy bt ng thc sau
1 + u v
1 + u + v

1 v
1 + v

u(2 + v)
(1 + v)(1 + v + u)
_
_
1 + u v
1 + u + v
+
_
1 v
1 + v
_
,
2uv
(1 + u + v)(1 + v)

u(2 + v)
(1 + v)(1 + v + u)
_
_
1 + u v
1 + u + v
+
_
1 v
1 + v
_
.
Nu u = 0 th bt ng thc trn hin nhin ng. Nu u > 0, bt ng thc trn tng ng
vi
2v
2 + v

_
1 + u v
1 + u + v
+
_
1 v
1 + v
. ()
rng vi u > 0, ta c nh gi
1 + u v
1 + u + v

1 v
1 + v
,
do vy
_
1 + u v
1 + u + v
+
_
1 v
1 + v
2
_
1 v
1 + v
= 2
_
1 +
2
1 + v
.
Hn na, ta li c v
4
5
theo gi thit nn
_
1 + u v
1 + u + v
+
_
1 v
1 + v
2

1 +
2
1 +
4
5
=
2
3
.
Ngoi ra cng do v
4
5
< 1 nn
2v
2 + v
=
2
2
v
+ 1
<
2
3
,
do vy nh gi () ng, cng c ngha bt ng thc ban u c chng minh.
45
Bi ton hon tt.2
2.12 Cho a, b, c l cc s thc dng thay i bt k. Chng minh rng:
a
2
b
+
b
2
c
+
c
2
a
+ a + b + c
6(a
2
+ b
2
+ c
2
)
a + b + c
Li gii. Bt ng thc cn chng minh mang tnh hon v gia cc bin, do khng mt tnh
tng qut, ta gi s b l s hng nm gia a v c. Khi ta bin i bt ng thc nh sau

_
a
2
b
+ b 2a
_

6(a
2
+ b
2
+ c
2
)
a + b + c
2(a + b + c),

(a b)
2
b

6(a
2
+ b
2
+ c
2
)
a + b + c
2(a + b + c).
p dng bt ng thc Cauchy - Schwarz, ta c

(a b)
2
b

[(a b) + (b c) + (a c)]
2
b + c + a
=
4(a c)
2
a + b + c
.
Do ta ch cn chng minh c
2(a c)
2
3(a
2
+ b
2
+ c
2
) (a + b + c)
2
.
Sau khi thu gn, ta c bt ng thc hin nhin ng do b nm gia a v c
2(b c)(b a) 0.
Bi ton hon tt.2
2.13 Cho x, y, z l cc s thc tho mn x
2
+ y
2
+ z
2
= 1. Chng minh rng:
1 x
3
+ y
3
+ z
3
3xyz 1
Li gii 1. Ch rng ta c ng thc
(x
3
+ y
3
+ z
3
3xyz)
2
= (x + y + z)
2
(x
2
+ y
2
+ z
2
xy yz zx)
2
= (1 + 2t)(1 t)(1 t),
trong t = xy + yz + zx. n y ta p dng bt ng thc AM-GM c
(x
3
+ y
3
+ z
3
3xyz)
2

[(1 + 2t) + (1 t) + (1 t)]


3
27
= 1,
do vy 1 x
3
+ y
3
+ z
3
3xyz 1.
Php chng minh hon tt.2
Li gii 2. p dng bt ng thc Cauchy - Schwarz, ta c
(x
3
+ y
3
+ z
3
3xyz)
2
= [x(x
2
yz) + y(y
2
zx) + z(z
2
xy)]
2
(x
2
+ y
2
+ z
2
)[(x
2
yz)
2
+ (y
2
zx)
2
+ (z
2
xy)
2
].
46
Hn na, ta li c
(x
2
yz)
2
+ (y
2
zx)
2
+ (z
2
xy)
2
= (x
2
+ y
2
+ z
2
)
2
(xy + yz + zx)
2
(x
2
+ y
2
+ z
2
)
2
,
do vy
(x
3
+ y
3
+ z
3
3xyz)
2
(x
2
+ y
2
+ z
2
)
3
= 1.
T ta suy ra 1 x
3
+ y
3
+ z
3
3xyz 1.
Php chng minh hon tt.2
2.14 Cho x, y, z l cc s thc dng thay i bt k. Chng minh rng:
xyz
(1 + 3x)(z + 6)(x + 8y)(y + 9z)

1
7
4
Li gii. p dng bt ng thc AM-GM, ta c cc nh gi sau:
z + 6 = z + 1 + 1 + 1 + 1 + 1 + 1 7
7

z,
1 + 3x = 1 +
x
2
+
x
2
+
x
2
+
x
2
+
x
2
+
x
2
7
7
_
x
6
2
6
,
x + 8y = x +
4y
3
+
4y
3
+
4y
3
+
4y
3
+
4y
3
+
4y
3
7
7
_
xy
6
.
4
6
3
6
,
y + 9z = y +
3z
2
+
3z
2
+
3z
2
+
3z
2
+
3z
2
+
3z
2
7
7
_
yz
6
.
3
6
2
6
.
Nhn cc bt ng thc trn vi nhau, ta c
(z + 6)(1 + 3x)(x + 8y)(y + 9z) 7
4
7
_
z.
x
6
2
6
.xy
6
.
4
6
3
6
.yz
6
.
3
6
2
6
= 7
4
xyz,
t suy ra
xyz
(1 + 3x)(z + 6)(x + 8y)(y + 9z)

1
7
4
.
Php chng minh hon tt.2
2.15 Cho a, b, c l cc s thc dng thay i bt k. Chng minh rng:
a + b
ab + c
2
+
b + c
bc + a
2
+
a + c
ac + b
2

1
a
+
1
b
+
1
c
Li gii. p dng bt ng thc Cauchy - Schwarz, ta c
a
2
b(a
2
+ c
2
)
+
b
2
a(b
2
+ c
2
)

(a + b)
2
b(a
2
+ c
2
) + a(b
2
+ c
2
)
=
(a + b)
2
(a + b)(ab + c
2
)
,
t y ta suy ra
a + b
ab + c
2

a
2
b(a
2
+ c
2
)
+
b
2
a(b
2
+ c
2
)
. Thit lp hai bt ng thc tng t ri cng
li, ta c
a + b
ab + c
2
+
b + c
bc + a
2
+
a + c
ac + b
2

a
2
b(a
2
+ c
2
)
+
b
2
a(b
2
+ c
2
)
+
b
2
c(b
2
+ a
2
)
+
c
2
b(a
2
+ c
2
)
+
a
2
c(a
2
+ b
2
)
+
c
2
a(b
2
+ c
2
)
=
1
a
+
1
b
+
1
c
.
47
Php chng minh hon tt.2
2.16 Cho a, b, c l cc s thc khng m tho mn khng c bt k hai s no ng thi
bng 0. Chng minh rng:
a(b + c)
b
2
+ bc + c
2
+
b(a + c)
a
2
+ ac + c
2
+
c(a + b)
a
2
+ ab + b
2
2
Li gii. p dng bt ng thc Cauchy - Schwarz, ta c
a(b + c)
b
2
+ bc + c
2
+
b(a + c)
a
2
+ ac + c
2
+
c(a + b)
a
2
+ ab + b
2
=
a
2
a(b + c)
abc
b + c
+
b
2
b(a + c)
abc
a+c
+
c
2
c(a + b)
abc
a+b

(a + b + c)
2
2(ab + bc + ca)
abc
b + c

abc
c + a

abc
a + b
.
Nh vy, kt thc chng minh, ta cn ch ra rng
(a + b + c)
2
4(ab + bc + ca) 2abc.(
1
a + b
+
1
b + c
+
1
a + c
),
hay
a
2
+ b
2
+ c
2
+ 2abc.(
1
a + b
+
1
b + c
+
1
a + c
) 2(ab + bc + ca).
p dng nh gi c bn
1
x
+
1
y
+
1
z

9
x + y + z
, ta c
a
2
+ b
2
+ c
2
+ 2abc.(
1
a + b
+
1
b + c
+
1
a + c
) a
2
+ b
2
+ c
2
+
9abc
a + b + c
.
Cng vic cui cng ch cn chng minh
a
2
+ b
2
+ c
2
+
9abc
a + b + c
2(ab + bc + ca),
hay a
3
+b
3
+c
3
+3abc ab(a +b) +bc(b +c) +ca(c +a). Tuy nhin nh gi ny ng theo bt
ng thc Schur bc ba nn bt ng thc ban u c chng minh xong.
Bi ton kt thc.2
2.17 Cho a, b, c l cc s thc dng tho mn a
2
+ b
2
+ c
2
= 1. Chng minh rng:
a
b
2
+ c
2
+
b
a
2
+ c
2
+
c
a
2
+ b
2

3

3
2
Li gii. S dng gi thit, ta c
a
b
2
+ c
2
+
b
a
2
+ c
2
+
c
a
2
+ b
2
=
a
1 a
2
+
b
1 b
2
+
c
1 c
2
.
rng ta c nh gi
a
1 a
2

3

3
2
a
2
=
a(a

3 + 2)(a

3 1)
2
2(1 a
2
)
0,
48
do vy
a
1 a
2

3

3
2
a
2
. Thit lp hai nh gi tng t v cng li, ta c
a
1 a
2
+
b
1 b
2
+
c
1 c
2

3

3
2
(a
2
+ b
2
+ c
2
),
do vy
a
b
2
+ c
2
+
b
a
2
+ c
2
+
c
a
2
+ b
2

3

3
2
.
Php chng minh hon tt.2
2.18 Cho x, y, z l cc s thc dng thay i bt k. Chng minh rng:

x + y + z
_
x
y + z
+

y
x + z
+

z
x + y
_

3
2
Li gii. Bt ng thc cn chng minh mang tnh thun nht, do ta chun ha x+y +z = 1.
ng thi, ta t a =

x, b =

y, c =

z. Nh vy ta cn chng minh
a
1 a
2
+
b
1 b
2
+
c
1 c
2

3

3
2
.
Tuy nhin y l mt kt qu c chng minh bi 2.17 . 2
2.19 Cho a, b, c l cc s thc khng m tho mn khng c bt k hai s no ng thi
bng 0. Chng minh rng:
a
3
(2a
2
+ b
2
)(2a
2
+ c
2
)
+
b
3
(2b
2
+ a
2
)(2b
2
+ c
2
)
+
c
3
(2c
2
+ a
2
)(2c
2
+ b
2
)

1
a + b + c
Li gii. p dng bt ng thc Cauchy-Schwartz, ta c
(2a
2
+ b
2
)(2a
2
+ c
2
) = (a
2
+ a
2
+ b
2
)(a
2
+ c
2
+ a
2
) (a
2
+ ab + ac)
2
= a
2
(a + b + c)
2
.
Nh vy
a
3
(2a
2
+ b
2
)(2a
2
+ c
2
)

a
(a + b + c)
2
. Thit lp hai nh gi tng t ri cng li, ta c
a
3
(2a
2
+ b
2
)(2a
2
+ c
2
)
+
b
3
(2b
2
+ a
2
)(2b
2
+ c
2
)
+
c
3
(2c
2
+ a
2
)(2c
2
+ b
2
)

a + b + c
(a + b + c)
2
=
1
a + b + c
.
Php chng minh hon tt.2
2.20 Cho a, b, c l cc s thc khng m thay i bt k. Chng minh rng:
a
2
+ b
2
a + b
+
b
2
+ c
2
b + c
+
c
2
+ a
2
c + a

3(a
2
+ b
2
+ c
2
)
a + b + c
Li gii. Dy bt ng thc sau tng ng vi bt ng thc cn chng minh
2(a
2
+ b
2
+ c
2
) +
c(a
2
+ b
2
)
a + b
+
a(b
2
+ c
2
)
b + c
+
b(c
2
+ a
2
)
c + a
3(a
2
+ b
2
+ c
2
),
c[(a + b)
2
2ab]
a + b
+
a[(b + c)
2
2bc]
b + c
+
b[(c + a)
2
2ca])
c + a
a
2
+ b
2
+ c
2
,
2ab + 2bc + 2ca a
2
+ b
2
+ c
2
+ 2abc
_
1
a + b
+
1
b + c
+
1
a + c
_
.
49
nh gi cui cng l mt kt qu c chng minh bi 2.16 , do vy ta kt thc chng
minh.2
2.21 Cho x, y, z l cc s thc dng tho mn x + y + z = 1. Chng minh rng:
xy
1 + z
+
yz
1 + x
+
xz
1 + y

1
4
Li gii. Ch rng
xy
1 + z
=
xy
(x + z) + (y + z)
,
v theo mt nh gi quen thuc th
4
(x + z) + (y + z)

1
x + z
+
1
y + z
,
do vy
xy
1 + z

1
4
_
xy
x + z
+
xy
y + z
_
. Thit lp hai nh gi tng t ri cng li, ta c
xy
1 + z
+
yz
1 + x
+
xz
1 + y

1
4
_
xy + yz
x + z
+
yz + zx
x + y
+
zx + xy
y + z
_
=
x + y + z
4
,
t y ta thu c bt ng thc cn chng minh.
Bi ton kt thc.2
2.22 Cho a, b, c l cc s thc dng thay i bt k. Chng minh rng:
(
a
b
+
b
c
+
c
a
)(a + b + c) 3
_
3(a
2
+ b
2
+ c
2
)
Li gii. p dng bt ng thc Cauchy-Schwartz, ta c
a
b
+
b
c
+
c
a

(a + b + c)
2
ab + bc + ca
,
do vy (
a
b
+
b
c
+
c
a
)(a+b +c)
(a + b + c)
3
ab + bc + ca
. Nh vy, kt thc chng minh, ta cn ch ra rng
(a + b + c)
3
3(ab + bc + ca)
_
3(a
2
+ b
2
+ c
2
),
hay (a +b +c)
6
27(a
2
+b
2
+c
2
)(ab +bc +ca)
2
. Tuy nhin y l mt nh gi ng v theo bt
ng thc AM-GM, ta c
(a +b +c)
6
= [(a
2
+b
2
+c
2
) + (ab +bc +ca) + (ab +bc +ca)]
3
27(a
2
+b
2
+c
2
)(ab +bc +ca)
2
,
do vy bt ng thc ban u c chng minh xong.
Bi ton kt thc.2
2.23 Cho a, b, c l cc s thc dng thay i bt k. Chng minh rng:
a
2
b
+
b
2
c
+
c
2
a
+ a + b + c 2
_
3(a
2
+ b
2
+ c
2
)
Li gii. p dng bt ng thc AM-GM ta c
a
2
b
+
b
2
c
+
c
2
a
+ a + b + c 2
_
(
a
2
b
+
b
2
c
+
c
2
a
)(a + b + c).
50
Nh vy, kt thc chng minh, ta cn ch ra rng
(
a
2
b
+
b
2
c
+
c
2
a
)(a + b + c) 3(a
2
+ b
2
+ c
2
).
Tht vy, p dng bt ng thc Cauchy-Schwartz, ta c
a
2
b
+
b
2
c
+
c
2
a

(a
2
+ b
2
+ c
2
)
2
a
2
b + b
2
c + c
2
a
.
Cng vic cui cng ch cn chng minh
(a + b + c)(a
2
+ b
2
+ c
2
) 3(a
2
b + b
2
c + c
2
a),
hay (a
3
+ab
2
) + (b
3
+bc
2
) + (c
3
+ca
2
) 2(a
2
b +b
2
c +c
2
a). Tuy nhin y l mt nh gi ng
theo bt ng thc AM-GM, do vy bt ng thc ban u c chng minh.
Bi ton kt thc.2
2.24 Cho a, b, c l cc s thc dng thuc khong (0, 1) tho mn ab +bc +ca = 1. Chng
minh rng:
a
2
+ b
2
(1 a
2
)(1 b
2
)
+
b
2
+ c
2
(1 b
2
)(1 c
2
)
+
c
2
+ a
2
(1 a
2
)(1 c
2
)

9
2
Li gii. Bt ng thc ban u tng ng vi mi bt ng thc trong dy sau

_
a
2
+ b
2
(1 a
2
)(1 b
2
)
+
1
2
_
6,

(1 + a
2
)(1 + b
2
)
(1 a
2
)(1 b
2
)
12.
rng ta c
(1 a
2
)(1 b
2
) (1 ab)
2
= (a b)
2
,
do vy (1 a
2
)(1 b
2
) (1 ab)
2
. Mt khc, theo bt ng thc Cauchy - Schwarz, ta c
(1 + a
2
)(1 + b
2
) (1 + ab)
2
,
do vy ta suy ra
(1 + a
2
)(1 + b
2
)
(1 a
2
)(1 b
2
)

(1 + ab)
2
(1 ab)
2
.
n y ta thit lp hai nh gi tng t v cng li c

(1 + a
2
)(1 + b
2
)
(1 a
2
)(1 b
2
)

(1 + ab)
2
(1 ab)
2
.
Ta p dng tip bt ng thc AM-GM suy ra

(1 + a
2
)(1 + b
2
)
(1 a
2
)(1 b
2
)
3
3

_
(1 + ab)(1 + bc)(1 + ca)
(1 ab)(1 bc)(1 ca)
_
2
.
Do vy, kt thc chng minh, ta cn ch ra rng
(1 + ab)(1 + bc)(1 + ca) 8(1 ab)(1 bc)(1 ca).
51
t x = ab, y = bc, z = ca. Khi x, y, z > 0; x + y + z = 1 v ta cn chng minh
(1 + x)(1 + y)(1 + z) 8(1 x)(1 y)(1 z),
tng ng
9xyz 7(xy + yz + zx) 2.
Theo mt kt qu c chng minh bi 2.35 , ta c
x
2
+ y
2
+ z
2
+
9xyz
x + y + z
2(xy + yz + zx),
t s dng gi thit x +y +z = 1 suy ra 9xyz 4(xy +yz +zx) 1. Cng vic cui cng
l chng minh
4(xy + yz + zx) 1 7(xy + yz + zx) 2,
hay xy + yz + zx
1
3
. Tuy nhin y l mt nh gi ng v
xy + yz + zx
(x + y + z)
2
3
=
1
3
,
do vy bt ng thc ban u c chng minh xong.
Bi ton kt thc.2
2.25 Cho a, b, c l cc s thc dng thay i bt k. Chng minh rng:
3

1 + a
3
+ b
3
+
3

1 + b
3
+ c
3
+
3

1 + a
3
+ c
3

3
_
27 + 2(a + b + c)
3
Li gii. p dng bt ng thc Holder, ta c
(1 + a
3
+ b
3
)[27 + (a + b + c)
3
+ (a + b + c)
3
]
2
[9 + a(a + b + c)
2
+ b(a + b + c)
2
]
3
,
t ta suy ra
3

1 + a
3
+ b
3
.
3
_
[27 + 2(a + b + c)
3
]
2
9 + (a + b)(a + b + c)
2
.
Thit lp hai bt ng thc tng t v cng li, ta c
3
_
[27 + 2(a + b + c)
3
]
2
(
3

1 + a
3
+ b
3
+
3

1 + b
3
+ c
3
+
3

1 + a
3
+ c
3
) 27 + 2(a + b + c)
3
,
t ta thu c bt ng thc cn chng minh.
Bi ton kt thc.2
Nhn xt. Bt ng thc trn l h qu trc tip ca bt ng thc Minkowsky m rng:
3

a
3
+ b
3
+ c
3
+
3
_
d
3
+ e
3
+ f
3
+
3
_
g
3
+ h
3
+ k
3

3
_
(a + d + g)
3
+ (b + e + h)
3
+ (c + f + k)
3
.
Cch chng minh tng t nh li gii ca bi ton trn.
2.26 Cho a, b, c l cc s thc khng m thay i bt k. Chng minh rng:
bc
(a + b)(a + c)
+
ca
(b + c)(b + a)
+
ab
(c + a)(c + b)

2(a
2
+ b
2
+ c
2
) + ab + bc + ca
2(a
2
+ b
2
+ c
2
) + 2(ab + bc + ca)
52
Li gii. Bt ng thc cn chng minh tng ng vi
1
bc
(a + b)(a + c)

ca
(b + c)(b + a)

ab
(c + a)(c + b)
1
2(a
2
+ b
2
+ c
2
) + ab + bc + ca
2(a
2
+ b
2
+ c
2
) + 2(ab + bc + ca)
.
Mt khc, rng ta c cc ng thc sau:
1
bc
(a + b)(a + c)

ca
(b + c)(b + a)

ab
(c + a)(c + b)
=
2abc
(a + b)(b + c)(c + a)
,
1
2(a
2
+ b
2
+ c
2
) + ab + bc + ca
2(a
2
+ b
2
+ c
2
) + 2(ab + bc + ca)
=
ab + bc + ca
(a + b)
2
+ (b + c)
2
+ (c + a)
2
,
do ta cn chng minh
2abc
(a + b)(b + c)(c + a)

ab + bc + ca
(a + b)
2
+ (b + c)
2
+ (c + a)
2
,
hay
2(a + b)
(c + a)(c + b)
+
2(b + c)
(a + b)(a + c)
+
2(c + a)
(b + c)(b + a)

1
a
+
1
b
+
1
c
.
rng
1
c

2(a + b)
(c + a)(c + b)
=
(c a)(c b)
c(c + a)(c + b)
,
do bt ng thc cn chng minh tng ng vi
(a b)(a c)
a(a + b)(a + c)
+
(b a)(b c)
b(b + a)(b + c)
+
(c a)(c b)
c(c + a)(c + b)
0.
Tuy nhin nh gi ny ng theo bt ng thc Vornicu - Schur, do vy bt ng thc ban u
c chng minh xong.
Bi ton kt thc.2
2.27 Cho a, b, c l cc s thc dng tho mn abc = 1. Chng minh rng:
4

2a
2
+ bc +
4

2b
2
+ ac +
4

2c
2
+ ab
ab + bc + ca
4

3
.
_

a +

b +

c
Li gii. t x =
1
a
, y =
1
b
, z =
1
c
. Khi x, y, z > 0 v xyz = 1. ng thi ta cng c
4

2a
2
+ bc =
4
_
2
x
2
+
1
yz
=
4
_
2yz + x
2
x
,
v ab + bc + ca = x + y + z. Theo , bt ng thc cn chng minh tr thnh

4
_
2yz + x
2
x

(x + y + z)
4

x
+
1

y
+
1

z
,
hay
_

4
_
2yz + x
2
x
_
4

(x + y + z)
4
3
_
1

x
+
1

y
+
1

z
_
2
.
53
p dng bt ng thc Holder, ta c
3(2yz + x
2
+ 2zx + y
2
+ 2xy + z
2
)
_
1

x
+
1

y
+
1

z
_
2

4
_
2yz + x
2
x
_
4
.
Nh vy, kt thc chng minh, ta cn ch ra rng
3(2yz + x
2
+ 2zx + y
2
+ 2xy + z
2
)
(x + y + z)
4
3
,
hay x + y + z 3. Tuy nhin y l mt nh gi ng v theo bt ng thc AM-GM
x + y + z 3
3

xyz = 3,
do vy bt ng thc ban u c chng minh xong.
Bi ton kt thc.2
2.28 Cho a, b, c l cc s thc khng m i mt phn bit. Chng minh rng:
(ab + bc + ca)
_
1
(a b)
2
+
1
(b c)
2
+
1
(c a)
2
_
4
Li gii. Bt ng thc ban u mang tnh i xng gia cc bin, do khng mt tnh tng
qut, ta gi s a > b > c 0. Khi ta t a b = x; b c = y. T y ta suy ra x, y > 0 v
ab + bc + ca ab = (c + y)(c + x + y) y(x + y).
ng thi, cng t php t trn, ta c
1
(a b)
2
+
1
(b c)
2
+
1
(c a)
2
=
1
x
2
+
1
y
2
+
1
(x + y)
2
.
Nh vy, ta a bi ton v vic chng minh
y(x + y)
_
1
x
2
+
1
y
2
+
1
(x + y)
2
_
4,
hay
y(x + y)
x
2
+
x
y
+
y
x + y
3.
t t =
x
y
. Khi t > 0 v ta cn chng minh
t + 1
t
2
+ t +
1
t + 1
3.
Sau khi bin i tng ng, ta thu c mt nh gi hin nhin ng
(t
2
t 1)
2
0,
do vy bt ng thc ban u c chng minh xong.
Bi ton kt thc.2
54
2.29 Cho a, b, c l cc s thc dng tha mn abc = 1. Chng minh rng:
a
b
+
b
c
+
c
a
+ 3 ab + bc + ca + a + b + c
Li gii. Do abc = 1 nn tn ti cc s thc dng x, y, z sao cho
a =
x
y
, b =
y
z
, c =
z
x
.
Khi bt ng thc cn chng minh tr thnh
xz
y
2
+
xy
z
2
+
yz
x
2
+ 3
x
z
+
y
x
+
z
y
+
x
y
+
y
z
+
z
x
,
tng ng
x
3
y
3
+ y
3
z
3
+ z
3
x
3
+ 3x
2
y
2
z
2
xyz[xy(x + y) + yz(y + z) + zx(z + x)].
Tuy nhin y l mt h qu trc tip ca bt ng thc Schur bc 3:
m
3
+ n
3
+ p
3
+ 3mnp mn(m + n) + np(n + p) + pm(p + m),
y m = xy,n = yz v p = zx. Do vy bt ng thc ban u c chng minh xong.
Bi ton kt thc.2
2.30 Cho a, b, c > 0. Chng minh rng:
a + b
ab + c
2

1
a
Li gii.
Bt ng thc cho tng ng vi:
1
a

a + b
ab + c
2
=
(a c)(b c)
abc + c
3
+
(b a)(c a)
abc + a
3
+
(a b)(c b)
abc + b
3
0
t
1
abc + a
3
= x,
1
abc + b
3
= y,
1
abc + c
3
= z.
Bt ng thc c a v dng Vornicu Schur:
x(a c)(b c) + y(b a)(c a) + z(a b)(c b) 0()
Gi s a b c, th th abc + c
3
abc + b
3
.
Suy ra
1
abc + c
3

1
abc + b
3
hay z y
Mc khc, theo iu gi s th b c, do a b a c.
Kt hp vi z y > 0, suy ra z(a c) y(a b).
Vit li bt ng thc () nh sau:
x(a b)(b c) + (b c)[z(a c) y(a b)] 0
Bt ng thc ny ng theo cc iu gi s.
Php chng minh hon tt. Du bng xy ra khi a = b = c.2
2.31 Cho x, y, z l cc s thc tha mn x
2
+ y
2
+ z
2
= 1. Tm max ca biu thc:
P = x
3
+ y
3
+ z
3
3xyz
Li gii. Ta c: P = x
3
+ y
3
+ z
3
3xyz = (x + y + z)(x
2
+ y
2
+ z
2
xy yz zx).
Suy ra P
2
= (x + y + z)
2
(x
2
+ y
2
+ z
2
xy yz zx)
2
p dng bt ng thc AM-GM cho 3 s khng m:
55
P
2
= (x + y + z)
2
(x
2
+ y
2
+ z
2
xy yz zx)
2
= (x + y + z)
2
(x
2
+ y
2
+ z
2
xy yz zx)(x
2
+ y
2
+ z
2
xy yz zx)

_
(x + y + z)
2
+ (x
2
+ y
2
+ z
2
xy yz zx) + (x
2
+ y
2
+ z
2
xy yz zx)
3
_
3
= (x
2
+ y
2
+ z
2
)
3
= 1 (theo gi thit) .
Suy ra P 1.
Vy maxP = 1 (x+y +z)
2
= x
2
+y
2
+z
2
xy yz zx x = 1, y = z = 0 v cc hon v.2
2.32 Cho a, b, c > 0. Chng minh rng:
a
2
b + c
+
b
2
a + c
+
c
2
a + b

a
2
a + b
+
b
2
b + c
+
c
2
c + a
Li gii.
Cch 1
Ta c:
a
2
b
2
a + b
= a b + b c + c a = 0
Suy ra
2a
2
a + b
=
a
2
+ b
2
a + b
Khi ta cn chng minh:
2c
2
a + b

a
2
+ b
2
a + b
Bt ng thc ny tng ng vi:
2c
2
a
2
b
2
a + b
0
hay

_
c
2
a + b

a
2
a + b
+
a
2
b + c

c
2
b + c
_
0
hay
(c a)
2
(c + a)
(a + b)(b + c)
0 (ng)
Php chng minh hon tt. ng thc xy ra khi v ch khi a = b = c
Cch 2
Bt ng thc cho tng ng vi:

a
2
_
1
b + c

1
a + c
_
0
hay
(a
2
(a
2
c
2
) + b
2
(b
2
a
2
) + c
2
(c
2
b
2
) 0
hay
1
2
[(a
2
b
2
)
2
+ (b
2
c
2
)
2
+ (c
2
a
2
)
2
] 0 (ng) .
Php chng minh hon tt. ng thc xy ra khi v ch khi a = b = c.2
2.34 Cho x, y, z 0. Chng minh rng:
a
2
+ bc
(b + c)
2
+
b
2
+ ac
(a + c)
2
+
c
2
+ ab
(a + b
2

3
2
Li gii.
Bt ng thc cho tng ng vi:

_
a
2
+ bc
(b + c)
2

1
2
_
0
hay
56
2a
2
b
2
c
2
(b + c)
2
0
Gi s a b c.
Khi ta c hai dy cng chiu:
_

_
2a
2
b
2
c
2
2b
2
a
2
c
2
2c
2
a
2
b
2
1
(b + c)
2

1
(a + c)
2

1
(a + b)
2
p dng bt ng thc Chebychep cho hai dy trn:

_
(2a
2
b
2
c
2
).
1
(b + c)
2
_
[

(2a
2
b
2
c
2
)] .
_
1
(b + c)
2
_
= 0.
_
1
(b + c)
2
_
= 0
Php chng minh hon tt. ng thc xy ra khi v ch khi a = b = c.2
2.35 Cho a, b > 0. Chng minh rng:
(a
2
+ b
2
)(a + b)
2
+ (ab + 1)
2
2(a + b)
2
Li gii.
rng a
2
+ b
2
= (a + b)
2
2ab.
t a
2
+ b
2
= x, ab = y
Bt ng thc cn chng minh tng ng vi:
x(x + 2y) + (y + 1)
2
2(x + 2y)
Khai trin v rt gn, ta c:
x
2
+ y
2
+ 1 2x 2y + 2xy 0
hay
(x + y 1)
2
0 (ng)
Php chng minh hon tt. ng thc xy ra khi v ch khi a
2
+ b
2
+ ab = 1 (chng hn
a = b =
_
1
3
).2
2.36 Cho a, b, c R. Chng minh rng:
a
3
b
3
(a b)
3
+
b
3
c
3
(b c)
3
+
c
3
a
3
(c a)
3

9
4
Li gii.
Bt ng thc cn chng minh tng ng vi:
a
2
+ b
2
+ ab
(a b)
2

9
4
Nhn thy rng:
a
2
+ ab + b
2
(a b)
2
=
1
4
(a + b)
2
+
3
4
(a b)
2
(a b)
2

3
4
(a b)
2
(a b)
2
=
3
4
Thc hin tng t cho hai bt ng thc cn li.
Php chng minh hon tt. ng thc xy ra khi v ch khi a + b + c = 0.2
2.37 Cho a, b > 0. Chng minh rng:
1
a
2
+
1
b
2
+
4
a
2
+ b
2

32(a
2
+ b
2
)
(a + b)
4
Li gii.
S dng bt ng thc AM-GM cho hai s dng:
1
a
2
+
1
b
2
+
4
a
2
+ b
2
=
a
2
+ b
2
a
2
b
2
+
4
a
2
+ b
2
2
_
a
2
+ b
2
a
2
b
2
.
4
a
2
+ b
2
=
4
ab
57
Ta s chng minh:
4
ab

32(a
2
+ b
2
)
(a + b)
4
hay
8ab(a
2
+ b
2
) (a + b)
4
p dng bt ng thc 4xy (x + y)
2
:
8ab(a
2
+ b
2
) = 4.2ab.a
2
+ b
2
) (a
2
+ b
2
+ 2ab)
2
= (a + b)
4
Php chng minh hon tt. ng thc xy ra khi v ch khi a = b.2
2.38 Cho a, b > 0 tha mn a
2
+ b
2
+ c
2
+ abc = 4. Chng minh rng:
a + b + c 3
Li gii.
Cch 1:
Theo nguyn l Dirichlet, trong ba s a, b, c t s c hai s cng pha vi 1 trn trc s. G s hai
s l a v b. Th th:
(a 1)(b 1) 0
hay
ab a + b 1 .
Mt khc, theo gi thit v bt ng thc AM-GM cho hai s dng:
4 c
2
= a
2
+ b
2
+ abc 2ab + abc = ab(2 + c)
hay
(2 c)(2 + c) ab(2 + c)
hay
2 c ab
Kt hp vi bt ng thc ab a + b 1 (chng minh trn), suy ra:
2 c ab a + b 1
hay
a + b + c 3 .
Php chng minh hon tt. ng thc xy ra khi v ch khi a = b = c = 1
Cch 2:
t a =
2x
_
(x + y) (x + z)
, b =
2y
_
(y + z) (y + x)
, c =
2z
_
(z + y) (z + x)
Suy ra:
a + b + c =

2x

y + z
_
(x + y) (y + z) (z + x)
V th bt ng thc a + b + c 3 s tng ng vi:

2x

y + z 3
_
(x + y)(y + z)(z + x)
y chnh l bt ng thc Schur vi cc bin

y + z,

y + x,

z + x.
Php chng minh hon tt. ng thc xy ra khi v ch khi a = b = c = 1.2
Cch 3:
Gi s tn ti mt s (cho s l a) trong ba s a, b, c ln hn 2. Khi , v a, b, c dng nn:
a
2
+ b
2
+ c
2
+ abc = 4 > 4 + b
2
+ c
2
+ abc > 4 (v l!)
Do a, b, c (0; 2]
T gi thit suy ra:
a
2
+ abc +
b
2
c
2
4
= 4 +
b
2
c
2
4
b
2
c
2
58
hay
_
a +
bc
2
_
2
=
(4 b
2
)(4 c
2
)
4
Do b, c 2 nn suy ra:
a + b + c =
_
(4 b
2
) (4 c
2
)
4

bc
2
+ b + c
p dng bt ng thc AM-GM, ta c:
_
(4 b
2
) (4 c
2
)
4

bc
2
+ b + c
1
2
(4 b
2
+ 4 c
2
) bc
2
+ b + c
= 3
_
b + c
2
1
_
2
3 .
Php chng minh hon tt. ng thc xy ra khi v ch khi a = b = c = 1.2
Nhn xt:
Bt ng thc a + b + c 3 cng ng vi iu kin a
2
+ b
2
+ c
2
+
3
2
abc =
9
2
.
2.39 Cho x, y, z > 0. Chng minh rng:
1
x
+
1
y
+
1
z

36
9 + x
2
y
2
+ y
2
z
2
+ z
2
x
2
Li gii.
t xy = a, yz = b, xz = c, bt ng thc tr thnh:
(a + b + c)(a
2
+ b
2
+ c
2
+ 9) 36

abc
p dng bt ng thc AM-GM:
a + b + c 3
3

abc = 3
12
_
(abc)
4
a
2
+ b
2
+ c
2
+ 9 3
3
_
(abc)
2
+ 9 = 3
3
_
(abc)
2
+ 3 + 3 + 3 4
4
_
3
3
_
(abc)
2
.3.3.3 = 12
12
_
(abc)
2
Nhn v theo v hai bt ng thc trn:
(a + b + c)(a
2
+ b
2
+ c
2
+ 9) 3
3

abc.12
12
_
(abc)
2
= 36.

abc .
Php chng minh hon tt. ng thc xy ra khi v ch khi a = b = c = 1.2
2.40 Cho a, b, c > 0 tha mn a
2
+ b
2
+ c
2
= 3. Chng minh rng:
_
4
a
2
+ b
2
+ 1
__
4
b
2
+ c
2
+ 1
__
4
c
2
+ a
2
+ 1
_
3(a + b + c)
2
Li gii.
p dng bt ng thc Holder:
_
4
a
2
+ b
2
+ 1
__
4
b
2
+ c
2
+ 1
__
4
c
2
+ a
2
+ 1
_

_
3
_
64
(a
2
+ b
2
)(b
2
+ c
2
)(c
2
+ a
2
)
+ 1
_
3
p dng bt ng thc AM-GM v bt ng thc 3(x
2
+ y
2
+ z
2
) (x + y + z)
2
:
_
3
_
64
(a
2
+ b
2
)(b
2
+ c
2
)(c
2
+ a
2
)
+ 1
_
3

_
12
2(a
2
+ b
2
+ c
2
)
+ 1
_
3
= 27 = 9(a
2
+ b
2
+ c
2
) 3(a + b + c)
2
Php chng minh hon tt. ng thc xy ra khi v ch khi a = b = c =
_
1
3
.2
3.3 Bi 3.1 n bi 3.40
3.1 Cho a, b, c, x, y, z > 0 tha mn x + y + z = 1. Chng minh rng:
ax + by + cz + 2
_
(xy + yz + zx)(ab + bc + ca) a + b + c
Li gii.
59
p dng bt ng thc Cauchy-Schwarz:
ax + by + cz + 2
_
(xy + yz + zx)(ab + bc + ca)
_

a
2
.

x
2
+
_
2

xy.2

ab

_
(

a
2
+ 2

ab)(

x
2
+ 2

xy)
= a + b + c (do x + y + z = 1)
Php chng minh hon tt. ng thc xy ra khi v ch khi
a
x
=
b
y
=
c
z
=
1
a + b + c
=
a + b + c
x + y + z
=
a + b + c hay a + b + c = 1. 2
3.2 Cho a, b, c 0 tha mn a
3
+ b
3
+ c
3
= 3. Tm gi tr ln nht ca:
P = a
4
b
4
+ b
4
c
4
+ c
4
a
4
Li gii.
Ta chng minh gi tr ln nht ca biu thc l 3.
t a
3
= x, b
3
= y, c
3
= z, suy ra x + y + z = 3.
p dng AM-GM
3a
4
b
4
a
3
b
3
(a
3
+ b
3
+ 1)
Khi , ta ch cn chng minh:
xy(x + y + 1) + yz(y + z + 1) + zx(z + x + 1) 9
a v dng ng bc, ta cn chng minh
3

xy(x + y) + (x + y + z)(xy + yz + zx) (x + y + z)


3
Sau khi khai trin, bt ng thc tr thnh:
x
3
+ y
3
+ z
3
+ 3xyz

xy(x + y)
ng theo bt ng thc Schur.
Php chng minh hon tt. ng thc xy ra khi v ch khi a = b = c = 1.2
3.3 Cho a, b, c > 0 tha mn a
2
+ b
2
+ c
2
= 1. Tm gi tr nh nht ca:
P =
a
2
b + c
+
b
2
c + a
+
c
2
a + b
Li gii.
Cch 1:
Gi s a b c.
Ta s c hai dy cng chiu:
_
_
_
a
2
b
2
c
2
1
b + c

1
a + c

1
a + b
p dng ln lt bt cc bt ng thc Chebychep, gi thit a
2
+ b
2
+ c
2
= 1 v bt ng thc
1
x

9

x
, ta c:
P =
a
2
b + c
+
b
2
c + a
+
c
2
a + b

1
3
(a
2
+ b
2
+ c
2
)
_
1
b + c
+
1
c + a
+
1
a + b
_

3
2 (a + b + c)
Li theo bt ng thc:
3 = 3(a
2
+ b
2
+ c
2
) (a + b + c)
2
Suy ra:
a + b + c

3 .
Do :
P
3
2(a + b + c)

3
2
.
Vy minP =

3
2
a = b = c =
1

3
.
60
Cch 2:
p dng bt ng thc Cauchy-Schwarz v gi thit:
a
2
b + c
=
a
4
a
2
(b + c)

(a
2
+ b
2
+ c
2
)
2
a
2
(b + c) + b
2
(c + a) + c
2
(a + b)
=
1
a(b
2
+ c
2
) + b(c
2
+ a
2
) + c(a
2
+ b
2
)
Mt khc, theo bt ng thc AM-GM:

a(b
2
+ c
2
) =

_
1
2
.2a
2
(b
2
+ c
2
)(b
2
+ c
2
) 3

1
2
_
2a
2
+ 2b
2
+ 2c
2
3
_
2
=
2

3
Suy ra:
P
1
_
2

3
_ =

3
2
Vy minP =

3
2
a = b = c =
1

3
.2
3.4 Cho a, b, c > 0. Chng minh rng:
1
a(b + a)
+
1
b(c + 1)
+
1
c(c + a)

3
3

abc.(1 +
3

abc)
Li gii.
Cch 1:
Nhn v tri vi abc + 1, ta c:
1
a(b + a)
+
1
b(c + 1)
+
1
c(c + a)
=

_
bc
1 + b
+
1
a(1 + b)
_
=

_
b(1 + c)
1 + b
+
1 + a
a(1 + b)
1
_
=
b(1 + c)
1 + b
+
1 + a
a(1 + b)
3 = P
p dng bt ng thc AM-GM:
P 3
3

abc(1 + a)(1 + b)(1 + c)


(1 + a)(1 + b)(1 + c)
+ 3
3

(1 + a)(1 + b)(1 + c)
abc(1 + a)(1 + b)(1 + c)
3
= 3
(
3

abc)
2

abc + 1
3

abc
=
3(abc + 1)
3

abc(
3

abc + 1)
Php chng minh hon tt. ng thc xy ra khi v ch khi a = b = c.2
Cch 2:
Theo bt ng thc Holder th:
(1 + a)(1 + b)(1 + c) (1 +
3

abc)
3
.
p dng bt ng thc (x + y + z)
2
3(xy + yz + zx) v bt ng thc trn:
_
1
a(b + 1)
_
2
3
_
1
ab(1 + b)(1 + c)
_
=
3(a + b + c + ab + bc + ca)
abc(1 + b)(1 + c)(1 + a)
=
3
abc

3(abc + 1)
abc(1 + a)(1 + b)(1 + c)

3
abc

3(abc + 1)
abc(1 +
3

abc)
3
= 3
3
3

abc(1 +
3

abc)
abc(1 +
3

abc)
3
=
9
3

a
2
b
2
c
2
(1 +
3

abc)
2
Khai cn hai v, suy ra:
1
a(b + a)
+
1
b(c + 1)
+
1
c(c + a)

3
3

abc.(1 +
3

abc)
Php chng minh hon tt. ng thc xy ra khi v ch khi a = b = c.2
Cch 3:
t abc = k. Th th lun tn ti x, y, z > 0 sao cho a =
ky
x
, b =
kz
y
, c =
kx
z
.
Khi bt ng thc cn chng minh tng ng vi:
61
1
ky
x
_
kz
y
_
+ 1
+
1
kz
y
_
kx
z
_
+ 1
+
1
kx
z
_
ky
x
_
+ 1

3
k(k + 1)
hay
x
y + kz
+
y
z + kx
+
z
x + ky

3
k + 1
.
Theo bt ng thc Cauchy-Schwarz v bt ng thc (x + y + z)
2
3(xy + yz + zx):
x
y + kz
+
y
z + kx
+
z
x + ky

(x + y + z)
2
x(y + kz) + y(z + kx) + z(x + ky)
=
(x + y + z)
2
(k + 1)(xy + yz + zx)

3
k + 1
Php chng minh hon tt. ng thc xy ra khi v ch khi a = b = c.2
3.5 Cho a, b, c > 0 tha mn a + b + c = 3. Chng minh rng:
1
2 + a
2
+ b
2
+
1
2 + b
2
+ c
2
+
1
2 + c
2
+ a
2

3
4
Li gii.
Gi s a b c.
Trng hp 1: a
2
+ b
2
6
Bt ng thc cn chng minh tng ng vi:
a
2
+ b
2
a
2
+ b
2
+ 2

3
2

(a + b)
2
(a + b)
2
+
2(a + b)
2
a
2
+ b
2

3
2
Theo bt ng thc Cauchy-Schwarz :
(a + b)
2
(a + b)
2
+
2(a + b)
2
a
2
+ b
2

4(a + b + c)
2

(a + b)
2
+
2(a + b)
2
a
2
+ b
2
Nh vy ta ch cn chng minh:
(a + b)
2
+ (b + c)
2
+ (c + a)
2
+
2(a + b)
2
a
2
+ b
2
+
2(b + c)
2
b
2
+ c
2
+
2(c + a)
2
c
2
+ a
2
24
Ta li c:
12

(a + b)
2
=
4
3
(a + b + c)
2

(a + b)
2
=
1
3

(a b)
2
v 12
2(a + b)
2
a
2
+ b
2
=
2(a b)
2
a
2
+ b
2
Nn bt ng thc tng ng vi:

(a b)
2
(
6
a
2
+ b
2
1) 0 (ng)
Trng hp 2: a
2
+ b
2
6.
Khi ta c:
1
a
2
+ b
2
+ 2

1
8
v
1
a
2
+ c
2
+ 2
+
1
b
2
+ c
2
+ 2

1
a
2
+ 2
+
1
b
2
+ 2

1
8 b
2
+
1
b
2
+ 2

1
8
+
1
2
( v 0 b
2
6)
Khi :
1
a
2
+ b
2
+ 2

3
4
Php chng minh hon tt. ng thc xy ra khi v ch khi a = b = c.2
3.6 Cho a, b, c > 0 tha mn ab + bc + ca = 3. Chng minh rng:
1
a
2
+ 1
+
1
b
2
+ 1
+
1
c
2
+ 1

3
2
Li gii.
62
Gi s a b c.
Suy ra:
ab 1
Suy ra:
(a b)
2
(ab 1) 0
hay
1
1 + a
2
+
1
1 + b
2

2
1 + ab
Vy ta cn chng minh:
2
1 + ab
+
1
1 + c
2

3
2
Bt ng thc ny tng ng vi:
3 ab
ab + 1

2c
2
c
2
+ 1
hay
c
2
+ 3 ab 3abc
2
hay
c
2
+ ca + bc 3abc
2
hay
a + b + c 3abc
iu ny ng v:
_
_
_
(a + b + c)
2
3(ab + bc + ca) = 9
ab + bc + ca 3
3
_
(abc)
2
hay a + b + c 3 3abc .
Php chng minh hon tt. ng thc xy ra khi v ch khi a = b = c = 1.2
3.7 Cho a, b, c > 0 tha mn ab + bc + ca = 3. Chng minh rng:
1
2abc + ab
2
+
1
2abc + bc
2
+
1
2abc + ca
2

a + b + c
3
Li gii.
Nhn abc cho 2 v ca bt ng thc, ta cn chng minh:
ca
2ca + ba
+
ab
2ab + cb
+
bc
2bc + ac

abc(a + b + c)
3
t bc = x, ca = y, ab = z, suy ra x + y + z = 3, bt ng thc tr thnh:
y
2y + z
+
z
2z + x
+
x
2x + y

xy + yz + zx
3
t x
2
+ y
2
+ z
2
= m, xy + yz + zx = n, suy ra m + 2n = 9.
p dng bt ng thc Cauchy-Schwarz:
y
2y + z
+
z
2z + x
+
x
2x + y

(x + y + z)
2
2m + n
V vy ta cn chng minh:
(x + y + z)
2
2m + n

n
3
hay
(m + 2n)
2
3n(2m + n) (do m + 2n = 9)
Hay tc l
m
2
+ n
2
2mn (ng theo AM-GM) .
Php chng minh hon tt. ng thc xy ra khi v ch khi a = b = c = 1.2
63
3.8 Cho a, b, c R. Chng minh rng:
2(a
2
+ b
2
)(b
2
+ c
2
)(c
2
+ a
2
) (a b)
2
(b c)
2
(c a)
2
Li gii.
Ta c hng ng thc sau:
(x
2
+ y
2
)(m
2
+ n
2
) = (xm + yn)
2
+ (xn ym)
2
p dng hng ng thc trn, ta c:
2(a
2
+ b
2
)(b
2
+ c
2
)(c
2
+ a
2
) =
_
(a + b)
2
+ (a b)
2

_
(c
2
+ ab)
2
+ c
2
(a b)
2
_
=
_
(a + b)(c
2
+ ab) + c(a b)
2

2
+
_
c(a b)(a + b) (c
2
+ ab)(a b)

2
=
_
(a + b)(c
2
+ ab) + c(a b)
2

2
+ (a b)
2
(b c)
2
(c a)
2
(a b)
2
(b c)
2
(c a)
2
Php chng minh hon tt. ng thc xy ra khi v ch khi (a + b)(c
2
+ ab) = c(a b)
2
.2
3.9 Cho a, b, c > 0. Chng minh rng:
a
3
b
1 + ab
2
+
b
3
c
1 + bc
2
+
c
3
a
1 + ca
2

abc(a + b + c)
1 + abc
Li gii.
p dng bt ng thc Cauchy-Schwarz:
a
3
b
1 + ab
2
.
1 + ab
2
ab
(a + b + c)
2
hay
a
3
b
1 + ab
2
.
_
abc + 1
abc
_
(a + b + c) (a + b + c)
2
hay
a
3
b
1 + ab
2
+
b
3
c
1 + bc
2
+
c
3
a
1 + ca
2

abc(a + b + c)
1 + abc
Php chng minh hon tt. ng thc xy ra khi v ch khi a = b = c.2
3.10 Cho a, b, c [0; 2] tha mn a + b + c = 3. Chng minh rng:
A = a
3
+ b
3
+ c
3
9
Li gii.
Cch 1
Ta c ng thc sau:
A = a
3
+ b
3
+ c
3
= (a + b + c)
3
3(a + b)(b + c)(c + a)
= 27 3(3 a)(3 b)(3 c)
= 27 9(ab + bc + ca) + 3abc
Mt khc, do a, b, c [0; 2] nn (2 a)(2 b)(2 c) 0, hay:
8 4(a + b + c) + 2(ab + bc + ca) abc 0
Suy ra:
2(ab + bc + ca) abc 4(a + b + c) 8 = 4
Suy ra:
9(ab + bc + ca)
9
2
abc 18 .
Do :
A 27
9
2
abc 18 + 3abc = 9
3
2
abc 9 (do a, b, c 0)
Php chng minh hon tt. ng thc xy ra khi v ch khi a = 2, b = 1, c = 0 v cc hon v.
Cch 2
Gi s a = max {a; b; c}
64
Suy ra 3 = a + b + c 3a, hay a [1; 2], hay (a 1)(a 2) 0
Ta c:
A = a
3
+ b
3
+ c
3
a
3
+ 3bc(b + c) + b
3
+ c
3
= a
3
+ (b + c)
3
= a
3
+ (3 a)
3
= (a 1)(a 2) + 9 9
Php chng minh hon tt. ng thc xy ra khi v ch khi a = 2, b = 1, c = 0 v cc hon v.2
3.11 Cho dy s dng a
n
. Chng minh rng:
1
a
1
+
2
a
1
+ a
2
+ . . . +
n
a
1
+ a
2
+ ... + a
n
< 4
_
1
a
1
+ .... +
1
a
n
_
Li gii.
Theo bt ng thc Cauchy-Schwarz:
1
2
a
1
+
2
2
a
2
+ ... +
i
2
a
i

(1 + 2 +... + i)
2
a
1
+ a
2
+ ... + a
i
Th ln lt i = 1, 2, 3, ..., n ri cng v theo v:
A

n
i=1
[
i
(1 + 2 +... + i)
2
.(
1
2
a
1
+
2
2
a
2
+ ... +
i
2
a
i
)] =

n
i=1
(B
i
.
i
2
a
i
) = T
Trong :
B
i
=
i
(1 + 2 +... + i)
2
+
i + 1
[1 + 2 +... + (i + 1)]
2
+ ... +
n
(1 + 2 +... + n)
2
= 4.
_
i
[i(i + 1)]
2
+
i + 1
[(i + 1)(i + 2)]
2
+ ... +
n
[n(n + 1)]
2
_
= 4.
_
1
i(i + 1)
2
+ ... +
1
n(n + 1)
2
_
= 4
_
1
i + 1
_
1
i

1
i + 1
_
+ ... +
1
n + 1
_
1
n

1
n + 1
__
= 4(
1
i
2
C)
< 4.
1
i
2
Do
C =
1
i
(
1
i

1
i + 1
) + ... +
1
n
(
1
n

1
n + 1
) +
1
(n + 1)
2
> 0
Suy ra:
B
i
.i
2
< 4
Suy ra:
B
i
.
i
2
a
i
<
4
a
i
Suy ra:
A T < 4(
1
a
1
+ .... +
1
a
n
)
Php chng minh hon tt.2
3.12 Cho a, b, c l di ba cnh tam gic. Chng minh rng:
a
3
+ b
3
+ c
3
3abc [ab(a + b) + bc(b + c) + ca(c + a)]
Li gii.
Bt ng thc cho tng ng vi:
65
a
3
+ b
3
+ c
3
3abc 2 [ab(a + b) + bc(b + c) + ca(c + a) 6abc]
hay
(a + b + c) [(a b)
2
+ (b c)
2
+ (c a)
2
] 4 [c(a b)
2
+ a(b c)
2
+ b(c a)
2
]
hay
(3a b c)(b c)
2
+ (3b c a)(c a)
2
+ (3c a b)(a b)
2
0 (1)
Khng mt tnh tng qut, gi s a b c.
Xt hai trng hp:
Trng hp 1: c
a + b
3
Bt ng thc (1) c th bin i thnh:
(c + a b)(b c)
2
+ (b + c a)(c a)
2
+ (a + b 3c)(b c)(a c) 0
Bt ng thc ny ng theo iu gi s.
Trng hp 2: c >
a + b
3
Suy ra b >
a + b
3

a + c
3
Bin i bt ng thc (1) thnh:
(b + c a)(a b)
2
+ (a + b c)(b c)
2
+ (3b c a)(a b)(b c) 0
Bt ng thc ny cng ng theo iu gi s.
Php chng minh hon tt. ng thc xy ra khi v ch khi a + b = 3c hoc a + c = 3b.2
3.13 Cho a, b, c R. Chng minh rng:
S = a
2
1
+
_
a
1
+ a
2
2
_
2
+ ... +
_
a
1
+ a
2
+ ... + a
n
n
_
2
< 4(a
2
1
+ a
2
2
+ ... + a
2
n
)
Li gii.
t
k
=

k 1
Theo bt ng thc Cauchy-Schwarz:
_
a
2
1

1
+
a
2
2

2
+ ... +
a
2
k

k
_
(
1
+
2
+ ... +
k
) (a
1
+ a
2
+ .. + a
k
)
2
Suy ra:
_
a
1
+ a
2
+ ... + a
k
k
_
2

(
1
+
2
+ .. +
k
)
k
2
_
a
2
1

1
+
a
2
2

2
+ ... +
a
2
k

k
_
Suy ra:
S

n
k=1
_
c
k
a
2
k

k
_
vi c
k
=

nk
i=0

1
+
2
+ .. +
k+i
(k + i)
2
Suy ra:
c
k
=

k
k
2
+

k + 1
(k + 1)
2
+ ... +

n
n
2
=
1
k

k
+
1
(k + 1)

k + 1
+ ... +
1
n

n
Ta li c:
1
2

k
<
1
_
k +
1
2
+
_
k
1
2
=
_
k +
1
2

_
k
1
2
v
_
(k
1
2
).(k +
1
2
) < k
Suy ra:
66
1
k

k
< 2
_
_
_
_
1
_
k
1
2

1
_
k +
1
2
_
_
_
_
Suy ra:
c
k
< 2
_
_
_
_
1
_
k
1
2

1
_
n +
1
2
_
_
_
_
<
2
_
k
1
2
Suy ra:
c
k

k
<
2
_
k
1
2
.2
_
k
1
2
= 4
Suy ra:
S < 4(a
2
1
+ a
2
2
+ ... + a
2
n
)
Php chng minh hon tt.2
3.14 (VMO 2002) Cho a, b, c l cc s thc tha mn a
2
+ b
2
+ c
2
= 9. Chng minh rng:
2(a + b + c) abc 10
Li gii.
Cch 1:
Theo gi thit:
_

_
|a|, |b|, |c| 3
|a + b + c| 3

3
|abc| 3

3
Ta c th gi s a, b, c = 0 v a b c. Xt cc trng hp sau:
c < 0: Th th
2(a + b + c) abc abc 3

3 < 10
a b < 0 < c. Suy ra abc > 0. Suy ra
2(a + b + c) < 2c 6 < 10 + abc
hay
2(a + b + c) abc < 10
a < 0 < b c
Theo bt ng thc Cauchy-Schwarz:
(2b + 2c a)
2
(b
2
+ c
2
+ a
2
)[2
2
+ 2
2
+ (1)
2
] = 9.9 = 81
Suy ra:
2b + 2c a 9
Do :
2(a + b + c) = 2b + 2c a + 3a 9 + 3a
Ta cn chng minh:
9 + 3a abc + 10
hay
3a 1 abc
Theo bt ng thc AM-GM th bc
b
2
+ c
2
2
=
9 a
2
2
Tc ta phi chng minh:
67
3a 1
9 a
2
2
Bt ng thc ny tng ng vi:
(a + 1)
2
(a 2) 0 (ng do a < 0)
0 < a b c
S dng bt ng thc Cauchy-Schwarz tng t nh trn, ta co: 2b + 2c + a 9
Ta cng suy ra c:
2(a + b + c) 9 + a
Vy ta cn chng minh:
9 + a 10 + abc
hay
a 1 + abc
Vi a < 1 th bt ng thc hin nhin ng.
Vi a 1 th c b 1. Khi bt ng thc cng ng.
Php chng minh hon tt. ng thc xy ra khi v ch khi b = c = 2, a = 1 v cc hon v.
Cch 2:
t P = 2(a + b + c) abc.
Suy ra P
2
= [2(a + b + c) abc]
2
= [2(a + b) + z(2 ab)]
2
S dng bt ng thc Cauchy-Schwarz:
P
2
[(a + b)
2
+ c
2
)[4 + (2 ab)
2
] = 72 20ab + (ab)
2
+ 2(ab)
3
t t = ab. Ta s chng minh:
100 72 20t + t
2
+ 2t
3
hay
(t + 2)
2
(t 3.5) 0 (*)
Khng mt tnh tng qut, gi s |a| |b| |c|.
Suy ra:
3|c| 9
Suy ra:
a
2
+ b
2
6
Theo bt ng thc AM-GM th:
6 a
2
+ b
2
2ab
hay
ab 3 < 3.5
Suy ra (*) ng.
Php chng minh hon tt. ng thc xy ra khi v ch khi b = c = 2, a = 1 v cc hon v.2
3.15 Cho a, b, c 0 v khng c hai s no trong chng ng thi bng 0. Chng minh rng:
3
_
a
2
+ bc
b
2
+ c
2
+
3
_
b
2
+ ca
c
2
+ a
2
+
3
_
c
2
+ ab
a
2
+ b
2

9
3

abc
a + b + c
Li gii.
p dng bt ng thc AM-GM:

a(b
2
+ c
2
)
a
2
+ bc
=
a(b
2
+ c
2
)
a
2
+ bc
+ b + c 3
3
_
abc(b
2
+ c
2
)
a
2
+ bc
Suy ra:
68
3
_
a
2
+ bc
b
2
+ c
2

3
3

abc(a
2
+ bc)

a(b
2
+ c
2
)
Tng t vi
3
_
b
2
+ ca
c
2
+ a
2
v
3
_
c
2
+ ab
a
2
+ b
2
, ta c:
3
_
a
2
+ bc
b
2
+ c
2
+
3
_
b
2
+ ca
c
2
+ a
2
+
3
_
c
2
+ ab
a
2
+ b
2

3
3

abc(a
2
+ b
2
+ c
2
+ ab + bc + ca)

a(b
2
+ c
2
)
Vy ta ch cn chng minh:
3(a
2
+ b
2
+ c
2
+ ab + bc + ca)

a(b
2
+ c
2
)

9
a + b + c
hay
(a + b + c)[(a
2
+ b
2
+ c
2
+ ab + bc + ca)] 3[

a(b
2
+ c
2
)]
Gi s a b c. Bin i bt ng thc trn thnh:
a(a b)
2
+ c(b c)
2
+ (a + c b)(a b)(b c) 0 (ng theo iu gi s) .
Php chng minh hon tt. ng thc xy ra khi v ch khi a = b = c.2
3.16 Cho a, b, c R tha mn ab + bc + ca = 3. Chng minh rng:
a
2
+ b
2
+ c
2
+
9
4
a
2
b
2
c
2

21
4
Li gii.
T bi suy ra b + c = 0 v a =
3 bc
b + c
.
Khng mt tnh tng qut, gi s:
bc ca ab .
Bt ng thc cn chng minh tng ng vi:
_
3 bc
b + c
_
2
_
1 +
9
4
b
2
c
2
_
+ b
2
+ c
2

21
4
t b + c = S v bc = P.
Ta c:
(3 P)
2
(4 + 9P
2
) + 4S
2
(S
2
2P) 21S
2
hay
9P
4
54P
3
+ 85P
2
24P + 36 + 4S
4
8S
2
P 21S
2
0
hay
(9P
4
54P
3
+ 117P
2
108P + 36) + (4S
4
8S
2
P 32P
2
21S
2
+ 84P) 0
hay
9(P
2
3P + 2)
2
+ (S
2
4P)(4S
2
+ 8P 21) 0
Bt ng thc ny ng v 4S
2
+ 8P 16P + 8P 21 24 21 > 0.
Php chng minh hon tt. ng thc xy ra khi v ch khi a = b = c = 1 hoc a =
1
2

2
; b =
c =

2.2
3.17 Cho a, b, c [0; 1]. Chng minh rng:
a
2
+ b
2
+ c
2
1 + a
2
b + b
2
c + c
2
a
Li gii.
Bt ng thc cho tng ng vi:
(1 a
2
)(1 b
2
)(1 c
2
) + a
2
b
2
c
2
+ a
2
b(1 b) + b
2
c(1 c) + c
2
b(1 b) 0
Bt ng thc ny ng theo gi thit.
Php chng minh hon tt. ng thc xy ra khi v ch khi a = 0, b = c = 1 v cc hon v.2
69
3.18 Cho a, b, c > 0 tha mn a + b + c
1
2
. Chng minh rng:
1
a a
2
+
1
b b
2
+
1
c c
2

108
5
Li gii.
Cch 1:
T gi thit suy ra 0 < a, b, c
1
2
Xt hiu sau:
1
a a
2

36
5
+
144(6a 1)
25
=
(6a 1)(6a 5)
5(a a
2
)
+
144(6a 1)
25
=
(6a 1)
5
_
6a 5
a a
2
+
144
5
_
=
(6a 1)
5
_
144a
2
+ 174a 25
5(a a
2
)
_
=
(6a 1)
2
(25 24a)
25(a a
2
)
0
Suy ra:
1
a a
2

36
5

144(6a 1)
25
Mt cch tng t:
1
b b
2

36
5

144(6b 1)
25
1
c c
2

36
5

144(6c 1)
25
Cng v theo v:
1
a a
2
+
1
b b
2
+
1
c c
2

108
5

144
25
_
6a + 6b + 6c 3
25
_

108
25
(do a + b + c
1
2
)
Php chng minh hon tt. ng thc xy ra khi v ch khi a = b = c =
1
6
.
Cch 2:
Theo bt ng thc Cauchy-Schwarz:
1
a a
2

9
a + b + c a
2
b
2
c
2
t a + b + c = x. Th th x
1
2
v a
2
+ b
2
+ c
2

x
2
3
.
Do ta ch cn chng minh:
9
x
x
2
3

108
5
Bt ng thc ny tng ng vi:
(2x 1)(2x 5) 0 (ng do x
1
2
)
Php chng minh hon tt. ng thc xy ra khi v ch khi a = b = c =
1
6
.2
3.19 Cho a, b, c, d > 0 tha a + b + c + d = 3. Chng minh rng:
ab
3b + c + d + 3
+
bc
3c + d + a + 3
+
cd
3d + a + b + 3
+
da
3a + b + c + 3

1
3
Li gii.
Vi a + b + c + d = 3, bt ng thc c vit thnh:
ab
4b + 2c + 2d + a
+
bc
4c + 2d + 2a + b
+
cd
4d + 2a + 2b + c
+
da
4a + 2b + 2c + d

1
3
70
p dng bt ng thc Cauchy-Schwarz:
ab
4b + 2c + 2d + a

1
9
_
2ab
2b + d
+
ab
2c + a
_
bc
4c + 2d + 2a + b

1
9
_
2bc
2c + a
+
bc
2d + b
_
cd
4d + 2a + 2b + c

1
9
_
2cd
2d + b
+
cd
2a + c
_
da
4a + 2b + 2c + d

1
9
_
2da
2a + c
+
da
2b + d
_
Cng v theo v, ta c bt ng thc cn chng minh.
Php chng minh hon tt. ng thc xy ra khi v ch khi a = b = c = d =
3
4
. 2
3.20 Cho a, b, c > 0, Chng minh rng:
a
3
b(c + a)
+
b
3
c(a + b)
+
c
2
b + c
a +
b
2
Li gii.
p dng bt ng thc AM-GM:
a
3
b(c + a)
+
b
2
+
c + a
4

3a
2
b
3
c(a + b)
+
c
2
+
a + b
4

3b
2
c
2
b + c
+
b + c
4
c
Cng v theo v:
a
3
b(c + a)
+
b
3
c(a + b)
+
c
2
b + c
+
a
2
+ b + c
3a
2
+
3b
2
+ c
hay
a
3
b(c + a)
+
b
3
c(a + b)
+
c
2
b + c
a +
b
2
Php chng minh hon tt. ng thc xy ra khi v ch khi a = b = c.2
3.21 Cho x, y, z > 0 tha mn x
2
+ y
2
+ z
2
= 3. Tm gi tr nh nht ca:
P =
1
x
+
1
y
+
1
z
+
3(x + y + z)
2
Li gii.
T gi thit suy ra 0 < a, b, c

3
Ta chng minh bt ng thc sau:
1
a
+
3
2
a
a
2
+ 9
4
Tht vy, bt ng thc trn tng ng vi:
(a 1)
2
(4 a) 0 (ng)
Tng t vi b v c, ri cng v theo v, ta c:
P =
1
x
+
1
y
+
1
z
+
3(x + y + z)
2

a
2
+ b
2
+ c
2
+ 27
4
=
15
2
Php chng minh hon tt. ng thc xy ra khi v ch khi a = b = c = 1.2
3.22 (Iran 1996) Cho a, b, c > 0. Chng minh rng:
(ab + bc + ca)
_
1
(a + b)
2
+
1
(b + c)
2
+
1
(c + a)
2
_

9
4
Li gii.
71
t a + b = x; b + c = y; c + a = z. Bt ng thc cn chng minh tng ng vi:
(2xy + 2yz + 2zx x
2
y
2
z
2
)
_
1
x
2
+
1
y
2
+
1
z
2
_
9
hay
2

sym
x
y

sym
x
2
y
2
+ 2

sym
xy
z
2
12 0
hay

sym
(x y)
2
_
2
xy

1
z
2
_
0
Gi s x y z. Th th:
2
yz

1
x
2
0
hay
(x y)
2
_
2
yz

1
x
2
_
0
Li c:
(y z)(y + z x) 0
Suy ra;
x z
x y

y
z
Mt khc:
2
zx

1
y
2
>
2
z(z + y)

1
y
2
> 0
Suy ra:
(z x)
2
_
2
zx

1
y
2
_
+ (x y)
2
_
2
xy

1
z
2
_
(x y)
2
_
2y
2
z
3
x

1
z
2
+
2
xy

1
z
2
_
Vy ta cn chng minh:
2y
2
z
3
x

1
z
2
+
2
xy

1
z
2
0
hay
y
3
+ z
3
xyz 0
hay
(y + z)(y z)
2
+ yz(y + z x) 0 (ng)
Php chng minh hon tt. ng thc xy ra khi v ch khi a = b = c hoc a = 0, b = c v cc
hon v.2
3.23 Cho a, b, c > 0. Chng minh rng:
_
a
b + 2c
_
2
+
_
b
c + 2a
_
2
+
_
c
a + 2b
_
2

1
3
Li gii.
p dng bt ng thc x
2
+ y
2
+ z
2

1
3
(a + b + c)
2
:
_
a
b + 2c
_
2
+
_
b
c + 2a
_
2
+
_
c
a + 2b
_
2

1
3
_
a
b + 2c
+
b
c + 2a
+
c
a + 2b
_
2
Li theo bt ng thc Cauchy-Schwarz v bt ng thc (x + y + z)
2
3(xy + yz + zx):
a
b + 2c
+
b
c + 2a
+
c
a + 2b
=
a
2
ab + 2ac
+
b
2
bc + 2ab
+
c
2
ac + 2bc

(a + b + c)
2
3(ab + bc + ca)
1
Suy ra:
_
a
b + 2c
_
2
+
_
b
c + 2a
_
2
+
_
c
a + 2b
_
2

1
3
_
a
b + 2c
+
b
c + 2a
+
c
a + 2b
_
2

1
3
Php chng minh hon tt. n thc xy ra khi v ch khi a = b = c.2
72
3.24 Cho a, b, c, d > 0 tha mn (a+b +c +d)
_
1
a
+
1
b
+
1
c
+
1
d
_
= 20. Tm gi tr nh nht
ca:
A = (a
2
+ b
2
+ c
2
)
_
1
a
2
+
1
b
2
+
1
c
2
+
1
d
2
_
Li gii.
T gi thit suy ra:
a + b + c
d
= 16
p dng hng ng thc:
(a + b + c d)
2
+ (b + c + d a)
2
+ (c + d + a b)
2
+ (d + a + b c)
2
= 4(a
2
+ b
2
+ c
2
+ d
2
)
Theo bt ng thc Cauchy-Schwarz:
(a
2
+ b
2
+ c
2
+ d
2
).(
1
a
2
+
1
b
2
+
1
c
2
+
1
d
2
) =
1
4
.
_

(a + b + c d)
2
_
.
_

1
a
2
_

1
4
.
_

a + b + c d
d
_
2
=
1
4
_

a + b + c
d
4
_
2
=
1
4
.12
2
= 36
Vy minA = 36 a = c =
3

5
2
.b =
3

5
2
.d. 2
Nhn xt:
Ta c hng ng thc ng ch sau:
(a + b + c d)
2
+ (b + c + d a)
2
+ (c + d + a b)
2
+ (d + a + b c)
2
= 4(a
2
+ b
2
+ c
2
+ d
2
)
T nu c tm s thc dng a, b, c, d, x, y, z, t th p dng bt ng thc Cauchy-Schwarz v
hng ng thc trn:
[(a + b + c + d)(x + y + z + t) 2(at + bx + cy + dz)]
2
= [a(x + y + z t) + b(y + z + t x) + c(z + t + x y) + d(t + x + y z)]
2
(a
2
+ b
2
+ c
2
+ d
2
)[(x + y + z t)
2
+ (y + z + t x)
2
+ (z + t + x y)
2
+ (y + x + y z)
2
]
= 4(a
2
+ b
2
+ c
2
+ d
2
)(x
2
+ y
2
+ z
2
+ t
2
)
hay
(a + b + c + d)(x + y + z + t) 2(at + bx + cy + dz) 2
_
(a
2
+ b
2
+ c
2
+ d
2
)(x
2
+ y
2
+ z
2
+ t
2
)
Bi ton trn l trng hp ring khi a =
1
t
; b =
1
x
; c =
1
y
; d =
1
z
.
3.25 Cho a, b, c > 0. Chng minh rng:
64abc(a + b + c)
3
27[(a + b)(b + c)(c + a)]
2
Li gii.
Ta chng minh bt ng thc sau:
(a + b)(b + c)(c + a)
8
9
(a + b + c)(ab + +bc + ca)
Tht vy, sau khi khai trin, rt gn, ta c bt ng thc tng ng sau:
c(a b)
2
+ b(a c)
2
+ a(b c)
2
0 (ng)
p dng bt ng thc trn:
27[(a + b)(b + c)(c + a)]
2
27.
64
81
.(a + b + c)
2
.(ab + bc + ca)
2
hay
27[(a + b)(b + c)(c + a)]
2

64
3
(a + b + c)
2
(ab + bc + ca)
2
73
p dng bt ng thc (ab + bc + ca)
2
3abc(a + b + c):
64
3
(a + b + c)
2
(ab + bc + ca)
2

64
3
.(a + b + c)
2
.3abc.(a + b + c) = 64abc.(a + b + c)
3
Suy ra:
64abc(a + b + c)
3
27[(a + b)(b + c)(c + a)]
2
Php chng minh hon tt. ng thc xy ra khi v ch khi a = b = c.2
3.26 Cho a, b, c > 0 tha mn a
3
+ b
3
= c
3
. Chng minh rng:
a
2
+ b
2
c
2
> 6(c a)(c b)
Li gii.
t x =
a
c
, y =
b
c
. Theo gi thit th x
3
+ y
3
= 1 v 0 < x, y < 1.
Bt ng thc cn chng minh tng ng vi:
x
2
+ y
2
+ 1 > 6(1 x)(1 y)
T gi thit v bt ng thc AM-GM cho ba s dng:
x
3
y
3
= (1 x)(1 y)(1 + x + x
2
)(1 + y + y
2
) (1 x)(1 y).3x.3y = (1 x)(1 y)9xy
Suy ra:
xy 3
_
(1 x)(1 y
Do :
x
2
+ y
2
1 = x
2
(1 x) + y
2
(1 y) 2xy
_
(1 x)(1 y) 6(1 x)(1 y)
ng thc khng xy ra. Php chng minh hon tt.2
3.27 Cho a, b, c 0. Chng minh rng:
(a + b + c)
4
16(a
2
b
2
+ b
2
c
2
+ c
2
a
2
)
Li gii.
Khng mt tnh tng qut, gi s a = max {a, b, c}.
p dng bt ng thc AM-GM:
(a + b + c)
4
16a
2
(b + c)
2
= 16(a
2
b
2
+ a
2
c
2
+ 2a
2
bc) 16(a
2
b
2
+ b
2
c
2
+ a
2
c
2
)
Php chng mnh hon tt, ng thc xy ra khi v ch khi a = b, c = 0 v cc hon v.2
3.28 Cho a, b, c 0. Chng minh rng:
a

a
2
+ 3bc
b + c
+
b

b
2
+ 3ac
a + c
+
c

c
2
+ 3ab
a + b
a + b + c
Li gii.
Theo bt ng thc AM-GM:
a(b
2
+ 3bc)
(b + c)

b
2
+ 3bc

2a(b
2
+ 3bc)
b
2
+ 3bc + (b + c)
2
=
2a(a
2
+ 3bc)
a
2
+ b
2
+ c
2
+ 5bc
Xt hiu sau:

_
2a(a
2
+ 3bc)
a
2
+ b
2
+ c
2
+ 5bc
a
_
=
a(a
2
+ bc b
2
c
2
)
a
2
+ b
2
+ c
2
+ 5bc
=
a(b + c)(a
2
+ bc b
2
c
2
)
(b + c)(a
2
+ b
2
+ c
2
+ 5bc)
=
a
3
(b + c) a(b
3
+ c
3
)
(b + c)(a
2
+ b
2
+ c
2
+ 5bc)
=
ab(a
2
b
2
) + ac(a
2
b
2
)
(b + c)(a
2
+ b
2
+ c
2
+ 5bc)
= ab

(a
2
b
2
)
_
1
(b + c)(a
2
+ b
2
+ c
2
+ 5bc)

1
(a + c)(a
2
+ b
2
+ c
2
+ 5ac)
_
74
=
ab(a
2
b
2
)(a b)(a
2
+ b
2
+ c
2
+ 5ac + 5bc)
(a + c)(b + c)(a
2
+ b
2
+ c
2
+ 5bc)(a
2
+ b
2
+ c
2
+ 5ac)
0
Do :
a

a
2
+ 3bc
b + c
+
b

b
2
+ 3ac
a + c
+
c

c
2
+ 3ab
a + b
a + b + c
Php chng minh hon tt. ng thc xy ra khi v ch khi a = b = c.2
3.29 Cho x, y, z > 0 thon mn xyz = 1. Chng minh rng:
(a
3
+ b
3
)(b
3
+ c
3
)(c
3
+ a
3
) 2
_
2(2 +

x
3
y
3
+

x
3
)
Li gii.
t x
3
= a, y
3
= b, z
3
= c. Th th abc = 1.
Bt ng thc cn chng minh tng ng vi:
(a + b)(b + c)(c + a) 2
_
2(2 +

ab +

a)
hay
(a + b)(b + c)(c + a) 2
_
2(a + 1)(b + 1)(c + 1)
hay
[(a + b)(b + c)(c + a)]
2
8(a + 1)(b + 1)(c + 1)
Theo bt ng thc Holder th:
(a + b)
2
_
a +
1
b
_
2
(a + 1)
4
hay
a
2
(a + b)
2
(c + 1)
2
(a + 1)
4
Lp cc bt ng thc tng t:
b
2
(b + c)
2
(a + 1)
2
(b + 1)
4
c
2
(c + a)(b + 1)
2
(c + 1)
4
Nhn v theo v ba bt ng thc trn:
a
2
b
2
c
2
(a + b)
2
(b + c)
2
(c + a)
2
(a + 1)
2
(b + 1)
2
(c + 1)
2
(a + 1)
4
(b + 1)
4
(c + 1)
4
hay
[(a + b)(b + c)(c + a)]
2
(a + 1)
2
(b + 1)
2
(c + 1)
2
Li theo bt ng thc AM-GM:
(a + 1)
2
(b + 1)
2
(c + 1)
2
= [(a + 1)(b + 1)(c + 1)][(a + 1)(b + 1)(c + 1)]
8

abc.[(a + 1)(b + 1)(c + 1)]


= 8(a + 1)(b + 1)(c + 1)
Do :
[(a + b)(b + c)(c + a)]
2
8(a + 1)(b + 1)(c + 1)
Php chng minh hon tt. ng thc xy ra khi v ch khi x = y = z = 1.2
3.30 Cho z, y, z > 0. Chng minh rng:
i)
1
(x + 1)
2
+
1
(y + 1)
2

1
1 + xy
ii) Cho xyz = 1. Chng minh rng:
1
(x + 1)
2
+
1
(y + 1)
2
+
1
(z + 1)
2
+
2
(1 + x)(1 + y)(1 + z)
1
Li gii.
i) Tin hnh quy ng, thu gn, ta c bt ng thc tng ng:
x
3
y + xy
3
+ 1 x
2
y
2
+ 2xy
75
hay
xy(x y)
2
+ (xy 1)
2
0 (ng)
ii) t x =
ab
c
2
, y =
bc
a
2
, z =
ca
b
2
Bt ng thc cho tng ng vi:
c
4
(ab + c
2
)
2
+
b
4
(ac + b
2
)
2
+
a
4
(bc + a
2
)
2
+
2a
2
b
2
c
2
(ab + c
2
)(bc + a
2
)(ca + b
2
)
1
Theo bt ng thc Cauchy-Schwarz:
(ab + c
2
)
2
(a
2
+ c
2
).(b
2
+ c
2
)
(ab + c
2
)(bc + a
2
)(ca + b
2
) (a
2
+ b
2
)(b
2
+ c
2
)(c
2
+ a
2
)
Do :
c
4
(ab + c
2
)
2
+
b
4
(ac + b
2
)
2
+
a
4
(bc + a
2
)
2
+
2a
2
b
2
c
2
(ab + c
2
)(bc + a
2
)(ca + b
2
)

c
4
(a
2
+ b
2
) + b
4
(a
2
+ c
2
) + a
4
(c
2
+ b
2
) + 2a
2
b
2
c
2
(a
2
+ b
2
)(a
2
+ c
2
)(c
2
+ b
2
)
= 1
Php chng minh hon tt. ng thc xy ra khi v ch khi x = y = z = 1.2
3.31 Cho a, b, c 0. Chng minh rng:
a

a
2
+ bc + b

b
2
+ ac + c

c
2
+ ab

2(a + b + c)
Li gii.
Ta chng minh b sau:
Cho x, y, z 0. Khi :

x
4
+ xyz(x + y + z) 2

x
2
y
2
S dng bt ng thc AM-GM, ta c bt ng thc quen thuc sau:
xyz (x + y z)(y + z x)(z + x y)
Suy ra:
xyz(x + y + z) (x + y z)(y + z x)(z + x y)(x + y + z)
Khai trin v chuyn v bt ng thc trn, ta c b cn chng minh.
Tr li bi ton:
Bnh phng hai v bt ng thc cn chng minh:

a
4
+ abc

a + 2

ab
_
(a
2
+ bc)(b
2
+ ca) 2

a
2
b
2
+ 4abc

a
p dng bt ng thc Cauchy-Schwarz:
2

ab
_
(a
2
+ bc)(b
2
+ ca) 2

ab(ab + c

ab) = 2

a
2
b
2
+ 2abc

ab
p dng b trn:

a
4
+ abc

a 2

a
2
b
2
2

a
2
b
2
+ 2abc

ab 4abc

a
Cng v theo v hai bt ng thc trn, ta c bt ng thc cn chng minh.
Php chng minh hon tt. ng thc xy ra khi v ch khi a = b = c.2
3.32 Cho a, b, c 0. Chng minh rng:
a(a + b)(a + c)
(b + c)
3
+
b(b + c)(b + a)
(c + a)
3
+
c(c + a)(c + b)
(a + b)
3

(a + b + c)
4
6(ab + bc + ca)
2
Li gii.
Chun ha ab + bc + ca = 3.
Ta cn chng minh:
76
a(a
2
+ 3)
(b + c)
3

(a + b + c)
4
54
hay
a
3
(b + c)
3
+ 3
a
(b + c)
3

(a + b + c)
4
54
p dng bt ng thc Holder:
[

a(b + c)]
3
a
3
(b + c)
3
(

a)
4
[

a(b + c)]
3
a
(b + c)
3
(a + b + c)
4
(1 + 1 + 1) (

a)
2
(

a)
3

ab (

a)
2
(

a)
4
(

a)
4
Kt hp cc bt ng thc trn, ta c bt ng thc cn chng minh.
Php chng minh hon tt. ng thc xy ra khi v ch khi a = b = c.2
3.33 Cho a, b, c 0. Chng minh rng:
1
(b + c)(b
2
+ bc + c
2
)
+
1
(a + b)(a
2
+ ab + b
2
)
+
1
(a + c)(a
2
+ ac + c
2
)

4
(a + b)(b + c)(c + a)
Li gii.
S dng bt ng thc AM-GM:
1
(b + c)(b
2
+ bc + c
2
)
=
ab + bc + ac
(b + c)(b
2
+ bc + c
2
)(ab + bc + ac)

4(ab + bc + ac)
(b + c)
3
(a + b + c)
2
Tng t vi cc phn thc cn li, t ta cn chng minh:
(a + b)(a + c)
(b + c)
2

(a + b + c)
2
ab + bc + ac
Theo bt ng thc Cauchy-Schwarz:
2a
b + c
+
2b
a + c
+
2c
a + b

(a + b + c)
2
ab + bc + ac
Do ta cn chng minh:
(a + b)(a + c)
(b + c)
2

2a(b + c)
(b + c)
2
hay
(a b)(a c)
(b + c)
2
0
Bt ng thc trn ng theo bt ng thc Vonicur-Schur.
Php chng minh hon tt. ng thc xy ra khi v ch khi a = b = c.2
3.34 Cho a, b, c > 0. Chng minh rng:
a(b + c a)
a
2
+ 2bc
+
b(c + a b)
b
2
+ 2ac
+
c(a + b c)
c
2
+ 2ab
0
Li gii.
Gi s a b c.
Trng hp 1: a b + c.
Bt ng thc hin nhin ng.
Trng hp 2: a b + c.
Suy ra:
b
b
2
+ ac

a
a
2
+ bc
=
(a b)(ab ac bc)
(b
2
+ ac)(a
2
+ 2bc)
0
Suy ra:
b(a + c b)
b
2
+ 2ac
+
a(b + c a)
a
2
+ 2bc

2ac
a
2
+ bc
0
T suy ra bt ng thc cn chng minh.
77
Php chng minh hon tt. 2
3.35 Cho a, b, c > 0 tha a + b + c = 3. Chng minh rng:
a
3

ab + b
3

bc + c
3

ca 3
Li gii.
Theo bt ng thc AM-GM:
a
3

ab =
3

a
4
b
2a

ab + a
3
Tng t:
b
3

bc
2b

bc + b
3
c
3

ca
2c

ca + c
3
Do ta ch cn chng minh:
a

ab + b

bc + c

ca 3
hay
a

ab + b

bc + c

ca (a + b + c)
2
Bin i bt ng thc trn thnh bt ng thc tng ng:

(a b

ab

ca + 2

bc)
2
0 (ng)
Php chng minh hon tt. ng thc xy ra khi v ch khi a = b = c = 1.2
3.36 Cho a, b, c l ba cnh ca tam gic. Chng minh rng:
b + c
a
+
(a + b c)(b + c a)(c + a b)
abc
7
Li gii.
Bt ng thc cho tng ng vi:
bc(b + c)
abc
+
2

ab(a + b)

a
3
2abc
abc
4
hay
2

ab(a + b)

a
3
+ 9abc
y chnh l bt ng thc Schur theo cc bin a + b c, b + c a, c + a b.
Php chng minh hon tt. ng thc xy ra khi v ch khi a = b = c.2
3.37 Cho a, b, c > 0. Chng minh rng:

a
2
b
2
+ (c + a)
2

3

5
Li gii.
Bt ng thc cn chng minh tng ng vi:

a
2
5(b
2
+ (c + a)
2
)

3
5
S dng bt ng thc Cauchy-Schwarz:
[(b
2
+ (c + a)
2
] = (1 + 4)[b
2
+ (c + a)
2
] [b + 2(c + a)]
2
Suy ra:

a
2
5(b
2
+ (c + a)
2
)

a
b + 2(c + a)
Ta cn chng minh bt ng thc sau:
a
b + 2(c + a)
+
b
c + 2(a + b)
+
c
a + 2(b + c)

3
5
hay
78
b + 2c
b + 2(c + a)
+
c + 2a
c + 2(a + b)
+
a + 2b
a + 2(b + c)

9
5
Tht vy:

_
1
2

a
b + 2(c + a)
_
=
b + 2c
b + 2(c + a)
=
(b + 2c)
2
(b + 2c)(b + 2c + 2a)

9(a + b + c)
2

(b + 2c)(b + 2c + 2a)
=
9
5
Php chng minh hon tt. ng thc xy ra khi v ch khi a = b = c.2
3.38 Cho a, b, c 0. Chng minh rng:
4(a
3
+ b
3
+ c
3
)
a
2
+ b
2
+ c
2
+
9(a + b)(b + c)(c + a)
(a + b + c)
2
4(a + b + c)
Li gii.
S dng bt ng thc Cauchy-Schwarz:
(a
3
+ b
3
+ c
3
)(a + b + c) (a
2
+ b
2
+ c
2
)
2
Suy ra:
4(a
3
+ b
3
+ c
3
)
a
2
+ b
2
+ c
2

4(a
2
+ b
2
+ c
2
)
a + b + c
Li c:
8(a + b + c)(ab + bc + ca) = 8(a + b)(b + c)(c + a) + 8abc 9(a + b)(b + c)(c + a)
Nn
4(a
3
+ b
3
+ c
3
)
a
2
+ b
2
+ c
2
+
9(a + b)(b + c)(c + a)
(a + b + c)
2

4(a
2
+ b
2
+ c
2
)
a + b + c
+
8(ab + bc + ca)
a + b + c
= 4(a + b + c)
Php chng minh hon tt. ng thc xy ra khi v ch khi a = b = c.2
3.39 Cho a, b, c 0. Chng minh rng:
a
3
b
a
4
+ a
2
b
2
+ b
4
+
b
3
c
b
4
+ b
2
c
2
+ c
4
+
c
3
a
c
4
+ c
2
a
2
+ a
4
1
Li gii.
Theo bt ng thc AM-GM;
a
3
b
a
4
+ a
2
b
2
+ b
4

a
3
b
2a
3
b + b
4
=
a
3
2a
3
+ b
3
Ta cn chng minh:
a
3
2a
3
+ b
3
1
hay
a
3
a
3
+ 2c
3
1
Tht vy. Theo bt ng thc Cauchy-Schwarz:
a
3
a
3
+ 2c
3

_

a
3
_
2

a
6
+ 2

a
3
b
3
= 1
Php chng minh hon tt. ng thc xy ra khi v ch khi a = b = c.2
3.40 Cho a, b, c 0. Chng minh rng:
(a + b + c)
3
abc
+
ab + bc + ca
a
2
+ b
2
+ c
2
28
Li gii.
Ta c bt ng thc sau:
(a + b + c)
3
a
3
+ b
3
+ c
3
+ 24abc
79
Tht vy. Sau khi khai trin v rt gon, ta c bt ng thc tng ng:

c(a b)
2
0 (ng)
p dng bt ng thc trn, ta cn chng minh:
(a + b + c)
3
abc
+
ab + bc + ca
a
2
+ b
2
+ c
2

a
3
+ b
3
+ c
3
+ 24abc
abc
+
ab + bc + ca
a
2
+ b
2
+ c
2
28
hay
a
2
bc
+
ab + bc + ca
a
2
+ b
2
+ c
2
4
p dng bt ng thc Chebychep;
a
2
bc
+
ab + bc + ca
a
2
+ b
2
+ c
2

3(a
2
+ b
2
+ c
2
)
ab + bc + ca
+
ab + bc + ca
a
2
+ b
2
+ c
2
Ta cn chng minh:
3(a
2
+ b
2
+ c
2
)
ab + bc + ca
+
ab + bc + ca
a
2
+ b
2
+ c
2
4
Bt ng thc ny c to bi hai bt ng thc lun ng sau:
2(a
2
+ b
2
+ c
2
)
ab + bc + ca
2
a
2
+ b
2
+ c
2
ab + bc + ca
+
ab + bc + ca
a
2
+ b
2
+ c
2
2 (theo bt ng thc AM-GM)
Php chng minh hon tt. ng thc xy ra khi v ch khi a = b = c.2
3.4 Bi 4.1 n bi 4.40
4.1 Cho a, b, c > 0. Chng minh rng:
3

abc +
|a b| +|b c| +|c a|
3

a + b + c
3
Li gii.
Gi s a b c. Bt ng thc cn chng minh tr thnh:
3

abc +
2a 2c
3

a + b + c
3
hay
3

abc +
a b 3c
3
0
Ta c:
f

(a) =
1
3
+
1
2
3

bc
1
a
2
> 0
nn hm f(a) ng bin. Suy ra:
f(a) f(b) =
3

b
2
c c 0
Php chng minh hon tt. ng thc xy ra khi v ch khi a = b = c.2
Nhn xt:
Bi ton tng qut:
Vi mi a
1
, a
2
, ..., a
n
0 th
n
n

a
1
a
2
...a
n
+

i=j
|a
i
a
j
|

n
i=1
a
i
4.2 Cho a, b, c l ba cnh ca tam gic. Chng minh rng:
(2b
2
+ 2c
2
a
2
)(2c
2
+ 2a
2
b
2
)(2a
2
+ 2b
2
c
2
) (2a
2
+ bc)(2b
2
+ ca)(2c
2
+ ab)
Li gii.
Do a, b, c l ba cnh tam gic nn
(2a
2
+ bc)
2
(2a
2
+ 2b
2
c
2
)(2a
2
+ 2c
2
b
2
) = [(b + c)
2
a
2
](b c)
2
0
Suy ra:
80
(2a
2
+ bc)
2
(2a
2
+ 2b
2
c
2
)(2a
2
+ 2c
2
b
2
)
Tng t:
(2b
2
+ ca)
2
(2b
2
+ 2c
2
a
2
)(2b
2
+ 2a
2
c
2
)
(2c
2
+ ab)
2
(2c
2
+ 2a
2
b
2
)(2c
2
+ 2b
2
a
2
)
Nhn v theo v ba bt ng thc trn, ta c bt ng thc cn chng minh.
Php chng minh hon tt. ng thc xy ra khi v ch khi a = b = c.2
4.3 Cho a, b, c 0. Chng minh rng:
ab
(a + b)
2
+
bc
(b + c)
2
+
ca
(c + a)
2

1
4
+
4abc
(a + b)(b + c)(c + a)
Li gii.
Chun ha c = 1. Ta cn chng minh:
(a b)
2
4(a + b)
2
+
4ab
(a + b)(a + 1)(b + 1)

a
(a + 1)
2
+
b
(b + 1)
2
hay
(a b)
2
4(a + b)
+
1
(a + 1)
2
+
1
(b + 1)
2

2ab
(a + 1)(b + 1)

a + b
(a + 1)(b + 1)

4ab
(a + b)(a + 1)(b + 1)
hay
(a b)
2
(a + b)
2
+
4(a b)
2
(a + 1)
2
(b + 1)
2

4(a b)
2
(a + 1)(b + 1)(a + b)
Kt qu ny hin nhin ng theo bt ng thc AM-GM cho hai s dng.
Php chng minh hon tt. ng thc xy ra khi v ch khi c hai bin bng nhau.2
4.4 Cho a, b, c > 0 tha a
2
+ b
2
+ c
2
+ abc = 4. Chng minh rng:
bc
a
a
2
+ b
2
+ c
2
Li gii.
t x =
ab
c
; y =
bc
a
; z =
ca
b
.
Theo gi thit th xy + yz + zx + xyz = 4. Suy ra
x
x + 2
= 1
Bt ng thc cn chng minh tng ng vi:
x + y + z xy + yz + zx
p dng bt ng thc Cauchy-Schwarz:
_
x
x + 2
_
[

x (x + 2)] (

x)
2
]
Suy ra:

x(x + 2) (x + y + z)
2
hay
x + y + z xy + yz + zx
Php chng minh hon tt. ng thc xy ra khi v ch khi a = b = c = 1.2
4.5 Cho a, b, c > 0 tha a
2
+ b
2
= 2(a + b) + 1.
Tm gi tri nh nht, gi tr ln nht ca P = a
2
+ b
2
Li gii.
t a
2
+ b
2
= x > 0.
Theo gi thit v bt ng thc a + b
_
2(a
2
+ b
2
):
x = 2(a + b) + 1 2

2x + 1
81
Suy ra:
x 2

2x + 1 0
Suy ra:

2 1 x 1 +

2
Do :
3 2

2 x 3 +

2
Vy:
min(a
2
+ b
2
) = 3 2

2 a = b =
_
3
2

2
max(a
2
+ b
2
) = 3 + 2

2 a = b =
_
3
2
+

2 .2
4.6 Cho a, b, c > 0 tha mn a
2
+ b
2
+ c
2
= 3. Chng minh rng:
3
_
2a
3
a
2
+ b
2
+
3
_
2b
3
b
2
+ c
2
+
3
_
2c
3
c
2
+ a
2
3
Li gii.
Ta c:
a + b + c
_
3(a
2
+ b
2
+ c
2
) = 3
8(x + y + z)(xy + yz + zx) 9(x + y)(y + z)(z + x)
2a
3
a
2
+ b
2
= 2a.(a
2
+ c
2
).
a
2
(a
2
+ b
2
)(a
2
+ c
2
)
S dng cc ng thc bt ng thc trn v bt ng thc Holder:
_
3

2a
3
a
2
+ b
2
+
3

2b
3
b
2
+ c
2
+
3

2c
3
c
2
+ a
2
_
3

a
2
(a
2
+ b
2
)(a
2
+ c
2
)
.

(2a).

(a
2
+ c
2
)
=

a
2
(a
2
+ b
2
)(a
2
+ c
2
)
.

(2a).

(a
2
+ c
2
)
= 9(a + b + c) 27
Suy ra:
3
_
2a
3
a
2
+ b
2
+
3
_
2b
3
b
2
+ c
2
+
3
_
2c
3
c
2
+ a
2
3
Php chng minh hon tt. ng thc cy ra khi v ch khi a = b = c.2
4.7 Cho a, b, c > 0 tha mn abc = 1. Chng minh rng:
3(a
4
b
4
+ b
4
c
4
+ c
4
a
4
)
a
2
+ b
2
+ c
2
+
8a
3
(bc + a)
3
+
8b
3
(ac + b)
3
+
8c
3
(ba + c)
3
6[
Li gii.
Ta c:
3(a
4
b
4
+ b
4
c
4
+ c
4
a
4
)
a
2
+ b
2
+ c
2

3a
2
b
2
c
2
(a
2
+ b
2
+ c
2
)
a
2
+ b
2
+ c
2
= 3
V vy ta cn chng minh:
a
3
(a + bc)
3

3
8
hay
a
6
(a
2
+ 1)
3

3
8
(do abc = 1)
hay
82
x
3
(x + 1)
3

3
8
(vi x = a
2
, y = b
2
, x = c
2
, xyz = 1)
y l bt ng thc Vietnam TST 2005.
p dng bt ng thc AM-GM:

_
2.
x
3
(x + y)
3
+
1
8
_

3
2
.
x
2
(x + y)
2
Do ta ch cn chng minh:
x
2
(x + y)
2
+
y
2
(y + z)
2
+
z
2
(z + x)
2

3
4
Trc ht ta chng minh:
1
(x + 1)
2
+
1
(y + 1)
2

1
xy + 1
Tht vy. S dng bt ng thc Cauchy-Schwarz:
1
(x + 1)
2

1
(xy + 1)
_
x
y
+ 1
_ =
y
(xy + 1)(x + y)
nn
1
(x + 1)
2
+
1
(y + 1)
2

y
(xy + 1)(x + y)
+
x
(xy + 1)(x + y)
=
1
xy + 1
Cui cng, ta cn chng minh:
1
xy + 1
+
1
(z + 1)
2

3
4
hay
z
z + 1
+
1
(z + 1)
2

3
4
Bt ng thc ny ng v:
z
z + 1
+
1
(z + 1)
2

3
4
=
(z 1)
2
4(z + 1)
2
0
Php chng minh hon tt. ng thc xy ra khi v ch khi a = b = c = 1
Nhn xt: Bt ng thc
1
(x + 1)
2
+
1
(y + 1)
2

1
xy + 1
rt thng dng trong vic chng minh bt ng thc.
Hy ly v d l bi bt ng thc trong China MO 2004:
Vi x, y, z, t > 0 tha mn xyzt = 1. Chng minh rng:
1
(x + 1)
2
+
1
(y + 1)
2
+
1
(z + 1)
2
+
1
(t + 1)
2
1
S dng kt qu trn, ta c ngay:
1
(x + 1)
2

1
xy + 1
+
1
zt + 1
= 1. (do xyzt = 1) .
2
4.8 Cho a, b, c > 0 tha mn a
2
+ b
2
+ c
2
= 3. Chng minh rng:
1
a + b
+
1
b + c
+
1
c + a

4
a
2
+ 7
+
4
b
2
+ 7
+
4
c
2
+ 7
Li gii.
p dng bt ng thc Cauchy-Schwarz:
83
a
2
a
2
+ 7
+
1
a + b

(a + b + c)
2
24
+
9
2(a + b + c)
t a + b + c = x
_
3(a
2
+ b
2
+ c
2
) = 3. Ta chng minh:
x
2
12
+
9
2x

15
8
Bt ng thc ny tng ng vi:
(x 3)
_
x
3 + 3

17
2
__
x
3 3

17
2
_
0 (ng, do x 3)
Do :
a
2
a
2
+ 7
+
1
a + b

(a + b + c)
2
24
+
9
2(a + b + c)

15
8
Suy ra:
1
a + b

15
8

a
2
a
2
+ 7
=
7
a
2
+ 7

9
8
Ta cn chng minh:
7
a
2
+ 7

9
8

4
a
2
+ 7
hay
1
a
2
+ 7

3
8
Bt ng thc trn ng theo bt ng thc Cauchy-Schwarz:
1
x
2
+ 7

9

a
2
+ 21
=
3
8
Php chng minh hon tt. ng thc xy ra khi v ch khi a = b = c = 1.2
84
4.9 Cho x, y, z > 0 tho mn x + y + z = 1. Tm gi tr nh nht ca biu thc:
M =
x + y
xyz
Li gii.
Ta c:
M =
x + y
xyz
=
1
yz
+
1
xz
p dng cc bt ng thc Cauchy - Schwarz v AM - GM, ta c:
M
4
xz + yz
=
4
_
z(x + y)
2

4
_
z + x + y
2
_
2
= 16
Vy min M = 16 z =
1
2
; x = y =
1
4
. 2
4.10 Cho x, y, z l cc s thc tho mn x
4
+y
4
+z
4
= 3. Tm gi tr ln nht ca biu thc:
N = x
2
(y + z) + y
2
(z + x) + z
2
(x + y)
Li gii.
p dng bt ng thc Cauchy - Schwarz, ta c:
N
2
(x
4
+ y
4
+ z
4
)[(y + z)
2
+ (z + x)
2
+ (x + y)
2
] = 6(x
2
+ y
2
+ z
2
+ xy + yz + zx)
Ta c mt kt qu quen thuc:
xy + yz + zx x
2
+ y
2
+ z
2
Do
N
2
12(x
2
+ y
2
+ z
2
)
Li p dng bt ng thc Cauchy - Schwarz, ta c:
N
4
144.3(x
4
+ y
4
+ z
4
) = 1296
T y suy ra N 6.
Vy max N = 6 x = y = z = 1.2
4.11 Cho x, y, z > 0. Chng minh rng:
16xyz(x + y + z) 3
3
_
(x + y)(y + z)(z + x)
4
Li gii.
Li gii 1.
p dng bt ng thc AM - GM, ta c:
(x + y)(z + x) =
x(x + y + z)
3
+
x(x + y + z)
3
+
x(x + y + z)
3
+ yz 4
4
_
x
3
(x + y + z)
3
yz
27
y + z 2

yz
Nhn theo v hai bt ng thc trn, ta c:
(x + y)(y + z)(z + x) 8
4
_
x
3
y
3
z
3
(x + y + z)
3
27
3
3
_
(x + y)(y + z)(z + x)
4
16xyz(x + y + z) .
Php chng minh hon tt. Du "=" xy ra x = y = z.2
Li gii 2.
Ta vit li bt ng thc cn chng minh nh sau:
3
8
(x + y)(y + z)(z + x)
3
_
(x + y)(y + z)(z + x) 2xyz(x + y + z)
Trc ht, ta s chng minh:
9
8
(x + y)(y + z)(z + x) (x + y + z)(xy + yz + zx) (1)
Ta c ng thc (x + y)(y + z)(z + x) = (x + y + z)(xy + yz + zx) xyz
85
Do , (1) (x + y + z)(xy + yz + zx) 9xyz
iu ny ng v theo bt ng thc AM - GM th V T 3
3

xyz.3
3
_
x
2
y
2
z
2
= V P.
Vy (1) c chng minh.
Li p dng bt ng thc AM - GM, ta c:
(x + y + z)(xy + yz + zx) 3(x + y + z)
3
_
x
2
y
2
z
2
(2)
T (1) v (2) ta suy ra:
3
8
(x + y)(y + z)(z + x) (x + y + z)
3
_
x
2
y
2
z
2
By gi, ta ch cn chng minh c:
3
_
(x + y)(y + z)(z + x) 2
3

xyz
hay
(x + y)(y + z)(z + x) 8xyz
Nhng iu ny li ng theo bt ng thc AM - GM:
(x + y)(y + z)(z + x) 2

xy.2

yz.2

zx = 8xyz
Do php chng minh hon tt. Du "=" xy ra x = y = z.2
4.12 Cho a, b, c l di ba cnh ca tam gic. Chng minh rng:
a
3a b + c
+
b
3b c + a
+
c
3c a + b
1
Li gii.
D thy bt ng thc trn tng ng vi mi bt ng thc trong dy sau:
4a
3a b + c
1 +
4b
3b c + a
1 +
4c
3c a + b
1 1
a + b c
3a b + c
+
b + c a
3b c + a
+
c + a b
3c a + b
1
Bt ng thc cui cng ng v theo bt ng thc Cauchy - Schwarz:

a + b c
3a b + c
=
(a + b c)
2
(3a b + c)(a + b c)

[(a + b c)]
2
(3a b + c)(a + b c)
=
(a + b + c)
2
(a + b + c)
2
= 1 .
Vy php chng minh hon tt. Du "=" xy ra a = b = c.2
4.13 Cho a, b, c > 0. Chng minh rng:
a(b + c)
(a + b)(a + c)
+
b(c + a)
(b + c)(b + a)
+
c(a + b)
(c + a)(c + b)

(a + b + c)
2
2(a
2
+ b
2
+ c
2
)
Li gii.
Bt ng thc tng ng vi mi bt ng thc trong dy sau:
a(b + c)
2
+ b(c + a)
2
+ c(a + b)
2
(a + b)(b + c)(c + a)

(a + b + c)
2
2(a
2
+ b
2
+ c
2
)
1 +
4abc
(a + b)(b + c)(c + a)
1 +
2(ab + bc + ca) (a
2
+ b
2
+ c
2
)
2(a
2
+ b
2
+ c
2
)
8abc(a
2
+ b
2
+ c
2
)
(a + b)(b + c)(c + a)
2(ab + bc + ca) (a
2
+ b
2
+ c
2
)
Theo bt ng thc Schur, ta c:
2(ab + bc + ca) (a
2
+ b
2
+ c
2
) = 4(ab + bc + ca) (a + b + c)
2

9abc
a + b + c
By gi ta ch cn chng minh c:
8abc(a
2
+ b
2
+ c
2
)
(a + b)(b + c)(c + a)

9abc
a + b + c
iu ny tng ng vi
86
8(a + b + c)(a
2
+ b
2
+ c
2
) 9(a + b)(b + c)(c + a)
hay
8(a
3
+ b
3
+ c
3
) a
2
(b + c) + b
2
(c + a) + c
2
(a + b) + 18abc
hay
8(a
3
+ b
3
+ c
3
) ab(a + b) + bc(b + c) + ca(c + a) + 18abc
Bt ng thc ny ng t cc bt ng thc sau:
7(a
3
+ b
3
+ c
3
) 21abc (Theo bt ng thc AM - GM)
a
3
+ b
3
+ c
3
+ 3abc ab(a + b) + bc(b + c) + ca(c + a) (Theo bt ng thc Schur)
Vy php chng minh hon tt. Du "=" xy ra a = b = c.2
4.14 Cho a, b, c > 0. Chng minh rng:
b + c
a
+
c + a
b
+
a + b
c
3 +
(a
2
+ b
2
+ c
2
)(ab + bc + ca)
abc(a + b + c)
Li gii.
Bt ng thc trn tng ng vi mi bt ng thc trong dy sau:
bc(b + c) + ca(c + a) + ab(a + b) + 3abc
abc
6 +
(a
2
+ b
2
+ c
2
)(ab + bc + ca)
abc(a + b + c)
(a + b + c)(ab + bc + ca)
abc

(a
2
+ b
2
+ c
2
)(ab + bc + ca)
abc(a + b + c)
6
2(ab + bc + ca)
2
abc(a + b + c)
6
(ab + bc + ca)
2
3abc(a + b + c)
Bt ng thc cui cng ng theo mt kt qu quen thuc:
(x + y + z)
2
3(xy + yz + zx)
Vy php chng minh hon tt. Du "=" xy ra a = b = c.2
4.15 Cho ba s thc dng a, b, c tho mn a +b +c = 6; ab +bc +ca = 9; a < b < c. Chng
minh rng:
0 < a < 1 < b < 3 < c < 4
Li gii.
Theo bt ng thc AM - GM:
c(6 c) = c(a + b) = ac + bc = 9 ab 9
(a + b)
2
2
=
36 (6 c)
2
2
Suy ra c 4. Du "=" khng xy ra v a < b. Do c < 4.
Ta c: (b a)(b c) < 0
T suy ra
b
2
+ 9 < 2b(a + c) = 2b(6 b)
hay
(b 1)(b 3) < 0
hay
1 < b < 3 .
Vi cch lm tng t, ta c (a 1)(a 3) > 0 v (c 1)(c 3) > 0.
87
Do c > b > 1 nn c > 3.
Do a < b < 3 nn a < 1.
Vy ta c 0 < a < 1 < b < 3 < c < 4.2
4.16 Cho a, b, c > 0. Chng minh rng:
(a + b + c)
3
3(a
2
+ b
2
+ c
2
)

ab(a + b)
a
2
+ b
2
+
bc(b + c)
b
2
+ c
2
+
ca(c + a)
c
2
+ a
2
Li gii.
Bt ng thc tng ng vi mi bt ng thc trong dy sau:
(a + b + c)
3

3(a
2
+ b
2
+ c
2
)ab(a + b)
a
2
+ b
2
+
3(a
2
+ b
2
+ c
2
)bc(b + c)
b
2
+ c
2
+
3(a
2
+ b
2
+ c
2
)ca(c + a)
c
2
+ a
2
a
3
+ 3ab(a + b) + 6abc 3ab(a + b) +
3c
2
ab(a + b)
a
2
+ b
2
a
3
+ 6abc 6abc
c(a + b)
2(a
2
+ b
2
)
Theo bt ng thc Cauchy - Schwarz, ta c:
6abc
c(a + b)
2(a
2
+ b
2
)
6abc
c(a + b)
(a + b)
2
= 6abc
c
a + b
By gi ta ch cn chng minh c:
a
3
+ 6abc 6abc
c
a + b
iu ny tng ng vi
(a
3
+ 6abc)(a + b)(b + c)(c + a) 6abc[a
3
+ a
2
(b + c) + 3abc]
hay
a
3
(a + b)(b + c)(c + a) 6abc[a
3
+ a
2
(b + c) + 3abc (a + b)(b + c)(c + a)]
hay
a
3
(a + b)(b + c)(c + a) 6abc(a
3
+ abc)
hay
a
3
[(a + b)(b + c)(c + a) 6abc] 6a
2
b
2
c
2
iu ny ng theo bt ng thc AM - GM:
a
3
[(a + b)(b + c)(c + a) 6abc] 3abc(2

ab.2

bc.2

ca 6abc) = 6a
2
b
2
c
2
Php chng minh hon tt. Du "=" xy ra a = b = c.2
4.17 Cho a, b, c > 0 tho mn a + b + c = 1. Tm gi tr nh nht ca biu thc:
P =
a
2
+ b
b + c
+
b
2
+ c
c + a
+
c
2
+ a
a + b
Li gii.
Ta c:
P + 1 =
a
2
+ b
b + c
+ a +
b
2
+ c
c + a
+ b +
c
2
+ a
a + b
+ c
=
a(a + b + c) + b
b + c
+
b(a + b + c) + c
c + a
+
c(a + b + c) + a
a + b
=
a + b
b + c
+
b + c
c + a
+
c + a
a + b
p dng bt ng thc AM - GM, ta c ngay P + 1 3, suy ra P 2.
88
Vy min P = 2 a = b = c =
1
3
.2
4.18 Cho a, b, c, d > 0 tho mn a + b + c + d = 4. Chng minh rng:
a
2
bc + b
2
cd + c
2
da + d
2
ab 4
Li gii.
p dng bt ng thc AM - GM lin tip, ta c:
(ab + cd)(ac + bd) =
_
(ab + cd)(ac + bd)
2

_
ab + cd + ac + bd
2
_
2
=
1
4
.[(a + d)(b + c)]
2
[(a + d)(b + c)]
2
=
_
(a + d)(b + c)
4

_
a + b + c + d
2
_
4
= 16
T y suy ra:
(ab + cd)(ac + bd) 4
Tng t, ta chng minh c:
(bc + da)(bd + ac) 4
Ta c cc nh gi sau:
V T (ab + cd)(ac + bd) = bd(a c)(b d)
V T (bc + da)(bd + ac) = ac(a c)(b d)
D thy trong 2 biu thc trn c mt biu thc khng dng nn
V T max{(ab + cd)(ac + bd), (bc + da)(bd + ac)} 4
Php chng minh hon tt. Du "=" xy ra a = b = c = d = 1.2
4.19 Cho a, b, c > 0. Chng minh rng:
a
3
+ abc
b
3
+ c
3
+ abc
+
b
3
+ abc
c
3
+ a
3
+ abc
+
c
3
+ abc
a
3
+ b
3
+ abc
2
Li gii.
Do bt ng thc l thun nht nn ta chun ho abc = 1. Ta c th vit li bt ng thc cn
chng minh di dng:
a + 1
b + c + 1
+
b + 1
c + a + 1
+
c + 1
c + a + 1
2
p dng bt ng thc Cauchy - Schwarz, ta c:
V T =
(a + 1)
2
(a + 1)(b + c + 1)
+
(b + 1)
2
(b + 1)(c + a + 1)
+
(c + 1)
2
(c + 1)(c + a + 1)

(a + b + c + 3)
2
2(ab + bc + ca) + 3(a + b + c) + 3
Ta s chng minh:
(a + b + c + 3)
2
2(ab + bc + ca) + 3(a + b + c) + 3
2
iu ny tng ng vi:
a
2
+ b
2
+ c
2
+ 2(ab + bc + ca) + 6(a + b + c) + 9 4(ab + bc + ca) + 6(a + b + c) + 6
hay
3 2(ab + bc + ca) (a
2
+ b
2
+ c
2
)
Theo bt ng thc Schur, ta c:
2(ab + bc + ca) (a
2
+ b
2
+ c
2
) = 4(ab + bc + ca) (a + b + c)
2

9abc
a + b + c
=
9
a + b + c
Vy ta ch cn chng minh:
9
a + b + c
3
89
hay
3
3

abc a + b + c
iu ny ng theo bt ng thc AM - GM.
Php chng minh hon tt. Du "=" xy ra a = b = c.2
4.20 Cho a, b, c > 0. Chng minh rng:

a
3
5a
2
+ (b + c)
2
+

b
3
5b
2
+ (c + a)
2
+

c
3
5c
2
+ (a + b)
2

_
a + b + c
3
Li gii.
p dng bt ng thc Cauchy - Schwarz, ta c:
V T

(a + b + c)
_
a
2
5a
2
+ (b + c)
2
+
b
2
5b
2
+ (c + a)
2
+
c
2
5c
2
+ (a + b)
2
_
Ta s chng minh:
a
2
5a
2
+ (b + c)
2
+
b
2
5b
2
+ (c + a)
2
+
c
2
5c
2
+ (a + b)
2

1
3
Ta c:

a
2
5a
2
+ (b + c)
2
=
1
9
.
(3a)
2
a
2
+ b
2
+ c
2
+ 4a
2
+ 2bc
Li p dng bt ng thc Cauchy - Schwarz, ta c:

(3a)
2
a
2
+ b
2
+ c
2
+ 4a
2
+ 2bc

_
a
2
a
2
+ b
2
+ c
2
+
a
2
2a
2
+ bc
+
a
2
2a
2
+ bc
_
=
a
2
a
2
+ b
2
+ c
2
+
2a
2
2a
2
+ bc
Do :

a
2
5a
2
+ (b + c)
2

1
9
_

a
2
a
2
+ b
2
+ c
2
+
2a
2
2a
2
+ bc
_
=
1
9
_
1 +
2a
2
2a
2
+ bc
_
By gi ta ch cn chng minh c:
1
9
_
1 +
2a
2
2a
2
+ bc
_

1
3
Bt ng thc ny tng ng vi mi bt ng thc trong dy sau:
1 +
2a
2
2a
2
+ bc
3
4
bc
2a
2
+ bc
3

bc
2a
2
+ bc
1
Bt ng thc cui cng ng v theo Cauchy - Schwarz th:

bc
2a
2
+ bc
=
b
2
c
2
2a
2
bc + b
2
c
2

(ab + bc + ca)
2
a
2
b
2
+ 2abc(a + b + c)
= 1
Vy php chng minh hon tt. Du "=" xy ra a = b = c.2
4.21 Cho a, b, c > 0 tho mn a + b + c = 3. Chng minh rng:
(a + c)(b + 1) abc(a
2
+ b
2
+ c
2
+ 1)
Li gii.
Ta c:
(a c)
2
4ac

(a c)
2
2b
2
+ 2 + (a + c)
2
T suy ra
90
(a + c)
2
4ac
1
2(a
2
+ b
2
+ c
2
+ 1)
2b
2
+ 2 + (a + c)
2
1
hay
(a + c)
2
4ac

2(a
2
+ b
2
+ c
2
+ 1)
2b
2
+ 2 + (a + c)
2
hay
[2b
2
+ 2 + (a + c)
2
](a + c)
2
b
8
abc(a
2
+ b
2
+ c
2
+ 1)
By gi ta ch cn chng minh:
(a + c)(b + 1)
[2b
2
+ 2 + (a + c)
2
](a + c)
2
b
8
hay
8(b + 1) [2b
2
+ 2 + (a + c)
2
](a + c)b 0
Kt hp vi gi thit a + b + c = 3, ta suy ra cn phi chng minh:
8(b + 1) [2b
2
+ 2 + (3 b)
2
](3 b)b 0
hay
3b
4
15b
3
+ 29b
2
25b + 8 0
hay
(b 1)
2
(3b
2
9b + 8) 0 (lun ng)
Php chng minh hon tt. Du "=" xy ra a = b = c = 1.2
4.22 Cho x, y, z [1; 2]. Tm gi tr ln nht ca biu thc:
Q = (x + y + z)
_
1
x
+
1
y
+
1
z
_
Li gii.
Ta c:
Q = 3 +
x
y
+
x
z
+
y
x
+
y
z
+
z
x
+
z
y
Khng mt tnh tng qut, gi s x y z. Khi th
(x y)(y z) 0
hay
y
2
+ xz xy + yz
hay
y
x
+
z
y
1 +
z
x
v
y
z
+
x
y
1 +
x
z
V vy
Q 5 + 2
_
x
z
+
z
x
_
D thy 1
x
z
2 nn (x 2z)(x z) 0 hay
x
z
+
z
x

5
2
.
T suy ra Q 10.
Vy max Q = 10 x = y = 2; z = 1 hoc x = 2; y = z = 1 v cc hon v.2
4.23 Cho a, b, c 0. Chng minh rng:
_
a
b + c
+
_
b
c + a
+
_
c
a + b
+
ab
(a + b)
2
+
bc
(b + c)
2
+
ca
(c + a)
2

9
4
Li gii.
91
S dng bt ng thc Holder cng vi nh gi 3abc 0, ta c:

_
a
b + c
=

a
3
a
2
(b + c)

(a + b + c)
3
a
2
(b + c) + b
2
(c + a) + c
2
(a + b)

(a + b + c)
3
(a + b + c)(ab + bc + ca)
=
a + b + c

ab + bc + ca
p dng bt ng thc Cauchy - Schwarz lin tip, ta c:

ab
(a + b)
2

ab
2(a
2
+ b
2
)
=
(a + b)
2
4(a
2
+ b
2
)

3
4

4(a + b + c)
2
8(a
2
+ b
2
+ c
2
)

3
4
=
(a + b + c)
2
2(a
2
+ b
2
+ c
2
)

3
4
Cng theo v hai bt ng thc trn, ta c:

_
a
b + c
+
ab
(a + b)
2

a + b + c

ab + bc + ca
+
(a + b + c)
2
2(a
2
+ b
2
+ c
2
)

3
4
Vy ta ch cn chng minh
a + b + c

ab + bc + ca
+
(a + b + c)
2
2(a
2
+ b
2
+ c
2
)

3
4

9
4
hay
a + b + c
2

ab + bc + ca
+
a + b + c
2

ab + bc + ca
+
(a + b + c)
2
2(a
2
+ b
2
+ c
2
)
3
iu ny ng theo bt ng thc AM - GM:
V T 3
3

(a + b + c)
4
[2
_
2(ab + bc + ca)(a
2
+ b
2
+ c
2
)]
2
3
3

(a + b + c)
4
[2(ab + bc + ca) + (a
2
+ b
2
+ c
2
)]
2
= 3
Php chng minh hon tt. Du "=" xy ra a = b; c = 0 v cc hon v.2
4.24 Cho a, b, c > 0. Chng minh rng:
ab
(a + b)
2
+
bc
(b + c)
2
+
ca
(c + a)
2

4abc
(a + b)(b + c)(c + a)
+
1
4
Li gii.
Bt ng thc ny tng ng vi mi bt ng thc trong dy sau:
4ab
(a + b)
2
+
4bc
(b + c)
2
+
4ca
(c + a)
2

16abc
(a + b)(b + c)(c + a)
1
4ab
(a + b)
2
+ 1 +
4bc
(b + c)
2
+ 1 +
4ca
(c + a)
2
+ 1 +
16abc 2(a + b)(b + c)(c + a)
(a + b)(b + c)(c + a)
0
(a b)
2
(a + b)
2
+
(b c)
2
(b + c)
2
+
(c a)
2
(c + a)
2

2a(b c)
2
+ 2b(c a)
2
+ 2c(a b)
2
(a + b)(b + c)(c + a)
0

(b c)
2
b + c
_
1
b + c

2a
(c + a)(a + b)
_
0

(b c)
2
b + c
.
(a b)(a c)
(a + b)(b + c)(c + a)
0
(a b)(b c)(c a)
(a + b)(b + c)(c + a)
.
b c
b + c
0
(a b)(b c)(c a)
(a + b)(b + c)(c + a)
.
(a b)(b c)(a c)
(a + b)(b + c)(c + a)
0
(a b)
2
(b c)
2
(c a)
2
(a + b)
2
(b + c)
2
(c + a)
2
0 (lun ng)
Php chng minh hon tt. Du "=" xy ra a = b = c.2
92
4.25 Cho a, b, c > 0. Chng minh rng:
_
2a
b + c
+
_
2b
c + a
+
_
2c
a + b

(a + b + c)
2
a
2
+ b
2
+ c
2
Li gii.
Li gii 1.
p dng bt ng thc AM - GM, ta c:
2
_
2a(b + c) b + c + 2a
T suy ra
_
2a
b + c
=
4a
2
_
2a(b + c)

4a
b + c + 2a
Tng t, ta chng minh c:
_
2b
c + a

4b
c + a + 2b
_
2c
a + b

4c
a + b + 2c
Cng theo v ba bt ng thc trn, ta c:
V T
4a
b + c + 2a
+
4b
c + a + 2b
+
4c
a + b + 2c
Ta s chng minh
4a
b + c + 2a
+
4b
c + a + 2b
+
4c
a + b + 2c

(a + b + c)
2
a
2
+ b
2
+ c
2
p dng bt ng thc Cauchy - Schwarz, ta c:

4a
b + c + 2a
=
4a
2
ab + ac + 2a
2

4(a + b + c)
2
2(a
2
+ b
2
+ c
2
) + 2(ab + bc + ca)
=
2(a + b + c)
2
a
2
+ b
2
+ c
2
+ ab + bc + ca
By gi ta ch cn chng minh c
2
a
2
+ b
2
+ c
2
+ ab + bc + ca

1
a
2
+ b
2
+ c
2
hay
a
2
+ b
2
+ c
2
ab + bc + ca
y l mt kt qu quen thuc.
Php chng minh hon tt. Du "=" xy ra a = b = c.2
Li gii 2.
p dng bt ng thc Holder, ta c:
_
_
2a
b + c
+
_
2b
c + a
+
_
2c
a + b
_
2 _
a
2
(b + c)
2
+
b
2
(c + a)
2
+
c
2
(a + b)
2
_
(a + b + c)
3
hay
V T
2

2(a + b + c)
3
a
2
(b + c) + b
2
(c + a) + c
2
(a + b)
Ta s chng minh
2(a + b + c)
3
a
2
(b + c) + b
2
(c + a) + c
2
(a + b)

(a + b + c)
4
(a
2
+ b
2
+ c
2
)
2
Bt ng thc ny tng ng vi
2(a
2
+ b
2
+ c
2
)
2
(a + b + c)[a
2
(b + c) + b
2
(c + a) + c
2
(a + b)]
hay
2(a
4
+ b
4
+ c
4
) + 2(a
2
b
2
+ b
2
c
2
+ c
2
a
2
) ab(a
2
+ b
2
) + bc(b
2
+ c
2
) + ca(c
2
+ a
2
) + 2abc(a + b + c)
Ta c hai cch chng minh bt ng thc ny.
Cch 1: S dng bt ng thc Schur, ta c:
93
a
4
+ b
4
+ c
4
+ abc(a + b + c) ab(a
2
+ b
2
) + bc(b
2
+ c
2
) + ca(c
2
+ a
2
)
Vy ta ch cn chng minh
a
4
+ b
4
+ c
4
+ 2(a
2
b
2
+ b
2
c
2
+ c
2
a
2
) 3abc(a + b + c)
hay
(a
2
+ b
2
+ c
2
)
2
3abc(a + b + c)
Ta c nh gi
a
2
+ b
2
+ c
2

(a + b + c)
2
3
nn
(a
2
+ b
2
+ c
2
)
2

(a + b + c)
4
9
Ta cn chng minh
(a + b + c)
4
9
3abc(a + b + c)
hay
(a + b + c)
3
27abc
hay
a + b + c 3
3

abc
iu ny ng theo bt ng thc AM - GM.
Php chng minh hon tt. Du "=" xy ra a = b = c. 2
Cch 2: S dng bt ng thc AM - GM, ta c:
ab(a
2
+ b
2
)
(a
2
+ b
2
)
2
2
=
a
4
+ b
4
2
+ a
2
b
2
Tng t, ta chng minh c
bc(b
2
+ c
2
)
b
4
+ c
4
2
+ b
2
c
2
ca(c
2
+ a
2
)
c
4
+ a
4
2
+ c
2
a
2
Cng theo v ba bt ng thc trn, ta c:
ab(a
2
+ b
2
) + bc(b
2
+ c
2
) + ca(c
2
+ a
2
) a
4
+ b
4
+ c
4
+ a
2
b
2
+ b
2
c
2
+ c
2
a
2
Ta cn chng minh
2abc(a + b + c) a
4
+ b
4
+ c
4
+ a
2
b
2
+ b
2
c
2
+ c
2
a
2
iu ny ng v theo bt ng thc AM - GM:
(a
4
+ b
2
c
2
) + (b
4
+ c
2
a
2
) + (c
4
+ a
2
b
2
) 2a
2
bc + 2b
2
ca + 2c
2
ab = 2abc(a + b + c)
Php chng minh hon tt. Du "=" xy ra a = b = c.2
4.26 Cho a, b, c [0; 1]. Chng minh rng:
2(a
3
+ b
3
+ c
3
) (a
2
b + b
2
c + c
2
a) 3
Li gii.
Gi s c = min{a, b, c} th:
2c
3
b
2
c + c
2
a
hay
2c
3
(b
2
c + c
2
a) 0
By gi ta ch cn chng minh:
2(a
3
+ b
3
) a
2
b 3
94
Nu a b 0, ta c:
2(a
3
+ b
3
) a
2
b = 2a
3
+ b
3
+ b(b
2
a
2
) 2 + 1 + 0 = 3
Nu b a 0, ta c:
2(a
3
+ b
3
) a
2
b = a
3
+ 2b
3
+ a
2
(a b) 2 + 1 + 0 = 3
Php chng minh hon tt. Du "=" xy ra a = b = c = 1 hoc a = b = 1, c = 0 v cc hon
v.2
4.27 Cho
_
_
_
x, y, z > 0
x max{y, z}
. Tm gi tr nh nht ca biu thc:
M =
x
y
+ 2
_
1 +
y
z
+ 3
3
_
1 +
z
x
Li gii.
p dng bt ng thc AM - GM, ta c:
1 +
y
z
2
_
y
z
1 +
z
x
2
_
z
x
T ta suy ra:
M
x
y
+ 2

2.
4
_
y
z
+ 3
3

2.
6
_
z
x
=
1

2
_
x
y
+ 4
4
_
y
z
+ 6
6
_
z
x
_
+
_
1
1

2
_
x
y
+ (3
3

2 3

2)
6
_
z
x
Li p dng bt ng thc AM - GM, ta c:
1

2
_
x
y
+ 4
4
_
y
z
+ 6
6
_
z
x
_

2
.11
11
_
x
y
.
y
z
.
z
x
=
11

2
T gi thit x max{y, z}, ta suy ra:
_

_
x
y
1
z
x
1
Do :
_

_
_
1
1

2
_
x
y
1
1

2
(3
3

2 3

2)
6
_
z
x
3
3

2 3

2
T y ta suy ra:
M
1

2
_
x
y
+ 4
4
_
y
z
+ 6
6
_
z
x
_
+
_
1
1

2
_
x
y
+ (3
3

2 3

2)
6
_
z
x

11

2
+ 1
1

2
+ 3
3

2 3

2 = 1 + 2

2 + 3
3

2.
Vy min M = 1 + 2

2 + 3
3

2 x = y = z.2
4.28 Cho a, b, c > 0. Chng minh rng:
3
_
a.
a + b
2
.
a + b + c
3

a +

ab +
3

abc
3
Li gii.
p dng bt ng thc AM - GM, ta c:
95
3a
3
_
a.
a + b
2
.
a + b + c
3
= 3
3

_
a.a.a
a.
a + b
2
.
a + b + c
3

a
a
+
2a
a + b
+
3a
a + b + c
= 1 +
2a
a + b
+
3a
a + b + c
3

ab
3
_
a.
a+b
2
.
a+b+c
3
= 3
3

_
a.

ab.b
a.
a + b
2
.
a + b + c
3

a
a
+
2

ab
a + b
+
3b
a + b + c
2 +
3b
a + b + c
3
3

abc
3
_
a.
a + b
2
.
a + b + c
3
= 3
3

_
abc
a.
a + b
2
.
a + b + c
3

a
a
+
2b
a + b
+
3c
a + b + c
= 1 +
2b
a + b
+
3c
a + b + c
Cng theo v ba bt ng thc trn, ta c:
3a
3
_
a.
a + b
2
.
a + b + c
3
+
3

ab
3
_
a.
a + b
2
.
a + b + c
3
+
3
3

abc
3
_
a.
a+b
2
.
a+b+c
3
9
hay
a +

ab +
3

abc
3

3
_
a.
a + b
2
.
a + b + c
3
y chnh l iu phi chng minh. Du "=" xy ra a = b = c.2
4.29 Cho a, b, c > 0. Chng minh rng:
_
8 +
2

ab
a + b
_
.
3
_
a.
a + b
2
.
a + b + c
3
3(a +

ab +
3

abc)
Li gii.
p dng bt ng thc Holder, ta c:
3

_
_
a
3
+
a
3
+
a
3
_
_
a
3
+

ab
3
+
b
3
_
_
a
3
+
b
3
+
c
3
_

a +

ab +
3

abc
3
By gi ta ch cn chng minh:
_
8 +
2

ab
a + b
_
.
3
_
a.
a + b
2
.
a + b + c
3
9
3

_
_
a
3
+
a
3
+
a
3
_
_
a
3
+

ab
3
+
b
3
_
_
a
3
+
b
3
+
c
3
_
hay
_
8 +
2

ab
a + b
_
.
3
_
a + b
2
9
3
_
a +

ab + b
3
iu ny ng v theo bt ng thc AM - GM:
_
8 +
2

ab
a + b
_
.
3
_
a + b
2
=
_
3 + 3 +
2(a +

ab + b)
a + b
_
.
3
_
a + b
2
3
3

3
2
.
2(a +

ab + b)
a + b
.
a + b
2
= 9
3
_
a +

ab + b
3
Php chng minh hon tt. Du "=" xy ra a = b = c.2
4.30 Cho a, b, c > 0 tho mn abc = 1. Chng minh rng:
a
3
4 + 2b
2
(a + c) + c
3
+
b
3
4 + 2c
2
(b + a) + a
3
+
c
3
4 + 2a
2
(c + b) + b
3

1
3
Li gii.
96
Ta c mt b c bn:
Vi m, n, p, x, y, z l cc s thc dng bt k, ta lun c bt ng thc:
m
3
x
+
n
3
y
+
p
3
z

(m + n + p)
3
3(x + y + z)
Du "=" xy ra m = n = p, x = y = z.
B ny c th chng minh d dng bng bt ng thc Holder nh sau:
_
m
3
x
+
n
3
y
+
p
3
z
_
(x + y + z)(1 + 1 + 1) (m + n + p)
3
Chia c hai v cho 3(x + y + z), ta c ngay iu phi chng minh.
Tr li vi bi ton. p dng b trn, ta c:
V T
(a + b + c)
3
3[12 + a
3
+ b
3
+ c
3
+ 2b
2
(a + c) + 2c
2
(b + a) + 2a
2
(c + b)]
By gi ta ch cn chng minh:
(a + b + c)
3
3[12 + a
3
+ b
3
+ c
3
+ 2a
2
(c + b) + 2b
2
(a + c) + 2c
2
(b + a)]

1
3
Bt ng thc ny tng ng vi mi bt ng thc trong dy sau:
(a + b + c)
3
12 + a
3
+ b
3
+ c
3
+ 2b
2
(a + c) + 2c
2
(b + a) + 2a
2
(c + b)
a
2
(c + b) + b
2
(a + c) + c
2
(b + a) + 6abc 12
(a
2
b + a
2
c + b
2
a + b
2
c + c
2
a + c
2
b) + 6abc 12
Bt ng thc cui cng ng v theo bt ng thc AM - GM th:
(a
2
b + a
2
c + b
2
a + b
2
c + c
2
a + c
2
b) + 6abc 6
6

a
6
b
6
c
6
+ 6abc = 12abc = 12
Php chng minh hon tt. Du "=" xy ra a = b = c = 1.2
4.31 Cho a, b, c > 0 tho mn a + b + c = 3. Chng minh rng:
8
_
1
a
+
1
b
+
1
c
_
+ 9 10(a
2
+ b
2
+ c
2
)
Li gii.
Gi s a = max{a, b, c}, d thy a [1; 3).
Bt ng thc tng ng vi
8
a
+
8
b
+
8
c
+ 42a + 42b + 42c 117 10a
2
+ 10b
2
+ 10c
2
hay
_
10b
2
+ 42b
69
2
+
8
b
_
+
_
10c
2
+ 42c
69
2
+
8
c
_
10a
2
42a + 48
8
a
hay
(2b 1)
2
(16 5b)
b
+
(2c 1)
2
(16 5c)
c

(a 2)
2
(20a 4)
a
p dng bt ng thc Cauchy - Schwarz, ta c:
V T =
(1 2b)
2
b
165b
+
(1 2c)
2
c
16 5c

[2(1 b c)]
2
b
16 5b
+
c
165c
=
4(a 2)
2
b
16 5b
+
c
16 5c
Ta s chng minh:
(a 2)
2
b
16 5b
+
c
16 5c

(a 2)
2
(5a 1)
a
Ta c:
b
16 5b
+
c
16 5c

b
16 5a
+
c
16 5a
=
3 a
16 5a
97
Do :
(a 2)
2
b
16 5b
+
c
16 5c

(a 2)
2
3 a
16 5a
By gi ta ch cn chng minh:
(a 2)
2
3 a
16 5a

(a 2)
2
(5a 1)
a
iu ny tng ng vi:
(a 2)
2
(16a 5a
2
) (a 2)
2
(5a
2
+ 16a 3)
hay
3(a 2)
2
0 (lun ng)
Php chng minh hon tt. Du "=" xy ra (a, b, c) =
_
2;
1
2
;
1
2
_
v cc hon v.2
4.32 Cho a, b, c > 0. Chng minh rng:

_
a
3
a
3
+
1
4
abc + b
3
+

_
b
3
b
3
+
1
4
abc + c
3
+

_
c
3
c
3
+
1
4
abc + a
3
2
Li gii.
Do bt ng thc l thun nht nn ta chun ho abc = 1. Ta c th vit li bt ng thc cn
chng minh di dng:
_
a
a +
1
4
+ b
+

_
b
b +
1
4
+ c
+

_
c
c +
1
4
+ a
2
hay
_
a
4a + 4b + 1
+
_
b
4b + 4c + 1
+
_
c
4c + 4a + 1
1
p dng bt ng thc Cauchy - Schwarz, ta c:
V T
2
=
_
(4a + 4c + 1)a
(4a + 4b + 1)(4a + 4c + 1)
_
2
[(4a + 4c + 1)]
_

a
(4a + 4b + 1)(4a + 4c + 1)
_
=
(8a + 8b + 8c + 3)(8ab + 8bc + 8ca + a + b + c)
(4a + 4b + 1)(4b + 4c + 1)(4c + 4a + 1)
Ta s chng minh:
(8a + 8b + 8c + 3)(8ab + 8bc + 8ca + a + b + c)
(4a + 4b + 1)(4b + 4c + 1)(4c + 4a + 1)
1
hay
(8a + 8b + 8c + 3)(8ab + 8bc + 8ca + a + b + c) (4a + 4b + 1)(4b + 4c + 1)(4c + 4a + 1)
Ta c nhn xt:
V T = 64(a + b + c)(ab + bc + ca) + 8(a + b + c)
2
+ 24(ab + bc + ca) + 3(a + b + c)
= 64[(a + b)(b + c)(c + a) + abc] + 8(a + b + c)
2
+ 24(ab + bc + ca) + 3(a + b + c)
= 64(a + b)(b + c)(c + a) + 64 + 8(a + b + c)
2
+ 24(ab + bc + ca) + 3(a + b + c) (do abc = 1)
V P = 64(a + b)(b + c)(c + a) + 16(a + b)(b + c) + 16(a + b)(c + a) + 16(b + c)(c + a)
+4(a + b) + 4(b + c) + 4(c + a) + 1
= 64(a + b)(b + c)(c + a) + 16(3ab + 3bc + 3ca + a
2
+ b
2
+ c
2
) + 8(a + b + c) + 1
= 64(a + b)(b + c)(c + a) + 16(a + b + c)
2
+ 16(ab + bc + ca) + 8(a + b + c) + 1
98
nn
V P V T = 8(a + b + c)
2
+ 5(a + b + c) 8(ab + bc + ca) 63
By gi ta ch cn chng minh:
8(a + b + c)
2
+ 5(a + b + c) 8(ab + bc + ca) 63 0
hay
16(a + b + c)
2
+ 15(a + b + c) + 8[(a + b + c)
2
3(ab + bc + ca)] 189
Bt ng thc ny ng t cc bt ng thc sau:
16(a + b + c)
2
16(3
3

abc)
2
= 144 (Theo bt ng thc AM - GM)
15(a + b + c) 45
3

abc = 45 (Theo bt ng thc AM - GM)


(a + b + c)
2
3(ab + bc + ca)
Vy php chng minh hon tt. Du "=" xy ra a = b = c.2
4.33 Cho a, b, c l di ba cnh ca tam gic. Chng minh rng:
3

b + c a +
3

c + a b +
3

a + b c
3

a +
3

b +
3

c
Li gii.
p dng bt ng thc Holder, ta c:
(
3

c + a b +
3

a + b c)
3
(1 + 1)(1 + 1).2a = 8a
Do
3

c + a b +
3

a + b c 2
3

a
Tng t, ta chng minh c
3

a + b c +
3

b + c a 2
3

b
3

b + c a +
3

c + a b 2
3

c
Cng theo v ba bt ng thc trn, ta c:
2(
3

b + c a +
3

c + a b +
3

a + b c) 2(
3

a +
3

b +
3

c)
hay
3

b + c a +
3

c + a b +
3

a + b c
3

a +
3

b +
3

c
y chnh l iu phi chng minh. Du "=" xy ra a = b = c.2
4.34 Cho a, b, c > 0. Chng minh rng:
(b + c a)
2
(b + c)
2
+ a
2
+
(c + a b)
2
(c + a)
2
+ b
2
+
(a + b c)
2
(a + b)
2
+ c
2

3
5
Li gii.
V bt ng thc l thun nht nn ta chun ha a +b +c = 3. Khi y, bt ng thc cn chng
minh tr thnh:
(3 2a)
2
(3 a)
2
+ a
2
+
(3 2b)
2
(3 b)
2
+ b
2
+
(3 2c)
2
(3 c)
2
+ c
2

3
5
Bt ng thc ny tng ng vi:
4a
2
12a + 9
2a
2
6a + 9
+
4b
2
12b + 9
2b
2
6b + 9
+
4c
2
12c + 9
2c
2
6c + 9

3
5
Ta c nhn xt:
4x
2
12x + 9
2x
2
6x + 9
+
18x 23
25
=
36x
3
54x
2
+ 18
25(2x
2
6x + 9)
=
18
25
.
(x 1)
2
(2x + 1)
2x
2
6x + 9
0, x > 0
99
Do :
4x
2
12x + 9
2x
2
6x + 9

23 18x
25
, x > 0
S dng nhn xt trn, ta c ngay:
4a
2
12a + 9
2a
2
6a + 9
+
4b
2
12b + 9
2b
2
6b + 9
+
4c
2
12c + 9
2c
2
6c + 9

69 18(a + b + c)
25
=
3
5
Php chng minh hon tt. Du "=" xy ra a = b = c.2
Li gii 2
Bt ng thc tng ng vi:
1
2a(b + c)
(b + c)
2
+ a
2
+ 1
2b(c + a)
(c + a)
2
+ b
2
+ 1
2c(a + b)
(a + b)
2
+ c
2

3
5
hay
a(b + c)
(b + c)
2
+ a
2
+
b(c + a)
(c + a)
2
+ b
2
+
c(a + b)
(a + b)
2
+ c
2

6
5
p dng bt ng thc AM - GM, ta c:
(b + c)
2
+ a
2
=
3(b + c)
2
4
+
(b + c)
2
4
+ a
2

3(b + c)
2
4
+ a(b + c) = (b + c).
3b + 3c + 4a
4
T suy ra:
a(b + c)
(b + c)
2
+ a
2

a(b + c)
(b + c).
3b + 3c + 4a
4
=
4a
3b + 3c + 4a
p dng bt ng thc Cauchy - Schwarz, ta c:
1
a
+
27
a + b + c
=
1
a
+
9
a + b + c
3

10
2
a +
9(a + b + c)
3
=
100
3b + 3c + 4a
Do :
4a
3b + 3c + 4a

4a
100
.
_
1
a
+
27
a + b + c
_
=
1
25
+
27a
25(a + b + c)
Vy tm li, ta thu c:
a(b + c)
(b + c)
2
+ a
2

1
25
+
27a
25(a + b + c)
Tng t, ta cng chng minh c:
b(c + a)
(c + a)
2
+ b
2

1
25
+
27b
25(a + b + c)
c(a + b)
(a + b)
2
+ c
2

1
25
+
27c
25(a + b + c)
Cng theo v ba bt ng thc trn, ta c:
a(b + c)
(b + c)
2
+ a
2
+
b(c + a)
(c + a)
2
+ b
2
+
c(a + b)
(a + b)
2
+ c
2

27(a + b + c)
25(a + b + c)
+
3
25
=
6
5
Php chng minh hon tt. Du "=" xy ra a = b = c.2
4.35 Cho a, b, c > 0 tho mn
1
bc
+
1
ca
+
1
ab
1. Chng minh rng:
a
bc
+
b
ca
+
c
ab

3 .
Li gii.
T gi thit
1
bc
+
1
ca
+
1
ab
1, ta suy ra:
a + b + c abc
Ta c mt kt qu quen thuc:
100
3(a
2
+ b
2
+ c
2
) (a + b + c)
2
3(ab + bc + ca)
p dng cc bt ng thc trn, ta c:
(a
2
+ b
2
+ c
2
)
2
(ab + bc + ca)
2
3(ab
2
c + bc
2
a + ca
2
b) = 3abc(a + b + c) 3(abc)
2
iu ny tng ng vi:
a
2
+ b
2
+ c
2

3abc
hay
a
bc
+
b
ca
+
c
ab

3
Php chng minh hon tt. Du "=" xy ra a = b = c =

3.2
4.36 Cho x, y, z, t l cc s thc tho mn |x + y + z t| 1 v cc hon v. Chng minh
rng:
x
2
+ y
2
+ z
2
+ t
2
1
Li gii.
T gi thit |x + y + z t| 1, ta suy ra:
x
2
+ y
2
+ z
2
+ t
2
+ 2(xy + xz + yt xt yt zt) 1
Tng t i vi cc hon v, ta cng c:
x
2
+ y
2
+ z
2
+ t
2
+ 2(yz + yt + zt yx zx tx) 1
x
2
+ y
2
+ z
2
+ t
2
+ 2(zt + zx + tx zy ty xy) 1
x
2
+ y
2
+ z
2
+ t
2
+ 2(tx + ty + xy tz xz yz) 1
Cng theo v bn bt ng thc trn, ta c:
4(x
2
+ y
2
+ z
2
+ t
2
) 4
hay
x
2
+ y
2
+ z
2
+ t
2
1
Php chng minh hon tt. Du "=" xy ra x = y = z = t =
1
2
.2
4.37 Cho a, b, c R. Tm gi tr nh nht ca biu thc:
P = (a + b)
4
+ (b + c)
4
+ (c + a)
4

4
7
(a
4
+ b
4
+ c
4
)
Li gii.
t a =
y + z x
2
, b =
z + x y
2
, c =
x + y z
2
th:
P = z
4
+ x
4
+ y
4

1
28
[(y + z x)
4
+ (z + x y)
4
+ (x + y z)
4
]
t Q = (y + z x)
4
+ (z + x y)
4
+ (x + y z)
4
.
Ta c ng thc:
(u + v)
4
+ (u v)
4
= 2(u
4
+ v
4
+ 6u
2
v
2
)
Ta p dng ng thc trn nh sau:
(x + y + z)
4
+ (x + y z)
4
= 2[(x + y)
4
+ z
4
+ 6z
2
(x + y)
2
]
(z + x y)
4
+ (z + y x)
4
= 2[(x y)
4
+ z
4
+ 6z
2
(x y)
2
]
Cng theo v hai ng thc trn, ta c:
Q + (x + y + z)
4
= (x + y + z)
4
+ (x + y z)
4
+ (z + x y)
4
+ (z + y x)
4
= 4[(x
4
+ y
4
+ 6x
2
y
2
) + z
4
+ 3z
2
(2x
2
+ 2y
2
)]
= 4[x
4
+ y
4
+ z
4
+ 6(x
2
y
2
+ y
2
z
2
+ z
2
x
2
)]
101
Ta c mt kt qu quen thuc:
mn + np + pm m
2
+ n
2
+ p
2
Do
Q 28(x
4
+ y
4
+ z
4
) (x + y + z)
4
28(x
4
+ y
4
+ z
4
)
Suy ra
P = z
4
+ x
4
+ y
4

1
28
Q x
4
+ y
4
+ z
4
(x
4
+ y
4
+ z
4
) = 0
Vy min P = 0 x = y = z = 0, hay a = b = c = 0.2
4.38 Cho a, b, c > 0. Chng minh rng:
a
2
b
+
b
2
c
+
c
2
a

4

3.
4
_
a
3
+ b
3
+ c
3
abc
.

a
2
+ b
2
+ c
2
Li gii.
M 4 hai v, ta c:
_
a
2
b
+
b
2
c
+
c
2
a
_
4

3(a
3
+ b
3
+ c
3
)(a
2
+ b
2
+ c
2
)
2
abc
p dng bt ng thc Cauchy - Schwarz, ta c:
a
2
b
+
b
2
c
+
c
2
a
=
a
4
a
2
b
+
b
4
b
2
c
+
c
4
c
2
a

(a
2
+ b
2
+ c
2
)
2
a
2
b + b
2
c + c
2
a
Ta s chng minh
_
a
2
b
+
b
2
c
+
c
2
a
_
3

3(a
3
+ b
3
+ c
3
)(a
2
b + b
2
c + c
2
a)
abc
hay
_
a
2
b
+
b
2
c
+
c
2
a
_
6

9(a
3
+ b
3
+ c
3
)
2
(a
2
b + b
2
c + c
2
a)
2
(abc)
2
Ta c b sau:
Vi x, y, z l cc s thc dng bt k, ta lun c bt ng thc:
(x + y + z)
6
27(x
2
+ y
2
+ z
2
)(xy + yz + zx)
2
Du "=" xy ra x = y = z.
B ny c th chng minh d dng bng AM - GM nh sau:
3
_
27(x
2
+ y
2
+ z
2
)(xy + yz + zx)
2
= 3
3
_
(x
2
+ y
2
+ z
2
)(xy + yz + zx)
2
x
2
+ y
2
+ z
2
+ 2(xy + yz + zx)
= (x + y + z)
2
Lp phng hai v, ta c ngay iu phi chng minh.
Tr li vi bi ton. p dng b trn, ta c:
_
a
2
b
+
b
2
c
+
c
2
a
_
6
27
_
a
4
b
2
+
b
4
c
2
+
c
4
a
2
__
a
2
b
c
+
b
2
c
a
+
c
2
a
b
_
2
Vy ta ch cn chng minh:
27
_
a
4
b
2
+
b
4
c
2
+
c
4
a
2
__
a
2
b
c
+
b
2
c
a
+
c
2
a
b
_
2

9(a
3
+ b
3
+ c
3
)
2
(a
2
b + b
2
c + c
2
a)
2
(abc)
2
hay
3
_
a
4
b
2
+
b
4
c
2
+
c
4
a
2
_
(a
3
b
2
+ b
3
c
2
+ c
3
a
2
)
2
(a
3
+ b
3
+ c
3
)
2
(a
2
b + b
2
c + c
2
a)
2
Bt ng thc ny ng t cc bt ng thc sau:
102
_
a
4
b
2
+
b
4
c
2
+
c
4
a
2
_
(a
3
b
2
+ b
3
c
2
+ c
3
a
2
)(a
2
+ b
2
+ c
2
) (a
3
+ b
3
+ c
3
)
3
(Theo Holder)
(a
3
b
2
+ b
3
c
2
+ c
3
a
2
)(a + b + c) (a
2
b + b
2
c + c
2
a)
2
(Theo Cauchy - Schwarz)
3(a
3
+ b
3
+ c
3
) (a
2
+ b
2
+ c
2
)(a + b + c) (Theo Chebychev)
Vy php chng minh hon tt. Du "=" xy ra a = b = c.2
4.39 Cho a, b, c, d > 0. Chng minh rng:
_
a +
1
a
__
b +
1
b
__
c +
1
c
__
d +
1
d
_

_
a +
1
b
__
b +
1
c
__
c +
1
d
__
d +
1
a
_
Li gii.
Bt ng thc ban u tng ng vi mi bt ng thc trong dy sau:

(a
2
+ 1)

(ab + 1)
[

(a
2
+ 1)]
2
[

(ab + 1)]
2

(a
2
+ 1)(b
2
+ 1)

(ab + 1)
2
Bt ng thc cui cng ng theo bt ng thc Cauchy - Schwarz.
Do php chng minh hon tt. Du "=" xy ra a = b = c = d.2
4.40 Cho a, b, c > 0 tho mn ab + bc + ca = 3. Chng minh rng:
(a + b
2
)(b + c
2
)(c + a
2
) 8
Li gii.
p dng bt ng thc Cauchy - Schwarz, ta c:
(a + b
2
)(a + 1) (a + b)
2
Tng t, ta chng minh c:
(b + c
2
)(b + 1) (b + c)
2
(c + a
2
)(c + 1) (c + a)
2
Nhn theo v ba bt ng thc trn, ta c:
(a + b
2
)(b + c
2
)(c + a
2
)(a + 1)(b + 1)(c + 1) [(a + b)(b + c)(c + a)]
2
Ta ch cn chng minh:
[(a + b)(b + c)(c + a)]
2
(a + 1)(b + 1)(c + 1)
8
Ta c th chng minh bng cch ch ra:
_

_
(a + b)(b + c)(c + a) 8 (1)
(a + b)(b + c)(c + a)
(a + 1)(b + 1)(c + 1)
1 (2)
Hai iu ny c th chng minh n gin nh sau:
T gi thit ab + bc + ca = 3, ta d dng suy ra:
(a + b + c)
2
3(ab + bc + ca) = 9 hay a + b + c 3
3 = ab + bc + ca 3
3

a
2
b
2
c
2
hay abc 1 (Theo bt ng thc AM - GM)
Chng minh (1):
(a + b)(b + c)(c + a) = (a + b + c)(ab + bc + ca) abc = 3(a + b + c) abc 8
Chng minh (2):
103
Ta c:
(a + b)(b + c)(c + a) (a + 1)(b + 1)(c + 1)
= 3(a + b + c) abc (a + b + c + ab + bc + ca + abc + 1)
= 2(a + b + c) 2abc 4 0
Do
(a + b)(b + c)(c + a) (a + 1)(b + 1)(c + 1)
hay
(a + b)(b + c)(c + a)
(a + 1)(b + 1)(c + 1)
1
Vy php chng minh hon tt. Du "=" xy ra a = b = c = 1.2
3.5 Bi 5.1 n bi 5.40
5.1 Cho a, b, c 0 tho mn khng c hai s no ng thi bng 0. Chng minh rng:
a(bc + ca 2ab)
(2a + b)
2
+
b(ca + ab 2bc)
(2b + c)
2
+
c(ab + bc 2ca)
(2c + a)
2
0
Li gii.
Ta thy:
a(bc + ca 2ab)
(2a + b)
2
=
a[(2b
2
+ bc) + (2ac + bc) 2(2ab + b
2
)]
(2a + b)
2
=
ab(2b + c)
(2a + b)
2
+
ca
2a + b

2ab
2a + b
Do :

a(bc + ca 2ab)
(2a + b)
2
=
ab(2b + c)
(2a + b)
2
+
ca
2a + b

2ab
2a + b
=
ab(2b + c)
(2a + b)
2
+
ab
2b + c

2ab
2a + b
= ab(2b + c)
_
1
(2a + b)
2

2
(2a + b)(2b + c)
+
1
(2b + c)
2
_
= ab(2b + c)
_
1
2a + b

1
2b + c
_
2
0
Php chng minh hon tt. Du "=" xy ra a = b = c hoc (a, b, c) = (0; 1; 2) v cc hon v.2
5.2 Cho a, b, c 0 tho mn khng c hai s no ng thi bng 0. Chng minh rng:
a
2
+ b
2
+ c
2
ab + bc + ca
+
8abc
(a + b)(b + c)(c + a)
2
Li gii.
Li gii 1.
Khng mt tnh tng qut, gi s a b c 0.
Ta c mt b quen thuc:
Vi x y > 0 v z 0 th ta lun c bt ng thc:
x
y

x + z
y + z
Du "=" xy ra x = y hoc z = 0.
104
B ny c th chng minh d dng bng cch xt hiu nh sau:
x
y

x + z
y + z
=
z(x y)
y(y + z)
0 (do x y > 0 v z 0)
p dng b trn, ta c:
a
2
+ b
2
+ c
2
ab + bc + ca

a
2
+ b
2
+ 2c
2
c
2
+ ab + bc + ca
=
a
2
+ b
2
+ 2c
2
(a + c)(b + c)
By gi ta ch cn chng minh:
a
2
+ b
2
+ 2c
2
(a + c)(b + c)
+
8abc
(a + b)(b + c)(c + a)
2
Bt ng thc ny tng ng vi mi bt ng thc trong dy sau:
(a
2
+ b
2
+ 2c
2
)(a + b) + 8abc 2(a + b)(b + c)(c + a)
a
3
+ a
2
b + ab
2
+ b
3
+ 2ac
2
+ 2bc
2
+ 8abc 2a
2
b + 2b
2
c + 2c
2
a + 2ab
2
+ 2bc
2
+ 2ca
2
+ 4abc
a
3
a
2
b ab
2
+ b
3
2ac
2
+ 4abc 2bc
2
0
(a + b)(a b)
2
2c(a b)
2
0
(a + b 2c)(a b)
2
0
Bt ng thc cui cng ng do gi s a b c.
Php chng minh hon tt. Du "=" xy ra a = b = c hoc a = b, c = 0 v cc hon v.2
Li gii 2.
Khng mt tnh tng qut, gi s a b c 0.
Ta c hai ng thc sau:
a
2
+ b
2
+ c
2
ab bc ca = (a b)(a c)
(a + b)(b + c)(c + a) 8abc = (b + c)(a b)(a c)
Do bt ng thc cn chng minh c th vit li nh sau:
ab + bc + ca + (a b)(a c)
ab + bc + ca
+
(a + b)(b + c)(c + a) (b + c)(a b)(a c)
(a + b)(b + c)(c + a)
2
Bt ng thc ny tng ng vi:
(a b)(a c)
ab + bc + ca

(b + c)(a b)(a c)
(a + b)(b + c)(c + a)
0
hay

_
1
ab + bc + ca

1
(a + b)(a + c)
_
(a b)(a c) 0
Ta d thy ngay rng bt ng thc s ng theo bt ng thc Vornicu Schur nu ta ch ra c:
1
ab + bc + ca

1
(a + b)(a + c)

1
ab + bc + ca

1
(b + c)(b + a)
iu ny tng ng vi:
1
(a + b)(a + c)

1
(b + c)(b + a)
hay
(b + c)(b + a) (a + b)(a + c)
hay
b a (ng theo gi s)
Vy php chng minh hon tt. Du "=" xy ra a = b = c hoc a = b, c = 0 v cc hon v.2
5.3 Cho x, y l cc s thc. Tm gi tr ln nht ca biu thc:
P =
4(2x + y) 13
(x
2
+ 5)(y
2
+ 5)
Li gii.
105
Ta thy rng:
max P =
4
21
x = 4; y =
1
2
nn ta s chng minh P
4
21
vi du "=" xy ra x = 4; y =
1
2
. 2
Tht vy:
4(2x + y) 13
(x
2
+ 5)(y
2
+ 5)

4
21
Bt ng thc ny tng ng vi:
168x + 84y 273 4x
2
y
2
+ 20x
2
+ 20y
2
+ 100
hay
4(x
2
y
2
4xy + 4) + 16x
2
+ 4y
2
+ 289 + 16xy 136x 68y + 4(x
2
8x + 16) + 4(4y
2
4y + 1) 0
hay
4(xy 2)
2
+ (4x + 2y 17)
2
+ 4(x 4)
2
+ 4(2y 1)
2
0 (lun ng)
Vy max P =
4
21
x = 4; y =
1
2
.2
5.4 Cho a, b, c > 0 tho mn 12a
2
+ 3b
2
+ 2c
2
= 20. Tm gi tr ln nht ca biu thc:
M = (a + b)(1 + c)
Li gii.
t m =
1

5
, n =
4

5
, p = 2, k =

5
3
.
p dng bt ng thc Cauchy - Schwarz, ta c:
M
2
(m + n)
_
a
2
m
+
b
2
n
_
(1 + p)
_
1 +
c
2
p
_
= N
p dng bt ng thc AM - GM, ta c:
N =
(m + n)(1 + p)
k
.
_
a
2
m
+
b
2
n
__
k +
kc
2
p
_

(m + n)(1 + p)
4k
.
_
a
2
m
+
b
2
n
+ k +
kc
2
p
_
2
Theo cch t m, n, p, k nh trn, ta s c:
a
2
m
+
b
2
n
+ k +
kc
2
p
=
npa
2
+ pmb
2
+ kmnc
2
mnp
+ k =

5
12
.(12a
2
+ 3b
2
+ 2c
2
) +

5
3
= 2

5
Cui cng, ta thu c:
M
2
N
(m + n)(1 + p)
4k
.(2

5)
2
= 45
T suy ra:
M 3

5
Vy max M = 3

5 a =
1

5
, b =
4

5
, c = 2.2
5.5 Cho cc s thc dng a, b, c. Chng minh rng:
a
2
3a
2
+ (b + c)
2
+
b
2
3b
2
+ (c + a)
2
+
c
2
3c
2
+ (a + b)
2

1
2
Li gii.
106
p dng bt ng thc cauchySchawrz, ta c:
a
2
3a
2
+ (b + c)
2
=
a
2
a
2
+ b
2
+ c
2
+ 2a
2
+ 2bc

a
2
4a
2
+ 4b
2
+ 4c
2
+
a
2
8a
2
+ 8bc
=
1
4
+
a
2
8a
2
+ 8bc
Ta cn chng minh:
a
2
a
2
+ bc
2
bc
a
2
+ bc
1
Ta c :
bc
a
2
+ bc

(ab + bc + ca)
2
a
2
b
2
+ b
2
c
2
+ c
2
a
2
+ abc(a + b + c)
1
tng ng
abc(a + b + c) 0 ng
Php chng minh hon tt.2
5.6 Cho a, b, c l cc s thc dng tho mn abc = 1. Chng minh rng:
5
_
1
a
4
+
1
b
4
+
1
c
4
_
+
a
c
2
+
b
a
2
+
c
b
2
2
_
a
b
2
+
b
c
2
+
c
a
2
_
+ a
2
+ b
2
+ c
2
+ (a + b + c)
2
Li gii.
Bt ng thc tng ng vi:
5
_
1
a
4
+
1
b
4
+
1
c
4
_
+
a
c
2
+
b
a
2
+
c
b
2
2
_
a
b
2
+
b
c
2
+
c
a
2
_
+ 2(a
2
+ b
2
+ c
2
) + 2(ab + bc + ca)
S dng bt ng thc AM - GM cng vi gi thit abc = 1, ta c:
1
a
4
+
1
b
4
= b
4
c
4
+
1
b
4
2c
2
Tng t, ta chng minh c:
1
b
4
+
1
c
4
2a
2
1
c
4
+
1
a
4
2b
2
Cng theo v ba bt ng thc trn, ta c:
2
_
1
a
4
+
1
b
4
+
1
c
4
_
2(a
2
+ b
2
+ c
2
)
hay
4
_
1
a
4
+
1
b
4
+
1
c
4
_
4(a
2
+ b
2
+ c
2
)
Tip tc p dng bt ng thc AM - GM, ta c:
_
1
a
4
+ c
2
_
+
_
1
b
4
+ a
2
_
+
_
1
c
4
+ b
2
_

2c
a
2
+
2b
b
2
+
2b
c
2
Cng theo v hai bt ng thc trn, ta c:
5
_
1
a
4
+
1
b
4
+
1
c
4
_
3(a
2
+ b
2
+ c
2
) + 2
_
a
b
2
+
b
c
2
+
c
a
2
_
By gi ta ch cn chng minh:
a
2
+ b
2
+ c
2
+
a
c
2
+
b
a
2
+
c
b
2
2(ab + bc + ca)
Li p dng AM - GM, ta c:
a
2
+
c
b
2
+ bc = a
2
+
c
b
2
+ ab
2
c
2
3ca
Xy dng cc bt ng thc tng t, ta c:
b
2
+
a
c
2
+ ca 3ab
107
c
2
+
b
a
2
+ ab 3bc
Cng theo v ba bt ng thc ny, ta c:
a
2
+
c
b
2
+ bc + b
2
+
a
c
2
+ ca + c
2
+
b
a
2
+ ab 3(ab + bc + ca)
hay
a
2
+
c
b
2
+ b
2
+
a
c
2
+ c
2
+
b
a
2
2(ab + bc + ca)
y chnh l iu phi chng minh.
Php chng minh hon tt. Du "=" xy ra a = b = c = 1.2
5.7 Cho a, b, c > 0 tho mn abc = 1. Chng minh rng:
a
2
b + b
2
c + c
2
a +
1
6

a
3
+ b
3
+ c
3
3 +
1
6

3
Li gii.
Ta c mt b quen thuc:
Vi x, y, z l cc s thc dng bt k, ta lun c bt ng thc:
(x + y + z)
3

27
4
(xy
2
+ yz
2
+ zx
2
+ xyz)
Du "=" xy ra x = y = z.
p dng b trn v kt hp vi gi thit abc = 1, ta c:
(a
2
b + b
2
c + c
2
a)
3

27
4
(a
2
b
5
c
2
+ b
2
c
5
a
2
+ c
2
a
5
b
2
+ a
3
b
3
c
3
) =
27
4
(a
3
+ b
3
+ c
3
+ 1)
T y suy ra:
a
2
b + b
2
c + c
2
a 3
3
_
a
3
+ b
3
+ c
3
+ 1
4
Ta s chng minh:
3
3
_
a
3
+ b
3
+ c
3
+ 1
4
+
1
6

a
3
+ b
3
+ c
3
3 +
1
6

3
p dng bt ng thc AM - GM, ta c:
a
3
+ b
3
+ c
3
+ 1 = 3.
a
3
+ b
3
+ c
3
3
+ 1 4
4

_
a
3
+ b
3
+ c
3
3
_
3
Do :
3
3
_
a
3
+ b
3
+ c
3
+ 1
4
3
4
_
a
3
+ b
3
+ c
3
3
By gi ta ch cn chng minh:
3
4
_
a
3
+ b
3
+ c
3
3
+
1
6

a
3
+ b
3
+ c
3
3 +
1
6

3
t t =
12
_
a
3
+ b
3
+ c
3
3

12

abc = 1, bt ng thc tr thnh:


3t
3
+
1
6

3t
2
3 +
1
6

3
iu ny tng ng vi:
3
6

3t
5
3
6

3t
2
t
2
+ 1 0
hay
(t 1)(3
6

3t
4
+ 3
6

3t
3
+ 3
6

3t
2
t 1) 0 (lun ng v t 1).
Php chng minh hon tt. Du "=" xy ra a = b = c = 1.2
108
5.8 Cho x, y, z l ba s thc dng tha mn x
5
+ y
5
+ z
5
= 3. Chng minh rng:
x
4
y
3
+
y
4
z
3
+
z
4
x
3
3
Li gii.
Theo bt ng thc Cauchy Schwarz ta c:
x
4
y
3
+
y
4
z
3
+
z
4
x
3

(x
5
+ y
5
+ z
5
)
2
x
6
y
4
+ y
6
z
4
+ z
6
x
4

9
x
6
y
4
+ y
6
z
4
+ z
6
x
4
Vy nn ta cn chng minh:
x
6
y
4
+ y
6
z
4
+ z
6
x
4
3
T bt ng thc quen thuc:
(a + b + c)
2
3
ab + bc + ca ta c:
x
5
y
5
+ y
5
z
5
+ z
5
x
5
3
Suy ra:
15

(x
5
+ 3x
5
y
5
+ x
10
)
S dng bt ng thc AM GM:
x
5
+ 3x
5
y
5
+ x
10
5x
6
y
3
Lm tng t vi cc biu thc cn li ta c:
15 5(x
6
y
4
+ y
6
z
4
+ z
6
x
4
)
hay
x
6
y
4
+ y
6
z
4
+ z
6
x
4
3
Bt ng thc c chng minh.
Du ng thc xy ra khi v ch khi x = y = z = 1.2
5.9 Cho ba s thc dng x, y, z tha mn xy + yz + zx 3xyz. Chng minh rng:
3

x
2
+
3
_
y
2
+
3

z
2

x +

y +

z
Li gii.
109
T gi thit xy + yz + zx 3xyz ta c
1
x
+
1
y
+
1
z
3
p dng bt ng thc AM GM:
3
1
x
+
1
y
+
1
z

3
3

xyz
xyz 1
t a =
6

x, b =
6

y, a =
6

z (x, y, z > 0)
Bt ng thc cn chng minh tr thnh:
a
4
+ b
4
+ c
4
a
3
+ b
3
+ c
3
vi a, b, c l cc s dng sao cho abc 1
Theo bt ng thc Chebyshev ta c:
3(a
4
+ b
4
+ c
4
) (a
3
+ b
3
+ c
3
)(a + b + c)
Li c theo AM GM:
a + b + c 3
3

abc 3
Kt hp hai iu trn cho ta iu phi chng minh.
Du ng thc xy ra khi v ch khi x = y = z = 1..2
5.10 Cho a, b, c l cc s thc i mt khc nhau. Chng minh rng:
a
2
b
2
+ 1
(a b)
2
+
b
2
c
2
+ 1
(b c)
2
+
c
2
a
2
+ 1
(c a)
2

3
2
Li gii.
Ta ch ti bin i sau:

a
2
b
2
+ 1
(a b)
2
=
1
2
.

(1 ab)
2
+ (1 + ab)
2
(a b)
2
=
1
2
.(

(1 ab)
2
(a b)
2
+

(1 + ab)
2
(a b)
2
)
n y s dng 2 bt ng thc quen thuc:
x
2
+ y
2
+ z
2
2(xy + yz + zx); x
2
+ y
2
+ z
2
xy + yz + zx
ta c:
1
2
.(

(1 ab)
2
(a b)
2
+

(1 + ab)
2
(a b)
2
)
1
2
(2.

(1 ab)(1 bc)
(a b)(b c)
+

(1 + ab)(1 + bc)
(a b)(b c)
)
110
Bng php quy ng v khai trin trc tip ta c 2 ng thc sau:
(1 ab)(1 bc)
(a b)(b c)
+
(1 bc)(1 ca)
(b c)(c a)
+
(1 ca)(1 ab)
(c a)(a b)
= 1
(1 + ab)(1 + bc)
(a b)(b c)
+
(1 + bc)(1 + ca)
(b c)(c a)
+
(1 + ca)(1 + ab)
(c a)(a b)
= 1
Suy ra:
1
2
(2.

(1 ab)(1 bc)
(a b)(b c)
+

(1 + ab)(1 + bc)
(a b)(b c)
) =
3
2
Php chng minh hon tt.2
5.11 Cho a, b l cc s thc dng. Chng minh rng:
a + b + a
2
b
2
ab
+
54ab
(a + b)ab + 6ab + 1
9
Li gii 1.
S dng phng php i bin p, q, r.
t x =
1
a
, y =
1
b
, z = ab; x + y + z = p, xy + yz + zx = q, xyz = r = 1
Bt ng thc cn chng minh tng ng vi:
x + y + z +
54
xy + yz + xz + 6
9
pq + 6p 9q
Ta c bt ng thc quen thuc sau:
p
2
q + 3pr 4q
2
p
2
q + 3p 4q
2
p
3 +
_
9 + 16q
3
2q
Mt khc vi q 3 d thy:
3 +
_
9 + 16q
3
2q

9q
q + 6
Nh vy bt ng thc c chng minh.
Du ng thc xy ra khi v ch khi a = b = c = 1.2
5.12 Chng minh vi mi s thc a, b, c khng m ta c bt ng thc sau:
(a
2
+ b
2
)(b
2
+ c
2
)(c
2
+ a
2
)(a + b + c)
2
8(a
2
b
2
+ b
2
c
2
+ c
2
a
2
)
2
111
Li gii.
t a
2
= x, b
2
= y, c
2
= z(x, y, z 0).
Bt ng thc cn chng minh tr thnh:
(x + y)(y + z)(z + x)(

x +

y +

z)
2
8(xy + yz + zx)
2
Theo bt ng thc AM GM:

xy

4xy
x + y
Suy ra:
(

x +

y +

z)
2
=

x +

xy

x +

4xy
x + y
Mt khc ta li c:
(x + y)(y + z)(z + x)(

x +

4xy
x + y
) 8(xy + yz + zx)
2
=

xy(x y)
2
0
Vy bt ng thc chng minh xong.
Du ng thc xy ra khi v ch khi a = b = c.2
5.13 Cho cc s thc dng x, y, z tha mn x + y + z = xyz. Chng minh rng:
y
x
_
y
2
+ 1
+
z
y

z
2
+ 1
+
x
z

x
2
+ 1

3
2
Li gii 1.
T gi thit x + y + z = xyz suy ra
1
xy
+
1
yz
+
1
zx
= 1.
t
1
x
= a;
1
y
= b;
1
z
= c khi ab + bc + ca = 1.
Bt ng thc cn chng minh tr thnh :
a

1 + b
2
+
b

1 + c
2
+
c

1 + a
2

3
2
Theo bt ng thc AM GM v Cauchy Schwarz ta c:

1 + b
2
=

a
_
(a + b)(b + c)

2a
2b + a + c

2(a + b + c)
2
a
2
+ b
2
+ c
2
+ 3(ab + bc + ca)
= 2.
(a + b + c)
2
(a + b + c)
2
+ (ab + bc + ca)
2.
(a + b + c)
2
(a + b + c)
2
+
(a + b + c)
2
3
=
3
2
112
Bt ng thc chng minh xong.
Du ng thc xy ra khi v ch khi x = y = z =

3. 2
Li gii 2.
V x, y, z l cc s dng tha mn x +y +z = xyz nn ta t x = tanA; y = tanB; z = tanC vi
A, B, C l 3 gc ca mt tam gic nhn v A + B + C = .
Bt ng thc cn chng minh tr thnh:
tanB
tanA

tan
2
B + 1
+
tanC
tanB

tan
2
C + 1
+
tanA
tanC

tan
2
A + 1

3
2

cosAsinB
sinA
+
cosBsinC
sinB
+
cosCsinA
sinC

3
2
S dng nh l hm s Sine v Cosine a v 3 cnh tam gic ta c:

b
2
+ c
2
a
2
2ac

3
2
a
3
+ b
3
+ c
3
+ ab
2
+ bc
2
+ ca
2
3abc + a
2
b + b
2
c + c
2
a
Nhng bt ng thc trn lun ng do ab
2
+ bc
2
+ ca
2
3abc ( theo AM GM )
v a
3
+ b
3
+ c
3
a
2
b + b
2
c + c
2
a ( theo bt ng thc hon v ).
Bt ng thc chng minh xong.
Du ng thc xy ra khi v ch khi x = y = z =

3.2
5.14 Cho a, b, c l cc s thc tha mn a, b, c 1, a
2
+ b
2
+ c
2
= 4.Chng minh:
1
a
+
1
b
+
1
c

9
2(

a
2
1 +

b
2
1 +

c
2
1)
Li gii.
t

a
2
1 = x,

b
2
1 = y,

c
2
1 = z ta vit li gi thit thnh: x
2
+y
2
+z
2
= 1. Bt ng
thc cn chng minh tng ng vi:
(x + y + z)(
1

x
2
+ 1
+
1
_
y
2
+ 1
+
1

z
2
+ 1
)
9
2
Theo bt ng thc Cauchy Schwarz ta c:

x
2
+ 1

3x
2
2x
2
+ y
2
+ z
2

3
4

(
x
2
x
2
+ y
2
+
x
2
x
2
+ z
2
) =
3
2

y + z

x
2
+ 1

3(y + z)
2
2x
2
+ y
2
+ z
2

(
y
2
x
2
+ y
2
+
z
2
x
2
+ z
2
) = 3
113
Bt ng thc c chng minh.
ng thc xy ra khi v ch khi a = b = c =
2

3
.2
5.15 Cho x, y, z l cc s thc dng. Chng ming rng:
1
x
2
+ xy + y
2
+
1
y
2
+ yz + z
2
+
1
z
2
+ zx + x
2

9
(x + y + z)
2
Li gii.
Nhn c 2 v vi x
2
+ y
2
+ z
2
+ xy + yz + xz ta c:

x
2
+ y
2
+ z
2
+ xy + yz + xz
x
2
+ xy + y
2

9(x
2
+ y
2
+ z
2
+ xy + yz + xz)
(x + y + z)
2
rng:

x
2
+ y
2
+ z
2
+ xy + yz + xz
x
2
+ xy + y
2
= 3 +

z(x + y + z)
x
2
+ xy + y
2
Nn bt ng thc cn chng minh tr thnh:
3 + (x + y + z)(

z
x
2
+ xy + y
2
)
9(x
2
+ xy + y
2
+ yz + xz + z
2
)
(x + y + z)
2
Theo bt ng thc Cauchy Schwarz ta c:
z
x
2
+ xy + y
2
+
x
z
2
+ zy + y
2
+
y
x
2
+ xz + z
2

(x + y + z)
2
zx
2
+ zy
2
+ xz
2
+ xy
2
+ yx
2
+ yz
2
+ 3xyz
=
(x + y + z)
2
(x + y + z)(xy + yz + xz)
=
x + y + z
xy + yz + xz
Vy ta cn chng minh:
3 +
(x + y + z)
2
xy + yz + xz

9(x
2
+ y
2
+ z
2
+ xy + yz + xz)
(x + y + z)
2
3 +
(x + y + z)
2
xy + yz + xz

9((x + y + z)
2
(xy + yz + xz))
(x + y + z)
2

(x + y + z)
2
xy + yz + xz

3(2(a
2
+ b
2
+ c
2
) + ab + bc + ca)
(x + y + z)
2
)
114
(a + b + c)
4
3(ab + bc + ca)(2(a
2
+ b
2
+ c
2
) + ab + bc + ca)
D thy bt ng thc trn ng theo bt ng thc AM GM cho 2 s.
Php chng minh hon tt.
ng thc xy ra khi v ch khi x = y = z. 2
Bt ng thc trn cng l mt h qu trc tip suy ra t bt ng thc Iran 1996. Tht vy, s
dng bt ng thc AM GM ta c:
V T =

1
x
2
+ xy + y
2

4(xy + yz + zx)
(y
2
+ yz + z
2
+ xy + yz + zx)
2
=

4(xy + yz + zx)
(y + z)
2
(x + y + z)
2
Nh vy ta cn chng minh:
1
(x + y)
2
+
1
(y + x)
2
+
1
(z + x)
2

9
4(xy + yz + zx)
Nhng y l bt ng thc Iran 1996 quen thuc.
5.16 Chng minh vi mi s thc a, b, c ta c bt ng thc sau:
2(1 + abc) +
_
2(1 + a
2
)(1 + b
2
)(1 + c
2
) (1 + a)(1 + b)(1 + c)
Li gii. Bt ng thc cho tng ng vi:
_
2(1 + a
2
)(1 + b
2
)(1 + c
2
) (b + c)(1 + a) + (1 bc)(a 1)

_
((1 + a)
2
+ (a 1)
2
)((b + c)
2
+ (1 bc)
2
) (1 + a)(b + c) + (a 1)(bc 1)
Kt qu ny ng theo Cauchy Schwarz.
ng thc xy ra khi a = b = c = 1. 2
Cng c th p dng Cauchy Schwarz nh sau:
2(1 + abc) +
_
2(a
2
+ 1)(b
2
+ 1)(c
2
+ 1) = 2(1 + abc) +
_
2[(ab + bc + ca 1)
2
+ (a + b + c abc)
2
]
(ab + bc + ca 1) + (a + b + c abc) + 2(1 +abc)
= (1 + a)(1 + b)(1 + c)
Ta c iu phi chng minh.2
115
5.17 Cho a, b, c l cc s thc dng tha mn a + b + c = 3.Chng minh:
12(
1
a
+
1
b
+
1
c
) 4(a
3
+ b
3
+ c
3
) + 21
Li gii. Gi s c l s ln nht trong 3 s a, b, c. t t =
a + b
2
vi t (0, 1]
Xt f(a, b, c) = V T V P. D thy f(a, b, c) f(t, t, c) v (a + b)
2
ab
(a + b)
4
4
4.
Li c:
f(t, t, c) = 3(2t 1)
2
(
1
3 2t
+
8
t
+ 2t 30)
chng minh f(t, t, c) 0 ta s i chng minh: g(t) = 4t
3
26t
2
+ 45t 24 < 0t (0, 1]
Ta thy g(t) ng bin nn g(t) < g(1) = 1 < 0.
T y ta c iu phi chng minh.
Du ng thc xy ra khi v ch khi a = b = 0.5, c = 2 v cc hon v.2
5.18 Cho ba s thc dng a, b, c. Chng minh rng:
a(b + c)
b
2
+ c
2
+
b(c + a)
c
2
+ a
2
+
c(a + b)
a
2
+ b
2
+ 3 4
_
ab
ab + c
2
+
bc
bc + a
2
+
ca
ca + b
2
_
Gii
p dng bt ng thc CauchySchwarz , ta c:
4bc
bc + a
2
=
4bc(b + c)
(b + c)(bc + a
2
)
= bc(b + c)
(1 + 1)
2
b(c
2
+ a
2
) + c(a
2
+ b
2
)
bc(b + c)
_
1
b(c
2
+ a
2
)
+
1
c(a
2
+ b
2
)
_
=
c(b + c)
c
2
+ a
2
+
b(b + c)
a
2
+ b
2
Do :

cyc
4bc
bc + a
2

cyc
c(b + c)
c
2
+ a
2
+

cyc
b(b + c)
a
2
+ b
2
=

cyc
b(a + b)
b
2
+ c
2
+

cyc
c(c + a)
b
2
+ c
2
=

cyc
_
b(a + b)
b
2
+ c
2
+
c(c + a)
b
2
+ c
2
_
=

cyc
_
b(a + b) + c(c + a)
b
2
+ c
2
_
=

cyc
_
a(b + c) + b
2
+ c
2
b
2
+ c
2
_
=

cyc
_
a(b + c)
b
2
+ c
2
+ 1
_
=

cyc
a(b + c)
b
2
+ c
2
+ 3
ng thc xy ra a = b = c.2
5.19 Cho cc s dng a, b, c, x, y, z tho mn:
(a + b + c)(x + y + z) = (a
2
+ b
2
+ c
2
)(x
2
+ y
2
+ z
2
) = 4
Chng minh rng:
abcxyz
1
36
116
Li gii.
S dng bt ng thc AM GM ta c:
4(ab + bc + ca)(xy + yz + xz) = [(a + b + c)
2
(a
2
+ b
2
+ c
2
)][(x + y + z)
2
(x
2
+ y
2
+ z
2
)]
= 20 (a + b + c)
2
.(x
2
+ y
2
+ z
2
) (a
2
+ b
2
+ c
2
)(x + y + z)
2
20 2
_
(a + b + c)
2
.(x
2
+ y
2
+ z
2
).(a
2
+ b
2
+ c
2
)(x + y + z)
2
= 4
(ab + bc + ca)(xy + yz + xz) 1
Mt khc ta li c:
(ab + bc + ca)
2
3abc(a + b + c); (xy + yz + xz)
2
3xyz(x + y + z)
(ab + bc + ca)
2
.(xy + yz + xz)
2
9abcxyz(a + b + c)(x + y + z)
abcxyz
1
36
Php chng minh hon tt.
ng thc khng xy ra.2
5.20 Cho a, b, c l cc s thc dng tha mn abc = 1. Chng minh:
a + b + c
3

5
_
a
2
+ b
2
+ c
2
3
Li gii.
Ta vit li bt ng thc di dng thun nht:
(a + b + c)
5
81abc(a
2
+ b
2
+ c
2
)()
R rng ch cn chng minh bt ng thc (*) vi mi a,b,c dng.
Khng mt tnh tng qut, gi s c = min(a, b, c).
Ta c cc nh gi sau:
8abc(a
2
+ b
2
) c(a + b)
4
8abc
3
2c
3
(a + b)
2
Do vy, ta ch cn chng minh:
8(a + b + c)
5
81c(a + b)
2
(2c
2
+ (a + b)
2
)(1)
t a + b = 2t v chun ho cho a + b + c = 3. Thay c = 3 2t vo (1), ta c:
3 t
2
(3 2t)((3 2t)
2
+ 2t
2
)
4t
5
14t
4
+ 18t
3
9t
2
+ 1 0
(t 1)
2
(4t
3
6t
2
+ 2t + 1) 0
Bt ng thc ny hin nhin ng v t 1 ( Do c = min(a, b, c) ).
Php chng minh hon tt.
117
ng thc xy ra khi v ch khi a = b = c = 1.2
5.21 Cho a, b, c l cc s thc dng tha mn ab + bc + ca = 3. Chng minh:
a + b + c abc + 2
_
a
2
b
2
+ b
2
c
2
+ c
2
a
2
3
Li gii 1.
t x + y + z = p, xy + yz + zx = q, xyz = r.
Bt ng thc cn chng minh tng ng vi:
3(p r)
2
4(9 2rp)
Theo bt ng thc Schur bc 4 ta c:
r
(4q p
2
)(p
2
q)
6p
Mt khc hm s f(p, r) = 3(p r)
2
4(9 2rp) ng bin theo r nn ta c:
3(p r)
2
3
_
p
(12 p
2
)(p
2
3)
6p
_
2
4
_
9 2p
(12 p
2
)(p
2
3)
6p
_
=
(p 3)(p 1)(p + 1)(p + 3)(p
2
12)
2
12p
2
0
Bt ng thc ny ng v p 3.
Php chng minh hon tt.
ng thc xy ra khi v ch khi a = b = c = 1.2
Li gii 2.
Gi s b l s nm gia a v c. Khi , s dng bt ng thc n gin 4xy (x +y)
2
, ta s quy
bi ton v chng minh bt ng thc mnh hn l:
ab(a + b) + bc(b + c) + ca(c + a)
a
2
b
2
+ b
2
c
2
+ c
2
a
2
b
+ b(ab + bc + ca)
Tng ng vi:
ca(a b)(b c)
b
0
Nhng nh gi trn ng theo iu gi s.
Ta c iu phi chng minh.2
5.22 Cho 3 s thc dng thay i a, b, c sao cho:a + b + c = 3. Tm gi tr nh nht ca
biu thc:
A = (a + b + c)
2
+ (ab + bc + ca)
_
1 + a
2
b + b
2
c + c
2
a
a
2
b + b
2
c + c
2
a
_
+
81
(a + b) (b + c) (c + a) + abc
118
Li gii.
S dng bt ng thc AM GM ta c cc nh gi sau:
(1)
3(a
2
+ b
2
+ c
2
) = (a + b + c)(a
2
+ b
2
+ c
2
)
=

(a
3
+

ab
2
+

a
2
b)
3(a
2
b + b
2
c + c
2
a)
a
2
+ b
2
+ c
2
a
2
b + b
2
c + c
2
a
(2)
abc
(a + b + c)
3
27
= 1
(3)
(a + b)(b + c)(c + a)
8(a + b + c)
3
27
= 8
t a
2
+ b
2
+ c
2
= t ab + bc + ca =
9 t
2
.
Xt f(t) = t +
9 t
2
.(3 +
1
t
) vi t 3 Ta c:
f

(t) =
2(t
3
2t
2
9)
4t
2
Gii phng trnh f

(t) = 0 ta c t = 3. f(t) f(3) = 13(4).


T (2), (3) v (4), suy ra MinA = 22
ng thc xy ra khi v ch khi a = b = c = 1.2
5.23 Cho a, b, c l cc s thc khng m sao cho khng tn ti 2 s ng thi bng 0 v tha
mn a
2
+ b
2
+ c
2
= 2. Chng minh:
1 a
2
b
2
+ bc + c
2
+
1 b
2
c
2
+ ca + a
2
+
1 c
2
a
2
+ ab + b
2

1
2
Li gii.
T gi thit a
2
+ b
2
+ c
2
= 2 ta vit li bt ng thc cn chng minh thnh:
a
2
+ bc
b
2
+ bc + c
2
+
b
2
+ ca
c
2
+ ca + a
2
+
c
2
+ ab
a
2
+ ab + b
2
2
y l mt kt qu quen thuc, chng minh n nh sau:
Xt hiu:
a
2
+ bc
b
2
+ bc + c
2
2 =

(a
2
+ bc)(c
2
+ ca + a
2
)(c
2
+ ca + a
2
)

(a
2
+ ab + b
2
)
=

a
6
+

a
5
(b + c)

a
4
(b
2
+ c
2
)

a
3
b
3

(a
2
+ ab + b
2
)
n y s dng bt ng thc AM GM ta c:
a
5
b + 3b
5
a 4a
2
b
4
119
Hon ton tng t vi s hng cn li ri cng theo v ta c:
a
5
(b + c) + b
5
(c + a) + c
5
(a + b) a
4
(b
2
+ c
2
) + b
4
(a
2
+ c
2
) + c
4
(a
2
+ b
2
)(1)
Mt khc d thy rng:
a
6
+ b
6
+ c
6
a
3
b
3
+ b
3
c
3
+ c
3
a
3
(2)
T 2 bt ng thc (1) v (2) suy ra:

a
6
+

a
5
(b + c)

a
4
(b
2
+ c
2
)

a
3
b
3

(a
2
+ ab + b
2
)
0
Php chng minh hon tt.
ng thc xy ra khi v ch khi a = b = 1, c = 0 v cc hon v.2
5.24 Cho a, b, c l cc s thc khng m sao cho ab + bc + ca > 0. Chng minh rng:
a
3
+ abc
b
2
+ c
2
+
b
3
+ abc
c
2
+ a
2
+
c
3
+ abc
a
2
+ b
2
a + b + c
Li gii.
rng:
a
3
+ abc
b
2
+ c
2
a =
a
3
+ abc ab
2
ac
2
b
2
+ c
2
=
a(a b)(a c)
b
2
+ c
2
Vy nn bt ng thc cn chng minh tng ng:
a(a b)(a c)
b
2
+ c
2
+
b(b c)(b a)
c
2
+ a
2
+
c(c a)(c b)
a
2
+ b
2
0
Khng mt tnh tng qut, gi s a b c khi d thy rng:
a
b
2
+ c
2

b
c
2
+ a
2

c
a
2
+ b
2
Nn theo bt ng thc V ornicu Schur ta c iu phi chng minh.
ng thc xy ra khi v ch khi a = b = c.2
5.25 Cho a, b, c l cc s thc khng m sao cho ab + bc + ca > 0. Chng minh rng:
a
2
+ b
2
+ c
2

a(b
2
+ c
2
)
b + c
+
b(c
2
+ a
2
)
c + a
+
c(a
2
+ b
2
)
a + b
Li gii.
Bt ng thc cn chng minh tng ng vi:
2abc(
1
b + c
+
1
c + a
+
1
a + b
) 2(ab + bc + ca) (a
2
+ b
2
+ c
2
)
Theo bt ng thc Cauchy Schwarz ta c:
2abc(
1
b + c
+
1
c + a
+
1
a + b
)
9abc
a + b + c
120
Vy ta cn chng minh:
a
2
+ b
2
+ c
2
+
9abc
a + b + c
2(ab + bc + ca)
Nhng y li l bt ng thc Schur dng phn thc.
Chng minh hon tt.
ng thc xy ra khi v ch khi a = b = c hoc a = b, c = 0 v cc hon v.2
5.26 Cho a, b, c, d l cc s dng tha mn a
2
+ b
2
+ c
2
+ d
2
= 1. Chng minh:
(a 1)(b 1)(c 1)(d 1) abcd
Li gii.
Khng mt tnh tng qut, gi s d a c b.
S dng bt ng thc Cauchy Schwarz d thy:
2(a + b) a + b + c + d
_
4(a
2
+ b
2
+ c
2
+ d
2
) = 2
a + b 1
Xt 2 s a v b ta c nh gi sau:
1 a
a
.
1 b
b

_

2

a
2
+ b
2
1
_
2
(a b)
2
(1 a b) + 2ab(
_
2(a
2
+ b
2
) a b) 0
Nh vy ch cn chng minh bt ng thc khi a = b = c d tc l chng minh:
(1 a)(1 b) cd
2 2a 2b + 2ab 2cd
(a + b 1)
2
+ (c d)
2
0
Php chng minh hon tt.
ng thc xy ra khi v ch khi a = b = c = 1.2
5.27 Chng minh vi cc s thc dng x, y, z ta c bt ng thc sau:
xyz + x
2
+ y
2
+ z
2
+ 5 3(x + y + z)
Li gii 1.
S dng bt ng thc AM GM ta c:
xyz + xyz + 1 3
3
_
x
2
y
2
z
2

9xyz
x + y + z
Mt khc theo Schur bc 2 dng phn thc:
9xyz
x + y + z
2(xy + yz + xz) x
2
y
2
z
2
121
Nh vy ta cn phi chng minh:
2(x
2
+ y
2
+ z
2
) + 2(xy + yz + xz) x
2
y
2
z
2
+ 9 6(x + y + z)
(x + y + z)
2
+ 9 6(x + y + z)
Bt ng thc trn ng theo AM GM.2
Li gii 2.
t x = a + 1, y = b + 1, z = c + 1(a, b, c > 1).
Bt ng thc cn chng minh tr thnh:
abc + ab + bc + ca + a
2
+ b
2
+ c
2
0
Do (abc)
2
0, nn gi s ab 0. Kt hp vi c > 1 ta c:
ab(c + 1) 0(1)
Li c:
a
2
+ b
2
+ c
2
+ bc + ca = (a +
c
2
)
2
+ (b +
c
2
)
2
+
c
2
2
(2)
T (1) v (2) ta c iu phi chng minh.2
Li gii 3.
Ta s dng phng php dn bin.
t f(x, y, z) = abc + a
2
+ b
2
+ c
2
+ 5 3(a + b + c)
Khng mt tnh tng qut, gi s c = min{a, b, c}.
Ta c:
f(a, b, c) f
_

ab,

ab, c
_
=
_

b
_
2
_
a + b + 2

ab 3
_
f(a, b, c) f
_
a + b
2
,
a + b
2
, c
_
=
(a b)
2
4
(2 c)
T y ta thy rng:
- Nu c 1 th f(a, b, c) f
_

ab,

ab, c
_
- Nu c 1 th f(a, b, c) f
_
a + b
2
,
a + b
2
, c
_
tc l ta ch cn chng minh bi ton trong trng hp c hai bin bng nhau.
Xt f(x, x, c) = x
2
(c + 2) 6x + c
2
3c + 5
Xem f(x, x, c) 0 l mt bt phng trnh bc hai n x, bt phng trnh ny c:

= (c 1)
2
(c + 1) 0
Nn suy ra f(x, x, c) 0 x, c > 0 Php chng minh hon tt.
ng thc xy ra khi v ch khi a = b = c = 1.2
5.28 Cho a, b, c l cc s thc dng tho mn a + b + c = 3. Chng minh:
a
b
+
b
c
+
c
a

_
a
2
+ b
2
+ c
2
3
+ 2
122
Li gii 1.
t x + y + z = p, xy + yz + zx = q, xyz = r, d thy q 3
Theo bt ng thc Cauchy Schwarz ta c:
a
b
+
b
c
+
c
a

(a + b + c)
2
ab + bc + ca
=
9
q
Li c a
2
+ b
2
+ c
2
= p
2
2q. Vy ta cn chng minh:
9
q

_
p
2
2q
3
+ 2
(
9
q
2)
2

9 2q
3

(3 q)(9 2q)(q + 9)
3q
2
0
Bt ng thc trn ng do q 3.
Li gii 2.
Nhn hai v ca bt ng thc vi a + b + c ta c:
a
2
b
+
a
2
b
+
a
2
b
+
ab
c
+
bc
a
+
ca
b
+ a + b + c
_
3(a
2
+ b
2
+ c
2
) + 6
D dng chng minh c theo AM GM:
ab
c
+
bc
a
+
ca
b
3
Vy chng minh bt ng thc ban u ta cn chng minh:
a
2
b
+
b
2
c
+
c
2
a

_
3(a
2
+ b
2
+ c
2
)

(
a
2
b
2a + b)
_
3(a
2
+ b
2
+ c
2
) (a + b + c)

(a b)
2
b

(a b)
2
_
3(a
2
+ b
2
+ c
2
) + (a + b + c)

(a b)
2
(
1
b

1
_
3(a
2
+ b
2
+ c
2
) + (a + b + c)
) 0
Bt ng thc trn hin nhin ng vi a, b, c > 0.2
Li gii 3.
Trc ht ta c mt b quen thuc:
x
y
+
y
z
+
z
x

9(x
2
+ y
2
+ z
2
)
(x + y + z)
2
p dung b trn ta ch cn chng minh c:
9(a
2
+ b
2
+ c
2
)
(a + b + c)
2

_
a
2
+ b
2
+ c
2
3
+ 2
123
t x =
_
a
2
+b
2
+c
2
3
1 Bt ng thc tr thnh:
3x
2
x + 2
(x 1)(3x + 2) 0
Chng minh hon tt.
ng thc xy ra khi v ch khi a = b = c = 1.2
5.29 Chng minh vi a, b, c l cc s thc dng ta c bt ng thc sau:
a
2
(b + c)
2
+
b
2
(c + a)
2
+
c
2
(a + b)
2
+
10abc
(a + b)(b + c)(c + a)
2
Li gii.
t
a
b + c
= x,
b
c + a
= y,
c
a + b
= z. Khi ta c cc ng thc v bt ng thc quen thuc
sau:
_
_
_
xy + yz + zx + 2xyz = 1
x + y + z
3
2
Bt ng thc cn chng minh tr thnh:
x
2
+ y
2
+ z
2
+ 10xyz 2
T iu kin x + y + z
3
2
ta c:
x
2
+ y
2
+ z
2
+ 6xyz + 4xyz x
2
+ y
2
+ z
2
+
9xyz
x + y + z
+ 4xyz
Mt khc theo Schur ta li c:
x
2
+ y
2
+ z
2
+
9xyz
x + y + z
2(xy + yz + zx)
x
2
+ y
2
+ z
2
+ 10xyz 2(xy + yz + zx) + 4xyz = 1
Php chng minh hon tt.
ng thc xy ra khi v ch khi a = b = c.2
5.30 Cho cc s thc khng m a, b, c tha mn a + b + c = 1. Chng minh:

3
8
(a b)(b c)(c a)

3
8
Li gii.
R rng bi ton s c chng minh nu ta chng minh c:
|(a b)(b c)(c a)|

3
18
Ta c bc d on nh sau:
Bt ng thc l i xng nn ta d on ng thc xy ra khi c mt s bng 0, v y khng
124
th xy ra trng hp c hai s bng nhau c v khi (a b)(b c)(c a) = 0 v ta gi s
c = 0 thu c |ab(a b)|

3
18
, a+b = 1. Gi h ny ta tm c a =
3+

3
6
, b =
3

3
6
, c = 0.
V ta c li gii bng AM GM nh sau:
Gi s a b c. Khi s dng bt ng thc AM GM ta c:
|(a b)(b c)(c a)| = (a c)(b c)(a b)
(a + c).b.(a + c b)
=
1
2
.
_

3 + 1
_
(a + c).b(

3 1)(a + c b)

1
2
.
_
_
3 + 1
_
(a + c) + b(

3 1) + (a + c b)
3
_
3
=
1
2
.
_

3(a + b + c)
3
_
3
=

3
18
Php chng minh hon tt.
ng thc xy ra khi v ch khi a =
3+

3
6
, b =
3

3
6
, c = 0 v cc hon v.2
5.31 Cho a, b, c l cc s dng tha mn abc = 1. Chng minh:
a
3
4 + 2b
2
(a + c) + c
3
+
b
3
4 + 2c
2
(a + b) + a
3
+
c
3
4 + 2a
2
(b + c) + b
3

1
3
Li gii.
p dng bt ng thc Holder ta c:

a
3
4 + 2b
2
(a + c) + c
3

(a + b + c)
3
3(12 + 2a
2
(b + c) + 2b
2
(a + c) + 2c
2
(a + b) + a
3
+ b
3
+ c
3
)
Nh vy ta ch cn chng minh bt ng thc sau:
(a + b + c)
3
12 + 2a
2
(b + c) + 2b
2
(a + c) + 2c
2
(a + b) + a
3
+ b
3
+ c
3
iu ny tng ng vi :
a
2
(b + c) + b
2
(a + c) + c
2
(a + b) 6
Nhng nh gi trn hin nhin ng theo AM GM
Chng minh hon tt.
ng thc xy ra khi v ch khi a = b = c = 1.2
5.32 Cho a l s thc tha mn a
5
a
3
+ a 2 = 0. Chng minh:
S =
a
16
+ a
12
+ 7a
8
+ 12a
4
+ 12
a
12
+ 7a
8
+ 7a
4
+ 12
<
3

4
Li gii.
125
T gi thit ta c:
2
a
= a
4
a
2
+ 1 > 0 a > 0.
S dng bt ng thc AM GM:
a
3
+ 2 = a
5
+ a 2a
3
Do du ng thc xy ra khng tha mn phng trnh trn nn suy ra a <
3

2(1)
Bt ng thc cn chng minh tng ng :
a
16
+ a
12
+ 7a
8
+ 12a
4
+ 12 a
14
+ 7a
10
+ 7a
6
+ 12a
2
(a
4
a
2
+ 1)(a
12
7a
6
+ 12) < 0
a
12
7a
6
+ 12 < 0
Mt khc do a = 1 ta li c nh gi sau:
(a 1)
2
> 0
2(a
2
+ 1)
a
> 4 (a
2
+ 1)(a
4
a
2
+ 1) > 4 a
6
+ 1 > 4
Suy ra
a >
6

3(2)
T (1) v (2) ta c iu phi chng minh.2
5.33 Cho a, b, c l cc s thc dng tho mn a +b +c ab +ac +bc. Chng minh rng"
a
b
+
b
c
+
c
a
a + b + c
Li gii.
p dng trc tip bt ng thc Cauchy Schwarz ta c
a
b
+
b
c
+
c
a
=
a
2
ab
+
b
2
bc
+
c
2
ca

(a + b + c)
2
ab + bc + ca

(a + b + c)
2
a + b + c
= a + b + c
ng thc xy ra khi v ch khi a = b = c = 1.2
5.34 Cho cc s thc khng m a, b, c tha mn ab + bc + ac = 1. Chng minh rng :
a + b + c +
ab
b + c
+
bc
c + a
+
ca
a + b

3

3
2
Li gii.
Ta vit li bt ng thc thnh:
(a + b + c)(
b
b + c
+
c
c + a
+
a
a + b
)
3

3
2
S dng bt ng thc Cauchy Schwarz ta c:
(a + b + c)(
b
b + c
+
c
c + a
+
a
a + b
)
(a + b + c)
3
a
2
+ b
2
+ c
2
+ ab + bc + ca
=

(a
2
+ b
2
+ c
2
+ 2)
3
(a
2
+ b
2
+ c
2
+ 1)
2
126
t a
2
+ b
2
+ c
2
= t.
Theo bt ng thc AM GM ta c:

(t + 2)
3
(t + 1)
2
=

_
(
t + 1
2
+
t + 1
2
+ 1)
3
(t + 1)
2

_
27(
t + 1
2
)(
t + 1
2
)
(t + 1)
2
=
3

3
2
Php chng minh hon tt.
ng thc xy ra khi v ch khi a = b = c = 1.2
5.35 Cho cc s a, b, c dng tho mn abc = 1. Chng minh rng:
a
3
(1 + b)(1 + c)
+
b
3
(1 + a)(1 + c)
+
c
3
(1 + a)(1 + b)

3
4
Li gii.
p dng bt ng thc AM GM ta c:
a
3
(1 + b)(1 + c)
+
1 + b
8
+
1 + c
8

3a
4
Hon ton tng t ta cng c:
b
3
(1 + c)(1 + a)
+
1 + c
8
+
1 + a
8

3b
4
v
c
3
(1 + a)(1 + b)
+
1 + a
8
+
1 + b
8

3a
4
Cng theo v cc bt ng thc trn ta c:
a
3
(1 + b)(1 + c)
+
b
3
(1 + a)(1 + c)
+
c
3
(1 + a)(1 + b)
+
3 + a + b + c
4

3(a + b + c)
4

a
3
(1 + b)(1 + c)
+
b
3
(1 + a)(1 + c)
+
c
3
(1 + a)(1 + b)

2(a + b + c) 3
4

2 3
3

abc 3
4
=
3
4
Chng minh hon tt.
ng thc xy ra khi v ch khi a = b = c = 1.2
5.36 Cho a, b, c l cc s dng v
1
a
+
1
c
=
2
b
. Chng minh rng:
a + b
2a b
+
b + c
2c b
4
Li gii.
Do a, b, c > 0, ta vit li gi thit thnh:
b
a
+
b
c
= 2. Ta c :
a + b
2a b
+
b + c
2c b
=
1 +
b
a
2
b
a
+
1 +
b
c
2
b
c
=
1 +
b
a
b
c
+
1 +
b
c
b
a
=
1
b
c
+
1
b
a
+
b
a
b
c
+
b
c
b
a
127
S dng 2 bt ng thc quen thuc:
1
x
+
1
y

4
x + y
v
x
y
+
y
x
2 suy ra:
1
b
c
+
1
b
a
+
b
a
b
c
+
b
c
b
a
2 + 2 = 4
Chng minh hon tt.
ng thc xy ra khi v ch khi a = b = c = 1.2
5.37 Cho x, y, z l cc s thc thuc on [0; 1] tha mn xyz = (1 x)(1 y)(1 z). Tm
gi tr nh nht ca:
F = x
2
+ y
2
+ z
2
Li gii 1.
T gi thit suy ra:
xy + yz + zx = 2xyz + (x + y + z) 1
Theo bt ng thc AM GM ta c:
x
2
+ y
2
+ z
2
= (x + y + z)
2
2(xy + yz + zx)
= (x + y + z)
2
4xyz 2(x + y + z) + 2

4
27
.(x + y + z)
3
+ (x + y + z)
2
2(x + y + z) + 2
t t = x + y + z; t [0; 3] ta c:
F = x
2
+ y
2
+ z
2
=
4
27
.t
3
+ t
2
2t + 2
=
1
27
.(2a 3)
2
.(
15
4
a) +
3
4

3
4
Vy MinP =
3
4
.
ng thc xy ra khi v ch khi x = y = z =
1
2
. 2
Li gii 2.
t a = sin
2
, b = sin
2
, c = sin
2
.
T gi thit xyz = (1 x)(1 y)(1 z) suy ra cot
2
.cot
2
.cot
2
= 1
Ta s chng minh:
x
2
+ y
2
+ z
2

3
4

1
(1 + cot
2
)
2
+
1
(1 + cot
2
)
2
+
1
(1 + cot
2
)
2

3
4
t cot
2
= a, cot
2
= b, cot
2
= c th xyz = 1.
S dng bt ng thc quen thuc:
1
(1 + x)
2
+
1
(1 + x)
2

1
1 + xy
ta c:
1
(1 + x)
2
+
1
(1 + x)
2
+
1
(1 + z)
2

1
1 + xy
+
1
(1 + z)
2
=
z
z + 1
+
1
(1 + z)
2
128
Ta cn chng minh:
z
z + 1
+
1
(1 + z)
2

3
4
(z 1)
2
0
Chng minh hon tt.2
5.38 Cho x, y, z l cc s thc dng. Chng minh rng:

x + y + z.(

x
y + z
+

y
x + z
+

z
y + x
)
3

3
2
Li gii.
V bt ng thc dng thun nht nn ta chun ha x + y + z = 3.
Bt ng thc cn chng minh tr thnh:

x
3 x
+

y
3 y
+

z
3 z

3
2
Theo AM GM ta c:
x
2
+

x +

x 3
2

x x(3 x)

x
3 x

x
2
Hon ton tng t vi cc biu thc cn li ta c:

x
3 x
+

y
3 y
+

z
3 z

x + y + z
2
=
3
2
Chng minh hon tt.
ng thc xy ra khi v ch khi tam gic x = y = z.2
5.39 Cho a, b, c 1 tha mn a + b + c + 2 = abc. Chng minh rng:
bc

a
2
1 + ca

b
2
1 + ab

c
2
1
3

3
2
abc
Li gii 1.
T gi thit, chia c 2 v cho abc = 1 ta c
1
ab
+
1
bc
+
1
ca
+
2
abc
= 1.
Ta c nhn xt sau:
Nu c 3 s x, y, z > 0 tha mn x
2
+ y
2
+ z
2
+ 2xyz = 1 th khi tn ti tam gic ABC nhn
sao cho x = cosA, y = cosB, z = cosC.
Theo nhn xt trn, p dng vo bi ton ta thy tn ti tam gic ABC nhn sao cho:
1
bc
=
cos
2
A;
1
ca
= cos
2
B;
1
ab
= cos
2
C.
Bt ng thc ban u li tr thnh:
_
1
cos
2
Bcos
2
C
cos
2
A
+
_
1
cos
2
Ccos
2
A
cos
2
B
+
_
1
cos
2
Acos
2
B
cos
2
C

3

3
2
129
S dng bt ng thc Cauchy Schwarz ta c:

_
1
cos
2
Bcos
2
C
cos
2
A

_
3(3

cos
2
Bcos
2
C
cos
2
A
Mt khc, theo AM GM d thy:
cos
2
Bcos
2
C
cos
2
A
+
cos
2
Ccos
2
A
cos
2
B
+
cos
2
Acos
2
B
cos
2
C
cos
2
A + cos
2
B + cos
2
C
Nh vy, cui cng ta ch cn chng minh:
sin
2
A + sin
2
B + sin
2
C
9
4
Nhng bt ng thc trn hin nhin ng.
Php chng minh hon tt.
ng thc xy ra khi v ch khi a = b = c = 2.2
Li gii 2.
t t =
1
a
+
1
b
+
1
c
T ga thit p dng AM GM ta c:
1 =
2
abc
+
1
ab
+
1
bc
+
1
ca

2t
3
27
+
t
2
3
t
3
2
()
Bt ng thc cn chng minh tng ng:
ab
_
(3c 3)(c + 1) + bc
_
(3a 3)(a + 1) + ca
_
(3b 3)(b + 1)
9
2
abc.
Cng theo AM GM ta li c:

ab
_
(3c 3)(c + 1)

ab(2c 1) = 6abc

ab
9
2
abc
2

ab 3abc
t
3
2
(ng theo iu kin ())
Chng minh hon tt.2
5.40 Cho cc s thc dng x, y, z tha mn x + y + z = 1.Chng minh rng:
1
1 xy
+
1
1 yz
+
1
1 zx

27
8
Li gii 1.
Bng php quy ng v khai trin trc tip, bt ng thc cn chng minh tng ng:
19.xyz 11(xy + yz + zx) 27x
2
y
2
z
2
+ 3 0
Theo bt ng thc AM GM ta c:
xyz = xyz.(x + y + z)
3
27.x
2
y
2
z
2
130
19.xyz 11(xy + yz + zx) 27x
2
y
2
z
2
+ 3 19xyz 11(xy + yz + zx) xyz + 3 0(1)
Khng mt tnh tng qut, ta gi s rng z = min(x; y; z) z
x + y + z
3
=
1
3
. Suy ra:
(1) xy.(18z 11) 11z.(x + y) + 3 (
x + y
2
)
2
(18z 11) 11z.(1 z) + 3
Nh vy ta ch cn chng minh:
(
1 z
2
)
2
(18z 11) 11z.(1 z) + 3 0
(2z + 1).(3z 1)
2
0
Bt ng thc trn hin nhin ng do z > 0.
Php chng minh hon tt.
Du ng thc xy ra khi v ch khi x = y = z =
1
3
.2
Li gii 2.
Bi ton ny c th s dng phng php tip tuyn.
t x =
a
3
, y =
b
3
, z =
c
3
th bi ton c vit li thnh:
Cho cc s thc dng a, b, c tha mn a + b + c = 3.Chng minh rng:
1
9 ab
+
1
9 bc
+
1
9 ca

3
8
Theo bt ng thc AM GM ta c:
ab
(a + b)
2
4
=
(3 c)
2
4
T suy ra:
1
9 ab

4
c
2
+ 6c + 27
Mt khc t gi thit d thy a, b, c (0; 3] nn ta c nh gi sau:
4
c
2
+ 6c + 27

9 c
64
=
(c 1)
2
(c 3)
64(c
2
+ 6c + 27)
0
Suy ra:
1
9 ab

4
c
2
+ 6c + 27

9 c
64
Xy dng cc bt ng thc tng t ta cng c:
1
9 bc

9 a
64
;
1
9 ca

9 b
64
Cng theo v cc bt ng thc trn ta c:
1
9 ab
+
1
9 bc
+
1
9 ca

27 (a + b + c)
64
=
3
8
131
Php chng minh hon tt.
Ngoi ra bi ton ny cn c kh nhiu cch gii, v d nh s dng hm bc nht hoc s dng
nh gi sau:
1
1 xy
+
1
1 yz
+
1
1 zx

3
1
xy + yz + zx
3
3.6 Bi 6.1 n bi 6.40
6.1 Cho cc s a, b, c khng m tha mn: a
2
+b
2
+c
2
= 2(ab +bc +ca). Chng minh rng:
_
1 +
a
b + c
+
_
1 +
b
c + a
+
_
1 +
c
a + b
1 + 2

2
Li gii.
u tin ta s chng minh b sau:
Nu xy khng m th

1 + x +

1 + y 1 +

1 + x + y
Chng minh bng cch bnh phng 2 v, cui cng ta c: xy 0 (ng)
Khng mt tnh tng qut, gi s a b c.Ta c:
a
2
+ b
2
c(a + b) c
2
c(a + b) c a + b
t x =
a
b + c
; y =
b
c + a
; z =
c
a + b
th xy + yz + zx + 2xyz = 1(1).
Ngoi ra, t h thc u bi ta cng c c x + y + z = 2 + 3xyz(2)
T (1) v (2) suy ra x + y =
2z
2
+ 6z + 2
3z
2
+ 2z + 1
p dng b trn, ta c:

1 + x +
_
1 + y 1 +
_
1 + x + y
Nh vy, ta ch cn chng minh:
_
1 + x + y +

1 + z 2

2
Hay:
_
z
2
+ 8z + 3
3z
2
+ 2z + 1
+

1 + z 2

2(3)
Ta lun c (z 1)
3
0 nn theo AM GM
_
z
2
+ 8z + 3
3z
2
+ 2z + 1
+

1 + z
2

1 + z
+

1 + z 2

2
Php chng minh hon tt.
Du ng thc xy ra khi v ch khi x = 0; y = z = 1 hay a = 0; b = c v cc hon v.2
6.2 Cho cc s a, b, c dng tha mn: a + b + c = 3. Chng minh rng:
a

7b
2
+ b + c
+
b

7c
2
+ c + a
+
c

7a
2
+ a + b
1
132
6.3 Cho x
1
x
2
.... x
n
0 tha mn

x
i
400 v

x
2
i
10
4
. Chng minh rng:

x
1
+

x
2
10
Li gii.
t x
i
= 25y
i
vi mi i = 1, 2, . . . , n. Khi bi ton c chuyn v chng minh:

y
1
+

y
2
2
vi y
1
y
2
y
n
0 tha mn y
1
+ y
2
+ + y
n
16 v y
2
1
+ y
2
2
+ + y
2
n
16.
Nu y
1
4 th bt ng thc hin nhin ng.
Xt trng hp y
1
4: Do y
i
y
2
vi mi 2 i n nn
16 y
2
1
+y
2
2
+ +y
2
n
y
2
1
+y
2
2
+y
2
y
3
+ +y
2
y
n
= y
2
1
+y
2
(y
2
+y
3
+ +y
n
) y
2
1
+y
2
(16 y
1
)
T y ta c:
y
2

16 y
2
1
16 y
1
()
Hn na, t nh gi trn ta cng suy ra c y
1
1. Tht vy, do y
2
y
1
4 nn
16 y
2
1
+ y
2
(16 y
1
) y
2
1
+ y
1
(16 y
1
) = 16y
1
v ta d dng thu c y
1
1.
By gi, s dng bt ng thc (*) thu c trn, ta c th a bi ton v chng minh:

y
1
+

16 y
2
1
16 y
1
2
vi 1 y
1
4. Tht vy, ta t

y
1
= t, 1 t 2. Bt ng thc tr thnh:
_
16 t
4
16 t
2
2 t
16 t
4
(2 t)
2
(16 t
2
)
(2 + t)(4 + t
2
) (2 t)(4 t)(4 + t)
Bt ng thc trn ng do 4 + t
2
4 + t v 2 + t 3 3(2 t) (4 t)(2 t).
Php chng minh hon tt.2
6.4 Cho tam gic ABC. Chng minh rng:
cos A + cos B + cos C +
1
sinA
+
1
sinB
+
1
sinC
2

3 +
3
2
Li gii.
t f(A, B, C) = cosA + cosB + cosC +
1
sinA
+
1
sinB
+
1
sinC
(2

3 +
3
2
) trong A, B, C l
133
ln 3 gc ca tam gic ABC v A + B + C = .
Xt hiu :
f(A, B, C) f(A,
B + C
2
,
B + C
2
) = (cosB + cosC 2cos
B + C
2
) + (
1
sinB
+
1
sinC

2
sin
B+C
2
)
= 2cos
B + C
2
(cos
B C
2
1) + (
1
sinB
+
1
sinC

2
sin
B+C
2
)
Mt khc ch rng sinB, sinC l cc s dng cho nn theo bt ng thc AM GM :
1
sinB
+
1
sinC

2
sin
B+C
2

4
sinB + sinC

2
sin
B+C
2
=
4(1 cos
BC
2
)
sinB + sinC
Do ta c :
f(A, B, C) f(A,
B + C
2
,
B + C
2
) 2(1 cos
B C
2
)(
2
sinB + sinC
cos
B + C
2
)
= 2(1 cos
B C
2
).
1 sin
B+C
2
.cos
B+C
2
.cos
BC
2
sinB + sinC
0
Vy nn f(A, B, C) f(A,
B+C
2
,
B+C
2
)
Tc l ta ch cn chng minh bt ng thc trong trng hp tam gic ABC cn ti A, khi
B =

2

A
2
cosB = cosC = sin
A
2
, sinB = sinC = cos
A
2
.
Ta c :
f(A,
B + C
2
,
B + C
2
) = (cosA + 2sin
A
2

3
2
) + (
1
sinA
+
2
cos
A
2
2

3)
=
(2sin
A
2
1)
2
2
+
1 + 4sin
A
2
2

3sinA
sinA
D thy rng:
1 sin(
A
2
+

3
) 8sin
A
2
2

3sinA + 4sin
2
A
2
1 + 4sin
A
2
2

3sinA 4sin
2
A
2
4sin
A
2
+ 1 = (2sin
A
2
1)
2
Vy ta c :
f(A,
B + C
2
,
B + C
2
) (2sin
A
2
1)
2
(
1
sinA

1
2
) 0
Chng minh hon tt.
ng thc xy ra khi v ch khi A = B = C =

3
.2
6.5 Cho tam gic ABC khng vung. Chng minh rng:
3tan
2
Atan
2
Btan
2
B5(tan
2
A+tan
2
B+tan
2
C) 9+tan
2
Atan
2
B+tan
2
Btan
2
C+tan
2
Ctan
2
A
Li gii.
Bt ng thc cn chng minh tng ng vi:
4tan
2
Atan
2
Btan
2
B 4(tan
2
A + tan
2
B + tan
2
C) 8 (1 + tan
2
A)(1 + tan
2
B)(1 + tan
2
C)
4

(
1
cos
2
A
1) 4(

1
cos
2
A
3) 8
1

cos
2
A
134

cos
2
A
(

1
cos
2
Acos
2
B
)
1

cos
2
A
cos
2
A + cos
2
B + cos
2
C
3
4

1 + cos2A
2
+
1 + cos2B
2
+ cos
2
C
3
4
2(cos2A + cos2B) + 4cos
2
C + 1 0
2cos(A B)cos(A + B) + 4cos
2
C + 1 0
4cos
2
C 4cos(A B)cosC + 1 0
(2cosC cos(A B))
2
+ sin
2
(A B) 0
Bt ng thc trn hin nhin ng.
Chng minh hon tt.2
6.6 Cho x (0;

2
). Chng minh:
(
sinx
x
)
3
> cosx
Li gii.
Bt ng thc cn chng minh tng ng:
sin x
3

cos x
x > 0
t f(x) =
sin x
3

cos x
x vi x (0,

2
)
Ta c:
f

(x) =
cos x.
3

cos x + sin
2
x.
1
3

cos
2
x
3

cos
2
x
1
f

(x) =
3 cos
2
x + sin
2
x 3
3

cos
4
x
3
3

cos
4
x
f

(x) =
2 cos
2
x 3
3

cos
4
x + 1
3
3

cos
4
x
f

(x) =
(
3

cos
2
x 1)
2
(2
3

cos
2
x + 1)
3
3

cos
4
x
> 0
Khi f(x) ng bin trong x (0,

2
) f(x) > f(0) = 0
Php chng minh hon tt. 2
6.7 Cho x, y, z l cc s thc dng, a, b, c l 3 cnh v S l din tch ca mt tam gic.
Chng minh rng:
xa
2
+ yb
2
+ zc
2
4

xy + yz + zxS
Li gii.
Gi s BC l cnh ln nht trong tam gic ABC. Gi H l chn ng cao h t A xung BC.
Ta c:
xa
2
+ yb
2
+ zc
2
= xBC
2
+ y(HC
2
+ HA
2
) + z(HB
2
+ HA
2
)
= xBC
2
+ (y + z)AH
2
+ yHC
2
+ zHB
2
(1)
135
p dng bt ng thc CauchySchwarz ta c:
_
yHC
2
+ zHB
2
_
_
1
y
+
1
z
_
(HB + HC)
2
= a
2
Suy ra:
yHC
2
+ zHB
2

yza
2
y + z
(2)
T (1) v (2) suy ra:
xa
2
+ yb
2
+ zc
2

(xy + yz + zx) a
2
y + z
+ (y + z)AH
2
2

xy + yz + zx.AH.a
= 4

xy + yz + zxS
Chng minh hon tt.
ng thc xy ra khi v ch khi tam gic ABC u.2
y l mt h qu ca bt ng thc Finsler Hadwinger.
6.8 Chng minh rng trong tam gic ABC ta lun c:
1/a(b
2
+ c
2
a
2
) + b(c
2
+ a
2
b
2
) + c(a
2
+ b
2
c
2
) > 2abc
2/

p <

p a +

p b +

p c <

3p vi p l na chu vi.
3/0, 4 <
r
h
a
0, 5 vi a
2
+ b
2
c
2
4/a
4
+ b
4
+ c
4
16 bit S
ABC
= 1
5/
ab
l
c
+
bc
l
a
+
ac
l
b
6R vi l
a
, l
b
, l
c
l cc ng phn gic tng ng mi gc.
6/sinA.sinB + sinB.sinC + sinC.sinA 9.(
r
R
)
2
Li gii.
1/ Chia c 2 v cho 2abc v p dng nh l Cosine, ta c bt ng thc tng ng:
cos A + cos B + cos C > 1
Mt khc, ta c ng thc quen thuc sau:
cos A + cos B + cos C = 1 + 4 sin
A
2
sin
B
2
sin
C
2
D thy sin
A
2
, sin
B
2
, sin
C
2
> 0 do A, B, C l 3 gc ca tam gic nn suy ra:
cos A + cos B + cos C > 1
y chnh l iu phi chng minh.
2/ S dng bt ng thc

a +

b +

a + b + c ta c:

p a +
_
p b +

p c
_
3p (a + b + c) =

p
136
Li c theo Cauchy Schwarz:
(

p a +

p a +

p a)
2
3(3p (a + b + c)) = 3p

p a +

p a +

p a
_
3p
Bt ng thc c chng minh.
3/ Ta c:
S = pr =
1
2
c.h
r
h
=
c
a + b + c
Do a + b > c nn ta c
r
h
<
1
2
Mt khc ta li c:
c
2
a
2
+ b
2

(a + b)
2
2
a + b c

2
T ta c:
r
h

c
c(

2 + 1)
=

2 1 > 0, 4
Chng minh hon tt.
4/ S dng cng thc Heron ta c:
S
ABC
= p(p a)(p b)(p c) = 1
Suy ra:
16 = (a + b + c)(a + b c)(a + c b)(b + c a)
=
_
(a + b)
2
c
2
_
c
2
(a b)
2

_
a
2
+ b
2
+ 2ab c
2
_
.c
2
Kt hp vi AM GM ta c:
16 2a
2
.c
2
+ 2b
2
c
2
c
4
(a
4
+ c
4
) + (b
4
+ c
4
) c
4
= a
4
+ b
4
+ c
4
Chng minh hon tt.
5/ Theo cng thc ng phn gic, bt ng thc cn chng minh tng ng vi:
ab
l
c
+
bc
l
a
+
ac
l
b
6.
abc
4S
=
3abc
2S

1
al
a
+
1
bl
b
+
1
cl
c

3
2S

b + c
a

bc
_
(a + b + c)(b + c a)

3
2S

(b + c)

bc
_
(a + b c)(a + c b)
abc
6
Ta s chng minh:
(b + c)
_
(a + b c)(a + c b) 2a

bc

b + c
2

bc

2a
_
(a + b c)(a + c b)
137

(b + c)
2
4bc
1
a
2
a
2
(b c)
2
1
(b c)
2
(
1
4bc

1
a
2
(b c)
2
)
) 0
Suy ra:
(b + c)
_
(a + b c)(a + c b) 2abc
Hon ton tng t vi cc biu thc cn li ta c iu phi chng minh.
6/ Theo nh l hm s Sine ta c:
sin Asin B =
ab
4R
2
; sin Bsin C =
bc
4R
2
; sin C sin A =
ca
4R
2
Bt ng thc cn chng minh tr thnh:
ab + bc + ca 36r
2
T h thc quen thuc r
2
=
(p a)(p b)(p c)
p
36r
2
=
(p a)(p b)(p c)
p
= 9(a + b c)(b + c a)(c + a b)
Mt khc theo bt ng thc Schur th:
9abc 9(a + b c)(b + c a)(c + a b)
Cui cng ta ch cn chng minh:
(a + b + c)(ab + bc + ca) 9abc
iu ny ng theo bt ng thc AM GM:
a + b + c 3
3

abc
ab + bc + ca 3
3

a
2
b
2
c
2
Chng minh hon tt.
ng thc xy ra khi v ch khi tam gic ABC u.2
6.9 Gi s a, b, c l 3 s thc phn bit. Chng minh rng

a + b
a b
+
b + c
b c
+
c + a
c a

> 1.
Li gii.
t x =
a + b
a b
, y =
b + c
b c
, z =
c + a
c a
th ab + bc + ca = 1.
Khi theo bt ng thc C S ta c:
(x + y + z)
2
3 (xy + yz + zx) = 3. Suy ra |x + y + z|

3 > 1. 2
6.10 Cho a, b, c l di 3 cnh tam gic tha mn ab + bc + ca = 1. Chng minh rng:
1
a + b
+
1
b + c
+
1
c + a

5
a + b + c + abc
138
Li gii.
t p = a + b + c, q = ab + bbc + ca, r = abc.
Ta c p
2
3q = 3
Hn na a, b, c l 3 cnh tam gic nn a
2
+ b
2
+ c
2
< 2 (ab + bc + ca) p
2
< 4q = 4
Bt ng thc cn chng minh tng ng vi p
3
+ (p
2
+ 6) r 4p
Theo bt ng thc Schur ta c : r
4pq p
3
9
=
4p p
3
9
Do p
3
+ (p
2
+ 6) r p
3
+ (p
2
+ 6)
4p p
3
9
(1)
M p
3
+ (p
2
+ 6)
4p p
3
9
=
p (p
2
4) (p
2
3)
9
0 (2) T (1) , (2).2
6.11 Cho ba a, b, c, x, y, z l 6 s dng tha mn ax + by + cz = 1. Chng minh rng:
x + y + z >

a + b +

b + c +

c + a
Li gii.
Ta c x =
ax
yz
+
b
z
+
c
y
>
b
z
+
c
y
Tng t, c y >
a
z
+
c
x
, z >
a
y
+
b
x
Suy ra x + y + z >
b + c
x
+
c + a
y
+
a + b
z
(x + y + z) > x +
b + c
x
+ y +
c + a
y
+ z +
a + b
z
Theo BT AM-GM ta c
x +
b + c
x
+ y +
c + a
y
+ z +
a + b
z
2

b + c + 2

c + a + 2

a + b
Vy x + y + z >

b + c +

c + a +

a + b. 2
6.12 Cho ba s thc dng a, b, c. Chng minh rng:
4a
2
b
2
c
2
(a
3
+ b
3
+ c
3
+ abc) (a + b c) (b + c a) (c + a b)
Li gii.
Gi s a b c, nu b + c a 0 th bt ng thc hin nhin ng. Nu b + c a > 0 th
a, b, c l 3 cnh ca tam gic. Ta c:
a
3
+ b
3
+ c
3
+ abc
a + b + c
R
2
Khai trin
_
(a + b)

OC + (c + b)

OA + (a + c)

OB
_
2
0 vi O l tm ng trn ngoi tip tam
gic ta c bt ng thc cn chng minh.2
6.13 Co a, b, c l 3 s thc dng. Chng minh rng:
a
2
+ b
2
a + b
+
b
2
+ c
2
b + c
+
c
2
+ a
2
c + a

3 (a
2
+ b
2
+ c
2
)
a + b + c
Li gii.
Bt ng thc tng ng vi
(a + b + c)
_
a
2
+ b
2
a + b
+
b
2
+ c
2
b + c
+
c
2
+ a
2
c + a
_
3 (a
2
+ b
2
+ c
2
)
a
2
+ b
2
+ c
2
+ 2abc
_
1
a + b
+
1
b + c
+
1
c + a
_
2 (ab + bc + ca)
139
Mt khc theo bt ng thc Cauchy-Schwart th
1
a + b
+
1
b + c
+
1
c + a

9
2 (a + b + c)
Vy chng minh bi ton ta ch cn chng minh c
a
2
+ b
2
+ c
2
+
9abc
a + b + c
2 (ab + bc + ca)
y chnh l bt ng thc Schur dng phn thc. Bi ton c chng minh. ng thc xy ra
khi a = b = c hoc a = b, c = 0 v cc hon v.2
6.14 Cho a; b; c dng v a
2
+ b
2
+ c
2
= 3. Chng minh rng:
8 (2 a) (2 b) (2 c) (a + bc) (b + ca) (c + ab)
Li gii.
Ta c 2 (2 a) = 4 2a = b
2
+ c
2
+ (a
2
2a + 1) b
2
+ c
2
Tng t ta c 2 (2 b) a
2
+ c
2
, 2 (2 c) a
2
+ b
2
Suy ra 8 (2 a) (2 b) (2 c) (a
2
+ b
2
) (b
2
+ c
2
) (c
2
+ a
2
)
Cui cng ta ch cn chng minh (a
2
+ b
2
) (b
2
+ c
2
) (c
2
+ a
2
) (a + bc) (b + ca) (c + ab)
Chng minh bt ng thc ny khng kh khn, xin dnh cho bn c.2
6.15 Cho a, b, c l cc s thc dng tha mn a + b + c = 1. Chng minh rng:
1
a
2
(1 + a)
+
1
b
2
(1 + b)
+
1
c
2
(1 + c)

3
4abc
Li gii.
p dng bt ng thc Cauchy-Schwart ta c:
bc
a (1 + a)
=
b
2
c
2
abc (1 + a)

(

bc)
2
4abc
Mt khc d thy (

bc)
2
3abc (a + b + c)
Vy
bc
a (1 + a)

3
4
Do :
1
a
2
(1 + a)
+
1
b
2
(1 + b)
+
1
c
2
(1 + c)

3
4abc
ng thc xy ra ti a = b = c =
1
3
.2
6.16 Cho a, b, c, l 3 s thc dng. Chng minh rng:
2 (a
2
+ b
2
+ c
2
) + 3
3

a
2
b
2
c
2
(a + b + c)
2
Li gii.
Bt ng thc tng ng a
2
+ b
2
+ c
2
+ 3
3

a
2
b
2
c
2
2 (ab + bc + ca)
M 3
3

a
2
b
2
c
2

9abc
a + b + c
Do ta ch cn chng minh a
2
+ b
2
+ c
2
+
9abc
a + b + c
2 (ab + bc + ca)
Nhng y l BT Schur dng phn thc. Vy, bt ng thc u bi c chng minh. ng
thc xy ra ti a = b = c.2
6.17 Cho a, b, c l cc s thc dng. Chng minh rng:
a
2
b
2
(a b)
2
+ b
2
c
2
(b c)
2
+ c
2
a
2
(c a)
2
[(a b) (b c) (c a)]
2
140
Li gii.
Nu abc = 0 th bt ng thc hin nhin ng.
Xt abc > 0, bt dng thc c vit li nh sau
_
a b
c
_
2
+
_
b c
a
_
2
+
_
c a
b
_
2

_
a b
c
+
b c
a
+
c a
b
_
2
Ta c
_
a b
c
_
2
=

_
a b
c
_
2
+ 2
a b
c
.
b c
a
Nhng
a b
c
.
b c
a
=
a (a b) (a c) + b (b c) (b a) + c (c a) (c b)
abc
0
Nn

_
a b
c
_
2

_
a b
c
_
2
.
Bt ng thc c chng minh. ng thc xy ra ti a = b = c.2
6.18 Cho a, b, c l cc s thc tha mn a
2
+b
2
+c
2
= 1. Tm gi tr ln nht ca biu thc
P =
1
1 ab
Li gii.
Xt
P 3
2
=
1
2 2ab

1
2
+
1
2 2bc

1
2
+
1
2 2ac

1
2
=
ab
2a
2
+ 2b
2
+ 2c
2
2ab

ab
2c
2
+ a
2
+ b
2
Mt khc Theo BT Cauchy-Schwart ta c
ab
2c
2
+ a
2
+ b
2

1
4
(a + b)
2
a
2
+ c
2
+ b
2
+ c
2

1
4

_
a
2
a
2
+ c
2
+
b
2
b
2
+ c
2
_
=
3
4
.
Do GTLN ca P l
9
2
khi a = b = c =
_
1
3
.2
6.19 Cho x, y, z Tha mn
yz
x
+
zx
y
+
xy
z
= 1.
Tm GTLN ca
A =
1
1 x
+
1
1 y
+
1
1 z
Li gii.
Ta t
_
yz
x
= a,
_
xz
y
= b,
_
xy
z
= c
Ta c z = ab, y = ac, x = bc v a
2
+ b
2
+ c
2
= 1.
Khi A =
1
1 ab
+
1
1 bc
+
1
1 ca
.
Theo bi trn th GTLN ca A l
9
2
khi x = y = z =
1
3
. 2
6.20 Cho a, b, c 0. CMR:
a
3
+ b
3
+ c
3
+ 6abc
3

abc (a + b + c)
2
Li gii.
Ta xt hai trng hp
Trng hp 1:
3

abc
ab + bc + ca
a + b + c
Khi theo BT Schur c
a
3
+ b
3
+ c
3
+ 6abc (ab + bc + ca) (a + b + c)
3

abc (a + b + c)
2
141
Trng hp 2:
3

abc
ab + bc + ca
a + b + c
.
Chun ha a + b + c = 3, bt ng thc tng ng vi 27 9 (ab + bc + ca) + 9abc 9
3

abc
Hay 3 + abc
3

abc + ab + bc + ca.
Theo gi thit trn ta c 3
3

abc ab + bc + ca. Vy ta ch cn chng minh 3 + abc 4


3

abc.
D thy vi abc 1 th bt ng thc ny hin nhin ng.
Vy ta c iu phi chng minh. ng thc xy ra ti a = b = c.2
6.21 Cho 3 s thc dng a, b, c tha mn a
2
+ b
2
+ c
2
= 3. Chng minh rng :
a
bc (c
2
+ c
2
) + a

3
1 + 2abc
Li gii.
Bt ng thc cho tng ng vi
bc (b
2
+ c
2
)
bc (b
2
+ c
2
) + a

6abc
1 + 2abc
hay
b
2
+ c
2
abc (b
2
+ c
2
) + a
2

6
1 + 2abc
Khng mt tnh tng qut ta gi s a b c th
b
2
+ c
2
c
2
+ a
2
a
2
+ b
2
v
1
abc (b
2
+ c
2
) + a
2

1
abc (c
2
+ a
2
) + b
2

1
abc (a
2
+ b
2
) + c
2
p dng BBT Chebyshev ta c
V T
2
3
(a
2
+ b
2
+ c
2
)
_
1
abc (b
2
+ c
2
) + a
2
_
2.
9
2abc (a
2
+ b
2
+ c
2
) + a
2
+ b
2
+ c
2
= V P
Bt ng thc c chng minh, ng thc xy ra khi a = b = c.2
6.22 Cho a, b, c l cc s thc dng. Chng minh rng:
a
a + 2b + 2c

3
5
Li gii.
t a + 2b + 2c = x, b + 2c + 2a = y, c + 2b + 2a = z
Suy ra a =
2y + 2z 3x
5
.b =
2x + 2z 3y
5
, z =
2x + 2y 3z
5
Bt ng thc c a v dng
x
y
+
x
z
+
y
x
+
y
z
+
z
x
+
z
y
6 Nhng bt ng thc ny hin
nhin ng theo BT AM-GM. ng thc xy ra khia = b = c.2
6.23 Cho a, b, c dng, Chng minh rng:
a
2
b
+
b
2
c
+
c
2
a
a + b + c +
4 (a b)
2
a + b + c
.
Li gii.
Bt ng thc tng ng vi

_
a
2
b
2a + b
_

4 (a b)
2
a + b + c
hay
(a b)
2
b

4 (a b)
2
a + b + c
p dng BT Cauchy-Schwart ta c:
(b c)
2
c
+
(c a)
2
a

(a b)
2
a + c
142
Do , V T
(a b)
2
b
+
(a b)
2
a + c
= (a b)
2
_
1
b
+
1
a + c
_

4 (a b)
2
a + b + c
Bt ng thc c chng minh.2
6.24 Cho a, b, c l 3 s thc khng m tha mn ab + bc + ca = 1. Chng minh rng:
1
a
2
bc + 1
1
Li gii.
Bt ng thc c vit li
ab + bc + ca
a
2
bc + 1
1

_
1 2
ab + bc + ca
a
2
bc + 1
_
1

a
2
+ ab + ac
a
2
bc + 1
1 p dng BT Cauchy-Schwart c
a
2
+ ab + ac
a
2
bc + 1
= (a + b + c)
a
2
a
3
abc + a

(a + b + c)
3

(a
3
abc + a)
Mt khc vi ab + bc + ca = 1
th d thy
(a + b + c)
3

(a
3
abc + a)
= 1
Vy, bt ng thc c chng minh, ng thc xy ra khi a = b = c =
1
3
.2
6.25 Cho 3 s a, b, c > 0.Chng minh rng:
a
2
b
+
b
2
c
+
c
2
a
+
81
4
a
2
b
(2a + b)
2

13
4
(a + b + c)
Li gii.
Theo bt ng thc Cauchy-Schwart ta c
a
2
b
(2a + b)
2

(a + b + c)
2
(2a + b)
2
b
=
x
2
5x + 4y
( vi x = a + b + c v y =
a
2
b
+
b
2
c
+
c
2
a
Ta ch cn chng minh y +
81x
2
4 (5x + 4y)

13
4
x
4x + 5y +
81x
2
4 (5x + 4y)
18x
Nhng bt ng thc ny hin nhin ng theo bt ng thc AM-GM. ng thc xy ra
khia = b = c. 2
6.26 Cho cc s thc dng a, b, c tho mna + b + c = 1.Chng minh rng:
5 (a
2
+ b
2
+ c
2
) 6 (a
3
+ b
3
+ c
3
) + 1
Li gii.
ng bc 2 v ta c
5 (a
2
+ b
2
+ c
2
) (a + b + c) 6 (a
3
+ b
3
+ c
3
) + (a + b + c)
3
Hay 2 (a
3
+ b
3
+ c
3
) + 6abc 2 [2 (b
2
+ c
2
) + b (a
2
+ c
2
) + c (a
2
+ b
2
)]
Nhng y chnh l BT Schur bc 3. ng thc xy ra khi a = b = c.2
143
6.27 Cho cc s thc a, b, c 1.Chng minh rng:

a 1 +

b 1 +

c 1
_
a (bc + 1)
Li gii.
Ta c bc
_
b 1 +

c 1
_
2
Hay

bc

b 1 +

c 1
Vy,

a 1

b 1 +

c 1

bc +

a 1
_
a (bc + 1).2
6. 28 Cho a, b, c l di 3 cnh mt tam gic. Chng minh rng:
a
_
a
2
+
13
5
bc

_
5
2
Li gii.
p dng bt ng thc Hoder ta c

a (5a
2
+ 13bc) .
a

5a
2
+ 13abc
.
a

5a
2
+ 13abc
(a + b + c)
3
Nh vy ch cn chng minh (a + b + c)
3

1
2

a (5a
2
+ 13bc)
V a, b, c l di 3 cnh tam gic nn tn t x, y, z dng sao cho a = x +y, b = y +z, c = z +x
Thay vo v khai trin ta c
x
3
+ y
3
+ z
3
+ 3xyz x
2
(x + y) + y
2
(x + z) + z
2
(x + y)
Nhng y chnh l bt ng thc Schur nn ta c PCM. ng thc xy ra khi a = b = c.2
6. 29 Cho cc s thc khng m a, b, c tha mn a
2
+b
2
+c
2
= a +b +c. Chng minh rng:
_
a + b + c

2
_
2

a
2
b + b
2
c + c
2
a +

ab
2
+ bc
2
+ ca
2
Li gii.
Ta c:

a
2
b + b
2
c + c
2
a +

ab
2
+ bc
2
+ ca
2

_
2 [ab (a + b) + bc (b + c) + ca (c + a)]
Ta ch cn chng minh:
(a + b + c)
4
8ab (a + b) + 8bc (b + c) + 8ca (c + a)
(a + b + c)
4
8ab (a
2
+ b
2
+ c
2
c) + 8cb (a
2
+ b
2
+ c
2
a) + 8ac (a
2
+ b
2
+ c
2
b)
(a + b + c)
4
+ 24abc 4 (2ab + 2bc + 2ca) (a
2
+ b
2
+ c
2
)
(a
2
+ b
2
+ c
2
2ab 2bc 2ca)
2
+ 24abc 0.
Vy, bt ng thc c chng minh, ng thc xy ra khi a = b = c = 0.2
6. 30 Cho cc s thc a, b, c . CMR:
2 (1 + abc) +
_
2 (1 + a
2
) (1 + b
2
) (1 + c
2
) (1 + a) (1 + b) (1 + c)
Li gii.
Bt ng thc cho c vit li
_
2 (1 + a
2
) (1 + b
2
) (1 + c
2
) (b + c) (1 + a) + (1 bc) (a 1)

_
_
(1 + a)
2
+ (a 1)
2
_
(b + c)
2
+ (1 bc)
2

(b + c) (1 + a) + (1 bc) (a 1)
Nhng bt ng thc ny hin nhin ng theo BT Cauchy-Schwart.2
144
6. 31 Cho cc s thc dng a, b, c, d . Tm gi tr nh nht ca biu thc:
A =
b (a + c)
c (a + b)
+
c (b + d)
d (b + c)
+
d (c + a)
a (c + d)
+
a (d + b)
b (d + a)
Li gii.
Ta c:
A = (a + c)
_
b
c (a + b)
+
d
d (c + d)
_
+ (b + d)
_
c
d (b + c)
+
a
b (d + a)
_
= (abc + abd + acd + bcd)
_
a + c
ac (a + b) (c + d)
+
b + d
bd (b + c) (d + a)
_
=
_
1
a
+
1
b
+
1
c
+
1
d
_
_

_
1
a
+
1
c
_
1
a
+
1
b
__
1
d
+
1
c
_ +
1
b
+
1
d
_
1
c
+
1
b
__
1
d
+
1
a
_
_

_
Theo BT AM-GM ta c:
1
a
+
1
c
_
1
a
+
1
b
__
1
d
+
1
c
_
4
_
1
a
+
1
c
_
_
1
a
+
1
b
+
1
c
+
1
d
_
2
1
b
+
1
d
_
1
c
+
1
b
__
1
d
+
1
a
_
4
_
1
b
+
1
d
_
_
1
a
+
1
b
+
1
c
+
1
d
_
2
do A 4.Vy gi tr nh nht ca A l 4 khi a = c v b = d..2
6. 32 Cho cc s thc dng a, b, c . Chng minh rng:
1
8a
2
+ bc

1
ab + bc + ca
Li gii.
p dng BT Cauchy-Schwart ta c:
1
8a
2
+ bc
=
b
2
c
2
8a
2
b
2
c
2
+ b
3
c
3

(ab + bc + ca)
2
24a
2
b
2
c
2
+ a
3
b
3
+ b
3
c
3
+ c
3
a
3
Do vy ta ch cn chng minh
(ab + bc + ca)
2
24a
2
b
2
c
2
+ a
3
b
3
+ b
3
c
3
+ c
3
a
3

1
ab + bc + ca
abc (a + b) (b + c) (c + a)
.
= 8a
2
b
2
c
2
Nhng bt ng thc ny hin nhin ng theo BT AM-GM.
ng thc xy ra khi a = b = c..2
6. 33 Cho cc s thc dng a, b, c tha mn a + b + c = 3. Chng minh rng:
a
2
b
2a + b
+
b
2
c
2b + c
+
c
2
a
2c + a
1
Li gii.
Nhn xt thy
a
2
b
2a + b

2ab + a
2
9
(1)
Tht vy, (1)a
3
+ ab
2
2a
2
b( lun ng theo BT AM-GM).
Tng t, ta c
b
2
c
2b + c

2bc + b
2
9
,
c
2
a
2c + a

2ac + c
2
9
Cng 3 v ca 3 bt ng thc cng chiu vi iu kin a +b +c = 3 ta c bt ng thc cn
145
chng minh.ng thc c khi a = b = c = 1.2
6. 34 Cho cc s thc dng a, b, c tha mn a
2
+b
2
+c
2
= 1. Tm gi tr nh nht ca biu
thc:
P =
a
b
2
+ c
2
+
b
a
2
+ c
2
+
c
a
2
+ b
2
Li gii.
V a
2
+ b
2
+ c
2
= 1 nn a, b, c (0, 1). Suy ra 1 a
2
, 1 b
2
, 1 c
2
l cc s dng.
Ta c:
_
b
2
+ c
2
a
_
2
=
a
2
(1 a
2
)
2
a
4
=
2a
2
(1 a
2
) (1 a
2
)
2a
4

(2a
2
+ 1 a
2
+ 1 a
2
)
54a
4
=
4
27a
4
Suy ra
a
b
2
+ c
2

3

3a
2
2
.
Chng minh tng t ta c
b
c
2
+ a
2

3

3b
2
2
v
c
a
2
+ b
2

3

3c
2
2
.
Cng v 3 bt ng thc cng chiu vi iu kin a
2
+ b
2
+ c
2
= 1 ta c P
3

3
2
.
Vy, gi tr nh nht ca P l
3

3
2
, t c khi a = b = c =

3
3
..2
6. 35 Cho cc s thc dng x, y tha mn x
2
+y
2
= 1. Tm gi tr nh nht ca biu thc:
A = (1 + x)
_
a +
1
y
_
+ (1 + y)
_
1 +
1
x
_
Li gii.
Ta c A = 2 + x + y +
1
x
+
1
y
+
x
y
+
y
x
4 + x + y +
1
x
+
1
y
Mt khc, c
x +
1
2x

2
y +
1
2y

2
1
2
_
1
x
+
1
y
_

2
x + y

2
_
2 (x
2
+ y
2
)
=

2 .
T suy ra A 4 + 3

2
Vy, GTNN ca A l 4 + 3

2 khi x = y =

2
2
..2
6. 36 Cho cc s thc khng m x, y, z. Chng minh rng:
1
(x y)
2
+
1
(y z)
2
+
1
(z x)
2

4
xy + yz + zx
Li gii.
Khng mt tnh tng qut, gi s z = min{x, y, z}. Khi ta c cc nh gi
(z x)
2
= z
2
+ x
2
2xz x
2
(y x)
2
= y
2
+ z
2
2yz y
2
xy + yz + zx xy
Vy, ta c
1
(x y)
2
+
1
(y z)
2
+
1
(z x)
2

4
xy + yz + zx

1
(x y)
2
+
1
x
2
+
1
y
2

4
xy
=
(x
2
+ y
2
3xy)
x
2
y
2
(y z
2
)
0.
146
ng thc xy ra khi v ch khi
x
y
=
3

5
2
, z = 0. Php chng minh hon tt.2
6. 37 Cho cc s thc dng a, b, c. Chng minh rng:
a
2b + c
+
b
2c + a
+
c
2a + c
1
Li gii.
p dng BT Cauchy-Schwart ta c
V T =
a
2
2ab + ac
+
b
2
2bc + ab
+
c
2
2ac + bc

(a + b + c)
2
3 (ab + bc + ca)
1.
ng thc xy ra khi a = b = c..2
6. 38 Cho cc s thc x, y, z tha mn x
2
+ y
2
+ z
2
= 2. Tm GTLN ca :
P = x
3
+ y
3
+ z
3
3xyz
Li gii.
Ta c P
2
= (x + y + z)
2
(x
2
+ y
2
+ z
2
xy yz zx)
= (x
2
+ y
2
+ z
2
+ 2xy + 2yz + 2zx) (x
2
+ y
2
+ z
2
xy yz zx) (x
2
+ y
2
+ z
2
xy yz zx)
(x
2
+ y
2
+ z
2
)
3
= 8 P 2

2.
Vy,GTLN ca P l 2

2 khi x, y, z l hon v ca b
_
0, 0,

2
_
. 2
6. 39 Cho cc s thc a, b, c, d thuc
_
0,
1
2
_
. Chng minh rng:
(a + b + c + d)
4
(1 a) (1 b) (1 c) (1 a) abcd (4 a b c d)
4
Li gii.
Nhn xt: vi 0 a, b
1
2
ta c:
_
1
a
1
__
1
b
1
_

_
2
a + b
1
_
2
Tht vy, d thy bt ng thc trn tng ng vi (a b)
2
(1 a b) 0.
Vy, p dng tng t vi c, d v
a + b
2
,
c + d
2
, ta c PCM.
ng thc c khi a = b = c = d..2
6. 40 Cho cc s thc khng m a, b, c. Chng minh rng:
a
2
b
2
bc + c
2
+
b
2
a
2
ac + c
2
+
c
2
a
2
ab + b
2
2
Li gii.
Khng mt tnh tng qut, gi s a b c. Khi
a
2
b
2
bc + c
2

a
2
b
2
b
2
c
2
ac + a
2

b
2
a
2
c
2
a
2
ab + b
2
0
T suy ra V T
a
2
b
2
+
b
2
a
2
2
ng thc c khi a = b, c = 0 v cc hon v..2
147
3.7 Bi 7.1 n bi 7.40
7.1 Tm gi tr nh nht ca biu thc sau:
A = |x + 2000| +|x + y + 4| +|2x + y 6|
Li gii.
p dng bt ng thc c bn: |a| +|b| +|c| |a + b + c|
Ta c:
A = |x + 2000| +|x + y + 4| +|6 2x y| |(x + 2000) + (x + y + 4) + (6 2x y)| = 2010
ng thc xy ra khi
_

_
x + 2000 0
x + y + 4 0
6 2x y 0
C v s cp (x; y) tha mn, v d (1; 1), (2; 1)....2
7.2 Cho a, b, [0, 2]. Tm gi tr ln nht ca biu thc sau:
P =
8 + 6(a + b) + (a + b)
2
4 + 2(a + b) + ab
Li gii.
Trc ht, ta c th d on gi tr ln nht ca P l 3 khi (a; b) = (2; 0) hoc (0; 2).
Do a [0; 2] nn a(a 2) 0 hay a
2
2a. Tng t ta c b
2
2b.
V vy,
P =
8 + 6(a + b) + (a + b)
2
4 + 2(a + b) + ab
=
8 + 6(a + b) + a
2
+ b
2
+ 2ab
4 + 2(a + b) + ab

8 + 8(a + b) + 2ab
4 + 2(a + b) + ab
= 2 +
4(a + b)
4 + 2(a + b) + ab
Ta cn chng minh
4(a + b)
4 + 2(a + b) + ab
1 hay (a 2)(b 2) 0
Nhng bt ng thc trn hin nhin ng do a; b 2.2
7.3 Cho ba s thc dng a, b, c tha mn a + b + c = 3. Chng minh rng:
bc

a
2
+ 3
+
ca

b
2
+ 3
+
ab

c
2
+ 3

3
2
Li gii.
Do (a + b + c)
2
3(ab + bc + ca) nn ab + bc + ca 3.
Ta c:
bc

a
2
+ 3

bc

a
2
+ ab + bc + ca
=
bc
_
(a + b)(a + c)

1
2
(
bc
a + b
+
bc
c + a
)
Tng t vi cc biu thc cn li. Cng 3 v bt ng thc va chng minh, suy ra:
bc

a
2
+ 3
+
ca

b
2
+ 3
+
ab

c
2
+ 3

a + b + c
2
=
3
2
.
ng thc xy ra khi a = b = c = 1. 2
148
7.4 Cho hai s thc a, b 0 tha mn a + b = 2. Chng minh rng:
a
a
b
b
+ 3ab 4
Li gii.
p dng bt ng thc AM GM suy rng, ta c:
a
a + b
a +
b
a + b
b a
a
a+b
b
b
a+b
Do x + y = 2 nn:
a
2
+ b
2
2
a
a
2
b
b
2
hay a
a
b
b
(
a
2
+ b
2
2
)
2
= (2 ab)
2
V vy, a
a
b
b
+ 3ab 4 (2 ab)
2
+ 3ab 4 = ab(ab 1) 0 v ab (
a + b
2
)
2
= 1
Ta c iu phi chng minh. ng thc xy ra khi(a; b) = (1; 1), (2; 0), (0; 2)..2
7.5 Cho hai s thc a b > 0. Chng minh rng:
(2
a
+
1
2
a
)
b
(2
b
+
1
2
b
)
a
Li gii.
Bt ng thc cn chng minh tng ng
ln(1 + 4
a
)
a

ln(1 + 4
b
)
b
Xt hm s f(x) =
ln(1 + 4
x
)
x
vi x > 0
Ta c: f

(x) =
4
x
ln4
x
(1 + 4
x
)ln(1 + 4
x
)
x
2
(1 + 4
x
)
< 0, do f(x) nghch bin trn khong (0; + ).
Kt hp a b > 0 nn
ln(1 + 4
a
)
a

ln(1 + 4
b
)
b
ng thc xy ra khi a = b. 2
7.6 Cho a; b; c dng v abc = 1. Chng minh bt ng thc sau:
(a
2
+ b
2
)
3
a
3
+ b
3
+
(b
2
+ c
2
)
3
b
3
+ c
3
+
(c
2
+ a
2
)
3
c
3
+ a
3
12
Li gii.
Li gii 1:
t a =
1
x
, b =
1
y
, c =
1
z
, ta c xyz = 1. Khi bt ng thc cn chng minh tng ng:
(x
2
+ y
2
)
3
x
3
y
3
(x
3
+ y
3
)
12 hay
(x
2
+ y
2
)
3
x
2
y
2
(x + y)xy(x
2
xy + y
2
)
12
Mt khc, ta c xy(x
2
xy + y
2
)
(x
2
+ y
2
)
2
4
nn:

(x
2
+ y
2
)
3
x
2
y
2
(x + y)xy(x
2
xy + y
2
)
2

2(x
2
+ y
2
)
x
2
y
2
(x + y)
2

x + y
x
2
y
2
6
3

(x + y)(y + z)(z + x)
x
4
y
4
z
4
6
3
_
8xyz = 12
Ta c iu phi chng minh. ng thc xy ra khi a = b = c = 1.2
Li gii 2:
149
Ta s chng minh nhn xt: Vi 2 s x, y dng,
(x
4
+ y
4
)
3
x
6
+ x
6
4x
3
y
3
Tht vy,
(x
4
+ y
4
)
3
x
6
+ x
6
=
x
12
+ y
12
+ 3x
4
y
4
(x
4
+ y
4
)
x
6
+ y
6
=
(x
6
+ y
6
)
2
+ x
4
y
4
(x
2
y
2
)
2
+ 2x
4
y
4
(x
4
+ y
4
)
x
6
+ y
6

(x
6
+ y
6
)
2
+ 2x
4
y
4
(x
4
+ y
4
)
x
6
+ y
6

2(x
6
+ y
6
)
_
2x
4
y
4
(x
4
+ y
4
)
x
6
+ y
6
4x
3
y
3
Ly x =

a, y =

b, khi
(a
2
+ b
2
)
3
a
3
+ y
3
4ab

ab.
Tng t vi cc biu thc cn li, ta c
(a
2
+ b
2
)
3
a
3
+ b
3
4

ab

ab 12(AM GM.)
ng thc xy ra khi a = b = c = 1. 2
7.7 Cho a, b, c l cc s thc dng v k [0; 2] Chng minh rng
a
2
bc
b
2
+ c
2
+ ka
2
+
b
2
ac
a
2
+ c
2
+ kb
2
+
c
2
ab
a
2
+ b
2
+ kc
2
0
Li gii.
Cn chng minh:
(a
2
bc)(b + c)
(b
2
+ c
2
+ ka
2
)(b + c)
+
(b
2
ac)(a + c)
(a
2
+ c
2
+ kb
2
)(a + c)
+
(c
2
ab)(a + b)
(a
2
+ b
2
+ kc
2
)(a + b)
0
Khng mt tnh tng qut, gi s a b, khi :
(a
2
bc)(b + c) (b
2
ac)(a + c) = (ab + c
2
)(a b) + c(a
2
b
2
) 0
(b
2
+ c
2
+ ka
2
)(b + c) (a
2
+ c
2
+ kb
2
)(a + c) = (b a)(a
2
+ b
2
+ c
2
(k 1)(ab + bc + ca)) 0
Ta c cc b s cng chiu :
(a
2
bc)(b + c); (b
2
ac)(a + c); (c
2
ab)(a + b)
v
1
(b
2
+ c
2
+ ka
2
)(b + c)
;
1
(a
2
+ c
2
+ kb
2
)(a + c)
;
1
(a
2
+ b
2
+ kc
2
)(a + b)
p dng bt ng thc Chebychev vi ch (a
2
bc)(b+c)+(b
2
ac)(a+c)+(c
2
ab)(a+b) = 0,
ta c iu phi chng minh.
ng thc xy ra khi a = b = c.2
7.8 Cho a, b, c, l 3 cnh ca mt tam gic. Chng minh rng:
a + b c
3b + c a
+
b + c a
3c + a b
+
c + a b
3a + b c
1
Li gii.
Do a, b, c l 3 cnh ca tam gic, t a = x + y, b = y + z, c = z + x, bt ng thc tr thnh:
y
y + 2z
+
z
z + 2x
+
x
x + 2y
1
Bt ng thc trn c th chng minh n gin bng Cauchy Schwarz:
y
y + 2z
+
z
z + 2x
+
x
x + 2y

(x + y + z)
2
x
2
+ y
2
+ z
2
+ 2xy + 2yz + 2zx
= 1
ng thc xy ra khi a = b = c.2
150
7.9 Cho a, b, c l cc s thc khng m tha mn a + b + c > 0. Chng minh rng:
a
4a + 4b + c
+
b
4b + 4c + a
+
c
4c + 4a + b

1
3
Li gii.
Ta c:
4a(a + b + c)
4a + 4b + c
= a +
3ac
4a + 4b + c
Bt ng thc cn chng minh tng ng:
ac
4a + 4b + c
+
ab
4b + 4c + a
+
bc
4c + 4a + b

a + b + c
9
p dng bt ng thc Cauchy Schwarz dng Engel, ta c:
ca
4a + 4b + c
=
ca
(2b + c) + 2(2a + b)

ca
9
(
1
2b + c
+
2
2a + b
)
Tng t, cng cc v bt ng thc ta c:
ca
4a + 4b + c

1
9

(
ca
2b + c
+
2ca
2a + b
) =
1
9
(
ca
2b + c
+
2ab
2b + c
) =
a + b + c
9
.
ng thc xy ra khi a = b = c hoc a = 2b, c = 0 v cc hon v.2
7.10 Cho xyz = 1, x, y, z > 0. Tm gi tr ln nht ca biu thc:
P =
1
x
2
+ 2y
2
+ 3
+
1
y
2
+ 2z
2
+ 3
+
1
z
2
+ 2x
2
+ 3
Li gii.
p dng 2 bt ng thc n gin: x
2
+ y
2
2xy v y
2
+ 1 2y, ta c:
1
x
2
+ 2y
2
+ 3

1
2(xy + y + 1)
Tng t vi cc biu thc cn li, suy ra: P
1
2
(Ch ng thc
1
xy + y + 1
= 1 khi xyz = 1)
ng thc xy ra khi x = y = z = 1.2
7.11 Cho x, y, z khng m tha mn x
2
+ y
2
+ z
2
+ xyz = 4. Chng minh rng:
0 xy + yz + xz xyz 2
Li gii.
Chng minh bt ng thc v tri:
T gi thit ta thy rng c t nht mt trong 3 s x, y, z 1. (V iu ngc li v l). Gi s
x 1. Khi ta c:
xy + yz + xz xyz = x(y + z) + yz(1 x) 0
ng thc xy ra khi (x, y, z) = (2, 0, 0) v cc hon v.
Chng minh bt ng thc v phi:
Theo nguyn l Dirichle, tn ti 2 s cng nm v 1 bn so vi 1. Gi s 2 s l y, z. Khi
:(1 y)(1 z) 0.
Ta c: 4 = x
2
+ y
2
+ z
2
+ xyz x
2
+ 2yz + xyz hay yz(2 + x) 4 x
2
hay yz 2 x.
151
V vy ta c:
xy + yz + xz xyz x(y + z) + (2 x) xyz
= 2 x(1 + yz y z) = 2 a(1 b)(1 c)
2
ng thc xy khi (x, y, z) = (1, 1, 1); (0,

2,

2) v cc hon v.2
7.12 Cho a, b, c > 0. Chng minh rng:
a
2

b + c
+
b
2

c + a
+
c
2

a + b

b
2

b + c
+
c
2

c + a
+
a
2

a + b
Li gii.
Gi s a b c. Khi , a + b a + c b + c.
Bt ng thc cn chng minh ch l h qu ca bt ng thc hon v vi 3 b s c iu kin
nh trn:
a b c
1
b + c

1
a + c

1
a + b
Ta c iu phi chng minh. ng thc xy ra khi a = b = c.2
7.13 Cho 3 s thc dng x, y, z > 1 tha mn x + y + z = xyz. Tm gi tr nh nht ca
biu thc:
T =
y 2
x
2
+
z 2
y
2
+
x 2
z
2
Li gii.
Ta c
T =

(
y 2
x
2
+
1
x
)

1
x
=

(
(x 1) + (y 1)
x
2
+
1
x
)

1
x
=

[(x 1)(
1
x
2
+
1
z
2
)]

1
x
(x 1)(
2
xz
)

1
x
=

1
x
2
Mt khc :
1
x

_
3
1
xy
=

3. V vy, T

3 2
ng thc xy ra khi x = y = z =

3. .2
7.14 Cho a, b, c 0, a + b + c = 1. Tm gi tr ln nht ca biu thc:
P =
1 + a
2
1 + b
2
+
1 + b
2
1 + c
2
+
1 + c
2
1 + a
2
Li gii.
Li gii 1.
Khng mt tnh tng qut, gi s a =max{a, b, c}. Ta c:
152
a
2
+ 1
b
2
+ 1
= a
2
+ 1
b
2
(a
2
+ 1)
b
2
+ 1
a
2
+ 1
b
2
(a
2
+ 1)
2
Tng t vi cc s hng cn li, ta thu c:
P a
2
+ b
2
+ c
2
+ 3
a
2
(b
2
+ 1) + b
2
(c
2
+ 1) + c
2
(a
2
+ 1)
2
=
a
2
+ b
2
+ c
2
(a
2
b
2
+ b
2
c
2
+ c
2
a
2
)
2
+ 3

a
2
+ b
2
+ c
2
+ 2(ab + bc + ca)
2
+ 3 =
7
2
.
ng thc xy ra khi = 1, b = 0, c = 0 v cc hon v.2
Li gii 2.
Cho a = 1, b = c = 0. Khi , gi tr ln nht d on l
7
2
. Da trn d on , ta c li gii
nh sau:
Gi s c = min{a, b, c}, ta c
a
b
+
b
c
+
c
a
3 =
_
a
b
+
b
a
2
_
+
_
b
c
+
c
a
2
b
a
_
=
(a b)
2
ab
+
(a c)(b c)
ac
0
T ta c:
P 3 =
1 + a
2
1 + b
2
+
1 + b
2
1 + c
2
+
1 + c
2
1 + a
2
3 =
(a
2
b
2
)
2
(1 + a
2
)(1 + b
2
)
+
(a
2
c
2
)(b
2
c
2
)
(1 + a
2
)(1 + c
2
)
,
Vi gi s c nm gia a, b, ta c (a
2
c
2
)(b
2
c
2
) 0 suy ra P 3
(a
2
b
2
)
2
(1 + a
2
)(1 + b
2
)
Nh vy, chng minh gi tr ln nht l
7
2
. Ta cn chng minh:
(a
2
b
2
)
2
(1 + a
2
)(1 + b
2
)

1
2
hay 2(a
2
b
2
)
2
(1 + a
2
)(1 + b
2
)
S dng gi thit, ta suy ra 0 a, b 1, t suy ra bt ng thc trn ng v:
2(a
2
b
2
)
2
2 max {a
4
, b
4
} 2 max {a
2
, b
2
} max {1 + a
2
, 1 + b
2
} (1 + a
2
)(1 + b
2
).
Bt ng thc c chng minh..2
Nhn xt:
Bng cch chng minh tng t, ta c kt qu tng qut sau:
Vi mi s thc khng m a, b, c c tng bng 1 v vi mi k 1, ta c:
P =
1 + a
k
1 + b
k
+
1 + b
k
1 + c
k
+
1 + c
k
1 + a
k

7
2
7.15 Cho x, y, z dng tha mn xy + yz + 3xz = 1. Tm gi tr nh nht ca x
2
+ y
2
+ z
2
Li gii.
Nhng bi dng ny, chng ta c th gii bng phng php cn bng h s vi bt ng thc
153
AMGM, Cauchy Schwarz hoc bt ng thc Holder. y s dng bt ng thc AM
GM. Cch cn li xin dnh cho bn c.
Ta c:
x
2
+ y
2
+ z
2
=(ax
2
+
1
2
y
2
) + (az
2
+
1
2
y
2
) + (1 a)x
2
+ (1 a)z
2
2
_
a
2
xy + 2
_
a
2
yz + 2(1 a)xz
Cn tm a sao cho
_
a
2
=
1
3
(1 a), ta tm c gi tr nh nht ca biu thc.2
Nhn xt:
By gi ta a ra 2 bi ton dng tng t v cng rt th v, v phn chng minh s dnh cho
cc bn.
1: Cho cc s thc khng m x
1
; x
2
; x
3
tha mn x
1
+x
2
+x
3
= a.(a l hng s bit) Tm gi
tr ln nht ca:
f = k
1
xy + k
2
yz + k
3
xz (vi k
1
; k
2
; k
3
l hng s)
2: Cho x
1
; ...; x
n
0; x
1
+ ... + x
n
= k(k l hng s). Tm gi tr nh nht ca:
a
1
x
m
1
+ a
2
x
m
2
+ ... + a
n
x
m
n
7.16 cho a; b; c > 0, abc = 1. Tm gi tr ln nht ca biu thc:
T =
ab
a + b + ab
+
bc
b + c + bc
+
ca
c + a + ca
Li gii.
vi iu kin abc = 1, ta c:
ab
a + b + ab
=
1
1
a
+
1
b
+ 1
p dng bt ng thc quen thuc x
3
+ y
3
xy(x + y), ta c:
1
a
+
1
b
+ 1
3
_
1
ab
_
3
_
1
a
+
3
_
1
b
_
+ 1 =
3

c
_
3
_
1
a
+
3
_
1
b
_
+ 1
suy ra
1
1
a
+
1
b
+ 1

1
3

c
_
3
_
1
a
+
3
_
1
b
_
+ 1
=
3

ab
3

ab +
3

bc +
3

ca
Lm tng t vi cc biu thc cn li, ta c T 1.
ng thc xy ra khi v ch khi a = b = c = 1.2
154
7.17 Cho a, b, c > 0 v a + b + c = 1. Tm gi tr nh nht ca biu thc:
P =
3
_
(
1
ab
1)(
1
bc
1)(
1
ac
1)
Li gii.
t a =
1
x
; b =
1
y
; c =
1
z
, khi ta c: xy + yz + zx = xyz v xyz 27
Ta s chng minh:
(
1
ab
1)(
1
bc
1)(
1
ac
1) = (xy 1)(yz 1)(zx 1) 512
x
2
y
2
z
2
xyz(x + y + z) + xy + yz + zx 1 512
2x
2
y
2
z
2
2xyz(x + y + z) + 2xyz 2.513
x
2
y
2
z
2
+ [(xy + yz + zx)
2
2xyz(x + y + z)] + 2xyz 1026
x
2
y
2
z
2
+ x
2
y
2
+ y
2
z
2
+ z
2
x
2
+ 2xyz 1026
Mt khc, theo AM GM, ta c:
x
2
y
2
+ y
2
z
2
+ z
2
x
2
3
3
_
x
4
y
4
z
4
243; x
2
y
2
z
2
729; v xyz 27
Vy P 8. ng thc xy ra khi a = b = c =
1
3
.2
7.18 Cho cc s dng a, b, c c tch bng 1. Chng minh bt ng thc sau:
a + b + c
3

10
_
a
3
+ b
3
+ c
3
3
Li gii.
y l mt bi ton rt kh. Ta s s dng b sau:
B : Vi mi s thc dng a, b, c th:
(a + b + c)
6

729
5
abc(a
3
+ b
3
+ c
3
+ 2abc) .
Quay tr li bi ton. T b trn vi ch abc = 1, ta c :
a + b + c
3

6
_
a
3
+ b
3
+ c
3
+ 2
5
Mt khc theo bt ng thc AM GM ta c:
a
3
+ b
3
+ c
3
+ 2 = 3
a
3
+ b
3
+ c
3
3
+ 1 + 1 5
5
_
(a
3
+ b
3
+ c
3
)
3
27
Ta c iu phi chng minh. ng thc xy ra khi a = b = c = 1. .2
7.19 Cho x, y, z 0 tho x + y + z = 1. Tm GTLN ca biu thc:
P = x
n
y + y
n
z + z
n
x vi n N

Li gii.
Vi n = 1 th maxP =
1
3
.
Vi n > 1.
Khng mt tng qut, gi s x = max {x, y, z}, th th:
155
_

_
y x y
n
z x
n1
yz
z x z
n
x zx
n
z
n
x z
2
x
n1
n > 1
n 1
n

1
2

n 1
n
.z
z
2
Ta c:
P = x
n
y + y
n
z + z
n
x x
n
y + x
n1
yz +
1
2
.z
n
x +
1
2
.z
n
x
x
n
y + x
n1
yz +
zx
n
2
+
z
2
x
n1
2
= x
n1
(x + z)
_
y +
z
2
_
x
n1
(x + z)
_
y +
n 1
n
z
_
= n
n
.
_

_
x
n
.
x
n
...
x
n
. .
n1
.
x + z
n
.
_
y +
n 1
n
.z
_
_

_
(1)
Theo bt ng thc AM GM th:
x
n
.
x
n
...
x
n
. .
n1
.
x + z
n
.
_
y +
n 1
n
.z
_

_
(n 1).
x
n
+
x + z
n
+ y +
n 1
n
.z
n + 1
_

_
n+1
T em th vo (1), ta c:
P
n
n
(n + 1)
n+1
ng thc xy ra khi x = y =
n + 1
n
, z = 0 v cc hon v.2
7.20 Cho a, b, c tha mn
1
2
a; b; c 2.Chng minh rng:
a
a + b
+
b
b + c
+
c
c + a

22
15
Li gii.
Xt :f(a, b, c) =
a
a + b
+
b
b + c
+
c
c + a
.
Khng mt tnh tng qut gi s a = max{a, b, c} .Ta c:
f(a, b, c) f(a, b,

ab) = (

ab c)
2
(

b
(b + c)(c + a)
)
1

a +

b
0
t
_
a
b
= x (
1
2
x 2), ta c:
f(x) =
x
2
x
2
+ 1
+
2
x + 1
f

(x) =
2x
(x
2
+ 1)
2

2
(x + 1)
2
=
2(x 1)(1 x
3
)
(x
2
+ 1)
2
(x + 1)
2
0
Vy f(x) nghch bin minf(x) = f(2) =
22
15
156
7.21 Cho a, b, c dng v a + b + c = 1. Chng minh rng:

a
2
+ abc
c + ab
+

b
2
+ abc
a + bc
+

c
2
+ abc
b + ac

1
2

abc
Li gii.
Ta c:
a
2
+ abc = a(a + b)(a + c) v c + ab = (b + c)(c + a)
V th bt ng thc cn chng minh c th c vit di dng:

_
a(a + b)(a + c)
(b + c)(c + a)

1
2

abc
.
hay
a
_
bc(a + b)(a + c)
(b + c)(c + a)

1
2
.
S dng bt ng thc AM-GM ta c:
_
bc(a + b)(a + c)
b(a + c) + c(a + b)
2
=
ab + ac + 2bc
2
.
Theo ta ch cn chng minh c:
a(ab + ac + 2bc)
(b + c)(c + a)
1.
Bt ng thc ny tng ng vi

a(a + b)(ab + ac + 2bc) (a + b)(b + c)(c + a)(a + b + c).


Khai trin v rt gn ta c

a
3
(b + c) +

a
2
b
2
+ 5

a
2
bc

a
3
(b + c) + 2

a
2
b
2
+ 4

a
2
bc
hay a
2
b
2
+ b
2
c
2
+ c
2
a
2
abc(a + b + c).
Bt ng thc ny hin nhin ng.
ng thc xy ra khi a = b = c =
1
3
. 2
7.22 Cho a, b, c dng tha mn a + b + c = 1. Chng minh rng:
ab + bc + ca 2abc
7
27
Li gii.
Vit li bt ng thc di dng: (ab + bc + ca)(a + b + c) 2abc
7
27
t a + b + c = k, s dng bt ng thc quen thuc:
(a + b c)(b + c a)(c + a b) abc
hay (k 2a)(k 2b)(k 2c) abc.
Sau khi bin i tng ng ta thu c:
4k(ab + bc + ca) k
3
+ 9abc
(a + b + c)(ab + bc + ca)
1
4
+
9abc
4
Mt khc, theo AM GM ta c: abc (
a + b + c
3
)
3
.
V vy (ab + bc + ca)(a + b + c) 2abc
1
4
+
abc
4

7
27
157
Ta c iu phi chng minh. ng thc xy ra khi a = b = c =
1
3
.2
7.23 Cho a, b, c > 0 v a + b + c = 1. Chng minh rng:
ab
ab + c
+
bc
bc + a
+
ac
ac + b

3
4
Li gii.
Bt ng thc cn chng minh tng ng vi:

ab
(c + a)(c + b)

3
4
3(a + b)(b + c)(c + a) 4[

ab(a + b)]
Da vo 2 ng thc quen thuc:
(a + b + c)(ab + bc + ca) abc = (a + b)(b + c)(c + a
(a + b + c) + (ab + bc + ca) = ab(a + b) + bc(b + c) + ca(c + a) + 3abc
ta a bt ng thc pha trn v dng:
(ab + bc + ca)(a + b + c) 9abc

1
a
+
1
b
+
1
c
9
Nhng y li l mt bt ng thc ng, do a + b + c = 1
Vy ta c iu phi chng minh. ng thc xy ra khi a = b = c =
1
3
.2
7.24 Cho a, b, c > 0 tha a + b + c = 9. Chng minh rng:
a
_
1 +
7
b
2
+ b
_
1 +
7
c
2
+ c
_
1 +
7
a
2

7

3
6
(

a +

b +

c) +
3
2
Li gii.
Ta c:
a
_
1 +
7
b
2
+ b
_
1 +
7
c
2
+ c
_
1 +
7
a
2
=
_
a
2
+
7a
2
b
2
+
_
b
2
+
7b
2
c
2
+
_
c
2
+
7c
2
a
2

_
(a + b + c)
2
+ 7(
a
b
+
b
c
+
c
a
)
2

81 + 7.9 = 12
7

3
6
(

a +

b +

c) +
3
2

7.

3.
_
3(a + b + c)
6
+
3
2
=
21
2
+
3
2
= 12
Ta c iu phi chng minh. ng thc xy ra khi a = b = c = 3.2
158
7.25 Cho a,b,c dng tha mn a + b + c = abc. Chng minh rng:
ab
c(1 + ab)
+
bc
a(1 + bc)
+
ca
b(1 + ca)

3

3
4
Li gii.
t a =
1
x
; b =
1
y
; c =
1
z
xy + yz + zx = 1
hay
x
yz + 1
+
y
zx + 1
+
z
xy + 1

3

3
4
Xt trng hp: x y z yz + 1 zx + 1 xy + 1
S dng bt ng thc Chebyshev c :
x
yz + 1
+
y
zx + 1
+
z
xy + 1

1
3
(x + y + z)(
1
yz + 1
+
1
zx + 1
+
1
xy + 1
)

3
4
(x + y + z)
3

3
4
Xt trng hp:x z y. Bt ng thc c chng minh tng t.
ng thc xy ra khi x = y = z =
1

3
hay a = b = c =

3.2
7.26 Cho a; b; c > 0 v a + b + c =

3. Chng minh rng:


1

a
2
+ 1
+
1

b
2
+ 1
+
1

c
2
+ 1

3
2
Li gii.
S dng phng php tip tuyn, ta s chng minh:
1

a
2
+ 1

3
8

3
8
a()
Tht vy
()

3(5

3a)

a
2
+ 1 8
9a
4
30

3a
3
+ 84a
2
30

3a + 11 0
(a
1

3
)
2
(9a
2
24

3a + 33) 0
Bt ng thc trn ng do a (0,

3).
Tng t vi cc bt ng thc cn li. Cng v vi v ta c iu phi chng minh.
ng thc xy ra khi a = b = c =
1

3
.2
7.27 Cho a; b; c > 0. Chng minh:
(a + b + c)
3
(a + b c)(b + c a)(c + a b) 27a
2
b
2
c
2
Li gii.
Li gii 1.
Khng mt tnh tng qut ta gi s b a c.
D thy ta ch cn chng minh bt ng thc thc khi a, b, c l di 3 cnh ca 1 tam gic. t
2 v ca bt ng thc ban u ln lt l VT (v tri) v VP (v phi).
p dng bt ng thc AM GM, ta c:
159
3V T = 3(a + b + c)(a + b c)(c + a b) (a + b + c)
2
(b + c a)

_
3(a + b + c)(a + b c)(c + a b) + (a + b + c)
2
(b + c a)
2
_
2
.
Nh vy ta cn chng minh:
3(a + b + c)(a + b c)(c + a b) + (a + b + c)
2
(b + c a)
2
9abc.
Rt gn mt cht:
3(a + b + c)(a + b c)(c + a b) + (a + b + c)
2
(b + c a) = 2(a + b + c)(a
2
b
2
c
2
+ 4bc)
Nh vy, ta cn chng minh:
(a + b + c)(a
2
b
2
c
2
+ 4bc) 9abc
Do iu gi s, ta c (a c)(a b) 0 hay a
2
+ bc a(b + c). Suy ra:
(a + b + c)(a
2
b
2
c
2
+ 4bc) (a + b + c)
_
ab + bc + ca b
2
c
2
+ 2bc
_
= (a + b + c)(ab + bc + ca) (b c)
2
(a + b + c)
Hn na ta c ng thc:
(a + b + c)(ab + bc + ca) 9abc = a(b c)
2
+ b(c a)
2
+ c(a b)
2
V vy, ta cn ch ra rng:
a(b c)
2
+ b(c a)
2
+ c(a b)
2
(b c)
2
(a + b + c)
iu ny ng do vi iu gi s b a c th:
(b c)
2
= max
_
(a b)
2
, (b c)
2
, (c a)
2
_
Vy ta c iu phi chng minh.
Li gii 2.
t: x = a + b c; y = a + c b; z = b + c a
Bt ng thc cn chng minh tng ng:
27(x + y)
2
(y + z)
2
(z + x)
2
64xyz(x + y + z)
3
M ta c:
81(x + y)
2
(y + z)
2
(z + x)
2
64(x + y + z)
2
(xy + yz + zx)
2
64.3(x + y + z)
3
xyz
y l iu phi chng minh.
Li gii 3.
Ta thy rng nu (b +c a)(c +a b)(a +b c) < 0 th bt ng thc hin nhin ng v v tri
ca n l s m, trong khi v phi li l mt s dng.
T l lun ny suy ra, ta ch cn xt trng hp (b + c a)(c + a b)(a + b c) 0 l .
Khi d thy b +c a > 0, c +a b > 0, a +b c > 0. By gi, ta nhn 27abc vo hai v ca
bt ng thc v vit li n nh sau:
27abc(a + b + c)
3
(b + c a)(c + a b)(a + b c) 9
3
a
3
b
3
c
3
hay
160
[27 a(b + c a) b(c + a b) c(a + b c)] (a + b + c)
3
9
3
a
3
b
3
c
3
.
S dng bt ng thc AM GM, ta c:
27 a(b + c a) b(c + a b) c(a + b c) [a(b + c a) + b(c + a b) + c(a + b c)]
3
.
Do ta ch cn chng minh c:
(a + b + c) [2(ab + bc + ca) a
2
b
2
c
2
] 9abc.
hay
a
2
+ b
2
+ c
2
+
9abc
a + b + c
2(ab + bc + ca)
Nhng y li chnh l bt ng thc Schur quen thuc. Ta c iu phi chng minh.
Li gii 4.
Khng mt tnh tng qut, gi s a b c.
Ta vit bt ng thc li nh sau:
(a + b + c)
3
[abc (a + b c)(b + c a)(c + a b)] + abc [27abc (a + b + c)
3
0
(a +b +c)
3
[(a b)
2
(a + b c) + c(a c)(b c)] abc(a +b +c)[(a b)
2
+ (a c)(b c)] 0
(a b)
2
.M + (a c)(b c).N 0
vi M = (a + b + c)
3
(a + b c) abc(a + b + c) v N = (a + b + c)
3
c abc(a + b + c).
Ta ch cn chng minh M, N u khng m l c. Tht vy, ta c:
M = (a + b + c)
3
(a + b c) abc(a + b + c) = (a + b + c)
_
(a + b + c)
2
(a + b c) abc

(a + b + c)(a
2
.a abc) 0.
N = (a + b + c)
3
.c abc(a + b + c) = c(a + b + c)
_
(a + b + c)
2
ab

= c(a + b + c)
_
a
2
+ b
2
+ c
2
+ 2(bc + ca) + ab

> 0.
Php chng minh hon tt.2
7.28 Cho a, b, c l cc s thc dng tha mn
1
a
2
+
1
b
2
+
1
c
2
= 1. Tm gi tr ln nht ca:
1

5a
2
+ 2ab + 2b
2
+
1

5c
2
+ 2bc + 2c
2
+
1

5c
2
+ 2ca + 2a
2
Li gii.
Ta c nh gi: 5a
2
+ 2ab + 2b
2
(2a + b)
2
. (Bt ng thc tng ng: (a b)
2
)
T nh gi trn , ta c:
P
1
2a + b
=
1
9
9
a + a + b

1
3
(
1
a
+
1
b
+
1
c
)
Mt khc, ta c:
1
a
+
1
b
+
1
c

_
3(
1
a
2
+
1
b
2
+
1
c
2
) =

3
V vy, P
1

3
.
161
ng thc xy ra khi a = b = c =
1

3
.2
7.29 Cho cc s thc a, b, c tha mn a
2
+ b
2
+ c
2
= 6v ab + bc + ac = 3. Tm gi tr ln
nht ca biu thc:
P = a
6
+ b
6
+ c
6
Li gii.
T gi thit d dng suy ra:
_

_
a + b + c = 0
a
2
b
2
+ b
2
c
2
+ c
2
a
2
= (ab + bc + ca)
2
= 9
a
4
+ b
4
+ c
4
= (a
2
+ b
2
+ c
2
)
2
2(a
2
b
2
+ b
2
c
2
+ c
2
a
2
) = 18
T , ta c:
a
6
+ b
6
+ c
6
3a
2
b
2
c
2
= (a
2
+ b
2
+ c
2
)(a
4
+ b
4
+ c
4
a
2
b
2
b
2
c
2
c
2
a
2
) = 54 .
Nh vy, ta ch cn tm cc tr ca P = a
2
b
2
c
2
.
D dng c P 0. Du bng xy ra khi a = 0, b =

3, c =

3.
Cng vic cn li ta cn tm gi tr ln nht ca P.
p dng Cauchy Schwarz, ta c:
6 = a
2
+ b
2
+ c
2
a
2
+
(b + c)
2
2
=
3a
2
2
.
Suy ra 2 a 2. Bng cch tng t ta cng c 2 b, c 2. Suy ra:
(a 2)(b 2)(c 2) 0 v (a + 2)(b + 2)(c + 2) 0
Khai trin v da vo cc ng thc c, ta c 2 abc 2 suy ra a
2
b
2
c
2
4.
Du bng xy ra khi a = 2, b = c = 1 hoc a = 2, b = c = 1.
n y bi ton c gii quyt.2
Nhn xt:
Ta c mt bi ton tng t nhng th v hn nh sau:
Cho a, b, c l cc s thc tha mn ng thi a +b +c = 0 v a
2
+b
2
+c
2
= 3. Chng minh rng:
a
5
b + b
5
c + c
5
a 3
7.30 Cho cc s khng m a, b, c tho mn a+b +c = 1. Tm gi tr nh nht ca biu thc:
P =
1
b
2
+ bc + c
2
+ 3a
+
1
c
2
+ ca + a
2
+ 3b
+
1
a
2
+ ab + b
2
+ 3c
Li gii.
S dng gi thit ta suy ra b
2
+ bc + c
2
+ 3a = (a + b)
2
+ (a + b)(a + c) + (a + c)
2
.
T t x = a + b, y = b + c, z = c + a, ta i tm gi tr nh nht ca biu thc
1
x
2
+ xy + y
2
+
1
y
2
+ yz + z
2
+
1
z
2
+ zx + x
2
vi x + y + z = 2.
Trn thc t ta c bt ng thc rt quen thuc sau v c l cng khng cn phi chng minh
li:
1
x
2
+ xy + y
2
+
1
y
2
+ yz + z
2
+
1
z
2
+ zx + x
2

9
(x + y + z)
2
.
Vy ta c P
9
4
. ng thc xy ra khi a = b = c =
1
3
.2
162
7.31 Cho cc s thc x; y; z 0 tho mn xy + yz + zx = 3.Chng minh bt ng thc:
1
xyz
+
4
(x + y)(y + z)(z + x)

3
2
Li gii.
S dng bt ng thc AM GM, ta c:
3 = xy + yz + xz 3
3
_
x
2
y
2
z
2
hay xyz 1
V (xz + yz)(xy + zx)(zy + xy)
_
xz + yz + xy + zx + zy + xy
3
_
3
= 8.
T 2 bt ng thc trn, ta c:
1
xyz
+
4
(x + y)(y + z)(z + x)
=
1
2xyz
+
1
2xyz
+
4
(x + y)(y + z)(z + x)

1
2
+ 2

2
(xz + yz)(yx + zx)(zy + xy)

3
2
Bt ng thc c chng minh. ng thc xy ra khi x = y = z = 1.2
7.32 Cho a, b, c, d tha mn:
_
a, b, c, d > 0
a
2
+ b
2
+ c
2
+ d
2
= 1
Chng minh rng : abcd (1 a)(1 b)(1 c)(1 d).
Li gii.
Ta i chng minh: (1 a)(1 b) cd.
Tht vy, bt ng thc trn tng ng vi:
1 a b + ab cd
a
2
+ b
2
+ c
2
+ d
2
2a 2b + 2ab 2cd + 1 0
(a + b 1)
2
+ (c d)
2
0
Tng t, ta c: (1 c)(1 d) ab. Nhn 2 v ca 2 bt ng thc va chng minh, ta c kt
lun.
ng thc xy ra khi a = b = c = d =
1
2
. 2
7.33 Cho a; b; c 0. Chng minh rng:
(
a
b + c
)
2
+ (
b
c + a
)
2
+ (
c
a + b
)
2
+
10abc
(a + b)(b + c)(c + a)
2
Li gii.
t x =
2a
b + c
; y =
2b
c + a
; z =
2c
a + b
.
T ng thc (a +b)(b +c)(c +a) = ab(a +b) +bc(b +c) +ca(c +a) +2abc, ta c ng thc sau :
xy + yz + zx + xyz =
4ab
(b + c)(a + c)
+
4bc
(c + a)(a + b)
+
4ca
(b + c)(a + b)
+
8abc
(a + b)(b + c)(c + a)
= 4
163
Bt ng thc cn chng minh tr thnh: x
2
+ y
2
+ z
2
+ 5xyz 8.
n y nu nh gi x
2
+ y
2
+ z
2
xy + yz + zx th tt nhin khng i ti kt qu. V vy, ta
ngh n mi quan h vi xyz, t ta s ngh n bt ng thc Schur (ch n du = xy
ra 2 im ) c dng:
x
2
+ y
2
+ z
2
+
9xyz
x + y + z
2(xy + yz + zx)
Mt khc, ta c:
4 = xy + yz + zx + xyz
t
2
3
+
t
33
27

1
27
((t) 3)(t + 6)
2
0
t 3
vi t = x + y + z.
T , ta vit li v tri bt ng thc cn chng minh di dng:
x
2
+ y
2
+ z
2
+ 5xyz = x
2
+ y
2
+ z
2
+
9
3
xyz + 2xyz
x
2
+ y
2
+ z
2
+
9xyz
x + y + z
+ 2xyz
2(xy + yz + zx + xyz) = 8.
ng thc xy ra khi x = y = z = 1 hoc x = 0; y = z = 2 v cc hon v .
Suy ra a = b = c hoc a = 0; b = c v cc hon v. 2
7.34 Chng minh rng vi mi a, b, c 0 ta u c :
a
3
+ b
3
+ c
3
3abc 2
_
b + c
2
a
_
3
Li gii.
t f(a, b, c) = a
3
+ b
3
+ c
3
3abc 2
_
b + c
2
a
_
3
. Ta c:
f(a, b, c) f
_
a,
b + c
2
,
b + c
2
_
=
3(b c)
2
(b + c)
4
+
3a(b c)
2
4
0
Do , f(a, b, c) f
_
a,
b + c
2
,
b + c
2
_
.
Mt khc f
_
a,
b + c
2
,
b + c
2
_
= 3a
_
a
b + c
2
_
2
0.
Vy, ta c iu phi chng minh. ng thc xy ra khi a = b = c.2
7.35 Cho a, b, c l cc s dng tho: a + b + c = 3. Tm gi tr nh nht ca:
F =
a
3
+ b
3
ab + 3
+
b
3
+ c
3
bc + 3
+
c
3
+ a
3
ca + 3
Li gii.
Ta c 2 bt ng thc sau:
a
3
x
+
b
3
y
+
c
3
z

(a + b + c)
3
3(x + y + z)
164
v ab + bc + ca
(a + b + c)
2
3
= 3
p dng bt ng thc trn, ta c:
F =
_
a
3
ab + 3
+
b
3
bc + 3
+
c
3
ca + 3
_
+
_
b
3
ab + 3
+
c
3
bc + 3
+
a
3
ca + 3
_

2(a + b + c)
3
3(ab + bc + ca + 9)

3
2
V vy, gi tr nh nht ca F =
3
2
.
Nhn xt:
T bi ton trn, ta a ra mt bt ng thc tng qut c nhiu ng dng:
Cho a
i
; b
i
(i = 1; 2; ...; k) l cc s thc dng v s nguyn n 2. Ta c bt ng thc sau:
a
n
1
b
1
+
a
n
2
b
2
+ ... +
a
n
k
b
k

(a
1
+ a
2
+ ... + a
k
)
n
k
n2
(b
1
+ b
2
+ ... + b
n
)
.
Bt ng thc ny c th chng minh trc tip bng bt ng thc Holder. Vic chng minh s
li cho bn c.
7. 36 Cho cc s thc dng a, b, c. Chng minh rng:
2 (a
2
+ 1) (b
2
+ 1) (c
2
+ 1) (abc + 1) (a + 1) (b + 1) (c + 1)
Li gii.
Nhn xt vi mi s thc dng a ta c (a
2
+ 1)
3
(a
3
+ 1) (a + 1)
3
. ng thc c khi a = 1.
Chng minh bt ng thc ny kh n gin, xin dnh cho bn c.
p dng cho b v c ta c
(b
2
+ 1)
3
(b
3
+ 1) (b + 1)
3
(c
2
+ 1)
3
(c
3
+ 1) (c + 1)
3
T suy ra V T (a + 1) (b + 1) (c + 1)
3
_
(a
3
+ 1) (b
3
+ 1) (c
3
+ 1)
Mt khc theo BT Hoder ta c
3
_
(a
3
+ 1) (b
3
+ 1) (c
3
+ 1) abc + 1.
vy, ta c V T V P. ng thc c khi a = b = c = 1.2
7.37 Cho cc s thc a, b, c, x, y, z tha mn (a + b + c) (x + y + z) = 3 v
(a
2
+ b
2
+ c
2
) (x
2
+ y
2
+ z
2
) = 4. Chng minh rng:
ax + by + cz 0
Li gii.
t t =
4

a
2
+ b
2
+ c
2
x
2
+ y
2
+ z
2
th:
a
2
+ b
2
+ c
2
= 2t
2
x
2
+ y
2
+ z
2
=
2
t
2
165
Bt ng thc cn chng minh tng ng vi:
_
a
t
+ tx
_
2
+
_
b
t
+ ty
_
2
+
_
c
t
+ tz
_
2
4
Nhng ta c:
_
a
t
+ tx
_
2
+
_
b
t
+ ty
_
2
+
_
c
t
+ tz
_
2

1
3
_
a + b + c
t
+ t (x + y + z)
_
2

4
3
(a + b + c) (x + y + z)
= 4.
Vy, bt ng thc c chng minh.2
7.38 Cho a, b, c l di 3 cnh mt tam gic. Chng minh rng:
abc [2

a
2
(b + c) 2

a
3
+ 3abc] (a + b + c) (b + c a) (c + a b) (a + b c) (ab + bc + ca)
Li gii.
p dng hng ng thc:
(a + b c) (c + a b) (b + c a) =

a
2
(b + c)

a
3
2abc
Suy ra bt ng thc trn tng ng vi
2abc

(b + c a) + 7a
2
b
2
c
2
(a + b + c)

(b + c a) (ab + bc + ca)
Tuy nhin, ta c mt s ng thc tam gic sau y:
16S
2
= (a + b + c)

(b + c a)
abc = 4SR
S = pr
T d dng a bt ng thc v dng:
4
r
R
+ 7 4

sin Asin B
4

cos A + 3

sin Asin B
4

cos Acos B 3
Nhng bt ng thc trn hin nhin ng v

cos A
3
2
.
Vy, ta c iu phi chng minh. ng thc xy ra khi a = b = c.2
7.39 Cho a, b, c l cc s nguyn khng m tha mn (a + b) (b + c) (c + a) = 2. Chng minh
rng:
(a
2
+ bc) (b
2
+ ca) (c
2
+ ab) 1
Li gii.
Khng mt tnh tng qut ta c th gi s c l s nm gia a v b, Khi (a c) (b c) 0.
Ta c:
(a + b)
2
[(b + c) (c + a)]
2
= 4
tng ng
(a + b)
2
(c
2
+ ab + ca + cb) = 4
Hn na,
(c
2
+ ab + bc + ca) 4 (c
2
+ ab) (bc + ca)
Do vy
(c
2
+ ab) (bc + ca) (a + b)
2
1
166
Tng ng
c (a + b)
3
(c
2
+ ab) 1
Vy ta ch cn chng minh
(a
2
+ bc) (b
2
+ ca) (c
2
+ ab) c (a + b)
3
(c
2
+ ab)
tng ng
ab [(a c) (b c) 2ac 2bc] 0
Vy ta c iu phi chng minh. ng thc c khi v ch khi a = 0, b = c = 1 v cc hon v.2
7.40 Cho a, b, c l cc s thc dng. Chng minh rng:
(a
2
+ 1)(b
2
+ 1)(c
2
+ 1)
5
16
(a + b + c + 1)
2
Li gii.
p dng bt ng thc Cauchy-Schwarz ta c:
(a + b + c + 1)
2
=
_
a.1 +
1

2
.

2(b + c) +
1

2
.

2
_
2
(a
2
+ 1)[3 + 2(b + c)
2
].
Khi ta cn chng minh bt ng thc sau:
5
16
[3 + 2(b + c)
2
] (b
2
+ 1)(c
2
+ 1)
Hay:
16b
2
c
2
+ 6(b
2
+ c
2
) + 1 20ab.
Bt ng thc trn hin nhin ng theo bt ng thc AM-GM:
16b
2
c
2
+ 1 8bc, 6(b
2
+ c
2
) 12bc
Du ng thc xy ra khi v ch khi a = b = c =
1
2
.2
167
3.8 Bi 8.1 n bi 8.40
8.1 Tm hng s k tt nht (ln nht) bt ng thc sau ng vi mi s thc a, b, c
khng m:
2

a
3
+ k (ab + bc + ca) (

a) ab (a + b) + bc (b + c) + ca (c + a) + k (a
2
+ b
2
+ c
2
) (a + b + c)
Li gii. Cho c = 0, a = b = t > 0 4t
3
+ 2kt
3
2t
3
+ 4kt
3
k 1 k
max
= 1
Khi k = 1 ta cn chng minh:
2

a
3
+ (a + b + c)(ab + bc + ca) ab(a + b) + bc(b + c) + ca(c + a) + (a + b + c)(a
2
+ b
2
+ c
2
)
tng ng
a
3
+ b
3
+ c
3
+ 3abc ab(a + b) + bc(b + c) + ca(c + a)
( ng theo bt ng thc Schur).2
8.2 Cho x, y, z > 0 tha mn x
2
+ y
2
+ z
2
1.Chng minh:
x
3
y
+
y
3
z
+
z
3
x
1
Li gii. Li gii 1. p dng bt ng thc Cauchy - Schwarz, ta c:
x
3
y
+
y
3
z
+
z
3
x

(x
2
+ y
2
+ z
2
)
2
xy + yz + zx
.
Mt khc, ta cng c x
2
+ y
2
+ z
2
xy + yz + zx v
2
+ y
2
+ z
2
1
theo iu kin nn ta suy ra:
x
3
y
+
y
3
z
+
z
3
x
1.
Php chng minh hon tt. 2
Li gii 2. S dng bt ng thc AM-GM ta c:
x
3
y
+
x
3
y
+ y
2
3x
2
Thit lp 2 biu thc tng t, sau cng v theo v, ta c:
2
_
x
3
y
+
y
3
z
+
z
3
x
_
+ x
2
+ y
2
+ z
2
3 (x
2
+ y
2
+ z
2
)
hay
x
3
y
+
y
3
z
+
z
3
x
x
2
+ y
2
+ z
2
1
Bi ton c chng minh.ng thc xy ra khi v ch khi x = y = z =
1

3
. 2
Li gii 3. S dng bt ng thc Holder ta c:
_
x
3
y
+
y
3
z
+
z
3
x
__
x
3
y
+
y
3
z
+
z
3
x
_
(y
2
+ z
2
+ x
2
) (x
2
+ y
2
+ z
2
)
3
Do :
x
3
y
+
y
3
z
+
z
3
x
x
2
+ y
2
+ z
2
1
Bi ton c chng minh.ng thc xy ra khi v ch khi x = y = z =
1

3
.2
8.3 Cho a, b, c > 0 tha mn a
3
+ b
3
+ c
3
= 3. Chng minh rng:
a
c
+
b
a
+
c
b
a + b + c
168
Li gii. T iu kin suy ra 1 abc.
Ta c:
a
c
+
a
c
+
c
b
3
3
_
a
2
bc
3a
Tng t:
b
a
+
b
a
+
a
c
3b
c
b
+
c
b
+
b
a
3c
Cng 3 bt ng thc trn, ta c iu phi chng minh.2
8.4 Cho a, b, c l ba s thc khng m , chng minh rng:
a
2
2a
2
+ bc
+
b
2
2b
2
+ ac
+
c
2
2c
2
+ ab
1
Li gii. Ta vit li:
2a
2
2a
2
+ bc
+
2b
2
2b
2
+ ac
+
2c
2
2c
2
+ ab
2
tng ng
1
2a
2
2a
2
+ bc
+ 1
2b
2
2b
2
+ ac
+ 1
2c
2
2c
2
+ ab
1
tng ng
bc
2a
2
+ bc
+
ac
2b
2
+ ac
+
ab
2c
2
+ ab
1
M theo CauchySchawrz, ta c:
bc
2a
2
+ bc
+
ac
2b
2
+ ac
+
ab
2c
2
+ ab

(bc + ac + ab)
2
a
2
b
2
+ c
2
b
2
+ a
2
c
2
+ 2abc(a + b + c)
= 1
Php chng minh hon tt.2
8.5 Cho x, y, z khng m tha mn x + y + z = 1. Tm gi tr ln nht ca
P = x
n
y + y
n
z + z
n
x, n 2
Li gii. Gi (a, b, c)l mt hon v ca b (x, y, z) sao cho a b c a
n1
b
n1
c
n1
; ab
ac bc
Ta c:
169
x
n
y + y
n
z + z
n
x = x
n1
xy + y
n1
yz + z
n1
zx
a
n1
ab + b
n1
ac + c
n1
bc
= b(a
n
+ acb
n2
+ c
n
)
b(a
n
+ a
n1
c + c
n
)
b(a + c)
n
= b(a + c)
n
=
1
n
nb(a + c)
n

1
n
_
n(a + b + c)
n + 1
_
n+1
=
n
n
(n + 1)
n+1
Du = xy ra khi(x, y, z) l hoan v ca
_
n
n + 1
;
1
n + 2
; 0
_
.2
8.6 Cho a, b, cl cc s thc tha mn 3a
2
+ 2b
2
+ c
2
6. Tm GTLN v GTNN ca biu
thc sau:
P = 2(a + b + c) abc
Li gii. p dng bt ng thc Cauchy-Schwarz:
[2(a + b + c) abc]
2
= [a(2 bc) + 2(b + c)]
2
(a
2
+ 2) [(2 bc)
2
+ 2(b + c)
2
] = (a
2
+ 2)(b
2
+ 2)(c
2
+ 2)
Li theo bt ng thc
(a
2
+ 2)(b
2
+ 2)(c
2
+ 2) =
1
6
(3a
2
+ 6)(2b
2
+ 4)(c
2
+ 2)
1
6
_
3a
2
+ 2b
2
+ c
2
+ 12
3
_
3
36
tng ng [2(a + b + c) abc]
2
36 6 2(a + b + c) abc 6
Du "=" xy ra khi v ch khi:
_
a(b + c) = 2 bc
3a
2
+ 6 = 2b
2
+ 4 = c
2
+ 2 = 6
tng ng
_
a = 0; b = 1; c = 2
a = 0; b = 1; c = 2
Vy:
Min(2(a + b + c) abc) = 6 a = 0; b = 1; c = 2
Max(2(a + b + c) abc) = 6 a = 0; b = 1; c = 2..2
8.7 Cho a, b, c > 0 tha mn a + b + c = 1.Chng minh rng:
1

ab
+
1

bc
+
1

ca
2 +
_
22 +
1
abc
Li gii. Nhn

abc cho mi v, ta c bt ng thc tng ng l:

a +

b +

c 2

abc +

22abc + 1.
Bnh phng hai v, ta c bt ng thc tng ng l
a + b + c + 2
_

ab +

bc +

ca
_
26abc + 1 + 4
_
abc(22abc + 1).
V a + b + c = 1 nn bt ng thc trn tng ng vi
170

ab +

bc +

ca 13abc + 2
_
abc(22abc + 1).
p dng AM-GM cho ba s dng a, b, c ta c ngay iu sau
abc
_
a + b + c
3
_
3
=
1
27
.
Nh vy, ta c
2
_
abc(22abc + 1) 2

abc
_
22
27
+ 1
_
=
14
3

3
(abc)
1/2
.
Ta cng c theo AM-GM

ab +

bc +

ca 3
3
_

ab

bc

ca = 3(abc)
1/3
.
Ni cch khc, ta cn ch ra rng
3(abc)
1/3
13abc +
14
3

3
(abc)
1/2
.
Hay tng ng vi (rt gn (abc)
1/3
hai v)
3 13(abc)
2/3
+
14
3

3
(abc)
1/6
.
V abc 127nn bt ng thc trn l mt bt ng thc ng, tht vy, ta c
13(abc)
2/3
+
14
3

3
(abc)
1/6
13
_
1
27
_
2/3
+
14
3

3
_
1
27
_
1/6
= 3.
Ta c iu phi chng minh. 2
8.8 (USA TST 2011)Cho a, b, c [0, 1] tha mn a + b, b + c, c + a > 1. chng minh:
1 (1 a)
2
+ (1 b)
2
+ (1 c)
2
+
2

2abc

a
2
+ b
2
+ c
2
Li gii. Vit li bt ng thc nh sau:
a
2
+ b
2
+ c
2
+ 2 +
2

2abc

a
2
+ b
2
+ c
2
2(a + b + c).
p dng AM-GM, ta c
2(a + b + c)
(a + b + c)
2
2
+ 2.
Nn ta ch cn chng minh c
a
2
+ b
2
+ c
2
+
2

2abc

a
2
+ b
2
+ c
2

(a + b + c)
2
2
,
Hay
4

2abc

a
2
+ b
2
+ c
2
2(ab + bc + ca) (a
2
+ b
2
+ c
2
).
171
Bt ng thc cho l thun nht, do ta c th chun ho a
2
+ b
2
+ c
2
= 2.
Bt ng thc tr thnh:
4abc 2(ab + bc + ca) 2
Hay
2abc + 1 ab + bc + ca
Gi s c 1 s ln hn 1, chng hn l a, ta c bt ng thc trn tng ng:
4bc(a 1) + (a b c)
2
0
Xt trng hp ngc li, bt ng thc tng ng:
(1 a)(1 bc) + a(1 b)(1 c) 0
Vy bt ng thc c chng minh.2
8.9 Cho 3 s a, b, c tha mn iu kin a + b + c + abc = 0; a, b [1 1]. Chng minh rng:

a + 1 +

b + 1 +

c + 1 3
Li gii. Khng mt tnh tng qut, c th gi s ab 0 theo nguyn l Dirichlet.
Khi , t gi thit ta c:
(1 + c) (1 + c)(1 + ab) = (1 a)(1 b)

1 + c
_
(1 a)(1 b)
p dng bt ng thc Cauchy-Schwarz:
_
(1 a)(1 b) +

1 + a +

1 + b
_
(1 a + 1 + a + 1)(1 b + 1 + 1 + b) = 3
Bi ton c chng minh.2
8.10 Cho a, b, c l cc s dng. Chng minh:
_
a + 2b
3
+
_
b + 2c
3
+
_
c + 2a
3

a +

b +

c
Li gii. Bnh phng hai v, bt ng thc cn chng minh tng ng vi:

_
(a + 2b)(b + 2c) 3

ab
p dng bt ng thcCauchySchawrz, ta c:
_
(a + 2b)(b + 2c)
_
(

ab + 2

bc)
2
=

ab + 2

bc
Lp cc bt ng thc tng t ri cng v theo v, ta c bt ng thc cn chng minh. 2
8.11 Cho cc s thc dng x, y, z tha mn xy + yz + zx = 3. Chng minh rng:
1
xyz
+
4
(x + y)(y + z)(z + x)

3
2
Li gii.
Li gii 1. Trc tin, ta d dng c xyz 1..p dng AM-GM lin tc, ta s c
172
1
xyz
+
4
(x + y)(y + z)(z + x)
=
1
2xyz
+
_
1
2xyz
+
4
(x + y)(y + z)(z + x)
_

1
2xyz
+
2

2
_
xyz(x + y)(y + z)(z + x)
=
1
2xyz
+
2

2
_
(xy + xz) (yz + yx) (zx + zy)

1
2
+
2

_
xy + xz + yz + yx + zx + zy
3
_
3
=
3
2
.
Ta c iu phi chng minh. 2
Li gii 2. D thy xyz 1.S dng AM-GM ta c:
4xyz
(xy + xz)(yz + yx)(zx + zy)

4xyz
_
xy + xz + yz + yx + zx + zy
3
_
3
=
xyz
2
Nh vy ch cn chng minh:
1
xyz
+
xyz
2

3
2
.
Nhng BT ny ng v:
V T =
1
2xyz
+
_
1
2xyz
+
xyz
2
_

1
2
+ 1 =
3
2
.True
Php chng minh hon tt.2
8.12 Cho a, b, c > 0. Chng minh rng:
a
_
b
2
+
bc
4
+ c
2
2
Li gii. Bt ng thc tng ng vi
a

4b
2
+ bc + 4c
2
+
b

4c
2
+ ca + 4a
2
+
c

4a
2
+ ab + 4b
2
1.
p dng lin tip bt ng thc Cauchy Schwarz, ta c

4b
2
+ bc + 4c
2

(a + b + c)
2

4b
2
+ bc + 4c
2

(a + b + c)
2
_
(

a) [

a(4b
2
+ bc + 4c
2
)]
.
Ni tm li, ta ch cn chng minh rng:
(a + b + c)
3

a(4b
2
+ bc + 4c
2
).
Khai trin ra, ta s c ngay bt ng thc
a
3
+ b
3
+ c
3
+ 3abc ab(a + b) + bc(b + c) + ca(c + a).
y chnh l bt ng thc Schur. Ta c iu phi chng minh. 2
173
8.13 Cho 3 s khng m a, b, c tha mn a + b + c = 1. Chng minh rng:
a
2
+ b
2
+ c
2
+

12abc 1
Li gii. Li gii 1. Bt ng thc tng ng
(a + b + c)
2
2(ab + bc + ca) +

12abc 1
tng ng ab + bc + ca

3abc
tng ng (ab + bc + ca)
2
3abc(a + b + c)
tng ng (ab bc)
2
+ (bc ca)
2
+ (ca ab)
2
0 ng
Li gii 2. t x =
_
bc
a
, y =
_
ca
b
, z =
_
ab
c
Khi a = yz, b = zx, c = xy v xy + yz + zx = 1
Bt ng thc tng ng
x
2
y
2
+ y
2
z
2
+ z
2
x
2
+ 2

3xyz 1 x + y + z

3()
T iu kin ta thy tn ti tam gic ABC nhn sao cho :
x = tan
A
2
, y = tan
B
2
, z = tan
C
2
() tan
A
2
+ tan
B
2
+ tan
C
2

3
Bt ng thc trn hin nhin ng, ta c iu phi chng minh.2
8.14 Cho a, b, c l 3 s thc khng ng thi bng 0. Chng minh rng:

3a + b

a
2
+ 2b
2
+ c
2

6
Li gii. p dng Cauchy Schwarz, ta c ngay

3a + b

a
2
+ 2b
2
+ c
2

_
3
_

(3a + b)
2
a
2
+ 2b
2
+ c
2
_
.
Tip tc dng Cauchy Schwarz, ta s ngay iu phi chng minh

(3a + b)
2
a
2
+ 2b
2
+ c
2

_
9a
2
a
2
+ b
2
+ c
2
+
b
2
b
2
_
= 12.
Nn

3a + b

a
2
+ 2b
2
+ c
2

_
3
_

(3a + b)
2
a
2
+ 2b
2
+ c
2
_

3 12 = 6.
.2
8.15 (IRan 2011)Tm s thc k nh nht sao cho vi mi s thc x, y, z khng m ta c bt
ng thc:
x

y + y

z + z

x k
_
(x + y)(y + z)(z + x)
Li gii. Cho x = y = z ta c gi tr nh nht ca k l
3

2
4
.Ta i chng minh gi tr ca k
l ng
174
S dng bt ng thc Cauchy Schwarz :
V T
2
= (

xy +

yz +

zx)
2
(x + y + z)(xy + yz + zx)
Ta ch cn i chng minh
8(x + y + z)(xy + yz + zx) 9(x + y)(y + z)(z + x)True
Ta c iu phi chng minh .ng thc xy ra khi v ch khi x = y = z.2
8.16 Cho a; b; c > 0 tha mn a + b + c = 1. Tm gi tr ln nht ca biu thc:
_
ab
ab + c
+
_
bc
bc + a
+
_
ca
ca + b
Li gii. p dng bt ng thc AM-GM,Ta c:
_
ab
ab + c
=
_
ab
ab + c(a + b + c)
=
_
ab
(c + a)(c + b)

1
2
(
a
c + a
+
b
a + c
)
Suy ra:
_
ab
ab + c
+
_
bc
bc + a
+
_
ca
ca + b

1
2
(
a
c + a
+
b
c + b
+
a
b + a
+
c
b + c
+
b
a + b
+
c
b + c
) =
3
2
.2
8.17 Cho a, b, c l cc s thc khng m tha mn a + b + c = 1. Tm gi tr ln nht ca
biu thc
P =
1 + a
2
1 + b
2
+
1 + b
2
1 + c
2
+
1 + c
2
1 + a
2
Li gii. Trc ht t gi thit, ta suy ra 0 a, b, c 1, t c a
2
, b
2
, c
2
1.
Ta vit li biu thc P nh sau:
P =

1
1 + a
2
+

a
2
1 + b
2
,
t kt hp vi hai nh gi sau:
1
a
2

1 + 1 a
2
1 + a
2
+ 1 a
2
=
2 a
2
2
,
a
2
1 + b
2
a
2
,
ta suy ra
P

2 a
2
2
+

a
2
= 3 +
1
2

a
2
.
Mt khc, rng do a, b, c [0, 1] nn a
2
+ b
2
+ c
2
a + b + c = 1, do vy m
P 3 +
1
2
=
7
2
.
Cui cng vi a = b = 0 v c = 1 (tho mn iu kin) th P =
7
2
nn ta kt lun
7
2
l gi tr ln
nht ca biu thc P.
Bi ton kt thc.2
8.18 Cho a, b, c l cc s thc dng tha mn a +

b +
4

c = 3. Chng minh rng


A =
_
1 + a
4
+ (a b)
2
+
_
1 + b
2
+ (b c)
2
+
_
1 + c + (c a)
2
3

2
175
Li gii 1. p dng bt ng thc Minkowski, ta c
A
_
(

1 + a
4
+

1 + b
2
+

1 + c)
2
+ (|a b| +|b c| +|c a|)
2

_
9 + (a
2
+ b +

c)
2
n y ta s dng nh gi c bn sau vi mi ba s thc dng x, y, z
x
2
+ y
2
+ z
2

(x + y + z)
2
3
()
c
A

9 +
(a +

b +
4

c)
4
9
=

9 + 9 = 3

2.
Php chng minh hon tt.2
Li gii 2. T biu din ca A ta suy ra
A

1 + a
4
+

1 + b
2
+

1 + c.
n y ta s dng nh gi c bn sau vi mi hai s thc dng x, y
x
2
+ y
2

(x + y)
2
2
c
A
1 + a
2

2
+
1 + b

2
+
1 +

2
=
3 + a
2
+ b +

2
.
Tip tc s dng nh gi () trn, ta suy ra
A
3 +
(a +

b +
4

c)
2
3

2
= 3

2.
Php chng minh hon tt.2
8.19 Cho a, b, c, d l cc s thc khng m. Chng minh rng
a + b + c + d
4

_
ab + ac + ad + bc + bd + cd
6

3
_
abc + abd + acd + bcd
4

4

abcd
Li gii.
1. Chng minh
a + b + c + d
4

_
ab + ac + ad + bc + bd + cd
6
.
rng ta c ng thc sau:
2(ab + ac + ad + bc + bd + cd) = (a + b)(c + d) + (a + c)(b + d) + (a + d)(b + c).
T y ta p dng nh gi c bn xy
(x + y)
2
4
c
2(ab + ac + ad + bc + bd + cd)
3(a + b + c + d)
2
4
,
v do ta c bt ng thc cn chng minh.
176
2. Chng minh
3
_
abc + abd + acd + bcd
4

4

abcd.
S dng bt ng thc AM-GM, ta c ngay iu phi chng minh:
3
_
abc + abd + acd + bcd
4

3
_
4

a
3
b
3
c
3
d
3
=
4

abcd.
3. Chng minh
_
ab + ac + ad + bc + bd + cd
6

3
_
abc + abd + acd + bcd
4
.
Bt ng thc cn chng minh tng ng vi
(abc + bcd + cda + dab)
2

2(ab + ac + ad + bc + bd + cd)
3
27
.
Bt ng thc ny mang tnh i xng gia cc bin, nn khng mt tnh tng qut, ta gi
s a b c d. Vi iu kin , p dng bt ng thc Cauchy-Schwarz, ta c
(abc + bcd + cda + dab)
2
(ab + bc + cd + da)(abc
2
+ bcd
2
+ cda
2
+ dab
2
)
= (ab + bc + cd + da)[ac(bc + da) + bd(ab + cd)]. ()
Mt khc, t a b c d ta suy ra ac bd v
bc + da (ab + cd) = (a c)(b d) 0,
hay bc + da ab + cd. Do vy theo bt ng thc Chebyshev, ta c
ac(bc + da) + bd(ad + cd)
1
2
(ac + bd)(bc + da + ab + cd). ()
Kt hp hai nh gi () v (), ta c
(abc + bcd + cda + dab)
2

1
2
(ac + bd)(bc + da + ab + cd)
2
=
1
4
.2(ac + bd)(bc + da + ab + cd)(bc + da + ab + cd).
n y ta p dng bt ng thc AM-GM c
(abc + bcd + cda + dab)
2

1
4
_
2(ac + bd) + 2(ab + bc + cd + da)
3
_
3
=
2(ab + ac + ad + bc + bd + cd)
3
27
.
Php chng minh hon tt.2
8.20 Cho a, b, c l di ba cnh ca mt tam gic. Chng minh rng
a

a
2
+ 3bc
+
b

b
2
+ 3ca
+
c

c
2
+ 3ab

9
4
Li gii. Trc ht, do bt ng thc cn chng minh mang tnh i xng gia cc bin, nn
177
khng mt tnh tng qut, ta gi s c = max{a, b, c}.
p dng bt ng thc Cauchy - Schwarz, ta c
a

a
2
+ 3bc
+
b

b
2
+ 3ca
+
c

c
2
+ 3ab

(a + b + c)
2
a

a
2
+ 3bc + b

b
2
+ 3ca + c

c
2
+ 3ab
.
Mt khc cng theo bt ng thc Cauchy - Schwarz th
a

a
2
+ 3bc + b

b
2
+ 3ca + c

c
2
+ 3ab =

a
3
+ 3abc +

b
3
+ 3abc +

c
3
+ 3abc

_
(a + b + c)(a
3
+ b
3
+ c
3
+ 9abc),
do ta suy ra
a

a
2
+ 3bc
+
b

b
2
+ 3ca
+
c

c
2
+ 3ab

(a + b + c)
2
_
(a + b + c)(a
3
+ b
3
+ c
3
+ 9abc)
.
Nh vy kt thc chng minh, ta cn ch ra rng
(a + b + c)
3
a
3
+ b
3
+ c
3
+ 9abc

9
4
.
Thc hin php bin i tng ng ta thu c
12[ab(a + b) + bc(b + c) + ca(c + a)] 5(a
3
+ b
3
+ c
3
) + 57abc.
rng theo bt ng thc AM-GM th a
3
+ b
3
+ c
3
3abc, do vy
6(a
3
+ b
3
+ c
3
+ 9abc) 5(a
3
+ b
3
+ c
3
) + 57abc.
Do vy ta ch cn chng minh
2[ab(a + b) + bc(b + c) + ca(c + a)] a
3
+ b
3
+ c
3
+ 9abc,
hay (3cab)(ab)
2
+(a+bc)(ca)(cb) 0. Tuy nhin nh gi ny ng do c = max{a, b, c}
v do a, b, c l di ba cnh ca mt tam gic. Do vy bt ng thc ban u c chng minh.
Bi ton kt thc.2
8.21 Cho a, b, c l cc s thc dng tho mn a
2
+ b
2
+ c
2
= 3. Chng minh rng
a

b
+
b

c
+
c

a
a + b + c
Li gii. p dng bt ng thc Cauchy - Schwarz, ta c
a

b
+
b

c
+
c

a

(a + b + c)
2
a

b + b

c + c

a
.
Nh vy kt thc chng minh, ta cn ch ra rng
a

b + b

c + c

a a + b + c.
178
Bnh phng hai v v ch rng theo bt ng thc Cauchy - Schwarz:
(a

b + b

c + c

a)
2
(ab + bc + ca)(a + b + c),
ta cn chng minh
a + b + c ab + bc + ca.
Tip tc bnh phng hai v v ti nh gi c bn (a + b + c)
2
3(ab + bc + ca), ta cn
phi kim tra nh gi sau
ab + bc + ca 3.
Tuy nhin nh gi trn ng do ab +bc +ca a
2
+b
2
+c
2
= 3. Do vy bt ng thc ban u
c chng minh xong.
Bi ton kt thc.2
8.22 Cho x, y l cc s thc dng tho mn xy +x +y = 3. Tm gi tr ln nht ca biu
thc
P =
3x
y + 1
+
3y
x + 1
+
xy
x + y
x
2
y
2
Li gii. T gi thit, ta suy ra x =
3 y
y + 1
. Th vo P, khai trin v rt gn, ta c
P =
(y 3) y (y
4
+ 2y
3
+ 12y
2
+ 14y + 19)
4 (y + 1)
2
(y
2
+ 3)
=
(y 3) y
_
(y 1)
4
+ 6 (y + 1) (y
2
+ 3)

4 (y + 1)
2
(y
2
+ 3)
.
T , ta s dng nh gi hin nhin (y 1)
4
0 c
P
(y 3) y.6 (y + 1) (y
2
+ 3)
4 (y + 1)
2
(y
2
+ 3)
=
3
2
_
y (y 3)
y + 1
_
.
n y ch rng
y (y 3)
y + 1
1 =
(y 1)
2
y + 1
0,
do vy ta suy ra P
3
2
.
Cui cng vi x = 1 v y = 1 (tho mn iu kin) th P =
3
2
nn ta suy ra
3
2
l gi tr ln nht
ca biu thc P.
Bi ton kt thc.2
8.23 Cho a, b, c l cc s thc thay i trong on [0, 1]. Tm gi tr ln nht ca biu thc
P =
a
1 + bc
+
b
1 + ac
+
c
1 + ab
abc
Li gii. Ch rng t gi thit ta c abc 0 v
a
1 + bc

a
1 + abc

2a
2 + abc
,
179
do vy ta suy ra
P
2(a + b + c)
2 + abc
.
n y tr 2 vo mi v c
P 2
2(a + b + c abc 2)
2 + abc
=
2[(1 a)(1 b) + (1 c)(1 ab)]
2 + abc
,
nhng rng (1 a)(1 b) + (1 c)(1 ab) 0 do a, b, c [0, 1]. Do vy ta suy ra
P 2 0,
hay P 2.
Cui cng vi a = b = 1 v c = 0 (tho mn iu kin) th P = 2 nn ta kt lun 2 l gi tr ln
nht ca biu thc P.
Bi ton kt thc.2
8.24 Cho a, b, c l cc s thc thay i bt k. Chng minh rng
(a
2
+ 1)(b
2
+ 1)(c
2
+ 1) (ab + bc + ca 1)
2
Li gii. rng ta c ng thc sau
(b
2
+ 1)(c
2
+ 1) (b + c)
2
+ (bc 1)
2
,
do vy (a
2
+ 1)(b
2
+ 1)(c
2
+ 1) = (a
2
+ 1)[(b +c)
2
+ (bc 1)
2
]. n y ta p dng bt ng thc
Cauchy - Schwarz c
(a
2
+ 1)(b
2
+ 1)(c
2
+ 1) [a(b + c) + bc 1]
2
= (ab + bc + ca 1)
2
.
Php chng minh hon tt.2
8.25 Cho x, y, z l cc s thc thay i trong khong (0; 1]. Chng minh rng
1
1 + xy
+
1
1 + yz
+
1
1 + xz

5
x + y + z
Li gii. Bt ng thc cn chng minh mang tnh i xng gia cc bin, do khng mt tnh
tng qut, ta gi s 0 < z y x 1. Khi :
1 + yz 1 + zx 1 + xy.
Do vy ta c nh gi
1
1 + xy
+
1
1 + yz
+
1
1 + xz

x + y + z
1 + yz
.
Mt khc rng
x + y + z
1 + yz
=
x 1 (y 1)(z 1) yz
1 + yz
+ 2,
180
nn t gi thit ta suy ra
x + y + z
1 + yz
2. Nh vy
1
1 + xy
+
1
1 + yz
+
1
1 + xz
2. ()
Hn na, ta cng c ng thc
_
1
x + y
1 + xy
_
+
_
1
y + z
1 + yz
_
+
_
1
z + x
1 + zx
_
=
(1 x)(1 y)
1 + xy
+
(1 y)(1 z)
1 + yz
+
(1 z)(1 x)
1 + zx
,
t y ta kt hp vi gi thit c
x + y
1 + xy
+
y + z
1 + yz
+
z + x
1 + zx
3. ()
Cng v theo v hai nh gi () v (), ta c
x + y + z
1 + xy
+
x + y + z
1 + yz
+
x + y + z
1 + xz
5,
v do ta thu c bt ng thc ban u.
Php chng minh hon tt.2
8.26 Cho x, y l cc s thc dng tho mn x +y 4. Tm gi tr nh nht ca biu thc
P =
3x
2
+ 4
4x
+
2 + y
3
y
2
Li gii.
Ta vit li biu thc P nh sau:
P =
2x
2
+ (x
2
+ 4)
4x
+
2y
3
+ (y
3
+ y
3
+ 8)
4y
2
.
T y ta p dng bt ng thc AM-GM c x
2
+ 4 4x v y
3
+ y
3
+ 8 6y
2
, do vy
P
2x
2
+ 4x
4x
+
2y
3
+ 6y
2
4y
2
=
x + y
2
+
5
2
.
n y ta s dng gi thit x + y 4 c P
9
2
.
Cui cng, vi x = 2 v y = 2 (tho mn iu kin) th P =
9
2
nn ta kt lun
9
2
l gi tr nh
nht ca biu thc P.
Bi ton kt thc.2
8.27 Cho a, b, c l cc s thc dng tho mn
1

a
+
1

b
+
1

c
= 2. Chng minh rng
1

a + 3b
+
1

b + 3c
+
1

c + 3a
1
181
Li gii. p dng bt ng thc Cauchy - Schwarz, ta c
(
9
4
+
27
4
)(a + 3b) (
3
2

a +
9
2

b)
2
,
t suy ra

a + 3b

a + 3

b
2
. Thit lp hai nh gi tng t v cng li, ta c
1

a + 3b
+
1

b + 3c
+
1

c + 3a

a + 3

b
+
2

a + 3

b
+
2

a + 3

b
. ()
Li p dng bt ng thc Cauchy - Schwarz, ta c nh gi
1

a
+
1

b
+
1

b
+
1

16

a + 3

b
,
hay
1

a
+
3

16

a + 3

b
. Cng nh gi ny vi hai nh gi tng t khc, ta suy ra
1

a
+
1

b
+
1

c

_
1

a + 3

b
+
1

b + 3

c
+
1

c + 3

a
_
. ()
Kt hp hai nh gi () v () cho ta
1

a + 3b
+
1

b + 3c
+
1

c + 3a

1
2
_
1

a
+
1

b
+
1

c
_
= 1.
Php chng minh hon tt.2
8.28 Cho x, y, z l cc s thc khng m thay i bt k. Chng minh rng vi mi s thc
a, b, c thay i tho mn 0 a b c, ta lun c
(ax + by + cz)(
x
a
+
y
b
+
z
c
)
(a + c)
2
4ac
(x + y + z)
2
Li gii. Bt ng thc cn chng minh tng ng vi
4(ax + by + cz)(cx +
acy
b
+ az) [(a + c)(x + y + z)]
2
.
p dng nh gi c bn 4uv (u + v)
2
, ta c
4(ax + by + cz)(cx +
acy
b
+ az) (ax + by + cz + cx +
acy
b
+ az)
2
.
Nh vy kt thc chng minh, ta cn ch ra rng
ax + by + cz + cx +
acy
b
+ az (a + c)(x + y + z).
Thc hin php bin i tng ng, ta thu c y(c b)(b a) 0. Tuy nhin y l nh gi
ng do 0 a b c. Do vy bt ng thc ban u c chng minh.
Bi ton kt thc.2
182
8.29 Cho a, b, c l cc s thc dng thay i bt k. Chng minh rng
a
2
(a + c)
2
+
b
2
(b + a)
2
+
c
2
(c + b)
2

3
4
Li gii. t x =
c
a
, y =
a
b
, z =
b
c
. Khi x, y, z > 0, xyz = 1 v bt ng thc cn chng minh
tng ng vi
_
1
1 + x
_
2
+
_
1
1 + y
_
2
+
_
1
1 + z
_
2

3
4
.
rng ta c ng thc sau
_
1
1 + x
_
2
+
_
1
1 + y
_
2

1
1 + xy
=
xy(x y)
2
+ (1 xy)
2
(1 + x
2
)(1 + y
2
)(1 + xy)
,
t ta s dng gi thit ca x, y, z c
_
1
1 + x
_
2
+
_
1
1 + y
_
2

1
1 + xy
=
z
z + 1
.
Nh vy kt thc chng minh, ta cn ch ra rng
z
z + 1
+
1
(1 + z)
2

3
4
.
Thc hin php bin i tng ng, ta thu c mt nh gi hin nhin ng
(z 1)
2
0,
do vy bt ng thc ban u c chng minh.
Bi ton kt thc.2
8.30 Cho x, y, z l cc s thc dng tho mn xyz = 1. Chng minh rng
1
(1 + x)
3
+
1
(1 + y)
3
+
1
(1 + z)
3

3
8
Li gii 1. Trc ht ta s chng minh bt ng thc sau:
1
(1 + x)
3
+
1
(1 + y)
3
+
1
(1 + z)
3
+
5
(x + 1)(y + 1)(z + 1)
1. ()
t a =
1
x + 1
; b =
1
y + 1
; c =
1
z + 1
; m = x + y + z; n = xy + yz + zx. Khi a, b, c, m, n > 0 v
t gi thit xyz = 1, ta suy ra
abc = (1 a)(1 b)(1 c),
hay 2abc = n m + 1. Bn cnh rng ta cng c ng thc sau:
a
3
+ b
3
+ c
3
3abc = m
3
3mn.
Tr li vic chng minh bt ng thc (), ta thy rng n tng ng vi mi bt ng thc
trong dy sau:
a
3
+ b
3
+ c
3
+ 5abc 1,
183
a
3
+ b
3
+ c
3
3abc + 8abc 1,
m
3
3mn + 4(n m + 1) 1,
m
3
4m + 3 n(3m4). ()
chng minh nh gi (), ta xt cc trng hp ca m nh sau:
Trng hp 1. m < 1.
Khi m
3
4m + 3 = (1 m)(3 mm
2
) 0 > n(3m4).
Trng hp 2. 1 < m <
4
3
.
Khi rng do 2abc = n m + 1 nn n > m1 > 0, do vy
m
3
4m + 3 n(3m4) > m
3
4m + 3 + (4 3m)(m1) = (m1)
3
> 0.
Trng hp 3. m
4
3
.
Khi t nh gi c bn
(x + y + z)
2
3(xy + yz + zx),
ta suy ra m
2
3n. Do vy
m
3
4m + 3 (3m4)n m
3
4m + 3
m
2
(3m4)
3
=
(2m3)
2
3
0.
Tm li, trong mi trng hp, nh gi () lun ng. T bt ng thc () c chng
minh xong.
n y ta s dng nh gi (x + y)(y + z)(z + x) 8xyz = 8 c
1
(1 + x)
3
+
1
(1 + y)
3
+
1
(1 + z)
3
+
5
8
1,
v do vy ta chng minh c bt ng thc ban u.
Bi ton kt thc.2
Li gii 2. p dng bt ng thc Holder, ta c
3
_
1
(1 + x)
3
+
1
(1 + y)
3
+
1
(1 + z)
3
_
2

_
1
(1 + x)
2
+
1
(1 + y)
2
+
1
(1 + z)
2
_
3
.
Mt khc theo mt kt qu chng minh bi 8.13 th
1
(1 + x)
2
+
1
(1 + y)
2
+
1
(1 + z)
2

3
4
,
do vy ta suy ra
3
_
1
(1 + x)
3
+
1
(1 + y)
3
+
1
(1 + z)
3
_
2

27
64
,
184
tng ng
1
(1 + x)
3
+
1
(1 + y)
3
+
1
(1 + z)
3

3
8
.
Php chng minh hon tt.2
8.31 Cho x, y, z l cc s thc tho mn 2x
2
+ y
2
+ xy 1. Tm gi tr nh nht ca biu
thc
P = x
2
+ y
2
Li gii. Trc ht ta t
Q =
x
2
+ y
2
2x
2
+ y
2
+ xy
.
Vi y = 0 th Q =
1
2
, t P = Q(2x
2
+ y
2
+ xy)
1
2
.
Vi y = 0, ta chia c t v mu ca biu thc Q cho y
2
c
Q =
_
x
y
_
2
+ 1
2
_
x
y
_
2
+
x
y
+ 1
=
t
2
+ 1
2t
2
+ t + 1
,
trong t =
x
y
. T y ta thc hin php bin i tng ng thu c
(2Q1)t
2
+ Qt + Q1 = 0.
Xem biu thc trn l phng trnh theo n t. phng trnh ny c nghim, ta cn c
= Q
2
+ 12Q4 0,
t suy ra
6 2

2
7
Q
6 + 2

2
7
. Do vy
P = Q(2x
2
+ y
2
+ xy)
6 2

2
7
.
So snh
1
2
v
6 2

2
7
, ta suy ra P
6 2

2
7
.
Cui cng, vi x =
_
4 +

2
14
v y =
_
8 5

2
14
(tho mn iu kin) th P =
6 2

2
7
nn ta
kt lun
6 2

2
7
l gi tr nh nht ca biu thc P.
Bi ton kt thc.2
8.32 Cho a, b, c l cc s thc dng thay i bt k. Chng minh rng
1
a
2
+ bc
+
1
b
2
+ ac
+
1
c
2
+ ab

a + b + c
2abc
185
Li gii. p dng bt ng thc AM-GM, ta c
a
2
+ bc 2a

bc.
Nh vy
1
a
2
+ bc

1
2a

bc
. Thit lp hai bt ng thc tng t v cng li, ta c
1
a
2
+ bc
+
1
b
2
+ ac
+
1
c
2
+ ab

1
2a

bc
+
1
2b

ca
+
1
2c

ab
=

bc +

ca +

ab
2abc
.
Cui cng, ta ch cn chng minh

bc +

ca +

ab a + b + c,
tuy nhin y li l mt nh gi c bn.
Php chng minh hon tt.2
8.33 Cho a, b, c l cc s thc dng tho mn a
2
+ b
2
+ c
2

3
4
. Tm gi tr nh nht ca
biu thc
P = (a + b)(b + c)(c + a) +
1
a
2
+
1
b
2
+
1
c
2
Li gii. p dng bt ng thc AM-GM, ta c
1
a
2
+ 4
4
a
,
t ta suy ra
1
a
2

4
a
4. Cng nh gi ny vi hai nh gi tng t khc, ng thi lu
rng ta c nh gi c bn sau:
(a + b)(b + c)(c + a) 8abc,
ta c
P 8abc +
4
a
+
4
b
+
4
c
12 =
_
8abc +
1
2a
+
1
2b
+
1
2c
_
+
7
2
_
1
a
+
1
b
+
1
c
_
.
n y ta p dng bt ng thc AM-GM, ta c
8abc +
1
2a
+
1
2b
+
1
2c
4.
ng thi ta cng c nh gi
1
a
+
1
b
+
1
c

9
a + b + c
. T y
P 4 +
63
2(a + b + c)
.
Li lu rng (a + b + c)
2
3(a
2
+ b
2
+ c
2
)
9
4
nn a + b + c
3
2
, do vy
P 4 +
63
3
= 25.
186
Cui cng vi a = b = c =
1
2
(tho mn iu kin) th P = 25 nn ta kt lun 25 l gi tr nh
nht ca biu thc P.
Bi ton kt thc.2
8.34 Cho a, b, c l cc s thc dng thay i bt k. Chng minh rng
(a b c)
2
2a
2
+ (b + c)
2
+
(b c a)
2
2b
2
+ (c + a)
2
+
(c a b)
2
2c
2
+ (a + b)
2

1
2
Li gii 1. Bt ng thc cn chng minh mang tnh thun nht nn ta chun ha a+b +c = 3.
Khi bt ng thc c vit li thnh
(3 2a)
2
3a
2
6a + 9
+
(3 2b)
2
3b
2
6b + 9
+
(3 2c)
2
3c
2
6c + 9

1
2
Theo nguyn l Dirichlet, trong ba s a, b, c lun c hai s nm cng pha so vi 1 trn trc s.
Gi s hai s l b v c. Th th
(b 1)(c 1) 0,
t suy ra b
2
+ c
2
1 + (b + c 1)
2
= 1 + (2 a)
2
. ()
Mt khc theo bt ng thc Cauchy-Schwarz ta c
(3 2b)
2
3b
2
6b + 9
+
(3 2c)
2
3c
2
6c + 9

(6 2b 2c)
2
3(b
2
+ c
2
2b 2c + 6)
=
4a
2
3(b
2
+ c
2
+ 2a)
()
Kt hp hai nh gi () v (), ta suy ra
(3 2b)
2
3b
2
6b + 9
+
(3 2c)
2
3c
2
6c + 9

4a
2
3[1 + (2 a)
2
+ 2a]
=
4a
2
3(a
2
2a + 5)
Nh vy, kt thc chng minh, ta cn ch ra rng
(3 2a)
2
3a
2
6a + 9
+
4a
2
3(a
2
2a + 5)

1
2
.
Thc hin php bin i tng ng, ta thu c nh gi ng sau:
(a 1)
2
(13a
2
18a + 45) 0.
Php chng minh hon tt.2
Li gii 2. Dy bt ng thc sau l tng ng vi bt ng thc cn chng minh:

_
a
2
+ (b + c)
2
2a(b + c)
2a
2
+ (b + c)
2
1
_

5
2
,

a(a + 2b + 2c)
2a
2
+ (b + c)
2

5
2
.
187
Ch rng theo bt ng thc Cauchy - Schwarz, ta c
(2a
2
+ (b + c)
2
)(2 + 4) 4(a + b + c)
2
.
Do ta suy ra

a(a + 2b + 2c)
2a
2
+ (b + c)
2

3
2

a(a + 2b + 2c)
(a + b + c)
2
=
3
2
(a + b + c)
2
+ 2(ab + bc + ca)
(a + b + c)
2
.
Cui cng, cn phi ch ra rng
(a + b + c)
2
+ 2(ab + bc + ca)
(a + b + c)
2

5
3
.
Tuy nhin bt ng thc ny tng ng vi mt nh gi c bn:
(a + b + c)
2
3(ab + bc + ca),
do vy bt ng thc ban u c chng minh xong.
Bi ton kt thc.2
Li gii 3. S dng nh gi c bn (b + c)
2
2(b
2
+ c
2
), ta suy ra
2a
2
+ (b + c)
2
2(a
2
+ b
2
+ c
2
).
Nh vy
(a b c)
2
2a
2
+ (b + c)
2

(a b c)
2
2(a
2
+ b
2
+ c
2
)
. Thit lp hai nh gi tng t, ta suy ra
(a b c)
2
2a
2
+ (b + c)
2
+
(b c a)
2
2b
2
+ (c + a)
2
+
(c a b)
2
2c
2
+ (a + b)
2

(a b c)
2
+ (b c a)
2
+ (c a b)
2
2(a
2
+ b
2
+ c
2
)
.
Cui cng, ta cn ch ra rng
(a b c)
2
+ (b c a)
2
+ (c a b)
2
a
2
+ b
2
+ c
2
,
hay a
2
+ b
2
+ c
2
ab + bc + ca. y l mt nh gi ng, do vy bt ng thc ban u c
chng minh xong.
Php chng minh hon tt.2
8.35 Cho a, b, c l cc s thc dng tho mn a
2
+ b
2
+ c
2
= 3. Chng minh rng
_
a
2
a
2
+ b + c
+
_
b
2
b
2
+ c + a
+
_
c
2
c
2
+ a + b

3
Li gii. p dng bt ng thc Cauchy-Schwarz, ta c
(a
2
+ b + c)(1 + b + c) (a + b + c)
2
,
t ta c
a
2
a
2
+ b + c

a

1 + b + c
a + b + c
. Thit lp hai nh gi tng t, ta suy ra

a
2
a
2
+ b + c
+

b
2
b
2
+ c + a
+

c
2
c
2
+ a + b

a

1 + b + c + b

1 + c + a + c

1 + a + b
a + b + c
.
Mt khc cng theo bt ng thc Cauchy - Schwarz th
188
a

1 + b + c + b

1 + c + a + c

1 + a + b =

a + ab + ac +

b + bc + ba +

c + ca + cb

_
(a + b + c)[a + b + c + 2(ab + bc + ca)],
do vy ta suy ra
a

1 + b + c + b

1 + c + a + c

1 + a + b
_
(a + b + c)[a + b + c + 2(ab + bc + ca)]
a + b + c
=
_
1 +
2(ab + bc + ca)
a + b + c
.
Nh vy, kt thc chng minh, ta cn ch ra rng
ab + bc + ca a + b + c.
Bnh phng hai v v ti nh gi c bn (a + b + c)
2
3(ab + bc + ca), ta c
ab + bc + ca 3.
Tuy nhin nh gi trn ng do ab +bc +ca a
2
+b
2
+c
2
= 3. Do vy bt ng thc ban u
c chng minh xong.
Bi ton kt thc.2
8.36 Cho a, b, c l cc s thc dng tho mn abc = 1. Chng minh rng
a
3
+ b
3
+ c
3
a + b + c
Li gii. p dng bt ng thc AM-GM, ta c
a
3
+ 1 + 1 3a.
Thit lp hai nh gi tng t cho b v c v cng li, ta c
a
3
+ b
3
+ c
3
+ 6 3(a + b + c).
Mt khc, cng theo bt ng thc AM-GM th a + b + c 3
3

abc = 3, do vy
a
3
+ b
3
+ c
3
+ 6 a + b + c + 6.
Rt gn 6 hai v ta thu c bt ng thc cn chng minh.
Bi ton kt thc.2
8.37 Cho a, b, c l cc s thc khng m tho mn a
2
+ b
2
+ c
2
= 1. Chng minh rng
1
1 a
2
+
1
1 b
2
+
1
1 c
2
+
1
1 bc
+
1
1 ca
+
1
1 ab
9
Li gii. Bt ng thc cn chng minh tng ng vi
a
2
1 a
2
+
b
2
1 b
2
+
c
2
1 c
2
+
bc
1 bc
+
ca
1 ca
+
ab
1 ab
3.
189
p dng bt ng thc Cauchy - Schwarz, ta c

a
2
1 a
2
+

bc
1 bc

(

a
2
+

bc)
2

a
2
(1 a
2
) +

bc(1 bc)
=
(

a
2
+

bc)
2

a
2
(b
2
+ c
2
) +

bc(a
2
+ b
2
+ c
2
bc)
.
Nh vy, kt thc chng minh, ta cn ch ra rng
(

a
2
+

bc)
2
3

a
2
(b
2
+ c
2
) + 3

bc(a
2
+ b
2
+ c
2
bc).
Khai trin v rt gn, ta thu c
a
2
(a b)(a c) + b
2
(b a)(b c) + c
2
(c a)(c b) 0.
Tuy nhin y chnh l bt ng thc Schur bc bn nn l mt nh gi ng. Do vy bt
ng thc ban u c chng minh xong.
Bi ton kt thc.2
8.38 Cho a, b, c l cc s thc thay i bt k. Chng minh rng
a
2
+ b
2
+ c
2
ab + bc + ca +
(a b)
2
26
+
(b c)
2
6
+
(c a)
2
2009
Li gii. Dy bt ng thc sau l tng ng vi bt ng thc cn chng minh:
2(a
2
+ b
2
+ c
2
) 2ab + 2bc + 2ac +
(a b)
2
13
+
(b c)
2
3
+
2(c a)
2
2009
,
(a b)
2
+ (b c)
2
+ (c a)
2

(a b)
2
13
+
(b c)
2
3
+
2(c a)
2
2009
,
12(a b)
2
13
+
2(b c)
2
3
+
2007(c a)
2
2009
0.
nh gi cui cng hin nhin ng cho mi s thc a, b, c, do vy bt ng thc ban u c
chng minh xong.
Bi ton kt thc.2
8.39 Cho a, b, c l cc s thc tho mn ng thi a < b < c,a+b +c = 6 v ab +bc +ca = 9.
Chng minh rng
0 < a < 1 < b < 3 < c < 4
Li gii. Trc ht, t gi thit ta c
a
2
+ b
2
+ c
2
= (a + b + c)
2
2(ab + bc + ca) = 6
2
9.2 = 18.
u tin, ta chng minh a, b, c l cc s thc dng, nh s dng nh gi
9 = ab + bc + ca < a(b + c) +
(b + c)
2
4
= a(6 a) +
(6 a)
2
4
.
190
nh gi ny tng ng vi
3a
2
4
3a < 0,
t ta suy ra 0 < a < 4, do vy 0 < a < b < c. Khi ta cng c
18 = a
2
+ b
2
+ c
2
< ac + bc + c
2
= c(a + b + c) = 6c,
hay c > 3.
By gi ta s chng minh c < 4 bng phn chng. Gi s rng c 4, khi c
2
4c. T y ta
suy ra
18 = a
2
+ b
2
+ c
2
>
(a + b)
2
2
+ 4c =
(6 c)
2
2
+ 4c,
tng ng
c
2
2
2c < 0,
hay 0 < c < 4, tri vi iu gi s. Mu thun ny chng t iu gi s l sai, do vy c < 4, cng
c ngha a < b < c < 4.
Ta tip tc chng minh a < 1 cng bng phn chng. Gi s rng 1 a < b < c < 4, khi ta
c cc nh gi
(a 1)(a 4) 0,
(b 1)(b 4) < 0,
(c 1)(c 4) < 0
Cng v theo v cc nh gi trn, ta s c
a
2
+ b
2
+ c
2
< 5(a + b + c) 12,
t kt hp vi iu kin c 18 < 5.6 12 = 18. iu v l ny chng t iu gi s l sai,
do vy a < 1.
T a < 1 v c < 4, ta suy ra b = 6 a c > 6 4 1 = 1. Nh vy, kt thc chng minh, ta
ch cn phi ch ra rng b > 3. Gi s ngc li, tc l b 3, khi ta c
(b 3)(c 3) 0,
tng ng
bc 3(b + c) 9 = 3(6 a) 9 = 9 3a.
T y ta suy ra
9 = ab + bc + ca = a(b + c) + bc a(b + c) + 9 3a,
hay
a(b + c 3) 0.
Tuy nhin li l mt nh gi sai, do vy iu gi s ban u ca chng ta l sai, nn b < 3.
Tm li, ta chng minh c 0 < a < 1 < b < 3 < c < 4, do vy bi ton kt thc.2
191
8.40 Cho a, b, c l cc s thc dng thay i bt k. Chng minh rng
3
_
a
b + c
+
3
_
b
a + c
+
3
_
c
a + b
> 2
Li gii. t a = x
3
, b = y
3
, c = z
3
. Khi x, y, z > 0 v bt ng thc cn chng minh tr
thnh
3

x
3
y
3
+ z
3
+
3

y
3
z
3
+ x
3
+
3

z
3
x
3
+ y
3
> 2.
Trc ht ta s chng minh rng
3

x
3
y
3
+ z
3
>

x
2
y
2
+ z
2
. ()
Tht vy, bt ng thc ny tng ng vi mi bt ng thc trong dy sau
x
6
(y
3
+ z
3
)
2
>
x
6
(y
2
+ z
2
)
3
,
(y
2
+ z
2
)
3
> (y
3
+ z
3
)
2
,
3y
2
z
2
(y
2
+ z
2
) > 2y
3
z
3
.
Tuy nhin nh gi cui cng ng theo bt ng thc AM-GM:
3y
2
z
2
(y
2
+ z
2
) 6y
3
z
3
> 2y
3
z
3
,
do vy bt ng thc () c chng minh.
T ta suy ra
3

x
3
y
3
+ z
3
+
3

y
3
z
3
+ x
3
+
3

z
3
x
3
+ y
3
>

x
2
y
2
+ z
2
+

y
2
z
2
+ x
2
+

z
2
x
2
+ y
2
. ()
Mt khc, theo bt ng thc AM-GM, ta suy ra

x
2
y
2
+ z
2
=
x
2
_
x
2
(y
2
+ z
2
)

2x
2
x
2
+ y
2
+ z
2
. ( )
Kt hp () vi ( ) v hai nh gi tng t khc, ta suy ra
3

x
3
y
3
+ z
3
+
3

y
3
z
3
+ x
3
+
3

z
3
x
3
+ y
3
>
2(x
2
+ y
2
+ z
2
)
x
2
+ y
2
+ z
2
= 2.
Php chng minh hon tt.2
192
3.9 Bi 9.1 n bi 9.40
9.1 Cho x, y, z l cc s thc thay i trong khong [0, 1]. Chng minh rng
2(x
3
+ y
3
+ z
3
) (x
2
y + y
2
z + z
2
x) 3
Li gii. S dng gi thit, ta suy ra
(1 x
2
)(1 y) 0,
hay 1 + x
2
y x
2
+ y. Cng nh gi ny vi hai nh gi tng t khc, ta suy ra
3 + x
2
y + y
2
z + z
2
x x
2
+ y
2
+ z
2
+ x + y + z. ()
Mt khc, cng t gi thit ta c
x(1 + 2x)(1 x) 0,
hay x
2
+ x 2x
3
. Thit lp hai bt ng thc tng t v cng li, ta c
x
2
+ y
2
+ z
2
+ x + y + z 2(x
3
+ y
3
+ z
3
). ()
Kt hp () v (), ta suy ra
3 + x
2
y + y
2
z + z
2
x 2(x
3
+ y
3
+ z
3
),
t ta thu c bt ng thc cn chng minh.
Bi ton kt thc.2
9.2 Cho a, b, c l cc s thc thay i trong on
_
1
2
, 1
_
. Chng minh rng
2
a + b
1 + c
+
b + c
1 + a
+
c + a
1 + b
3
Li gii. t x = a + 1, y = b + 1, z = c + 1. Khi x, y, z
_
3
2
, 2
_
v ta cn chng minh
2
x + y 2
z
+
y + z 2
x
+
z + x 2
y
3.
Trc ht ta s chng minh bt ng thc v tri. Ta vit li bt ng thc nh sau
(x + y + z 2)
_
1
x
+
1
y
+
1
z
_
5.
t s = x + y + z, khi theo mt nh gi quen thuc, ta c
1
x
+
1
y
+
1
z

9
s
.
Nh vy, ta cn chng minh
9(s 2)
s
5,
193
hay s
9
2
. Tuy nhin y l mt nh gi ng v theo iu kin, ta c
s = x + y + z
3
2
+
3
2
+
3
2
=
9
2
,
do vy bt ng thc cn chng minh.
By gi ta s chng minh bt ng thc v phi. Bt ng thc ny tng ng vi
_
x
y
+
y
x
_
+
_
y
z
+
z
y
_
+
_
z
x
+
x
z
_
3 +
2
x
+
2
y
+
2
z
.
Khng mt tnh tng qut, ta gi s
3
2
x y z 2. Khi rng
x
y
+
y
x

_
x
2
+
2
x
_
=
(2 y)(x
2
2y)
2xy
0,
do
x
y
+
y
x

x
2
+
2
x
. Hon ton tng t, ta cng c
y
z
+
z
y

y
2
+
2
y
,
v
z
x
+
x
z

x
2
+
2
x
.
Cng v theo v cc nh gi trn li, ta c
_
x
y
+
y
x
_
+
_
y
z
+
z
y
_
+
_
z
x
+
x
z
_
x +
4
x
+
y
2
+
2
y
.
Cui cng, ta cn ch ra rng
x +
4
x
+
y
2
+
2
y
3 +
2
x
+
2
y
+
2
z
,
tng ng
x +
2
x
+
y
2
3 +
2
z
.
y l mt nh gi ng v l h qu ca hai nh gi sau
x +
2
x
3 =
(x 1)(x 2)
3
0,
y
2
1
2
z
.
Do vy bt ng thc ban u c chng minh xong.
Tm li c hai bt ng thc u c chng minh hon tt. Bi ton kt thc.2
9.3 Cho a, b, c l di ba cnh ca mt tam gic. Chng minh rng

a + b c +

b + c a +

c + a b

a +

b +

c
194
Li gii. S dng nh gi c bn (x + y)
2
2(x
2
+ y
2
), ta c
(

a + b c +

b + c a)
2
2(a + b c + b + c a) = 4b,
t suy ra

a + b c +

b + c a 2

b. Cng bt ng thc trn vi hai nh gi tng t


khc, ta thu c bt ng thc cn chng minh.
Bi ton kt thc.2
9.4 Cho a, b, c l cc s thc thay i trong khong [1, 2]. Chng minh rng
a
3
+ b
3
+ c
3
5abc
Li gii 1. Bt ng thc cn chng minh mang tnh i xng gia cc bin, do khng mt
tnh tng qut, ta gi s 2 a b c 1.
t f(a, b, c) = a
3
+ b
3
+ c
3
5abc. Xt hiu sau
f(a, b, c) f(a, b, 1) = c
3
5abc 1 + 5ab = (c 1)(1 + c + c
2
5ab).
rng c 1 0 v
1 + c + c
2
5ab 1 + c 4c
2
= 4(c 1)
2
3c + 1 0,
do vy (c 1)(1 + c + c
2
5ab) 0 hay f(a, b, c) f(a, b, 1).
Tip tc xt hiu sau:
f(a, b, 1) f(a, 1, 1) = (b 1)(1 + b
2
+ b 5a),
v vi ch (1+b
2
+b5a) 14b+b
2
= (b1)(b2)b1 0, ta suy ra f(a, b, 1) f(a, 1, 1).
Nh vy
f(a, b, c) f(a, 1, 1) = a
3
5a + 2 = (a 2)(a
2
+ 2a 1),
t ta suy ra f(a, b, c) 0, hay a
3
+ b
3
+ c
3
5abc.
Bi ton kt thc.2
Li gii 2. Bt ng thc cn chng minh mang tnh i xng gia cc bin, do khng mt
tnh tng qut, ta gi s 2 a b c 1. Khi d thy
(b a)(b c) 0,
hay b
2
b(a + c) ac. T y ta suy ra
b
3
b [b(a + c) ac] = b
2
(a + c) abc
[b(a + c) ac] (a + c) abc = b(a + c)
2
ac(a + c) abc.
195
Nh vy, kt thc chng minh, ta cn ch ra rng
a
3
+ c
3
+ b(a + c)
2
ac(a + c) abc 5abc,
hay
(a c)
2
(a + c) + b(a + c)
2
6abc.
Lu rng do a, b, c [1, 2] nn a 2 2c b + c, t dn n
0 a c b.
V nh vy ta chng minh c nh gi trn v
(a c)
2
(a + c) + b(a + c)
2
b(a c)(a + c) + b(a + c)
2
= 2ab(a + c) 2ab(2c + c) = 6abc.
Chng minh hon tt2
Li gii 3. Bt ng thc cn chng minh tng ng vi
a
2
bc
+
b
2
ca
+
c
2
ab
5.
Bt ng thc ny mang tnh i xng gia cc bin, do khng mt tnh tng qut, ta gi s
1 a b c 2. Khi ta c nh gi
(a b)(b
2
c
2
) 0,
t y suy ra b
3
ab
2
+ bc
2
c
2
a hay
b
2
ca

b
c
+
c
a

c
b
.
Mt khc ta cng c
a
2
bc

a
2
ca
=
a
c
,
v
c
2
ab

2c
ab

2c
b
.
T cc nh gi trn, ta thu c
a
2
bc
+
b
2
ca
+
c
2
ab

_
b
c
+
c
b
_
+
_
a
c
+
c
a
_
V b c 2 nn
2b
c
1 v
c
b
1 >
1
2
. Do vy
_
2b
c
1
__
c
b

1
2
_
0,
hay
b
c
+
c
b

5
2
. Hon ton tng t, ta cng c
a
c
+
c
a

5
2
. T hai nh gi ny, ta c
a
2
bc
+
b
2
ca
+
c
2
ab

_
b
c
+
c
b
_
+
_
a
c
+
c
a
_
5.
196
Php chng minh hon tt.2
9.5 Cho a, b, c l cc s thc khng m thay i bt k. Chng minh rng
_
1 +
4a
b + c
__
1 +
4b
a + c
__
1 +
4c
b + a
_
25
Li gii. Dy bt ng thc sau l tng ng vi bt ng thc cn chng minh
(4a + b + c)(4b + c + a)(4c + a + b) 25(a + b)(b + c)(c + a),
a
3
+ b
3
+ c
3
+ 7abc ab(a + b) + bc(b + c) + ca(c + a).
nh gi cui cng ng do abc 0 v bt ng thc Schur bc ba, do vy bt ng thc ban
u c chng minh xong.
Bi ton kt thc.2
9.6 Cho a, b, c l cc s thc dng tho mn a
2
+ b
2
+ c
2
= 3. Chng minh rng
ab
2
+ bc
2
+ ca
2
2 + abc
Li gii. Bt ng thc ban u mang tnh hon v gia cc bin nn khng mt tnh tng qut,
ta gi s b nm gia a v c. T ta c nh gi
a(b c)(b a) 0,
hay ab
2
+ ca
2
a
2
b + abc. Nh vy, kt thc chng minh, ta cn ch ra rng
a
2
b + bc
2
2.
Thay a
2
+ c
2
bi 3 b
2
, ng thi thc hin bin i tng ng, ta c nh gi
b
3
+ 2 3b.
y l mt nh gi ng bi theo AM-GM, b
3
+ 1 + 1 3
3

b
3
= 3b. Do vy bt ng thc ban
u c chng minh xong.
Bi ton kt thc.2
9.7 Cho a, b, c l cc s thc dng thay i bt k. Chng minh rng
(2a + b + c)
2
2a
2
+ (b + c)
2
+
(a + 2b + c)
2
2b
2
+ (c + a)
2
+
(a + b + 2c)
2
2c
2
+ (a + b)
2
8
Li gii 1. Bt ng thc ban u mang tnh thun nht nn ta chun ha a + b + c = 3. Khi
bt ng thc cn chng minh tr thnh
a
2
+ 6a + 9
3a
2
6a + 9
+
b
2
+ 6b + 9
3b
2
6b + 9
+
c
2
+ 6c + 9
3c
2
6c + 9
8.
197
rng ta c nh gi sau
a
2
+ 6a + 9
3a
2
6a + 9
=
1
3
_
1 +
2(4a + 3)
2 + (a 1)
2
_

4(a + 1)
3
.
Thit lp hai nh gi tng t v cng li, ta c
a
2
+ 6a + 9
3a
2
6a + 9
+
b
2
+ 6b + 9
3b
2
6b + 9
+
c
2
+ 6c + 9
3c
2
6c + 9

4(a + b + c + 3)
3
= 8.
Php chng minh hon tt.2
Li gii 2. Ch rng
3
(2a + b + c)
2
2a
2
+ (b + c)
2
=
2(b + c a)
2
2a
2
+ (b + c)
2
,
do ta cn phi chng minh
(b + c a)
2
2a
2
+ (b + c)
2
+
(c + a b)
2
2b
2
+ (c + a)
2
+
(a + b c)
2
2c
2
+ (a + b)
2

1
2
.
Tuy nhin y l mt kt qu c chng minh bi 8.18 . 2
Li gii 3. t x =
b + c
a
, y =
c + a
b
, z =
a + b
c
. Khi bt ng thc cn chng minh tr thnh
(x + 2)
2
x
2
+ 2
+
(y + 2)
2
y
2
+ 2
+
(z + 2)
2
z
2
+ 2
8,
hay
(x 1)
2
x
2
+ 2
+
(y 1)
2
y
2
+ 2
+
(z 1)
2
z
2
+ 2

1
2
.
p dng bt ng thc Cauchy-Schwarz, ta c:
(x 1)
2
x
2
+ 2
+
(y 1)
2
y
2
+ 2
+
(z 1)
2
z
2
+ 2

(x + y + z 3)
2
x
2
+ y
2
+ z
2
+ 6
Nh vy, kt thc chng minh, ta cn ch ra rng
2(x + y + z 3)
2
x
2
+ y
2
+ z
2
+ 6,
hay (x + y + z 6)
2
+ 2(xy + yz + zx 12) 0. Tuy nhin y l mt nh gi ng nh nh
gi sau:
(x + y + z 6)
2
0
v nh gi c c theo bt ng thc AM-GM:
xy + yz + zx =
(b + c)(c + a)
ab
+
(b + c)(c + a)
ab
+
(b + c)(c + a)
ab
3
3

_
(a + b)(b + c)(c + a)
abc
_
2
12,
198
do vy bt ng thc ban u c chng minh xong.
Bi ton kt thc.2
9.8 Cho x, y, z l cc s thc dng tho mn 13x + 5y + 12z = 9. Tm gi tr ln nht ca
biu thc
P =
xy
2x + y
+
3yz
2y + z
+
6xz
x + z
Li gii. Ta vit li biu thc P nh sau:
P =
1
2
y
+
1
x
+
1
2
3x
+
1
3y
+
1
1
3x
+
1
6z
.
rng ta c nh gi c bn sau
a + b + c
9

1
1
a
+
1
b
+
1
c
,
t ta p dng c
P
2y + x + 6z + 3y + 12x + 6z
9
=
13x + 5y + 12z
9
= 1.
Cui cng, vi x = y = z =
3
10
(tho mn iu kin) th P = 1 nn ta kt lun 1 l gi tr ln
nht ca biu thc P.
Bi ton kt thc.2
9.9 Cho a, b, c l cc s thc khng m tho mn a + b + c = 1. Chng minh rng
10(a
3
+ b
3
+ c
3
) 9(a
5
+ b
5
+ c
5
) 1
Li gii. Dy bt ng thc sau l tng ng vi bt ng thc cn chng minh:
_
10a
3
9a
5
a
_
+
_
10b
3
9b
5
b
_
+
_
10c
3
9c
5
c
_
0,
a(1 a
2
)(9a
2
1) 0.
rng ta c nh gi sau
(1 + a)(9a
2
1)
8
3
.(3a 1) =
1
3
.(3a + 5)(3a 1)
2
0,
t ta a bi ton v vic chng minh
a(1 a)(3a 1) + b(1 b)(3b 1) + c(1 c)(3c 1) 0,
hay 4(a
2
+ b
2
+ c
2
) 3(a
3
+ b
3
+ c
3
) 1. S dng gi thit ta thu c bt ng thc
4(a
2
+ b
2
+ c
2
)(a + b + c) 3(a
3
+ b
3
+ c
3
) (a + b + c)
3
.
199
Thc hin php bin i tng ng, ta thu c nh gi c bn sau
ab(a + b) + bc(b + c) + ca(c + a) 6abc.
Do vy bt ng thc ban u c chng minh xong.
Bi ton kt thc.2
9.10 Cho a, b, c l cc s thc trong khong [1; 2]. Chng minh rng
(a + b + c)(
1
a
+
1
b
+
1
c
) 10
Li gii 1. Trc ht ta c
(a + b + c)(
1
a
+
1
b
+
1
c
) = 3 +
a
b
+
b
a
+
b
c
+
c
b
+
c
a
+
a
c
.
Bt ng thc ban u mang tnh i xng gia cc bin nn khng mt tnh tng qut, ta gi
s 2 a b c 1. T y ta c cc nh gi sau
a
c
+ 1
a
b
+
b
c
=
_
a
b
1
_
_
b
c
1
_
0,
c
a
+ 1
b
a
+
c
b
=
_
b
a
1
_
_
c
b
1
_
0.
Nh vy ta suy ra
(a + b + c)(
1
a
+
1
b
+
1
c
) 5 + 2(
a
c
+
c
a
),
v do kt thc chng minh, ta cn ch ra rng
a
c
+
c
a

5
2
,
hay (2a c)(a 2c) 0. Tuy nhin y l nh gi ng do 2a > c v a 2 2c, nn bt ng
thc ban u c chng minh xong.
Bi ton kt thc. 2
Nhn xt: Khi cc bin a, b, c thuc on[m, n] no , ta thng hay nh gi
(a m)(b m)(c m) 0 hoc (a n)(b n)(c b) 0 tm ra cc mi lin h khc. nh
gi trn c th c xem l " mnh" (khng phi tt c trng hp u dng c), v du
bng xy ra khi mt bin bng m hoc n. (c hiu qu khi chng minh bt ng thc i xng
m du bng t ti bin).
Li gii 2. Bt ng thc ban u mang tnh i xng gia cc bin nn khng mt tnh tng
qut, ta gi s 1 a b c 2.
D thy bt ng thc ban u tng ng vi mi bt ng thc trong dy sau:
(a + b + c)(ab + bc + ca) 10abc,
ab(a + b) + bc(b + c) + ca(c + a) 7abc,
200
(a b)(b c)(c + a) + b(2a c)(2c a) 0.
Tuy nhin nh gi cui cng ng do a b c, ng thi c 2 2a v a 2 2c. T ta
suy ra bt ng thc ban u c chng minh xong.
Bi ton kt thc.2
9.11 Cho x, y, z l cc s thc dng.Chng minh vi mi 0 a b c, ta c:
(ax + by + cz)(
x
a
+
y
b
+
z
c
)
(a + c)
2
4ac
(x + y + x)
2
.
Li gii.
p dng bt ng thc AM-GM ta c:
4ac(ax + by + cz)(
x
a
+
y
b
+
z
c
)
_
(ax + by + cz) + ac(
x
a
+
y
b
+
z
c
)
_
2
Khi ta cn chng minh:
ax + by + cz + ac(
x
a
+
y
b
+
z
c
) (a + c)(x + y + z)
Tng ng vi:
ay + cy by +
acy
b
y(a b)(b c) 0 .2
9.12 Cho ba s thc khng m a, b, c tha mn ab + bc + ca = 1. Chng minh rng:
a
2
+ b
2
+ c
2
+ 3
a + b + c abc

5
2
Li gii.
Bin i nh sau:
a
2
+ b
2
+ c
2
+ 3 = a
2
+ b
2
+ c
2
+ 3(ab + bc + ca) = (a + b)(b + c) + (b + c)(c + a) + (c + a)(a + b)
a + b + c abc = (a + b + c)(ab + bc + ca) abc = (a + b)(b + c)(c + a)
Ta a bt ng thc v dng:
1
a + b
+
1
b + c
+
1
c + a

5
2
Khng mt tnh tng qut, gi s c = max {a; b; c}
Khi :(a + b)
2
2c(a + b) 2 a + b < 2. By gi :
1
c + a
=
a + b
a
2
+ 1
= a + b
a
2
(a + b)
a
2
+ 1
a + b
a(a + b)
2
1
b + c
a + b
b(a + b)
2
Ta quy v chng minh:
2
x
+ x(4 x) 5 vi a + b = x
Tng ng:(x 1)
2
(x 2) 0 (ng)
Ta c iu phi chng minh. ng thc xy ra ch khi c = 1; a = 1; b = 0 v cc hon v.2
9.13 Cho ba s thc dng a, b, c tha mn a
2
+ b
2
+ c
2
3. Chng minh rng:
a
2
+ b
2
+ c
2
+ 3abc 2(a + b + c)
Li gii.
u tin ta xt trng hp a
2
+ b
2
+ c
2
= 3
Khi , ta c: 3(a
2
+ b
2
+ c
2
) (a + b + c)
2
> a
2
+ b
2
+ c
2
3 a + b + c >

3
201
Bt ng thc cn chng minh tng ng vi:
a + b + c 3abc 3 (a + b + c) Hay
1
3
(a + b + c)(a
2
+ b
2
+ c
2
) 3abc 3 (a + b + c)
Ta c:
1
3
(a+b+c)(a
2
+b
2
+c
2
)3abc
a
3
+ b
3
+ c
3
3abc
3
=
(a + b + c)(a
2
+ b
2
+ c
2
ab bc ca)
3
V: 3 (a + b + c) =
3(a
2
+ b
2
+ c
2
) (a + b + c)
2
3 + a + b + c
=
2(a
2
+ b
2
+ c
2
ab bc ca)
3 + a + b + c
Nh vy ta ch cn chng minh: (a + b + c)(a + b + c + 3) 6
iu ny hin nhin ng do

3(

3 + 3) > 6
Du bng ch xy ra khi a = b = c = 1
Nu a
2
+ b
2
+ c
2
< 3 th tn ti k > 1 sao cho: (ka)
2
+ (kb)
2
+ (kc)
2
= 3
Theo trn, ta c: k
2
(a
2
+ b
2
+ c
2
) + 3k
3
abc 2k(a + b + c)
hay k(a
2
+ b
2
+ c
2
) + 3k
2
abc 2(a + b + c)
M k > 1 nn a
2
+ b
2
+ c
2
+ 3abc < k(a
2
+ b
2
+ c
2
) + 3k
2
abc
Vy ta c iu phi chng minh.2
9.14 Cho ba s thc a, b, c [0; 1]. Chng minh rng:
a
1 + bc
+
b
1 + ac
+
c
1 + ab
+ abc
5
2
Li gii.
Ta c cc bt ng thc sau: (1 a)(1 bc) + (1 b)(1 c) 0 abc + 2 a + b + c
a
1 + bc

a
1 + abc
Nh vy ta ch cn chng minh:
x + 2
x + 1
+ x
5
2
vi x = abc
Tng ng: (x 1)(2x + 1) 0 ng thc ch xy ra khi a = b = c = 1.2
9.15 Cho cc s thc khng m a; b; c tha mn a +b +c = 1. Tm gi tr ln nht ca biu
thc:
M =
1 + a
2
1 + b
2
+
1 + b
2
1 + c
2
+
1 + c
2
1 + a
2
Li gii.
t x = a
2
+ 1, y = b
2
+ 1, z = c
2
+ 1
T gi thit ta c: a, b, c [0, 1] x, y, z [1, 2]
Tm Max: M =
x
y
+
y
z
+
z
x
Khng mt tnh tng qut:
Gi s x y z (y x)(y z) 0
x
y
+
y
z

x
z
+ 1
Do x, y, z [1, 2] (2x z)(2z x) 0
x
z
+
z
x

5
2
do : M
5
2
+ 1 =
7
2
.2
9.16 Cho cc s thc dng a; b; c tha mn abc = 1.Chng minh rng:
1
a + b
2
+ c
3
+
1
b + c
2
+ a
3
+
1
c + a
2
+ b
3
1
Li gii.
p dng bt ng thc CauchySchwarz, ta c: (a + b
2
+ c
3
)(a + 1 +
1
c
) (a + b + c)
2
202
Do

cyc
1
a + b
2
+ c
3

cyc
a + 1 +
1
c
(a + b + c)
2
=

a +

ab + 3
(a + b + c)
2
v abc = 1
By gi chng ta ch cn chng minh:

cyc

a +

ab + 3
(a + b + c)
2
1 (a + b + c)
2
a + b + c + ab + bc + ca + 3
Tng ng a
2
+ b
2
+ c
2
+ ab + bc + ca a + b + c + 3 Bt ng thc cui ng v:
a
2
+ b
2
+ c
2

(a + b + c)
2
3

3
3

abc(a + b + c)
3
= a + b + c v ab + bc + ca 3
3

a
2
b
2
c
2
= 3.
ng thc xy ra khi v ch khi a = b = c = 1.2
Cu hi m:Cc bn c th gii quyt bi ny theo cch trn c khng? V sao?
Cho cc s thc dng a; b; c tha mn abc = 1.Chng minh rng:
a
a + b
2
+ c
3
+
b
b + c
2
+ a
3
+
c
c + a
2
+ b
3
1
9.17 Cho cc s thc khng m a; b; c tha mn a
2
+ b
2
+ c
2
= 1. Chng minh rng:
1
1 a
2
+
1
1 b
2
+
1
1 c
2

9
2
Li gii.
Chng ta a bt ng thc v :
ab
1 ab
+
bc
1 bc
+
ca
1 ca

3
2
p dng bt ng thc Cauchy Schwaz:
ab
1 ab
=
2ab
(1 + c
2
) + (a b)
2

2ab
1 + c
2

1
2
.
(a + b)
2
(c
2
+ a
2
) + (c
2
+ b
2
)

1
2
.(
a
2
c
2
+ a
2
+
b
2
c
2
+ b
2
)
Tng t ta c :
ab
1 ab

3
2
.2
9.18 Cho cc s thc dng a; b; c tha mn a
3
+ b
3
+ c
3
= 3. Chng minh rng:
4(
1
a
+
1
b
+
1
c
) + 5(a
2
+ b
2
+ c
2
) 27
Li gii.
Ta c nh gi sau:
4
a
+ 5a
2
2a
3
7 =
(a 1)
2
(4 + a 2a
2
)
a
0
Bt ng thc trn ng do a <
3

3 nn 4+a2a
2
= a
2
_
4
a
2
+
1
a
2
_
> a
2
_
4
3

9
+
1
3

3
2
_
> 0.
Do ta c:
4
a
+ 5a
2
2a
3
+ 7.
Tng t ta c:
4
b
+ 5b
2
2b
3
+ 7;
4
c
+ 5c
2
2c
3
+ 7.
Cng cc bt ng thc trn li, ta c:
4
_
1
a
+
1
b
+
1
c
_
+ 5
_
a
2
+ b
2
+ c
2
_
2(a
3
+ b
3
+ c
3
) + 21 = 27.
203
Ta c iu phi chng minh. ng thc xy ra khi v ch khi a = b = c = 1. 2
9.19 Cho cc s thc dng a; b; c tha mn a + b + c 6. Tm gi tr ln nht ca biu
thc:
ab
a + 3b + 2c
+
bc
b + 3c + 2a
+
ac
c + 3a + 2b
Li gii.
S dng bt ng thc Cauchy-Schwarz ta c:
9
a + 3b + 2c
=
(1 + 1 + 1)
2
b + c + c + a + 2b

1
b + c
+
1
c + a
+
1
2b
Do :
ab
a + 3b + 2c

1
9
_
ab
b + c
+
ab
c + a
+
ab
2b
_
Xy dng cc bt ng thc tng t, ri cng li ta c:
A
1
9
_
ab
b + c
+
ab
c + a
_
+
a + b + c
18
Mt khc:

_
ab
b + c
+
ab
c + a
_
= a + b + c
nn A
a + b + c
6
1
Nh vy, gi tr ln nht ca A = 1 khi a = b = c = 2.2
9.20 Cho cc s thc dng a; b; c. Chng minh rng:
a

a
2
+ b
2
+
b

b
2
+ c
2
+
c

a
2
+ c
2

2
Li gii.
t x = a
2
, y = b
2
, z = c
2
. Khi ta cn chng minh:
_
x
x + y
+
_
y
y + z
+
_
z
z + x

3

2
p dng bt ng thc Cauchy Schwarz:
_

_
x
x + y
_
2
(

(x + z))
_
x
(x + y)(x + z)
_
=
4(x + y + z)(xy + yz + xz)
(x + y)(y + z(z + x)
.
Vy ta ch cn chng minh:
8(x + y + z)(xy + yz + xz) 9(x + y)(y + z)(x + z)
y l bt ng thc quen thuc . Vy ta c iu phi chng minh. 2
9.21 Cho cc s thc dng x; y; zc tha mn abc = 1. Chng minh rng:
x
3
+ 1
_
x
4
+ y + z
+
y
3
+ 1
_
y
4
+ z + x
+
z
3
+ 1
_
z
4
+ x + y
2

xy + yz + zx
Li gii.
rng x
4
+ y + z = x [x
3
+ yz(y + z)] p dng bt ng thc AM-GM, ta c:

sym
x
3
+ 1
2
_
x
4
+ y + z

xy + yz + zx
=

sym
x
3
+ 1
2
_
x
3
+ yz(y + z)
_
x
2
(y + z) + xyz


sym
x
3
+ 1
x
3
+ yz(y + z) + x
2
(y + z) + xyz
=

sym
x
3
+ 1
(x
2
+ yz)(x + y + z)
=

sym
x
3
+ xyz
(x
2
+ yz)(x + y + z)
=

sym
x
x + y + z
= 1.iu phi chng
minh. 2
9.22 Cho cc s thc dng a; b; c . Chng minh rng:
a
4
(b 1)
2
+
b
4
(c 1)
2
+
c
4
(a 1)
2
48
204
Li gii.
p dng bt ng thc Cauchy-swachz, ta c:
a
4
(b 1)
2
+
b
4
(c 1)
2
+
c
4
(a 1)
2

1
3
_
a
2
b 1
_
2
Ta chng minh:
1
3
48
_
a
2
b 1
_
2

a
2
b 1
12
Ta c: b
b
2
+ 4
4
b 1
b
2
4

a
2
b 1
4.
a
2
b
2
T ta c:
a
2
b 1
4
a
2
b
2
12 (ng theo AM-GM2
9.23 Cho cc s thc dng a; b; c tha mn:a + b + c = 3. Chng minh rng:
a
b
2
+ 1
+
b
c
2
+ 1
+
c
a
2
+ 1

3
2
Li gii.
p dng k thut Cauchy ngc du ta c:
a
1 + b
2
= a
ab
2
1 + b
2
a
ab
2
2b
= a
ab
2
Tng t vi 2 bt ng thc cn li, ta c:
a
1 + b
2
+
b
1 + c
2
+
c
1 + a
2
a + b + c
ab + bc + ca
2

3
2
.
ng thc xy ra khi v ch khi a = b = c = 1. 2
9.24 Cho cc s thc dng a; b; c tha mn a + b + c = 3 . Chng minh rng:
_
a + 3
a + bc
+
_
b + 3
b + ac
+
_
c + 3
c + ab
3

2
Li gii.
Ta c:
_
2(a + 3)
a + bc
=

2
_
a + b
a
2
+ bc
+
a + c
a
2
+ bc
_

_
a + b
a
2
+ bc
+
_
a + c
a
2
+ bc
Nh vy v tri khng nh hn:

cyc

a + b
_
1

a + bc
+
1

b + ca
_
M
1

a + bc
+
1

b + ca

_
8
a + bc + b + ca
=
2

2
_
(c + 1)(a + b)
Nh vy ch cn chng minh:
1

c + 1

2
Bt ng thc ny c suy ra t 2 bt ng thc:
1

c + 1

c + 1 +

b + 1 +

a + 1
V

a + 1 +

b + 1 +

c + 1
_
3(a + b + c + 3) = 3

2
ng thc xy ra ch khi a = b = c = 1.2
NHN XT: Bt ng thc tng quat sau cng ng vi cung iu kin v k 0
_
a + 1
a + bc
_
k
+
_
b + 1
b + ca
_
k
+
_
c + 1
c + ab
_
k
3
Chng minh trng hp tng qut:
S dng bt ng thc csi cho 3 s, ta ch cn chng minh:
205
(a + 1)(b + 1)(c + 1) (a + bc)(b + ac)(c + ab)
S dng bt ng thc c si cho 2 s :
(a + 1)(b + c) = (ab + c) + (ac + b) 2
_
(ab + c)(ac + b) .
(b + 1)(a + c) = (ab + c) + (bc + a) 2
_
(ab + c)(bc + a)
(c + 1)(a + b) = (ac + b) + (bc + a) 2
_
(ac + b)(bc + a)
Nhn v theo v ta ch cn chng minh:
(a + b)(b + c)(c + a) 8 .
Vy bt ng thc c chng minh.ng thc xy ra khi a = b = c = 1.2
9.25 Cho cc s thc khng m x; y; zc tha mn x + y + z = 3.Tm gi tr ln nht ca
biu thc:
x
2
+ x + 2y + 3z
Li gii.
Ta c: x
2
+ x + 2y + 3z x
2
+ x + 3(y + z) = x(x 2) + 9 3.(3 2) + 9 = 12
Du bng xy ra khi v ch khi x = 3; y = z = 0. 2
9.26 Cho cc s thc dng a; b; c . Chng minh rng:
(b + c) + b
3
(a + c) + c
3
(a + b)
2
3
(a
2
+ b
2
+ c
2
)
2
Li gii.
Bt ng thc cn chng minh tng ng vi:
2(a
4
+ b
4
+ c
4
) + 4(a
2
b
2
+ b
2
c
2
+ c
2
a
2
) 3ab(a
2
+ b
2
) + 3bc(b
2
+ c
2
) + 3ca(c
2
+ a
2
)
Ta tch ra chng minh n gin vi 2 bin nh sau
a
4
+ b
4
+ 4a
2
b
2
3ab(a
2
+ b
2
) (a b)
4
+ ab(a b)
2
0
Tng t ta cng c: a
4
+ c
4
+ 4a
2
c
2
3ac(a
2
+ c
2
) v b
4
+ c
4
+ 4b
2
c
2
3bc(b
2
+ c
2
)
Vy bt ng thc c chng minh.ng thc xy ra khi v ch khi a = b = c.2
9.27 Cho cc s thc dng a; b; c . Chng minh rng:
a
2
ab + b
2

b
4
+ c
4
+
b
2
bc + c
2

c
4
+ a
4
+
c
2
ca + a
2

b
4
+ c
4

2
2
Li gii.
p dng bt ng thc AM-GM ta c:
V T 3
3

(a
2
ab + b
2
)(b
2
bc + c
2
)(c
2
ca + a
2
)
_
(c
4
+ a
4
)(b
4
+ c
4
)(c
4
+ a
4
)
Nn ta ch cn chng minh:
8[(a
2
ab + b
2
)(b
2
bc + c
2
)(c
2
ca + a
2
)]
2
(c
4
+ a
4
)(b
4
+ c
4
)(c
4
+ a
4
)
Ta c bt ng thc: 2(a
2
ab + b
2
)
2
a
4
+ b
4
(a b)
4
0
Nhn 3 bt ng thc ny li ta c:
8[(a
2
ab + b
2
)(b
2
bc + c
2
)(c
2
ca + a
2
)]
2
(c
4
+ a
4
)(b
4
+ c
4
)(c
4
+ a
4
) .
Php chng minh hon tt.2
206
9.28 Cho cc s thc dng a; b; c . Chng minh rng:
15
15
_
32ab
(3a + b)(5a + 3b)
+ 6
3
_
a
5a + 3b
18
Li gii.
p dng AM-GM ta c:
V T
4a
3a + b
+
8b
5a + 3b
+ 13 +
8a
5a + b
+ 2
By gi ta chng minh:
4a
3a + b
+
8(a + b)
5a + 3b
3
Tc l 8(a + b)(3a + b) (5a + 3b)
2
(a b)
2
0. (Lun ng)2
9.29 Cho hai s thc dng a; b tha mn a
9
+ b
9
= 2. Chng minh rng:
a
2
b
+
b
2
a
2
Li gii.
Bt ng thc cn chng minh tng ng vi:
a
3
+ b
3
2ab 3(a
3
+ b
3
)
4
8.3a
3
b
3
(a
3
+ b
3
)
Tng ng
3(a
3
+ b
3
)
4
8((a
3
+ b
3
)
3
a
9
b
9
)
Tng ng
3(a
3
+ b
3
)
4
+ 16 8(a
3
+ b
3
)
3
ng theo C si 4 s).ng thc xy ra khi v ch khi a = b = 1.2
9.30 Cho hai s thc dng a; b . Chng minh rng:
(

a +

b)(
1

a + 3b
+
1

b + 3a
) 2
Li gii.
p dng bt ng thc AM-GM, ta c:
2

a + 3b

a
a + b
+
a + b
a + 3b
2

a + 3b

1
2
+
2b
a + 3b
Cng li ta c:

a +

a + 3b

3
4
+
a
2(a + b)
Lm tng t cng li c ngay:
(

a +

b)(
1

a + 3b
+
1

b + 3a
) 2 .2
9.31 Cho ba s thc khng m a; b; c tha mn a
2
+b
2
+c
2
+ (a +b +c)
2
4. Chng minh
rng:
ab + 1
(a + b)
2
3
Li gii.
Ta c:
207
4ab + 4
(a + b)
2

4ab + a
2
+ b
2
+ c
2
+ (a + b + c)
2
(a + b)
2
= 2
(c + a)(b + c)
(a + b)
2
+ 6
Nh vy ch cn chng minh:
(a + b)(b + c)
(c + a)
2
3
Bt ng thc cui ng theo AM-GM.
ng thc xy ra khi v ch khi a = b = c =

3
3
.2
9.32 Cho ba s thc a; b; c i mt khc nhau. Chng minh rng:
(
a + b
a c
)
2
+ (
b + c
b a
)
2
+ (
c + a
c b
)
2
1
Li gii.
t x =
a + b
a c
; y =
b + c
b a
; z =
c + a
c b
Khi : xyz = (x 1)(y 1)(z 1) =
(a + b)(b + c)(c + a)
(a c)(b a)(c b)
Suy ra: xy + yz + zx = x + y + z 1
T bt ng thc: (x + y + z 1)
2
0, ta c:
x
2
+ y
2
+ z
2
2(x + y + z xy yz zx) 1
Hay x
2
+ y
2
+ z
2
1 .
Php chng minh hon tt. 2
9.33 Cho hai s thc dng a; b; c tha mn a + b + c = 2. Tm gi tr ln nht ca biu
thc:
P =
ab

ab + 2c
+
bc

bc + 2a
+
ca

ca + 2b
Li gii.
Theo bt ng thc AM-GM:
ab

ab + 2c
=
ab
_
ab + c(a + b + c)
=
ab
_
(b + c)(c + a)

ab
2(b + c)
+
ab
2(c + a)
L tng t cho cc biu thc cn li, vi ch :
ab
2(b + c)
+
ca
2(b + c)
=
a
2
Ta tm c max ca P = 1 t c khi v ch khi a = b = c.2
9.34 Cho hai s thc dng a; b; ctha mn
1
a
+
1
b
+
1
c
= 1 . Chng minh rng:
a
2
a + bc
+
b
2
b + ca
+
c
2
c + ab

a + b + c
4
Li gii.
T gi thit ta c:
ab + bc + ca = abc
V T =
a
3
a
2
+ abc
=
a
3
(a + b)(a + c)
Ta c:
a
3
(a + b)(a + c)
+
a + b
8
+
b + c
8

3
4
(a + b + c)
Do ta c:
a
2
a + bc
+
b
2
b + ca
+
c
2
c + ab

a + b + c
4
.
208
Php chng minh hon tt.2
9.35 Cho ba s thc dng a; b; c tha mna + b + c = 3. Chng minh rng:
1
1 + ab

9
2(

a +

b +

c)
Li gii.
Bt ng thc cho tng ng vi:
ab
1 + ab
3
9
2(

a +

b +

c)
p dng bt ng thc AM-GM, ta c:
ab
1 + ab

ab
2

ab
=
1
2

ab
By gi ta ch cn chng minh:

ab +
9

a +

b +

c
6
n gin ta t x =

a; y =

b; z =

c.Khi : x
2
+ y
2
+ z
2
= 3 v:
xy + yz + zx +
9
x + y + z
6
Tng ng xy + yz + zx +
(x
2
+ y
2
+ z
2
)
_
3(x
2
+ y
2
+ z
2
)
x + y + z
2(x
2
+ y
2
+ z
2
)
(em thun nht bt ng thc). Do bt ng thc thun nht nn chun ha a + b + c = 1.
t xy + yz + zx = A. Ta c:x
2
+ y
2
+ z
2
= 1 2A, A
1
3
<
2
5
Bt ng thc c vit li:
A +
(1 2A)
_
3(1 2A)
1
2(1 2A)
Tng ng 3(1 2A)
3
(2 5A)
2
tng ng 0 (1 3A)(1 + A 8A
2
)
Bt ng thc cui ng v 1 3A 0; 1 + A 8A
2
> 1 9A
2
0
ng thc xy ra khi v ch khi a = b = c = 1.2
9.36 Cho hai s thc x; y tha mn xy = 0 v xy(x+y) = x
2
xy +y
2
. Tm gi tr ln nht
ca biu thc:
A =
1
x
3
+
1
y
3
Li gii.
Vi ch :
A =
1
x
3
+
1
y
3
=
x
3
+ y
3
x
3
y
3
=
(x + y)(x
2
xy + y
2
)
x
3
y
3
=
_
1
x
+
1
y
_
2
t a =
1
x
, b =
1
y
.iu kin ca bi ton c th vit li nh sau :
a + b = a
2
ab + b
2
v ta cn tm gi tr nh nht ca biu thc :A = (a + b)
2
vi nh gi:
a
2
ab + b
2
=
1
4
(a + b)
2
+
3
4
(a b)
2

1
4
(a + b)
2
Ta thu c :a + b
1
4
(a + b)
2
hay l 0 a + b 4
T suy ra A 16. Vy gi tr nh nht ca A l16 t c khi a = b hay x = y =
1
2
.2
209
9.37 Cho bai s thc dng x; y; zc tha mnxy + yz + zx = xyz . Chng minh rng:
1
x
2
+
1
y
2
+
1
z
2

3
x
2
y
+
3
y
2
z
+
3
z
2
x
Li gii.
t a =
1
x
, b =
1
y
, c =
1
z
th a + b + c = 1
Bt ng thc cho tng ng vi:
a
2
+ b
2
+ c
2
3(a
2
b + b
2
c + c
2
a)
(a
2
+ b
2
+ c
2
)(a + b + c) 3(a
2
b + b
2
c + c
2
a) (do a + b + c = 1).
(a
3
+ab
2
) +(b
3
+bc
2
) +(c
3
+ca
2
) 2a
2
b+2b
2
c+2c
2
a(ng theo bt Cauchy cho 2 s dng).2
9.38 Cho hai s thc dng a; b; c . Chng minh rng:
1
a + b
+
1
b + c
+
1
c + a
+
1
2
3

abc

(a + b + c +
3

abc)
2
(a + b)(b + c)(c + a)
Li gii.
p dng bt ng thc cauchySchawrz, ta c:
1
a + b
+
1
b + c
+
1
c + a
+
1
2
3

abc
=
c
2
c
2
(a + b)
+
3

abc
2
2abc

(a + b + c +
3

abc)
2
(a + b)(b + c)(c + a)
.
Php chng minh hon tt.2
9.39 Cho hai s thc a; b; c tha mn

a
2
+ b
2
+ c
2
=
3

ab + bc + ac . Chng minh rng:


a + b + c

3
Li gii.
D thy ab + bc + ca a
2
+ b
2
+ c
2
T ta c:
2

a
2
+ b
2
+ c
2

a
2
+ b
2
+ c
2
tng ng vi
(a
2
+ b
2
+ c
2
)
3
(a
2
+ b
2
+ c
2
)
2
tng ng
a
2
+ b
2
+ c
2
1
tng ng
1 a
2
+ b
2
+ c
2

(a + b + c)
2
3
hay a + b + c

3.2
9.40 Cho ba s thc dng a; b; c cng ln hn 2 v tho mn
1
x
+
1
y
+
1
z
= 1. Chng minh
rng:
xyz + 9 4.(x + y + z)
Li gii.
Trong 3 s x, y, z phi c 2 s cng ln hn hoc cng nh hn so vi 3, gi s l x vy.
Khi (x 3)(y 3) 0 xy + 9 3(x + y)
Mt khc t gi thit suy ra xyz = xy + z + zx
210
Do vy ta c xyz + 9 z(x + y) + 3(x + y)
Ta cn chng minh z(x + y) + 3(x + y) 4(x + y + z)
Hay z
x + y
x + y 4
Gi s ngc li z <
x + y
x + y 4
th khi :
1 =
1
x
+
1
y
+
1
z
>
1
x
+
1
y
+
x + y 4
x + y

4
x + y
+
x + y 4
x + y
= 1(v l)
Do vy ta c iu phi chng minh.2
3.10 Bi 10.1 n bi 10.40
10.1 Cho ba s thc khng m a; b; c . Chng minh rng:
a
2
+ b
2
+ c
2
ab + bc + ca
+
4abc
a
2
b + b
2
c + c
2
a + abc
2
Li gii.
Khng mt tnh tng qut, gi s b l s nm gia a v c. Khi ta c:
a
2
b + b
2
c + c
2
a + abc = b(c + a)
2
+ c(a b)(c b) b(c + a)
2
Ta cn chng minh bt ng thc:
a
2
+ b
2
+ c
2
ab + bc + ca
+
4ca
(c + a)
2
2

a
2
+ b
2
+ c
2
ab + bc + ca

2(c
2
+ a
2
)
(c + a)
2

a
2
+ b
2
+ c
2
c
2
+ a
2

2(ab + bc + ca
(c + a)
2

b
2
c
2
+ a
2
+
a
2
+ c
2
(c + a)
2
d
2b
c + a
(ab + bc a
2
c
2
)
2
0
Bt ng thc ny hin nhin ng . ng thc ch xy ra khi a = b = c.2
10.2 Cho hai s thc dng x; y tha mnx + y = 2. Chng minh rng:
(x
4
+ y
4
)(x
2
+ y
2
)
4

32
x
10
y
10
Li gii.
Cch gi 1:
(x
4
+ y
4
)(x
2
+ y
2
)
4
x
10
y
10
=
1
8
(x
4
+ y
4
)(x
3
y + xy
3
)
4
.(2x
2
y
2
)
3

1
8
.
((x + y)
4
)
8
8
8
= 32 (Theo c si 8
s)
Cch gii 2:
Bt ng thc trn thc cht l h qu ca 2 bt ng thc sau: 1) x
6
y
6
(x
4
+ y
4
) 2
2) xy(x
2
+ y
2
) 2
Chng minh 2): Ta c:xy(x
2
+ y
2
)
(x
2
+ y
2
+ 2xy)
2
8
= 2
211
Chng minh 1): Ta c:x
6
y
6
(x
4
+ y
4
)
x
4
y
4
(x
4
+ y
4
+ 2x
2
y
2
)
2
8
=
(xy(x
2
+ y
2
))
4
8
2
T ta c iu phi chng minh.
ng thc ch xy ra khi x = y = 1.2
10.3 Cho ba s thc khng m a, b, c tha mn
a
2
+ b
2
+ c
2
= 3.
Chng minh rng:
a
3
b
2
+ b
3
c
2
+ c
3
a
2
3
Li gii.
Gi (x; y; z) l mt hon v ca (a; b; c) sao cho x y z.
Khi theo bt ng thc hon v ta c:
a
3
b
2
+ b
3
c
2
+ c
3
a
2
x
3
y
2
+ x
2
yz
2
+ z
3
y
2
do 2 dy x; y; z v x
2
y
2
; z
2
x
2
; y
2
z
2
n iu cng chiu.
Nh vy ta s chng minh:
x
3
y
2
+ yx
2
z
2
+ z
3
y
2
3
p dng bt ng thc AM-GM:
2x
3
y
2
y(x
4
+ x
2
y
2
)
2z
3
y
2
y(z
4
+ z
2
y
2
)
Ch cn cn chng minh (vi ch x
2
+ y
2
+ z
2
= a
2
+ b
2
+ c
2
= 3):
y(x
4
+ x
2
y
2
) + y(z
4
+ z
2
y
2
) + 2yz
2
x
2
6
y(x
2
+ z
2
)(x
2
+ y
2
+ z
2
) 6
y(3 y
2
) 2
(y 1)
2
(y + 2) 0
Bt ng thc cui lun ng, nn ta c iu phi chng minh.
ng thc xy ra khi v ch khi x = y = z hay a = b = c = 1.2
10.4 Cho ba s thc dng a, b, c tha mn a + b + c = 1. Tm gi tr ln nht ca
_
ab
c + ab
+
_
bc
a + bc
+
_
ca
b + ca

3
2
Li gii.
Vi gi thit a + b + c = 1 th:
_
ab
c + ab
=
_
ab
1 a b + ab
=
_
ab
(1 a)(1 b)
=
_
ab
(b + c)(c + a)
Theo bt ng thc AM-GM ta c:
_
ab
c + ab
=
_
ab
(b + c)(c + a)

1
2
_
a
c + a
+
b
b + c
_
Tng t ta cng c:
_
bc
a + bc

1
2
_
b
a + b
+
c
c + a
_
;
_
ca
b + ca

1
2
_
c
b + c
+
a
a + b
_
Cng v vi v ca 3 bt ng thc trn ta c:
_
ab
c + ab
+
_
bc
a + bc
+
_
ca
b + ca

1
2
_
a
c + a
+
b
b + c
+
b
b + a
+
c
c + b
+
c
c + a
+
a
a + b
_
=
3
2
chnh l iu cn chng minh.
212
ng thc xy ra khi v ch khi a = b = c =
1
3
.2
10.5 Cho ba s thc dng a, b, c. Chng minh rng:
(a
2
+ b
2
+ c
2
)
2
3(a
3
b + b
3
c + c
3
a)
Li gii.
Li gii 1.
Bt ng thc cn chng minh tng ng vi:
2(a
2
+ b
2
+ c
2
)
2
6(a
3
b + b
3
c + c
3
a) 0

(a
2
2ab + bc c
2
+ ca)
2
0
iu ny hin nhin ng. Bt ng thc c chng minh.
Du ng thc xy ra khi a = b = c hoc trong b 3 s sau v cc hon v (a, b, c) =
k
_
sin
2 4
7
, sin
2 2
7
, sin
2
7
_
.2
Li gii 2.
Ta bit vi mi s thc x, y, z th
(x + y + z)
2
3(xy + yz + zx)
chn
x = a
2
+ bc ab, y = b
2
+ ca bc, z = c
2
+ ab ca
ta thu c
[

(a
2
+ bc ab)]
2
3

(a
2
bc ab)(b
2
+ ca bc)
Mt khc, ta thy rng

(a
2
+ bc ab) = a
2
+ b
2
+ c
2
v

(a
2
bc ab)(b
2
+ ca bc) = a
3
b + b
3
c + c
3
a
Nn ta c
(a
2
+ b
2
+ c
2
)
2
3(a
3
b + b
3
c + c
3
a).
Php chng minh hon tt.
Du ng thc xy ra khi a = b = c hoc trong b 3 s sau v cc hon v (a, b, c) =
k
_
sin
2 4
7
, sin
2 2
7
, sin
2
7
_
.2
Li gii 3.
Bng cch xt hiu hai v ca bt ng thc, ta c
_

a
2
_
2
3

a
2
b
=

_
1
2
a
2

_
3
4
+

5
4
_
.ab +

5
2
ca +
_

5
4

1
4
_
.b
2
+
_
3
4

5
4
_
.bc
_
1
4
+

5
4
_
.c
2
_
2
=

_
1
3
.a
2

_
1
2
+

15
6
_
.ab +

15
3
.ca +
_

15
6

1
6
_
.b
2
+
_
1
2

15
6
_
.bc
_
1
6
+

15
6
_
.c
2
_
2
=

_
1
4
.a
2

_
3
8
+

29
8
.ab
_
+

29
4
.ca +
_

29
8

1
8
_
.b
2
+
_
3
8

29
8
_
.bc
_
1
8
+

29
8
_
c
2
_
2
0.
T suy ra iu phi chng minh.
213
Du ng thc xy ra khi a = b = c hoc trong b 3 s sau v cc hon v (a, b, c) =
k
_
sin
2 4
7
, sin
2 2
7
, sin
2
7
_
.2
10.6 Cho ba s thc dng x, y, z tha mn
xy + yz + zx = 3.
Chng minh rng:
1
xyz
+
4
(x + y)(y + z)(z + x)

3
2
Li gii.
Theo Bt ng thc AM-GM ta c:
1
xyz
+
4
(x + y)(y + z)(z + x)
=
1
2xyz
+
1
2xyz
+
4
(x + y)(y + z)(z + x)

1
2xyz
+
2

2
_
xyz(x + y)(y + z)(z + x)
.
Mt khc, cng theo Bt ng thc AM-GM ta thy rng:
xyz =

xy.yz.zx

_
xy + yz + zx
3
_
3
= 1
v
xyz(x + y)(y + z)(z + x) = (xz + yz)(yx + zx)(zy + xy)

_
xz + yz + yx + zx + zy + xy
3
_
3
= 8.
T
1
xyz
+
4
(x + y)(y + z)(z + x)

1
2xyz
+
2

2
_
xyz(x + y)(y + z)(z + x)
.

1
2
+
2

8
=
3
2
.
Bi ton c chng minh xong.
ng thc xy ra khi v ch khi x = y = z = 1.2
10.7 Cho ba s thc dng x, y, z tha mn
x + y + z
3
2
.
Tm gi tr nh nht ca:
P =
_
x
2
+
1
y
2
+
4
z
2
+
_
y
2
+
1
z
2
+
4
x
2
+
_
z
2
+
1
x
2
+
4
y
2
Li gii.
S dng ln lt bt ng thc Mincopski, Cauchy-Schwarz, AM-GM v kt hp vi gi thit, ta
c:
214
P

(x + y + z)
2
+
_
1
x
+
1
y
+
1
z
_
2
+
_
2
x
+
2
y
+
2
z
_
2

(x + y + z)
2
+
81
(x + y + z)
2
+
324
(x + y + z)
2
=

_
(x + y + z)
2
+
81
16(x + y + z)
2
_
+
6399
16(x + y + z)
2

_
2.
9
4
+
6399.4
16.9
=
27
2
.
ng thc xy ra khi v ch khi x = y = z =
1
2
.
Vy gi tr nh nht ca P l
27
2
khi x = y = z =
1
2
.2
10.8 Cho ba s thc dng a, b, c
Chng minh rng:
ab
a + 9b + 6c
+
bc
b + 9c + 6a
+
ca
c + 9a + 6b

a + b + c
16
Li gii.
S dng bt ng thc Cauchy-Schwarz, ta c:
16
a + 9b + 6c
=
(1 + 3)
2
(3c + a) + 3(3b + c)

1
3c + a
+
3
3b + c
.
T ta suy ra
ab
a + 9b + 6c

1
16
_
ab
3c + a
+
3ab
3b + c
_
Chng minh tng t vi 2 biu thc cn li, sau cng v vi v, ta c:
ab
a + 9b + 6c
+
bc
b + 9c + 6a
+
ca
c + 9a + 6b

1
16
_
ab
3c + a
+
3ab
3b + c
+
bc
3a + b
+
3bc
3c + a
+
ca
3b + c
+
3ca
3a + b
_
=
a + b + c
16
.
Bi ton c chng minh xong.
ng thc xy ra khi v ch khi a = b = c.2
10.9 Cho a; b; c dng v a + b + c = 3. Chng minh rng
_
2a
2
+
2
a + 1
+ b
4
+
_
2b
2
+
2
b + 1
+ c
4
+
_
2c
2
+
2
c + 1
+ a
4
6
Li gii.
S dng bt ng thc Minkowski, kt hp vi bt ng thc AM-GM v gi thit, ta c
215
_
2a
2
+
2
a + 1
+ b
4
+
_
2b
2
+
2
b + 1
+ c
4
+
_
2c
2
+
2
c + 1
+ a
4

2(a + b + c)
2
+ 2
_
1

a + 1
+
1

b + 1
+
1

c + 1
_
2
+ (a + b + c)
2
=

27 + 2
_
1

a + 1
+
1

b + 1
+
1

c + 1
_
2

_
27 + 2
_
3
6
_
(a + 1)(b + 1)(c + 1)
_
2
=

27 +
18
3
_
(a + 1)(b + 1)(c + 1)

_
27 +
54
a + 1 + b + 1 + c + 1
= 6.
Nh vy ta c iu phi chng minh.
ng thc xy ra khi v ch khi a = b = c = 1.2
10.10 Cho tam gic ABC nhn. Chng minh rng
cos
_
A B
2
_
+ cos
_
B C
2
_
+ cos
_
C A
2
_

2
2
_
a + b

a
2
+ b
2
+
b + c

b
2
+ c
2
+
c + a

c
2
+ a
2
_
Li gii.
Bng 1 s php bin i lng gic trong tam gic, ta c:
cos
_
A B
2
_
=
cos
_
A B
2
_
cos
_
A + B
2
_
sin
C
2
=
cos A + cos B
2
_
1 cos C
2
=
b
2
+ c
2
a
2
2bc
+
a
2
+ c
2
b
2
2ac
2

_
1
a
2
+ b
2
c
2
2ab
2
=
(a + b)(c
2
(a b)
2
)
abc
2
_
c
2
(a b)
2
ab
=
(a + b)
_
c
2
(a b)
2
2c

ab
Ta s chng minh
216
(a + b)
_
c
2
(a b)
2
2c

ab

a + b
_
2(a
2
+ b
2
)

2abc
2
c
2
(a b)
2
a
2
+ b
2

2ab(a b)
2
c
2
(a b)
2
(a b)
2
a
2
+ b
2
c
2
.
Bt ng thc cui lun ng do ABC l tam gic nhn.
Vy:
cos
_
A B
2
_

a + b
_
2(a
2
+ b
2
)
Thit lp 2 biu thc tng t v cng v vi v, ta c:
cos
_
A B
2
_
+ cos
_
B C
2
_
+ cos
_
C A
2
_

2
2
_
a + b

a
2
+ b
2
+
b + c

b
2
+ c
2
+
c + a

c
2
+ a
2
_
.
chnh l iu m ta cn chng minh.2
10.11 Cho ba s thc dng a, b, c.
Chng minh rng:
a
2
b
+
b
2
c
+
c
2
a

a
2
ab + b
2
+

b
2
bc + c
2
+

c
2
+ a
2
ac
Li gii.
Cch 1:
S dng Bt ng thc AM-GM ta c:
a
2
ab + b
2
b
+ b 2

a
2
ab + b
2
Tung t ta cng c
b
2
bc + c
2
c
+ c 2

b
2
bc + c
2
c
2
ca + a
2
a
+ a 2

c
2
ca + a
2
Cng v vi v ba bt ng thc trn, ta c:
a
2
b
+
b
2
c
+
c
2
a
2
_
a
2
ab + b
2
+

b
2
bc + c
2
+

c
2
+ a
2
ac
_
a b c
Li c theo Bt ng thc Mikowsyki, ta c:

a
2
ab + b
2
+

b
2
bc + c
2
+

c
2
+ a
2
ac

_
a
b
2
+ b
c
2
+ c
a
2
_
2
+
3
4
(a + b + c)
2
= a + b + c.
T ta c iu phi chng minh.
ng thc xy ra khi v ch khi a = b = c.2
Cch 2:
Ta vit bt ng thc cn chng minh li nh sau:
2
_
a
2
b
+
b
2
c
+
c
2
a
_
2
_
a
2
ab + b
2
+

b
2
bc + c
2
+

c
2
ca + a
2
_
S dng bt ng thc Cauchy-Schwarz, ta c:
217
a
2
b
+
b
2
c
+
c
2
a

(a + b + c)
2
a + b + c
= a + b + c
Vy chng minh bi ton ta cn chng minh bt ng thc mnh hn sau y :
a
2
b
+
b
2
c
+
c
2
a
+ a + b + c 2
_
a
2
ab + b
2
+

b
2
bc + c
2
+

c
2
ca + a
2
_
hay l:
_
a
2
ab + b
2
b
+ b
_
+
_
b
2
bc + c
2
c
+ c
_
+
_
c
2
ca + a
2
a
+ a
_
2
_

a
2
ab + b
2
+

b
2
bc + c
2
+

c
2
ca + a
2
_
Bt ng thc cui cng hin hin ng theo bt ng thc AM-GM nn ta c iu phi chng
minh.
ng thc xy ra khi v ch khi a = b = c.2
10.12 Cho ba s thc a, b, c (0; 1).
Chng minh rng:
(a a
2
)(b b
2
)(c c
2
) (a bc)(b ca)(c ab)
Li gii.
Ta c:
(a a
2
)(b b
2
)(c c
2
) = abc abc
2
+ abc.(ab + bc + ca) abc.(a + b + c)
v:
(a bc)(b ca)(c ab) = abc abc
2
+ abc.(a
2
+ b
2
+ c
2
) (a
2
b
2
+ b
2
c
2
+ c
2
a
2
)
Khi , bt ng thc tng ng vi:
abc.(a
2
+ b
2
+ c
2
) (a
2
b
2
+ b
2
c
2
+ c
2
a
2
) abc.(a + b + c) abc.(a + b + c)
abc.(a
2
+ b
2
+ c
2
) abc.(ab + bc + ca) (a
2
b
2
+ b
2
c
2
+ c
2
a
2
) abc.(a + b + c)
abc.[(a b)
2
+ (b c)
2
+ (c a)
2
] b
2
.(c a)
2
+ c
2
.(a b)
2
+ a
2
.(b c)
2
S
a
.(b c)
2
+ S
b
.(c a)
2
+ S
c
.(a b)
2
0 (1).
Vi: S
a
= a
2
abc; S
b
= b
2
abc; S
c
= c
2
abc.
M:
S
a
+ S
b
+ S
c
= a
2
+ b
2
+ c
2
3a
2
b
2
c
2
3.(abc)
2
3
3(abc)
2
0.(do: abc (0, 1)).
V:
S
a
.S
b
+ S
b
.S
c
+ S
c
.S
a
=

a
2
b
2
+ 3a
2
b
2
c
2
2abc.(ab + bc + ca) 0.
(v: a
2
b
2
+ a
2
b
2
c
2
2a
2
b
2
c; b
2
c
2
+ a
2
b
2
c
2
2b
2
c
2
a; c
2
a
2
+ a
2
b
2
c
2
2c
2
a
2
b)
Nn theo nh l S.O.S ta c Bt ng thc (1) ng.
Do ta c iu phi chng minh.
ng thc xy ra khi v ch khi a = b = c.2
10.13 Cho ba s thc dng a, b, c.
Tm gi tr nh nht ca:
P =
a + b
a + b + c
+
b + c
b + c + 4a
+
c + a
c + a + 16b
Li gii.
t
218
_

_
a + b + c = x
b + c + 4a = y
c + a + 16b = z

_
a =
y x
3
b =
z x
15
c =
21x 5y z
15

_
a + b =
5y + z 6x
15
b + c =
4x y
3
c + a =
16x z
15
Khi ta c:
P =
y
3x
+
4x
3y
+
z
15x
+
16x
15z

4
5
S dng bt ng thc AM-GM, ta c:
y
3x
+
4x
3y
=
1
3
(
y
x
+
4x
y
)
4
3
v
z
15x
+
16x
15z
=
1
15
(
z
x
+
16x
z
)
8
15
Do :
P
4
3
+
8
15

4
5
=
16
15
.
ng thc xy ra 4x = 2y = z a =
5b
3
=
5c
7
.
Vy gi tr nh nht ca P l
16
15
.2
10.14 Cho ba s thc dng a, b, c tha mn abc = 1
Chng minh rng:
a

b
2
+ 3
+
b

c
2
+ 3
+
c

a
2
+ 3

3
2
Li gii.
Li gii 1.
Theo bt ng thc AM-GM ta c: ab + bc + ca 3
Mt khc, cng theo bt ng thc AM-GM th
a

b
2
+ 3

b
2
+ ab + bc + ca
=
a
_
(b + a)(b + c)

2a
a + 2b + c
Thit lp 2 biu thc tng t, ri cng v vi v, ta c:
a

b
2
+ 3
+
b

c
2
+ 3
+
c

a
2
+ 3

2a
a + 2b + c
+
2b
b + 2c + a
+
2c
c + 2a + b
M theo bt ng thc Cauchy-Schwarz ta c:
2a
a + 2b + c
+
2b
b + 2c + a
+
2c
c + 2a + b

2(a + b + c)
2
a
2
+ b
2
+ c
2
+ 3 (ab + bc + ca)
=
2(a + b + c)
2
(a + b + c)
2
+ (ab + bc + ca)

3
2
.
Do ta c iu phi chng minh.
ng thc xy ra khi v ch khi a = b = c = 1.2
Li gii 2.
V a
2
b
2
c
2
= 1 nn ta c th thay b (a, b, c) bi
_
x
y
,
z
x
,
y
z
_
.
Khi ta a bt ng thc v dng ng bc l
219
x

3xy + yz
+
y

3yz + zx
+
z

3zx + xy

3
2
S dng bt ng thc Holder, ta c
_
x

3xy + yz
_
2
[

x(3xy + yz)] (x + y + z)
3
Vy, ta cn chng minh c
(x + y + z)
3

27
4
(x
2
y + y
2
z + z
2
x + xyz)
Gi s z l s nm gia 3 s x, y, z. Khi ta c:
x(z x)(z y) 0
xz
2
+ x
2
y x
2
z + xyz
S dng nh gi trn v kt hp vi bt ng thc AM-GM, ta c:
x
2
y + y
2
z + z
2
x + xyz z(x + y)
2
AMGM

4
27
_
z + 2.
x + y
2
_
3
=
4(x + y + z)
3
27
.
Bi ton c chng minh xong.
ng thc xy ra khi v ch khi a = b = c = 1.2
10.15 Cho ba s thc dng a, b, c.
Chng minh rng:
2(a
3
+ b
3
+ c
3
)
abc
+
9(a + b + c)
2
a
2
+ b
2
+ c
2
33
Li gii.
Li gii 1. Bt ng thc cn chng minh tng ng vi:
2 (a
3
+ b
3
+ c
3
3abc)
abc
9
3(a
2
+ b
2
+ c
2
) (a + b + c)
2
a
2
+ b
2
+ c
2
(a
2
+ b
2
+ c
2
ab bc ca)
_
a + b + c
abc

9
a
2
+ b
2
+ c
2
_
0
(a
2
+ b
2
+ c
2
ab bc ca)
_
(a + b + c)(a
2
+ b
2
+ c
2
) 9abc

0
y l 1 iu hin nhin ng theo AM-GM, do php chng minh ca ta hon tt.
ng thc xy ra khi v ch khi a = b = c.2
Li gii 2. t p = a + b + c, q = ab + bc + ac, r = abc
Bt ng thc cn chng minh tng ng vi:
2
p
3
3pq + 3r
r
+ 9
p
2
p
2
2q
33
2
p(p
2
3q)
r
+ 9
p
2
p
2
2q
27
Ta c r
pq
9
nn:
2
p(p
2
3q)
r
+ 9
p
2
p
2
2q
18
p
2
3q
q
+ 9
_
1 +
2q
p
2
2q
_
Ta s chng minh
18
p
2
3q
q
+ 9
_
1 +
2q
p
2
2q
_
27

p
2
q
+
p
p
2
2q
4
(p
2
3q)
2
0
Bt ng thc cui ng, vy ta c iu phi chng minh.
220
ng thc xy ra khi v ch khi a = b = c.2
10.16 Cho ba s thc dng a, b, c.
Chng minh rng:
a + b + c
3

abc
+
8abc
(a + b)(b + c)(c + a)
4
Li gii.
Bt ng thc c th vit li nh sau
a + b
2
3

abc
+
b + c
2
3

abc
+
c + a
2
3

abc
+
8abc
(a + b)(b + c)(c + a)
4
Hin nhin ng theo bt ng thc AM-GM.
ng thc xy ra khi v ch khi a = b = c.2
10.17 Cho ba s thc dng x, y, z.
Chng minh rng:
x + y + z
3

xyz
+
4xyz
x
2
y + y
2
z + z
2
x + xyz
4
Li gii.
Gi s z l s nm gia 3 s x, y, z. Khi ta c:
x(z x)(z y) 0
xz
2
+ x
2
y x
2
z + xyz
S dng nh gi trn v kt hp vi bt ng thc AM-GM, ta c:
x
2
y + y
2
z + z
2
x + xyz z(x + y)
2
AMGM

4
27
_
z + 2.
x + y
2
_
3
=
4(x + y + z)
3
27
.
S dng kt qu trn, v theo bt ng thc AM-GM, ta c:
x + y + z
3

xyz
+
4xyz
x
2
y + y
2
z + z
2
x + xyz
3.
x + y + z
3
3

xyz
+
27xyz
(x + y + z)
3
AMGM
4.
Bi ton c chng minh xong.
ng thc xy ra khi v ch khi x = y = z.2
10.18 Cho ba s thc khng m a, b, c tha mn a + b + c = 3.
Chng minh rng:
a + 1
b + 1
+
b + 1
c + 1
+
c + 1
a + 1

25
3
3

4ab + 4bc + 4ca


Li gii.
S dng bt ng thc AM-GM, ta c
3
3
_
4(ab + bc + ca) 2+2+(ab+bc+ca) abc+ab+bc+ca+a+b+c+1 = (a+1)(b+1)(c+1).
Vy, ta cn chng minh c
a + 1
b + 1
+
b + 1
c + 1
+
c + 1
a + 1

25
(a + 1)(b + 1)(c + 1)
Hay l
(a + 1)
2
(c + 1) + (b + 1)
2
(a + 1) + (c + 1)
2
(b + 1) 25
ab
2
+ bc
2
+ ca
2
+ (a + b + c)
2
+ 3(a + b + c) + 3 25
ab
2
+ bc
2
+ ca
2
4
By gi ta gi s b l s nm gia 3 s a, b, c. Khi ta c:
221
a(b a)(b c) 0
ab
2
+ a
2
c abc + a
2
b
S ng nh gi trn, kt hp vi bt ng thc AM-GM, ta c:
ab
2
+ bc
2
+ ca
2
a
2
b + bc
2
+ abc = b(a
2
+ ac + c
2
)
b(a + c)
2

4
27
_
b + 2.
a + c
2
_
3
=
4(a + b + c)
3
27
= 4.
Php chng minh hon tt.
ng thc xy ra khi (a, b, c) l mt hon v ca (0, 1, 2).2
10.19 Cho ba s thc a, b, c tha mn a
2
+ b
2
+ c
2
= 2.
Chng minh rng:
|a
3
+ b
3
+ c
3
abc| 2

2
Li gii.
t t = ab th ta c t = ab; |t|
a
2
+ b
2
2

a
2
+ b
2
+ c
2
2
= 1
S dng bt ng thc Cauchy-Schwarz ta c:
_
a
3
+ b
3
+ c(c
2
ab)

_
(a + b)
2
+ c
2

_
(a
2
ab + b
2
)
2
+ (c
2
ab)
2
_
= 2(1 + t)
_
(c
2
ab)
2
+ (2 c
2
ab)
2
_
= 2(1 + t)
_
2c
4
+ 2a
2
b
2
+ 4 4c
2
4ab
_
= 4(1 + t)
_
t
2
2t + 2 + c
2
(c
2
2)

4(t + 1)(t
2
2t + 2)
Do ta ch cn chng minh
(t + 1)(t
2
2t + 2) 2 t
2
(t 1) 0 .
Bt ng thc cui lun ng, php chng minh hon tt.
ng thc xy ra khi v ch khi a = b = 0, c =

2.2
10.20 Cho ba s thc dng a, b, c.
Chng minh rng:
a
4
+ b
4
+ c
4
ab + bc + ca
+
3abc
a + b + c

2
3
(a
2
+ b
2
+ c
2
)
Li gii.
Trc ht, ta c 2 bt ng thc ph sau:
Ta c:
a
3
+ b
3
+ c
3
a + b + c
+
1
3
(ab + bc + ca)
2
3
(a
2
+ b
2
+ c
2
)
3(a
3
+ b
3
+ c
3
) + (a + b + c)(ab + bc + ca) 2(a + b + c)(a
2
+ b
2
+ c
2
)
a
3
+ b
3
+ c
3
+ 3abc ab(a + b) + bc(b + c) + ca(c + a)
Lun ng theo Schur bc 3.
222
Ta cng c:
a
4
+ b
4
+ c
4
ab + bc + ca
+
2
3
(ab + bc + ca) a
2
+ b
2
+ c
2
3(a
4
+ b
4
+ c
4
) + 2(ab + bc + ca)
2
3(a
2
+ b
2
+ c
2
)(ab + bc + ca)
a
4
+ b
4
+ c
4
+ abc(a + b + c) ab(a
2
+ b
2
) + bc(b
2
+ c
2
) + ca(c
2
+ a
2
)
Bt ng thc cui ng do s dng Schur bc 4 v v a
4
+b
4
+2a
2
b
2
= (a
2
+b
2
)
2
2ab(a
2
+b
2
).
Tr li bi ton, bt ng thc ca bi ton m ta cn chng minh tng ng vi
a
4
+ b
4
+ c
4
ab + bc + ca
+
a
3
+ b
3
+ c
3
a + b + c

a
3
+ b
3
+ c
3
3abc
a + b + c
+
2
3
(a
2
+ b
2
+ c
2
)

a
4
+ b
4
+ c
4
ab + bc + ca
+
a
3
+ b
3
+ c
3
a + b + c
+ ab + bc + ca
5
3
(a
2
+ b
2
+ c
2
)
Tht vt bt ng thc ny ng sau khi cng v theo v ca 2 bt ng thc ph m ta
chng minh trn.
Php chng minh hon tt.
ng thc xy ra khi v ch khi a = b = c.2
10.21 Cho ba s thc khng m a, b, c, d, e.
Chng minh rng:
a
6
b + b
6
c + c
6
d + d
6
e + e
6
a abcde(a
2
+ b
2
+ c
2
+ d
2
+ e
2
)
Li gii.
Nu abcde = 0 th bt ng thc hin nhin ng.
Vi abcde = 0 ta c bt ng thc tng ng vi
a
5
cde
+
b
5
dea
+
c
5
eab
+
d
5
abc
+
e
5
bcd
a
2
+ b
2
+ c
2
+ d
2
+ e
2
S dng bt ng thc AM-GM, ta c
a
5
cde
+
a
5
cde
+ c
2
+ d
2
+ e
2
5
5
_
a
5
cde
.
a
5
cde
.c
2
.d
2
.e
2
= 5a
2
Thc hin tng t cho cc hng t cn li, sau cng v theo v ta c iu phi chng minh.
ng thc xy ra khi v ch khi a = b = c = d. 2
10.22 Cho ba s thc dng a, b, c.
Chng minh rng:
a
a
2
+ bc
+
b
b
2
+ ac
+
c
c
2
+ ab

3(a + b + c)
2(ab + bc + ca)
Li gii.
Bt ng thc cn chng minh tng ng vi:
a
2
(b + c)
a
2
+ bc
+
b
2
(c + a)
b
2
+ ca
+
c
2
(a + b)
c
2
+ ab
+ abc
1
a
2
+ bc

3(a + b + c)
2
Ta nhn thy rng bt ng thc ny c suy trc tip t 2 kt qu sau:
1)
a
2
(b + c)
a
2
+ bc
+
b
2
(c + a)
b
2
+ ca
+
c
2
(a + b)
c
2
+ ab
a + b + c
2)
2
a
2
+ bc
+
2
b
2
+ ca
+
2
c
2
+ ab

1
ab
+
1
bc
+
1
ca
Chng minh 1):
t (x; y; z) (a
1
; b
1
; c
1
) , ta chuyn bt ng thc thnh:
x + y
z
2
+ xy
+
y + z
x
2
+ yz
+
z + x
y
2
+ zx

1
x
+
1
y
+
1
z
Khng mt tnh tng qut, gi s x y z, khi ta c:
223
V P V T =
_
1
z

x + y
z
2
+ xy
_
+
_
1
x

z + x
y
2
+ zx
_
+
_
1
y

y + z
x
2
+ yz
_
=
(z x)(z y)
z
3
+ xyz
+
(y
2
x
2
)(y x)(zx + yz xy)
xy(x
2
+ yz)(y
2
+ zx)
0.
Nh vy, bt ng thc 1) c chng minh.
Chng minh 2):
p dng bt ng thc AM-GM:
4
a
2
+ bc

2
a

bc

1
ab
+
1
ac
Cng v theo v cc bt ng thc tng t ta c bt ng thc 2) c chng minh.
Php chng minh hon tt.
ng thc xy ra khi v ch khi a = b = c.2
10.23 Cho ba s thc dng a, b, c tha mn a + b + c = 3.
Chng minh rng:
1
a
2
+
1
b
2
+
1
c
2
a
2
+ b
2
+ c
2
Li gii.
Cch 1. S dng hng ng thc quen thuc
(a + b + c)
2
= a
2
+ b
2
+ c
2
+ 2(ab + bc + ca)
Ta a bt ng thc v dng
1
a
2
+
1
b
2
+
1
c
2
+ 2(ab + bc + ca) (a + b + c)
2
Theo bt ng thc AM-GM v gi thit ta c
1
a
2
+
1
b
2
+
1
c
2
+ 2(ab + bc + ca)
1
ab
+
1
bc
+
1
ca
+ 2
_
3abc(a + b + c)
= 3
_
1
abc
+

abc +

abc
_
9 = (a + b + c)
2
.
Php chng minh hon tt.
ng thc xy ra khi v ch khi a = b = c = 1.2
Cch 2. S dng bt ng thc AM-GM ta c
1
a
2
+
1
b
2
+
1
c
2

1
ab
+
1
bc
+
1
ca
=
a + b + c
abc
=
3
abc
V th ta cn phi chng minh
3
abc
a
2
+ b
2
+ c
2
Hay
abc(a
2
+ b
2
+ c
2
) 3
n y ta c hai hng tn cng.
Hng 1. Dn bin
Gi s c = min{a, b, c} th 3 = a + b + c 3c, tc c 1 dn n
a + b
2
1
t
f(a, b, c) = abc(a
2
+ b
2
+ c
2
)
Ta c
224
f (a, b, c) f
_
a + b
2
,
a + b
2
, c
_
= c
__
ab(a
2
+ b
2
)
(a + b)
4
8
_
+
_
abc
2

(bc + ca)
2
4
__
M theo bt ng thc AM-GM th
(a + b)
4
= (a
2
+ b
2
+ 2ab)
2
8ab(a
2
+ b
2
)
(bc + ca)
2
4bc.ca = 4abc
2
nn ta c
f (a, b, c) f
_
a + b
2
,
a + b
2
, c
_
cui cng ta ch cn chng minh
f
_
a + b
2
,
a + b
2
, c
_
3
t x =
a + b
2
1, t gii thit ta rt ra c c = 3 2x. Xt
f
_
a + b
2
,
a + b
2
, c
_
3 = (4x
5
14x
4
+ 8x
3
9x
2
1) = (x 1)
2
[2x(x 1)(2x 1) + 1] 0
T suy ra iu phi chng minh.2
Hng 2. Dng bt ng thc c in
Ta vit bt ng thc cn chng minh li nh sau
27 3.abc(a + b + c)(a
2
+ b
2
+ c
2
)
S dng bt ng thc quen thuc
(ab + bc + ca)
2
3abc(a + b + c)
Ta a bt ng thc v chng minh
27 (a
2
+ b
2
+ c
2
)(ab + bc + ca)
2
Tht vy, theo bt ng thc AM-GM th
(a
2
+ b
2
+ c
2
)(ab + bc + ca)
2

_
a
2
+ b
2
+ c
2
+ ab + bc + ca + ab + bc + ca
3
_
3
= 27
Chng minh hon tt.
ng thc xy ra khi v ch khi a = b = c = 1. 2
Cch 3. t x = ab + bc + ca. Khi s dng cc bt ng thc quen thuc
(a + b + c)
2
3(ab + bc + ca), (ab + bc + ca)
2
3abc(a + b + c)
Ta c
0 < x 3 v abc
x
2
9
V th
1
a
2
+
1
b
2
+
1
c
2
=
_
1
a
+
1
b
+
1
c
_
2
2
_
1
ab
+
1
bc
+
1
ca
_
=
x
2
a
2
b
2
c
2

6
abc
Ta s chng minh
x
2
6abc (9 2x)a
2
b
2
c
2
Tht vy
V T V P x
2

2x
2
3

x
2
(9 2x)
81
=
x
2
(x 3)
2
(2x + 3)
81
0
Bi ton c chng minh hon ton.
ng thc xy ra khi v ch khi a = b = c = 1.2
10.24 Cho ba s thc dng a, b, c tha mn a
2
+ b
2
+ c
2
= 3.
Chng minh rng:
a + b + c a
2
b
2
+ b
2
c
2
+ c
2
a
2
Li gii.
225
Cch 1. t x = a
2
, y = b
2
, z = c
2
th x + y + z = 3 v bt ng thc tr thnh

x +

y +

z xy + yz + zx
Tng ng vi
2
_

x +

y +

z
_
+ x
2
+ y
2
+ z
2
(x + y + z)
2
Theo bt ng thc AM-GM th

x +

x + x
2
3x
V th m
V T 3(x + y + z) = (x + y + z)
2
Bi ton c chng minh xong.
ng thc xy ra khi v ch khi a = b = c = 1.2
Cch 2. S dng bt ng thc Holder, ta c
_

x +

y +

z
_
2
(x
2
+ y
2
+ z
2
) (x + y + z)
3
= 27
Vy ta ch cn chng minh c
(x
2
+ y
2
+ z
2
).(xy + yz + zx)
2
27
Tht vy, theo bt ng thc AM-GM th
(x
2
+ y
2
+ z
2
)(xy + yz + zx)
2

_
x
2
+ y
2
+ z
2
+ xy + yz + zx + xy + yz + zx
3
_
3
= 27
Chng minh hon tt.
ng thc xy ra khi v ch khi a = b = c = 1. 2
10.25 Cho ba s thc dng a, b, c.
Chng minh rng:
a
2
+ b
2
+ c
2
+ 2abc + 1 2(ab + bc + ca)
Li gii.
Trong 3 s a, b, c th lun tn ti 2 s nm cng pha so vi 1.
Gi s 2 s l a v b.
Khi ta c:
c(a 1)(b 1) 0
Mt khc, ta thy rng:
a
2
+ b
2
+ c
2
+ 2abc + 1 2ab 2bc 2ca = (a b)
2
+ (c 1)
2
+ 2c(a 1)(b 1) 0.
chnh l iu ta cn chng minh.
ng thc xy ra khi v ch khi a = b = c = 1.2
10.26 Cho ba s thc a, b, c.
Chng minh rng:
(2 + a
2
)(2 + b
2
)(2 + c
2
) 9(ab + bc + ca)
Li gii.
Ta c: (2 + a
2
)(2 + b
2
)(2 + c
2
) = 4 (a
2
+ b
2
+ c
2
) + 2 (a
2
b
2
+ b
2
c
2
+ c
2
a
2
) + 8 + a
2
b
2
c
2
rng ta c cc bt ng thc sau:
_
a
2
+ b
2
+ c
2
_
+ a
2
b
2
c
2
+ 2
_
a
2
+ b
2
+ c
2
_
+ 3
3

a
2
b
2
c
2

_
a
2
+ b
2
+ c
2
_
+
9abc
a + b + c
Schur
2 (ab + bc + ca) .
226
v:
3 (a
2
+ b
2
+ c
2
) 3 (ab + bc + ca).
2 (a
2
b
2
+ b
2
c
2
+ c
2
a
2
+ 3) 2 (2ab + 2bc + 2ca).
Cng v theo v cc bt ng thc trn ta thu ngay iu phi chng minh.
ng thc xy ra khi v ch khi a = b = c.2
10.27 Cho ba s thc dng x, y, z tha mn x + y + z = 1.
Chng minh rng:
x
1 + x
2
+
y
1 + y
2
+
z
1 + z
2

9
10
Li gii.
Cch 1:
S dng bt ng thc AM-GM ta c:
x
2
+ 1 = x
2
+
1
9
+
1
9
+
1
9
+
1
9
+
1
9
+
1
9
+
1
9
+
1
9
+
1
9
10
10
_
x
2
9
9
= 10
5
_
x
3
9
Thit lp 2 biu thc tng t, sau cng v theo v, ta c:
x
1 + x
2
+
y
1 + y
2
+
z
1 + z
2

3
10
_
5
_
(3x)
4
+
5
_
(3y)
4
+
5
_
(3z)
4
_
.
Mt khc, cng theo bt ng thc AM-GM, ta nhn thy rng:
3x + 3x + 3x + 3x + 1 5
5
_
(3x)
4
.
Tng t vi y, z, v ch x + y + z = 1, ta suy ra:
5
_
(3x)
4
+
5
_
(3y)
4
+
5
_
(3z)
4
3
T :
x
1 + x
2
+
y
1 + y
2
+
z
1 + z
2

9
10
Bi ton c chng minh xong.
ng thc xy ra khi v ch khi x = y = z =
1
3
.2
Cch 2:
Ta c:
x
1 + x
2

72x
100
+
3
50
Tht vy, bt ng thc trn tng ng vi 1 bt ng thc lun ng (vi mi x dng) sau:
(4x + 3)(3x 1)
2
0
Tng t vi y, z, sau cng v vi v, ta c:
x
1 + x
2
+
y
1 + y
2
+
z
1 + z
2

72
100
(x + y + z) +
9
50
=
9
10
.
Php chng minh hon tt.
ng thc xy ra khi v ch khi x = y = z =
1
3
. 2
10.28 Cho ba s thc khng m a, b, c tha mn a
2
+ b
2
+ c
2
= 1.
Chng minh rng:
a
1 + bc
+
b
1 + ca
+
c
1 + ab

2
Li gii.
Ta s chng minh rng:
(a + b + c)
2
2(1 + bc)
2
227
Tht vy, kt hp vi gi thit a
2
+ b
2
+ c
2
= 1 th bt ng thc trn s tng ng vi:
2(ab + bc + ca) 1 + 4bc + 2b
2
c
2
2a(b + c) a
2
+ (b + c)
2
+ 2b
2
c
2
(b + c a)
2
+ 2b
2
c
2
0
Bt ng thc cui lun ng.
T ta suy ra:
a
1 + bc

a

2
a + b + c
Tng t 2 biu thc cn li v cng v theo v ta c:
a
1 + bc
+
b
1 + ca
+
c
1 + ab

a

2
a + b + c
+
b

2
a + b + c
+
c

2
a + b + c
=

2
ng thc xy ra khi v ch khi a = b =
1

2
, c = 0 hoc cc hon v tng ng. 2
10.29 Cho a, b, c l di 3 cnh ca 1 tam gic.
Chng minh rng:
_
abc
a + b c
+
_
abc
b + c a
+
_
abc
c + a b
ab + bc + ca
Li gii.
Li gii 1.
t x = b + c a, y = c + a b, z = a + b c, bt ng thc khi tng ng vi
_
(x + y)(y + z)(z + x)
8

_
1

x
+
1

y
+
1

z
_
x + y + z.
Bnh phng hai v v quy ng, ta c
(x + y)(y + z)(z + x)
_

xy +

yz +

zx
_
2
8xyz(x + y + z)
2
.
t tip m =

x, n =

y, p =

z, bt ng thc tr thnh
(m
2
+ n
2
)(n
2
+ p
2
)(p
2
+ m
2
)
8m
2
n
2
p
2

_
m
2
+ n
2
+ p
2
mn + np + pm
_
2
.
rng ta c nhn xt sau:
Vi x y > 0 v z > 0 th ta c
x
y

x + z
y + z
T nhn xt suy ra
m
2
+ n
2
2mn

m
2
+ n
2
+ p
2
2mn + p
2
m
2
+ p
2
2mp

m
2
+ n
2
+ p
2
2mp + n
2
p
2
+ n
2
2pn

m
2
+ n
2
+ p
2
2pn + m
2
Vy ta ch cn chng minh
(m
2
+ n
2
+ p
2
)(mn + mp + np)
2
(m
2
+ 2np)(n
2
+ 2mp)(p
2
+ 2mn)
(mn)
2
(mp)
2
(n p)
2
0
Bt ng thc cui lun ng, vy ta c iu phi chng minh.
ng thc xy ra khi v ch khi a = b = c.2
Li gii 2.
Theo Bt ng thc Schur bc 4, ta c
abc(a + b + c)

(a + b c)c
3
Mt khc, theo Bt ng thc Holder ta c:
228

(a + b c)c
3
=
c
3
__
1
a + b c
_
2

(a + b + c)
3
_
1

a + b c
_
2
Kt hp 2 iu trn, ta suy ra
abc(a + b + c)

(a + b + c)
3
_
1

a + b c
_
2

_
abc
a + b c
+
_
abc
b + c a
+
_
abc
c + a b
a + b + c
chnh l iu cn chng minh.
ng thc xy ra khi v ch khi a = b = c.2
10.30 Cho k 1.
Chng minh rng:
k
k
(k + 1)
k1
Li gii.
V k = 1 th bt ng thc tr thnh ng thc nn ta ch cn xt k > 1
Ly Logarit Nepe hai v, ta c
k ln k (k 1) ln(k + 1).
Hay vit li di dng
ln k
k 1

ln(k + 1)
k
.
n y c th thy ngay l ta cn chng minh hm sau nghch bin
f(x) =
ln x
x 1
vi x > 1.
Ly o hm f(x) ta c
f

(x) =
1
1
x
ln x
(x 1)
2
=
g(x)
(x 1)
2
.
Ly o hm g(x), ta c
g

(x) =
1
x
2

1
x
< 0.
Suy ra
g(x) < lim
x1
+
g(x) = 0.
Suy ra
f

(x) < 0.
T ta c ngay hm f(x) nghch bin trn (1; +).
T suy ra iu phi chng minh. 2.
10.31 Cho a, b, c l cc s thc dng tha mn abc = 1.
Chng minh rng:
1
a(a + b)
+
1
b(b + c)
+
1
c(c + a)

3
2
Li gii.
Vit bt ng thc li thnh
bc
a + b
+
ca
b + c
+
ab
c + a

3
2
.
229
Dng bt ng thc hon v, ta c
(hoc cng c th chng minh bng phn tch dng M(a b)
2
+ N(a c)(b c) 0)
bc
a + b
+
ca
b + c
+
ab
c + a

ab
a + b
+
bc
b + c
+
ca
c + a
.
Nh vy (bc cui dng AM-GM)
bc
a + b
+
ca
b + c
+
ab
c + a

1
2
_
bc
a + b
+
ca
b + c
+
ab
c + a
_
+
1
2
_
ab
a + b
+
bc
b + c
+
ca
c + a
_
=
1
2
_
b(c + a)
a + b
+
c(a + b)
b + c
+
a(b + c)
c + a
_

3
2

3
_
b(c + a)
a + b

c(a + b)
b + c

a(b + c)
c + a
=
3
2
.
Ta c iu phi chng minh.
ng thc xy ra khi v ch khi a = b = c = 1.2
10.32 Cho a, b, c l cc s thc dng tha mn
1
a + b + 1
+
1
b + c + 1
+
1
c + a + 1
1
.
Chng minh rng:
a + b + c ab + bc + ca
Li gii.
Cch 1:
S dng bt ng thc Cauchy-Schwarz ta c:
(a + b + 1)(a + b + c
2
) (a + b + c)
2
Suy ra:
1

1
a + b + 1

a + b + c
2
(a + b + c)
2
(a + b + c)
2
2(a + b + c) + a
2
+ b
2
+ c
2
ab + bc + ca a + b + c
hay ta c iu phi chng minh.
ng thc xy ra khi v ch khi a = b = c = 1.2
Cch 2:
S dng bt ng thc Cauchy-Schwarz ta c:
2

(1
1
a + b + 1
) =

a + b
a + b + 1

(a + b + b + c + c + a)
2
(a + b)(a + b + 1) + (b + c)(b + c + 1) + (c + a)(c + a + 1)
a
2
+ b
2
+ c
2
+ ab + bc + ca + a + b + c (a + b + c)
2
ab + bc + ca a + b + c
hay ta c iu phi chng minh.
ng thc xy ra khi v ch khi a = b = c = 1. 2
Cch 3:
Gi s tn ti cc s dng a,b,c sao cho:
230
1
a + b + 1
1 v a + b + c < ab + bc + ca.
Khi ta c:
1
a + b + 1
<
ab + bc + ca
a + b + c
a + b + c +
ab + bc + ca
a + b + c
=
ab + bc + ca
(a + b)(a + b + c) + ab + bc + ca
Suy ra:

ab + bc + ca
(a + b)(a + b + c) + ab + bc + ca
> 1
1 >

_
1
2(ab + bc + ca)
(a + b)(a + b + c) + ab + bc + ca
_
1 >

a
2
+ ab + b
2
(a + b)(a + b + c) + ab + bc + ca

3
4

(a + b)
2
(a + b)(a + b + c) + ab + bc + ca

3(a + b + c)
2

[(a + b)(a + b + c) + ab + bc + ca]


=
3(a + b + c)
2
2(a + b + c)
2
+ 3(ab + bc + ca)
1
iu cui cng l v l, do bi ton ca ta ng.
Php chng minh hon tt.
ng thc xy ra khi v ch khi a = b = c = 1. 2
10.33 Cho a, b, c l cc s thc dng.
Chng minh rng:
a + b + c
3

(a b)
2
+ (b c)
2
+ (c a)
2
12(a + b + c)
+
3

abc
Li gii.
Cch 1:
Ta c bt ng thc cn chng minh tng ng vi
2(a + b + c)
2
6(a + b + c)

a
2
+ b
2
+ c
2
ab bc ca
6(a + b + c)

3

abc
Hay:
(a + b + c)
2
+ 3(ab + bc + ca) 6
3

abc(a + b + c)
Chun ha cho a + b + c = 3, bt ng thc tr thnh
3 + ab + bc + ca 6
3

abc
S nh gi
ab + bc + ca
_
3abc(a + b + c) = 3

abc
ta a bi ton v chng minh
1 +

abc 2
3

abc
t t =
6

abc 1, ta c bt ng thc trn tng ng vi


t
3
+ 1 2t
2
hay l
(1 t)(1 + x x
2
) 0
Bi ton c chng minh xong. 2
Cch 2:
231
Nhn 12(a + b + c) cho hai v, ta s c bt ng thc tng ng l
a
2
+ b
2
+ c
2
+ 5(ab + bc + ca) 6
3

abc(a + b + c).
Hay vit li l
(a + b + c)
2
+ 3(ab + bc + ca) 6
3

abc(a + b + c).
p dng AM-GM hai ln ta s c ngay iu phi chng minh
(a + b + c)
2
+ 3ab + bc + ca) 2(a + b + c)
_
3(ab + bc + ca)
6
3

abc(a + b + c).
ng thc xy ra khi v ch khi a = b = c. 2
10.34 Cho cc s thc dng x; y; z tha mn xy + yz + zx = 3.
Chng minh rng:
x + 2y
2x + 4y + 3z
2
+
y + 2z
2y + 4z + 3x
2
+
z + 2x
2z + 4x + 3y
2
1
Li gii.
V
x + 2y
2x + 4y + 3z
2
=
1
3

z
2
3(2x + 4y + 3z
2
)
Nn bt ng thc trn tng ng vi
x
2
2y + 4z + 3x
2
+
y
2
2z + 4x + 3y
2
+
z
2
2x + 4y + 3z
2

1
3
S dng bt ng thc Cauchy-Schwarz, ta c
z
2
2x + 4y + 3z
2

_

x
3
+
_
y
3
+

z
3
_
2
3(x
3
+ y
3
+ z
3
) + 6(xy + yz + zx)
Vy, ta cn ch ra rng
_

x
3
+
_
y
3
+

z
3
_
2
3(x
3
+ y
3
+ z
3
) + 6(xy + yz + zx)

1
3
hay l
_
(xy)
3
+
_
(yz)
3
+
_
(zx)
3
xy + yz + zx
Tht vy, s dng bt ng thc AM-GM, ta c:
_
(xy)
3
+
_
(yz)
3
+
_
(zx)
3
=
_
_
(xy)
3
+

xy
_
+
_
_
(yz)
3
+

yz
_
+
_
_
(zx)
3
+

zx
_

xy +

yz +

zx
_
2 (xy + yz + zx)
_

xy +

yz +

zx
_
(xy + yz + zx) + (xy + yz + zx)
_
3 (xy + yz + zx)
= (xy + yz + zx) + 3 3
= (xy + yz + zx)
Bi ton c chng minh xong.
ng thc xy ra khi v ch khi x = y = z = 1. 2
10.35 Cho cc s thc dng a, b, c.
Chng minh rng:
_
a
2
4a
2
+ ab + 4b
2
+
_
b
2
4b
2
+ bc + 4c
2
+
_
c
2
4c
2
+ ca + 4a
2
1
Li gii.
Cch 1:
232
S dng bt ng thc Cauchy-Schwarz, ta c :

_
a
2
4a
2
+ ab + 4b
2
[

(4a
2
+ ac + 4c
2
)]
_
a
2
(4a
2
+ ab + 4b
2
)(4a
2
+ ac + c
2
)
_
Do ta i chng minh:
[

(4a
2
+ ac + 4c
2
)]
_
a
2
(4a
2
+ ab + 4b
2
)(4a
2
+ ac + c
2
)
_
1
iu ny tng ng vi:
(8

a
2
+

ab) [8

a
2
b
2
+ abc(a + b + c)]

(4a
2
+ ab + 4b
2
)
Hay:
66a
2
b
2
c
2
8abc(a
3
+ b
3
+ c
3
) + 8(a
3
b
3
+ b
3
c
3
+ c
3
a
3
) + 3abc [a
2
(b + c) + b
2
(a + c) + c
2
(a + b)]
iu ny ng theo AM-GM.
Vy bi ton c chng minh xong.
ng thc xy ra khi v ch khi x = y = z = 1. 2
Cch 2:
rng:
(x + 1)
2
(4x
2
+ x + 4) 4(x
2
+ x + 1)
2
= x(x 1)
2
0
Nn ta c:
1

4x
2
+ x + 4

1
2

x + 1
x
2
+ x + 1
.
Thit lp 2 biu thc tng t, ri cng v theo v, ta c:
1

4x
2
+ x + 4
+
1
_
4y
2
+ y + 4
+
1

4z
2
+ z + 4

1
2
_
x + 1
x
2
+ x + 1
+
y + 1
y
2
+ y + 1
+
z + 1
z
2
+ z + 1
_
Nh vy ta cn chng minh
x + 1
x
2
+ x + 1
+
y + 1
y
2
+ y + 1
+
z + 1
z
2
+ z + 1
2
Tng ng vi
x
2
x
2
+ x + 1
+
y
2
y
2
+ y + 1
+
z
2
z
2
+ z + 1
1.
Bt ng thc ny lun ng theo Vasile Cirtoaje.
Bi ton c chng minh xong.
ng thc xy ra khi v ch khi x = y = z = 1. 2
10.36 Cho a, b, c l cc s thc khng m tha mn a
2
+ b
2
+ c
2
= 3.
Chng minh rng:
(a b)(b c)(c a) + 2
2
3
(ab + bc + ca).
Li gii.
Ta s chng minh bt ng thc mnh hn l nh sau
2
2
3
(ab + bc + ca) |(a b)(b c)(c a)| .
n y, ta c th gi s a b c 0.
rng
2
2
3
(ab + bc + ca) =
(a b)
2
+ (b c)
2
+ (c a)
2
3
.
Nn ta cn chng minh
(a b)
2
+ (b c)
2
+ (c a)
2
3
|(a b)(b c)(c a)| .
p dng bt ng thc AM-GM, ta c
233
(a b)
2
+ (b c)
2
+ (c a)
2
3

3
_
(a b)
2
(b c)
2
(c a)
2
.
Nh vy ta cn ch ra rng
3
_
(a b)
2
(b c)
2
(c a)
2
. |(a b)(b c)(c a)| .
Hay tc l (sau khi xt trng hp hai bin bng nhau)
1 |(a b)(b c)(c a)|
Bt ng thc ny ng v theo AM-GM v iu gi s ta c
|(a b)(b c)(c a)| = (a b)(a c)(b c)
ab(a b) =
_
ab ab (a b)
2

_
_
ab + ab + (a b)
2
3
_
3
=

_
a
2
+ b
2
3
_
3
1.
Nh vy, ta c iu phi chng minh. 2
10.37 Cho a, b, c l cc s thc dng.
Chng minh rng:
(a
3
+ b
3
+ c
3
)
2
(a
4
+ b
4
+ c
4
)(ab + bc + ca)
Li gii.
Ta c ng thc
(a
3
+ b
3
+ c
3
)
2
(a
4
+ b
4
+ c
4
)(ab + bc + ca) =
1
2

[(a
2
b
2
)
2
+ c
4
](a b)
2
0
T suy ra iu phi chng minh.
ng thc xy ra khi v ch khi a = b = c. 2
10.38 Cho a, b, c, d l cc s thc dng.
Chng minh rng:
4.
16

32a(a + b)(a + b + c)
3(a + b + c + d)
3
+
4
_
24bcd
(a + b)(a + b + c)(a + b + c + d)
5
Li gii.
S dng bt ng thc AM-GM ta c:
16.
16

32a(a + b)(a + b + c)
3(a + b + c + d)
3
= 16.
16

2a
a + b
.
3(a + b)
2(a + b + c)
.
3(a + b)
2(a + b + c)
.
4(a + b + c)
3(a + b + c + d)
.
4(a + b + c)
3(a + b + c + d)
.
4(a + b + c)
3(a + b + c + d)

2a
a + b
+ 2.
3(a + b)
2(a + b + c)
+ 3.
4(a + b + c)
3(a + b + c + d)
+ 10
=
2a
a + b
+
3(a + b)
a + b + c
+
4(a + b + c)
a + b + c + d
+ 10. (1)
Mt khc, cng theo bt ng thc AM-GM th:
234
4.
4

24bcd
(a + b)(a + b + c)(a + b + c + d)
= 4.
4
_
2b
a + b
.
3c
a + b + c
,
4d
a + b + c + d

2b
a + b
+
3c
a + b + c
+
4d
a + b + c + d
+ 1. (2)
Cng v theo v (1) v (2) suy ra iu phi chng minh.
ng thc xy ra khi v ch khi a = b = c = d. 2
10.39 Cho x, y, z, t l cc s thc khng m.
Chng minh rng:
3(x
2
+ y
2
+ z
2
+ t
2
) + 4

xyzt (x + y + z + t)
2
Li gii.
Ta chng minh bt ng thc tng ng (Tukervici):
x
4
+ y
4
+ z
4
+ t
4
+ 2xyzt x
2
y
2
+ y
2
z
2
+ z
2
x
2
+ t
2
x
2
+ t
2
y
2
+ z
2
t
2
Khng mt tnh tng qut, gi s t = min {x; y; z; t}
Nu t = 0 th ta c:
x
4
+ y
4
+ z
4
x
2
y
2
+ y
2
z
2
+ z
2
x
2
(x
2
y
2
)
2
+ (y
2
z
2
)
2
+ (z
2
x
2
)
2
0
Nu t > 0 ,chun ho t = 1. Ta cn chng minh:
x
4
+ y
4
+ z
4
+ 2xyz + 1 x
2
y
2
+ y
2
z
2
+ z
2
x
2
+ x
2
+ y
2
+ z
2
Mt khc, ta c bt ng thc vi 3 bin dng:
x
2
+ y
2
+ z
2
+ 2xyz + 1 2(xy + yz + zx)
nn ta cn ch ra rng
x
4
+ y
4
+ z
4
x
2
y
2
y
2
z
2
z
2
x
2
2(x
2
+ y
2
+ z
2
xy yz zx)
(x y)
2
[(x + y)
2
2] + (y z)
2
[(y + z)
2
2] + (z x)
2
[(z + x)
2
2] 0
Nh vy, php chng minh hon tt.
C 2 trng hp ca ng thc :x = y = z = t hoc x = y = z; t = 0.2
10.40 Cho a, b, c l cc s thc dng.
Chng minh rng:
(a + b + c)
_
1
a
+
1
b
+
1
c
_
9 + 8.
(a b)
2
+ (b c)
2
+ (c a)
2
(a + b + c)
2
Li gii.
Bt ng thc cn chng minh tng ng vi:
(b c)
2
bc
+
(c a)
2
ca
+
(a b)
2
ab
8.
(a b)
2
+ (b c)
2
+ (c a)
2
(a + b + c)
2

(b c)
2
_
a(a + b + c)
2
8abc

0
Khng mt tnh tng qut gi s a b c
Ta c:
S
b
+ S
a
= (a + b)(a + b + c)
2
16abc 4c(a + b)
2
16abc 0
S
b
+ S
c
= (b + c)(a + b + c)
2
16abc 4a(b + c)
2
16abc 0
2S
b
S
b
+ S
c
0 S
b
0
Nn theo nh l S.O.S ta c iu phi chng minh.
ng thc xy ra khi v ch khi a = b = c.2
235

You might also like